Sei sulla pagina 1di 218

1000 MMR Author: Victor A. Tondo Jr.

, LPT

1000
MMR
Victor A. Tondo Jr., LPT

This is a free reviewer. All rights reserved.


1000 MMR Author: Victor A. Tondo Jr., LPT

About the Author

Victor A. Tondo Jr. was a consistent Math


contest champion, a national finalist for MTAP and
PMO, a national awardee for the Intel Science Fair,
a number theorist, a coach for international
competitions, a review specialist, a licensed
teacher, a regional board top notcher, an
accomplished gamer, and a pyrographer.

This is a free reviewer. All rights reserved.


1000 MMR Author: Victor A. Tondo Jr., LPT

Acknowledgment
To my mom, Analyn N. Torizuka, for your all-out support in my entire life, even after I spent more
than four years playing games on the computer. I love you so much! Without you, I am nothing.
To my sister, Vera-Lee A. Tondo, for always sensing my emotions and knowing everything that I
am going through. Your unwavering support kept me sane, especially after the lowest points in my life.
To my daughter, Aira Quineisha R. Tondo, for being the sweetest, most loving daughter I could
ever ask for. I have been loving you since the day you were born, and I will always be very proud of you.
To my teachers and professors, for molding me into the person I am today. You have taken good
care of me, even with my special needs. You did your best to understand me, brought out the best in me,
and inspired me to be in the same profession as you. I will always be grateful for your gift.
To Ma’am Romana Valdecasa, for turning a poor nobody into someone who speaks and breathes
Mathematics. You saw my potential and made me shine brighter than any star. You really are my second
mother – you figuratively gave birth to the successful Victor that I am now. I can never thank you enough.
To Ma’am Joaquina Birung, for always looking after me. You have been a very crucial factor to my
success during high school and even during college. You took me under your wings. You have been my
teacher, my coach, my mentor, my cooperating teacher, and of course, one of my favorite mothers.
To Sir Pedro Suyu, to Sir Manuel Belango, and to the beautiful Ma’am Apolinaria Andres, for
instilling in me the principles that a teacher must live by.
To my beloved Ma’am Sally Caldez, for never giving up on me. You are perhaps my second lola, and
I will always love you. May you rest in peace.
To Ma’am Loida Calonia, for teaching me your Christian ways. While others have given up when
they knew I am an atheist, you stood by my side patiently. When I was close to committing suicide, you
were there to guide me. You extended my life past the age of 30.
To my “daughters” Marone Khyme and Marby Alzate, for treating me not only as your tutor but
also as a friend, as your tatay, and as your mentor. I love you both, and I have always been proud of you.
To my students, tutees, reviewees, subscribers, and friends, for always believing in me.
At sa aking Inang Bayan, inaalay ko sa iyo itong unang lathalain ko. Ang tanging hangad ko lang ay
ang ika-uunlad mo.

Victor A. Tondo Jr., LPT

This is a free reviewer. All rights reserved.


1000 MMR Author: Victor A. Tondo Jr., LPT
Tips for Reviewing

1. Expand your Mathematical vocabulary. You will find it difficult to learn concepts when your
Mathematical vocabulary is limited.
2. Do not memorize the answers. Instead, know how to use the solution, how it works, and when it should
be used.
3. Remember that there are many ways to deal with problems. Nobody wants a long solution when there
are several shortcuts out there.
4. Try answering the items first before reading the solutions or explanations. This will give you an idea on
your current standing and at the same time, what kinds of questions you should expect in the exam.
5. Familiarize yourself with your own scientific calculator. Know where the different buttons are, like nPr,
nCr, and n!. You should also know what these buttons do, and how to make them function.

Strategies in Answering the Mathematics Test


During the Licensure Exam

1. Keep track of the time. Before answering the questions, take note of the number of items and the time
allotted for the exam. Estimate the average time you should allot for each item.
2. Scan the items. Go over the test booklet quickly to get an idea of the types of questions or problems in
the test (such as word problems, analysis of graphs and tables, simple equations). This will help you
determine the appropriate pace to take in answering the test.
3. Pace yourself. Work quickly on topics you have mastered without being rash and careless. When you
encounter a difficult question, skip it and move on to less difficult ones. When you have finished
answering all the easy questions, go back to the skipped items.
4. Keep your pen going. On your second reading of the item, create a picture of the problem and draw that
picture on your scratch paper. Take down the relevant facts (numbers, units, dimensions), specially what
is asked for in the problem. Many items that seem difficult on first reading become easier to handle after
the facts have been taken down.
5. Visualize, draw diagrams. These help in answering word problems. Familiarize yourself with the types
of diagrams in every topic as these may prove useful in solving problems.
6. Eliminate choices. The right answer can be determined if all but one of the choices are eliminated.
Don’t easily give up on difficult items.
7. Work backwards. This method is useful if you find it difficult to start a solution to a problem. To work
backwards is similar to a trial-and-error on the choices. This can also be used to check if your answer is
correct.

This is a free reviewer. All rights reserved.


1000 MMR Author: Victor A. Tondo Jr., LPT

1000
Questions

This is a free reviewer. All rights reserved.


1000 MMR Author: Victor A. Tondo Jr., LPT
1. How many line segments can be made from 30 8. What are the missing terms in the series
non-collinear points? 5, 20, 80, ___,1280, ___, 20480?
A. 900 B. 870 A. 50; 210 B. 40; 160
C. 450 D. 435 C. 35; 135 D. 320; 5120

2. Calculate the mean absolute deviation of the 9. At what rate per annum should P2400 be
following numbers: 60, 80, 100, 75 and 95 invested so that it will earn an interest of P800 in
A. 12.4 B. 14.2 8 years?
C. 16.1 D. 18.9 A. 6 ½ % B. 5 ½ %
C. 4.17 % D. 6 %

3. Which of the following is the factorization of


the binomial x2 - 42? 10. The area of a rectangle is (x2 + 2x - 8). If its
A. (x + 4)(x + 2) length is x + 4, what is its width?
B. (x – 4)2 A. x + 2 B. x - 2
C. x(x + 2x + 2) C. x + 1 D. x + 6
D. (x – 4)(x + 4)

11. What is the value of 12⅙ - 3 ⅜ - 5 ⅔ + 20 ¾?


4. What value of x will satisfy the equation: A. 21 B. 22 C. 23 D. 21
0.4(5x – 1470) = x?
A. 490 B. 2,130
C. 1470 D. 588
12. The vertex angle of an isosceles triangle is
20°. What is the measure of one of the base
angles?
5. Which of the following is ALWAYS true?
A. 150° B. 60° C. 75° D. 80°
A. Vertical pairs of angles are supplementary.
B. Vertical pairs of angles are complementary.
C. Linear pairs of angles are congruent.
13. Ana and Beth do a job together in three
D. Linear pairs of angles are supplementary.
hours. Working alone, Ana does the job in 5
hours. How long will it take Beth to do the job
alone?
6. The average of 5 different counting numbers is
A. 3 and 1/3 hours B. 2 and 1/3 hours
20. What is the highest possible value that one of
C. 3 hours D. 7 and 1/2 hours
the numbers can have?
A. 20 B. 40 C. 30 D. 90
14. How much greater is the sum of the first 100
counting numbers than the sum of the first 50
7. Three brothers inherited a cash amount of
counting numbers?
P62,000 and they divided it among themselves in
A. 110 B. 3,775
the ratio of 5:4:1. How much more is the largest
C. 3,155 D. 1200
share than the smallest share?
A. P75,000 B. P30,000
C. P24,800 D. P37,200
15. Which of the following has the largest value?
A. 85 B. 39 C. 65 D. 94

This is a free reviewer. All rights reserved.


1000 MMR Author: Victor A. Tondo Jr., LPT
16. A water tank contains 18 liters when it is 23. One side of a 45° - 45° - 90° triangle
20% full. How many liters does it contain when measures x cm. What is the length of its
50% full? hypotenuse?
A. 60 B. 30 C. 58 D. 45 A. x √3 cm B. x cm
C. (x √3)/2 cm D. x √2 cm

17. The edges of a rectangular solid have these


measures: 1.5 feet by 1½ feet by 3 inches. What 24. The legs of one right triangle are 9 and 12,
is its volume in cubic inches? while those of another right triangle are 12 and
A. 324 B. 225 C. 972 D. 27 16. How much longer is the perimeter of the
larger triangle than the perimeter of the smaller
triangle?
18. In a certain school, the ratio of boys to girls is A. 84 B. 7 C. 12 D. 14
5 is to 7. If there are 180 boys and girls in the
school, how many boys are there?
A. 105 B. 90 25. An online shop sells a certain calculator for
C. 45 D. 75 P950 and charges P150 for shipping within
Manila, regardless of the number of calculators
ordered. Which of the following equations shows
19. Ruben’s grades in 6 subjects are 88, 90, 97, the total cost (y) of an order as a function of the
90, 91 and 86. What is the grade that he should number of calculators ordered (x)?
aim for in the 7th subject if he has to have an A. y = (950 + 150)x B. y = 150x +950
average of 91? C. x = 950y + 150 D. y = 950x + 150
A. 97 B. 95 C. 92 D. 89

26. Which of these has the longest perimeter?


20. On a certain day, three computer technicians A. A square 21 cm on a side
took turns in manning a 24-hour internet shop. B. A rectangle 19 cm long and 24 cm wide
The number of hours Cesar, Bert, and Danny C. An equilateral triangle whose side is 28 cm
were on duty was in the ratio 3:4:5, respectively. D. A right triangle whose two legs are 24 and 32
The shop owner pays them P50 per hour. How cm
much would Danny receive for that day?
A. P 230 B. P500
C. P160 D. P480 27. How many square inches are in 2 square
yards?
A. 900 B. 144
21. A retailer buys candies for P90.25. The pack C. 1296 D. 2,592
has 35 pieces of candies. If she sells each candy
for P3.25, how much profit does she make?
A. P11.50 B. P23.50 28. In a playground for Kindergarten kids, 18
C. P37.50 D. P18.75 children are riding tricycles or bicycles. If there
are 43 wheels in all, how many tricycles are
there?
22. Determine the midpoint of the line segment A. 8 B. 9 C. 7 D. 11
joining the points (7, -3) and (-1, 6).
A. (2, 3/2) B. (2, -3/2)
C. (3, 3/2) D. (1, 5/2)

This is a free reviewer. All rights reserved.


1000 MMR Author: Victor A. Tondo Jr., LPT
29. Aira takes ¾ hour to dress and get ready for 36. Find the domain of f(x) = .
school. It takes 4/5 hour to reach the school. If
her class starts promptly at 8:00 am; what is the A. x B. x = 1
latest time she can jump out of bed in order not C. x = -1 D. x ,x -1
to be late for school?
A. 6:42 am B. 6:27 am
C. 6:57 am D. 7:02 am 37. A car travels D km in H hours. Which of the
following expressions shows the distance
travelled by the car after M minutes?
30. Which common fraction is equivalent to A. MD/H B. 60MD/H
C. MD/60H D. 60HD/M
0.215?
A. 43/200 B. 27/125
C. 21/50 D. 108/375
38. Find the surface area of a rectangular box
whose dimensions are 30 cm x 40 cm x 50 cm.
31. What are the next three terms in the A. 4700 cm2 B. 7050 cm2
progression 1, 4, 16 …? C. 9400 cm2 D. 11750 cm2
A. 64, 256, 1024 B. 67, 259, 1027
C. 48, 198, 1026 D. 65, 257, 1025
39. If x – y = 3, then (y-x)-3 = ___.
A. 9 B. -9
32. A man is 3 times as old as his son now. Four C. 1/27 D. -1/27
years ago, the sum of their ages was 36. Find the
man’s age now.
A. 33 B. 11 40. Factorize (x4 – 81) completely.
C. 29 D. 36 A. (x-3)4
B. (x – 3)2 (x + 3)2
C. (x+3) (x-3) (x2+9)
33. What is the least common multiple of 12, 24 D. (x+3)3 (x-3)
and 72?
A. 12 B. 72 C. 144 D. 36
41. √8 + √18 √2 = ____
A. 4√2 B. 5√2 C. √24 D. 2√6
34. The hypotenuse of a right triangle is 25 feet.
If one leg is 24 feet, what is the length of the
other leg? 42. By which property can we state the
A. 6 ft. B. 5 ft. C. 20 ft. D. 7 ft. following:
“If ax + b = c, then ax + b - b = c – b.”
A. transposition B. transitive
35. If two variables X and Y are directly related, C. additive inverse D. addition property
which of these is NOT true?
A. When X is low, Y is also low.
B. As X increases, Y also increases. 43. The midpoint of P and (-7, 4) is (-3, 1). What
C. When X increases, Y decreases. are the coordinates of P?
D. A high Y is associated with a high X. A. (-5, 5/2) B. (-11, 7)
C. (1, -2) D. (-2, 3/2)

This is a free reviewer. All rights reserved.


1000 MMR Author: Victor A. Tondo Jr., LPT
44. What is the slope of the line 3x – y = 11? 52. There are 33 red bags, 25 green bags, and 17
A. -1/3 B. 1/3 C. -3 D. 3 blue bags in a store. What percent of the bags is
red?
A. 33% B. 44%
45. What is the minimum value of C. 66% D. 67%
f(x) = 3x2 + 6x + 7?
A. 1 B. -1 C. 4 D. -4
53. Given sin θ = 0.28, which of the following
could possibly be cos θ?
46. If xy = 23 and x2 + y2 = 75, find x + y. A. 0.72 B. -0.86
A. 10.7845 B. 11 C. 0.96 D. 1.14
C. 11.2155 D. 11.7845

54. If the sum of the supplement and the


47. How much water must be evaporated from complement of an angle is 130 degrees, what is
90 ml of a 50% salt solution to increase its the angle?
concentration to 75%? A. 65o B. 70o
A. 40 ml B. 38 ml C. 50o D. 25o
C. 35 ml D. 30 ml

55. If today is a Saturday, what day is 125 days


48. A and B form a vertical pair. If m A = 3x from now?
and m B = 5x – 44, what is the value of x? A. Thursday B. Friday
A. 50.5 B. 28 C. 22 D. 16.75 C. Sunday D. Monday

49. The angle of elevation from an observer to 56. Car A is traveling towards the east at a speed
the top of a building is 30o. If the building is 50 of 35 kph, while car B is traveling towards the
meters high, how far is the observer from the west at 45 kph. If they left the same point at 1:00
building? PM, how far apart are they at 3:45 PM?
A. 25 B. 25√3 C. 50√3 D. 100 A. 240 km B. 220 km
C. 200 km D. 180 km

50. 1 and 3 are opposite angles in a


parallelogram. If m 1 = 40o, what is m 3? 57. Mr. Santos left the house at 1:00 PM and
A. 40o B. 50o C. 70o D. 140o traveled east at an average speed of 40 kph. His
wife Mrs. Santos left the at 2:00 PM and traveled
west at an average speed of 30 kph. How far
51. Two parallel lines are cut by a transversal, apart are they at 4:00 PM?
forming H and K. If the two angles are A. 180 km B. 140 km
exterior angles on the same side of the C. 100 km D. 60 km
transversal, what is the measure of H if the
measure of K is 50o?
A. 25o B. 50o 58. Five consecutive even numbers have a sum of
C. 100 o D. 130o 120. What is the sum of the 2nd and 5th even
numbers?
A. 46 B. 48 C. 50 D. 52

This is a free reviewer. All rights reserved.


1000 MMR Author: Victor A. Tondo Jr., LPT
59. If x = 3, which of the following is equal to 13? 68. If A and B are the roots of x2 + 7x + 15, what
A. 5x + 2 B. x2 + 2x + 1 is AB?
C. x – 4x – 2
3 D. x2 + x + 2 A. 7√3 + 2 B. 2√3 + 7
C. 3√2 + 2√3 D. 15

60. If f(x) = x2 + 4x + 3, which of the following is


equal to 99? 69. 1 + 2 + 4 + 8 + … + 2048 = ____
A. f(11) B. f(-12) A. 4095 B. 4096
C. f(12) D. f(-8) C. 4097 D. 4098

61. Given f(x) = ln , what is f ‘(x)? 70. 24 + 12 + 6 + 3 + 1.5 + … = ____


A. B. A. 48 B. 50 C. 54 D. 60
C. (2x+2) ln (x2+2x) D. 2x + 2
71. How many terms are there in the sequence
5, 13, 21, 29, …, 357?
62. Which of the following could be the value of x
A. 40 B. 44
if x 3(mod 11)?
C. 45 D. 70
A. 33 B. 47 C. 52 D. 2

72. How many ways can a group of 5 be selected


63. If = 6x2 + 8x – 7, which could be u? from 5 boys and 5 girls if the group must contain
A. 12x + 8 B. 3x3 + 4x2 – 7x + 11 3 boys and 2 girls?
C. 2x + 4x -7x +1 D. 12x2 + 8x - 10
3 2 A. 151,200 B. 1200
C. 252 D. 100

64. What is the center of x2 + y2 – 8x + 6y = 0?


A. (-8.6) B. (8, -6) 73. What is the probability of getting a sum of 9
C. (-4, 3) D. (4, -3) when rolling 2 dice?
A. 1/9 B. 5/36
C. 1/6 D. 7/36
65. Which of the following is a parabola that
opens to the right?
A. 6y = (x+9)2 - 8 B. -4y = (x-6)2 + 3 ̅̅̅̅ where A is at (-3,4)
74. C is the midpoint of AB
C. -5x + 3 = (y-2)2 D. 2x + 6 = (y+3)2 and B is at (7,-10). Find the coordinates of C.
A. (5,-7) B. (-5,7) C. (2,-3) D. (-2,3)

66. Factorize: 12x2 – 7x – 10.


A. (6x + 5) (2x – 2) B. (6x – 2) (2x + 5) 75. It is a line segment formed by connecting two
C. (3x + 2) (4x – 5) D. (3x – 2) (4x + 5) non-consecutive vertices of a polygon.
A. side B. apothem
C. altitude D. diagonal
67. For which value of k does 4x2 + kx + 49 have
only one root?
A. -28 B. -14 C. 7/2 D. -7/4

This is a free reviewer. All rights reserved.


1000 MMR Author: Victor A. Tondo Jr., LPT
76. Find the equation of the line perpendicular to 84. Given ̅̅̅̅
BT bisects ABC and m ABT = 40o,
2x – 3y = 7, passing through (1,2). find m ABC.
A. 2x + 3y = 8 B. 3x + 2y = 7 A. 20o B. 40o C. 60o D. 80o
C. 2x – 3y = -4 D. 3x – 2y = -1

85. A cone has a radius of 9 cm and a slant height


77. Two parallel lines are cut by a transversal to of 15 cm. Find its volume.
form X, Y, and Z. Given that X and Y are A. 243 π cm3 B. 324 π cm3
alternate interior angles while Y and Z are C. 405 π cm 3 D. 486 π cm3
interior angles on the same side of the
transversal, find m Z if m X = 40o.
A. 40o B. 50o 86. If f(x) = x2 + 4x + 4 and g(x) = x-2, find
C. 130 o D. 140o f(g(x)).
A. x2 B. x3 – 6x2 + 6x – 9
C. x + 8x + 16
2 D. x2 – 8x + 16
78. The measure of each interior angle of a
regular polygon is 144o. How many vertices does
it have? 87. A 10 ft ladder leans against a wall, forming a
A. 36 B. 24 C. 12 D. 10 30o angle with it. How high on the wall does it
reach?
A. 5 ft B. 5 √3 ft
79. Solve: (x + 9) (x – 3) < 0 C. 10 √3 ft D. 10 √6 ft
A. -9 < x < 3 B. x < -3 x > 9
C. x < -9 x > 3 D. x ; x -9, 3
88. How many ways can a committee of 5 be
selected from 9 people?
80. The product of two consecutive even A. 126 B. 120
counting numbers is 3248. Find the smaller C. 3024 D. 15120
number.
A. 42 B. 46 C. 52 D. 56
89. What is 60% of 80% of 500?
A. 480 B. 240
81. Solve for x: 2log2 3 – log2 18 = x C. 120 D. 60
A. ½ B. -1 C. -2 D. 1

90. If 3x = 7 and 2y = 5, what is 6(x-y)?


82. Twinkle Bucks has four serving sizes for their A. -1 B. 1-√35
milk tea: Small, Medium, Large, and Extra Large.
C. √7 - √5 D.
What level of data are they using for their
serving sizes?
A. nominal B. ordinal
C. interval D. ratio 91. If two numbers have a product of 71 and the
sum of their squares is 147, what is their sum?
A. -17 B. 5
83. After receiving a 20% markup, a bag was sold C. 12√3 + √5 D. 12 + √3
for P960. How much was it originally?
A. P1152 B. P4800
C. P800 D. P1200

This is a free reviewer. All rights reserved.


1000 MMR Author: Victor A. Tondo Jr., LPT
92. Find the median: 7, 9, 11, 10, 9, 13, 17, 14 99. Which of the following has two diagonals that
A. 10 and 11 B. 9 and 10 are perpendicular bisectors of each other?
C. 10.5 D. 9.5 A. kite B. rectangle
C. rhombus D. isosceles trapezoid

93. How many 3-digit numbers can be formed


using the digits 0, 1, 2, 3, 4 and 5 if repetition is 100. A pipe can fill a pool in 6 hours while
not allowed? another pipe can drain empty the pool in 15
A. 60 B. 80 C. 100 D. 120 hours. How long will it take to fill the pool if both
pipes are open?
A. 9 hours B. 9.125 hours
94. How many ml of 20% acid must be added to C. 9.45 hours D. 10 hours
400 ml of 50% acid to make a 30% acid solution?
A. 1000 ml B. 900 ml
C. 800 ml D. 750 ml 101. If log n – 1 = 2, find n.
A. 3 B. 1000 C. e3 D. 3e

95. How many ml each of 10% and 50% solution


should be mixed to make 500 ml of 18% 102. log2 3 + 2 log2 7 – log2 5 = ______.
solution? A. log2 B. log2
A. 400 ml of 10% and 100 ml of 50%
B. 350 ml of 10% and 150 ml of 50% C. log2 D. log2
C. 300 ml of 10% and 200 ml of 50%
D. 200 ml of 10% and 300 ml of 50%
103. The surface areas of two spheres are 12 π
cm2 and 108 π cm2. What is the ratio of their
96. It takes 28 men a total of 24 days to build a volumes?
house. How long would it take 32 men to build a A. 1:3√3 B. 1:9
similar house? C. 1:27 D. 2:3√3
A. 28 days B. 27 days
C. 21 days D. 19 days
104. The volume of a regular hexahedron is 64
in3. How long is each side?
A. 2 in B. 4 in C. 6 in D. 8 in
97. Evaluate: lim
A. undefined B. limit does not exist
C. 8 D. + 105. Which of the following statements is
ALWAYS true?
A. The square of a prime number is odd.
98. A box contains 7 red, 8 blue, and 9 white B. The sum of two consecutive even numbers is
balls. When taking two balls in succession, what divisible by 4.
is the probability that both balls are white? C. Any even number is composite.
A. 9/64 B. 9/69 D. The product of two consecutive even numbers
C. 7/64 D. 7/69 is divisible by 8.

This is a free reviewer. All rights reserved.


1000 MMR Author: Victor A. Tondo Jr., LPT
106. Find the volume of a steel cylinder of radius 114. Find the area of the triangle whose vertices
5 cm and height 12 cm. are (1,4), (2,3), and (3,0).
A. 300 π cm3 B. 250 π cm3 A. 0 B. 1 C. 5/3 D. 3/4
C. 200 π cm3 D. 100 π cm3

115. Find the tenth term: 3, 10, 17, 24, …


107. A cube sits perfectly inside a sphere of A. 66 B. 67 C. 68 D. 69
volume 108 √3 π cm3. Find the volume of the
cube.
A. 27 cm3 B. 54 cm3 116. Find the remainder when
C. 108 cm3 D. 216 cm3 x4 – 5x3 + 6x2 + 2x + 1 is divided by (x – 2).
A. 17 B. 13 C. 9 D. 5

108. Find the distance in cm of an 80 cm chord


from the center of a circle whose radius is 41 cm. 117. The sum of Fe’s age and Sita’s age is 60.
A. 41 - 2√10 B. 41 - 4√10 Twelve years ago, Fe was twice as old as Sita.
C. 9√2 D. 9 How old is Sita now?
A. 18 B. 24 C. 30 D. 36

109. Which quadrilateral has two congruent


diagonals that bisect each other? 118. If the length of a rectangle is increased by
A. kite B. isosceles trapezoid 20% while the width is decreased by 10%, what
C. rectangle D. rhombus will happen to its area?
A. decreased by 10%
B. increased by 10%
110. What is the longest side of ∆MTC if m M = C. increased by 8%
40o and m C = 60o? D. decreased by 2%
̅̅̅̅
A. MC ̅̅̅̅
B. TC ̅̅̅̅
C. MT ̅̅̅̅
D. CT
119. The 19th term of an arithmetic sequence is
111. Find the altitude to the hypotenuse of a 85 and the 12th term is 43. Find the common
right triangle whose legs measure 10 cm and 24 difference.
cm. A. 5 B. 6 C. 7 D. 8
A. 120 cm B. cm
C. 120√2 cm D. 24√5 cm 120. If 2x = 3y and 4y = 5z, what is z in terms of
x?
A. z = x B. z = x
112. Find the inverse of y = x2 + 10x.
C. z = x D. z = x
A. y-1 = √ 25 + 5
B. y-1 = √ 25 – 5
C. y-1 = √ + 25 + 5
121. Victor had an average of 94 on his first four
D. y-1 = √ + 25 – 5
Math tests. After taking the next test, his average
dropped to 93. Find his most recent grade.
113. Find the intersection of y = 2x + 3 and
A. 88 B. 89 C. 90 D. 91
y = 4x – 11.
A. (-4/3, 0) B. (4/3, 0)
C. (7, 17) D. (-7,-17)

This is a free reviewer. All rights reserved.


1000 MMR Author: Victor A. Tondo Jr., LPT
122. X is of Y and Y is of Z. What part of Z is X? 129. Evaluate when x = ¾ and y = .
A. X = Z B. X = Z A. -38 B. -19 C. 19 D. 38
C. X = Z D. X = Z
130. Today, Vic is 11 years old while his father is
37. How many years from now will his father be
123. Two buses leave the same station at 8:00 twice as old as he?
pm. One bus travels north at the rate of 30 kph A. 15 B. 13 C. 11 D. 10
and the other travels east at 40 kph. How many
kilometers apart are the buses at 10 pm?
A. 140 km B. 100 km 131. Carla and Diana are on a seesaw. Carla
C. 70 km D. 50 km weighs 50 kg and sits 168 cm to the left of the
fulcrum. If Diana weighs 60 kg, how far to the
right of the fulcrum must she sit to balance the
124. A bus drove for 6 hours at 75 kph and 4 seesaw?
hours at 80 kph. What was its average speed? A. 140 cm B. 170.8 cm
A. 76 kph B. 77 kph C. 201.6 cm D. 210 cm
C. 77.5 kph D. 78 kph

132. Twenty guests shake hands with each other.


125. 18 students failed a quiz. They represent If each guest is to shake hands with all the other
30% of the class. How many students passed the guests, how many handshakes will be made?
quiz? A. 400 B. 380 C. 200 D. 190
A. 60 B. 42 C. 36 D. 24

133. How many line segments can be made from


126. Rationalize: 30 non-collinear points?

√ A. 900 B. 870 C. 450 D. 435
A. +1 B. 2√5 – 4
√ √
C. D.
134. The longest chord of a circle is 80 cm. How
long is its radius?
A. 20 cm B. 30 cm
127. RNHS has 130 quizzers. 67 of them are
C. 20√2 cm D. 40 cm
Math, 60 are Science, and 20 are quizzers for
both Math and Science. How many quizzers are
neither Math nor Science?
A. 0 B. 13 C. 17 D. 23 135. Find k such that 34k67 is divisible by 9.
A. 5 B. 6 C. 7 D. 8

128. Mr. Tondo has P100,000 to invest, from


which he wants to earn P5600 per year. Bank A 136. Find the largest area of a rectangle whose
offers 5% per annum while Bank B offers 6%. perimeter is 100 cm.
How much should he invest at Bank B? A. 2500 cm2 B. 2499 cm2
A. P45,000 B. P50,000 C. 625 cm 2 D. 624 cm2
C. P55,000 D. P60,000

This is a free reviewer. All rights reserved.


1000 MMR Author: Victor A. Tondo Jr., LPT
137. What time is 200 minutes past 10:30 PM? 146. Insert one term between 18 and 32 to make
A. 12:30 AM B. 12:30 PM a geometric sequence.
C. 1:50 AM D. 1:50 PM A. 20 B. 24 C. 25 D. 27

138. Find the product of two numbers whose 147. There are 100 pigs and chickens in a farm,
GCF is 24 and LCM is 120. all of which are healthy. If there are 340 legs in
A. 2880 B. 1440 C. 720 D. 360 total, how many pigs are there?
A. 70 B. 65
C. 60 D. 55
139. The salary of 4 men for 5 days is P9,000.
How much is the salary of 5 men for 6 days?
A. P12,000 B. P12,600 148. Adam can do a job alone in 8 hours, while
C. P13,500 D. P14,400 Bam can do the same job in 12 hours. One day,
they worked together for 1 hour before Bam left
Adam to finish the job. How long will it take
140. The average grade of eleven students is 83. Adam to finish the remaining job?
If the average of six of these students is 88, what A. 6 hrs 50 mins B. 6 hrs 40 mins
is the average of the other 5 students? C. 6 hrs 30 mins D. 6 hrs 20 mins
A. 77 B. 78 C. 79 D. 80

149. Find x if 2748 = 9x.


141. If x is 80% of y, what percent of y is x? A. 144 B. 81 C. 72 D. 60
A. 120% B. 125%
C. 130% D. 135%
150. Solve for x: 49x = 343
A. 1.142857 B. 7
142. Bus X left the terminal at 1 PM and traveled C. 1.5 D. √7
at a speed of 60 kph. Bus Y left the same terminal
2 hours later and traveled 80 kph on the same
route. What time will Bus B catch up with Bus A? 151. What is the highest possible product of two
A. 6 PM B. 9 PM numbers if their sum is 45?
C. 11 PM D. 1 AM A. 506 B. 506.25
C. 506.5 D. 506.725

143. What is the degree of the polynomial


-3 x2y3 + 21 x3y4 – 7 x5y6 – 15? 152. Which statistical test is used for comparing
A. 4 B. 5 C. 11 D. 21 observed frequencies to expected frequencies?
A. ANOVA B. t-test
C. Pearson R D. Chi Square
144. The average of x+5, 2x-4, and x+7 is 20.
Find x.
A. 18 B. 13 C. 9 D. 8 153. The product of two consecutive odd
counting numbers is 1443. What is their sum?
A. 76 B. 78 C. 80 D. 82
145. Mia is 16 years younger than Kia. 13 years
ago, Kia was thrice as old as Mia. What is Kia’s
present age?
A. 43 B. 40 C. 37 D. 34
This is a free reviewer. All rights reserved.
1000 MMR Author: Victor A. Tondo Jr., LPT
2 +1 4 162. Which of the following angles in standard
154. Given ( ) = { 4 = 4, position is coterminal with 40o?
7 4 A. 2200o B. 1760o
find lim ( ). C. 1520o D. 1360o
A. 4 B. 9
C. 0 D. limit does not exist
163. Find the equation of the line passing
through (2,7) and (-3,-3).
155. If today is a Saturday, what day is 125 days A. y = 4x -1 B. y = 3x + 1
from now? C. y = 3x + 6 D. y = 2x + 3
A. Friday B. Sunday
C. Monday D. Tuesday
164. In which quadrant can we find θ if tan θ < 0
and sin θ > 0?
156. If the sum of the supplement and the A. First Quadrant
complement of an angle is 124, what is the B. Second Quadrant
angle? C. Third Quadrant
A. 71 B. 72 C. 73 D. 74 D. Fourth Quadrant

157. Find + given x + y = 20 and xy = 81. 165. Find the equation of the line passing
through the point of origin and (3,4).
A. B. C. D.
A. y = x B. y= x
C. y = x + D. y = x + 1
158. What is the remainder when
534,214,557,989,215 is divided by 4?
A. 0 B. 1 C. 2 D. 3 166. Find the range of f(x) = -2x2 + 4x.
A. y 2 B. y 2
C. y -2 D. y -2
159. Dividing by 0.125 is the same as multiplying
by which number?
A. 5 B. 8 C. 10 D. 16 167. If a3/2 – 1 = 7, what is a?
A. 4 B. 8 C. 9 D. 18

160. Find the surface area of a sphere whose


radius is 6 cm. 168. Which of the following is true?
A. 72 π cm2 B. 108 π cm2 A. A rectangle is a square.
C. 144 π cm 2 D. 192 π cm2 B. A rhombus is a rectangle.
C. A trapezoid is a rhombus.
D. A square is a rhombus.
161. Which of the following is the reference
angle of 216o?
A. 84o B. 66o C. 54o D. 36o 169. What is the measure of each exterior angle
of a pentagon?
A. 108o B. 72o
C. 60o D. 36o

This is a free reviewer. All rights reserved.


1000 MMR Author: Victor A. Tondo Jr., LPT
170. How many diagonals does a nonagon have? 178. Which of the following has its incenter,
A. 27 B. 36 C. 45 D. 54 circumcenter, centroid, and orthocenter in just
one point?
A. Right Triangles B. Equilateral Triangles
171. What is the fractional equivalent of C. Isosceles Triangles D. Scalene Triangles
0.123123123123…?
A. B. C. D.
179. Dexter is twice as heavy as Pablo. Ming is
4kg heavier than Pablo. The sum of their masses
is 164kg. How heavy is Dexter?
172. Mrs. Pasay saved P250 after buying a phone
A. 40 kg B. 44 kg
with a 10% discount. How much did she pay for
C. 80 kg D. 88 kg
the phone?
A. P2500 B. P2250
C. P2000 D. P1750
180. A circle is drawn inside a triangle such that
it is tangent to the sides of the triangle. Its center
will be the triangle’s ___________________.
173. A book was sold for P270 after a 10% A. Incenter B. Circumcenter
discount was given. How much was the book C. Centroid D. Orthocenter
originally?
A. P330 B. P300
C. P297 D. P280 181. Rayon can do a job in 3 hours, while Carlyn
can do the same job in 7 hours. How long will it
take them to finish the job by working together?
174. Find the area of an equilateral triangle A. 2.1 hours B. 2.5 hours
whose sides measure 12 cm each. C. 5 hours D. 10 hours
A. 36√3 cm2 B. 48√3 cm2
C. 60√3 cm2 D. 72√3 cm2
182. This line is perpendicular to one side of the
triangle passing through the opposite vertex.
175. This is located at the intersection of the A. Longitude B. Median
angle bisectors of a triangle. C. Altitude D. Bisector
A. Incenter B. Circumcenter
C. Centroid D. Orthocenter
183. How many ways can Lola Leonor arrange
her six meals on the Lazy Susan (the rotating
̅̅̅̅ is 9 cm long
176. ∆ABC is similar to ∆DEF. AB circular wooden server on top of the table)?
̅̅̅̅
while DE is 12 cm long. If the area of ∆ABC is A. 720 B. 120 C. 36 D. 30
27 cm2, what is the area of ∆DEF?
A. 36 cm2 B. 48 cm2
C. 60 cm2 D. 72 cm2 184. In parallelogram
MATH, m M = 7x – 12 and
m T = 5x + 32. Find m A.
177. Find the remainder when x4 – 3x3 + 2x2 + A. 22 B. 38
3x – 9 is divided by (x-3). C. 44 D. 142
A. -18 B. -9 C. 9 D. 18

This is a free reviewer. All rights reserved.


1000 MMR Author: Victor A. Tondo Jr., LPT
185. Find the equation of the line perpendicular 193. Which of the following is false?
to 2x + 5y = 7, passing through (1, 2). A. sin2 θ + cos2 θ = 1
A. 2x + 5y = 12 B. 2x – 5y = -8 B. sin θ (csc θ) = 1
C. 5x + 2y = 9 D. 5x – 2y = 1 C. sin θ ÷ cos θ = tan θ
D. sin θ (tan θ) = cos θ

186. How many ways can the letters of the word


BANANA be rearranged? 194. If three-fourths of a number is 33 more than
A. 720 B. 240 C. 120 D. 60 its one-fifth, what is that number?
A. 240 B. 120
C.90 D. 60
187. “The temperature in Baguio City is 20o
while the temperature in Tuguegarao City is
40o”. What level of data is temperature in 195. Which of the following has the greatest
degrees Celsius? value:
A. Nominal B. Ordinal A. 3 + 32 + (3 + 3)2 B. 33
C. Interval D. Ratio C. [(3 + 3) ]
2 2 D. (3 + 3 + 3)2

188. What is formed by the intersection of two 196. Which of the following has an undefined
planes? slope?
A. a point B. a line A. a vertical line
C. a plane D. space B. a horizontal line
C. a line parallel to the x-axis
D. a diagonal line
189. What is formed when a plane intersects a
cone parallel to its circular base?
A. ellipse B. hyperbola 197. In solid geometry, what do you call a solid
C. circle D. parabola bound by polygons?
A. multigon B. tessellation
C. porygon D. polyhedron
190. In which non-Euclidean model for geometry
can we have any given line ℓ and a point A which
is not on ℓ, wherein all lines through A will 198. Tchr. Victor needs to randomly get 10 out of
intersect ℓ? his 50 students for drug testing. He proceeds by
A. hyperbolic B. elliptic making the students count off from 1 to 5. He
C. Saccheri D. Pythagorean then randomly picks a number from 1 to 5.
Which sampling method did he use?
A. stratified B. cluster
191. Which numerical system is sexagesimal C. systematic D. convenience
(base-60)?
A. Mayan B. Roman
C. Babylonian D. Hindu-Arabic 199. Which statistical test must be used in
testing the significance of group differences
between 2 or more groups?
192. Which numerical system makes use of dots A. Chi Square B. t-test
and horizontal lines, and shell shapes for zero? C. ANOVA D. Pearson R
A. Egyptian B. Roman
C. Greek D. Mayan
This is a free reviewer. All rights reserved.
1000 MMR Author: Victor A. Tondo Jr., LPT
200. Which Mathematician is famous for the 207. Victor deposited an amount of P200,000 in
Fibonacci sequence? a bank that offers 5% interest compounded per
A. Ptolemy annum. How much will he have in his account
B. Leonardo Pisano Bigollo after 3 years?
C. Pierre de Fermat A. P230,000 B. P231,525
D. Luca Pacioli C. P233,050 D. P234,575

201. Which Mathematician is famous for his last 208. Find the remainder when the polynomial
theorem? x4 – 3x3 + 2x2 – 5x + 8 is divided by (x – 3).
A. Pythagoras B. Isaac Newton A. 5 B. 8 C. 11 D. 14
C. Daniel Bernoulli D. Pierre de Fermat

209. What is 60% of 120?


202. Which of the following is a square? A. 50 B. 72 C. 180 D. 200
A. Polygon ABCD which has 4 congruent sides.
B. Polygon MATH which has 4 perpendicular
sides. 210. What percent of 80 is 55?
C. Quadrilateral HEAD which has one pair of A. 145.45% B. 135%
congruent perpendicular bisecting diagonals. C. 68.75% D. 44%
D. Quadrilateral FROG which has 4 right angles.

211. The hypotenuse of a right triangle measures


203. Which of the following is the set of points 40 cm. Find its area if one angle measures 30o.
whose sum of distance to two fixed points is A. 100√3 cm2 B. 200√2 cm2
constant? C. 200√3 cm2 D. 400√2 cm2
A. parabola B. circle
C. ellipse D. hyperbola
212. Nine cans of soda and four hamburgers cost
a total of P257. Five cans of soda and seven
204. Which of the following is not a triangle hamburgers cost a total of P224. How much is a
congruence postulate? can of soda?
A. SAS B. ASA C. SAA D. AAA A. P17 B. P19 C. P21 D. P23

205. If A is at (-8,5) and B is at (4,-11), find C if C 213. The product of two consecutive even
is three-fourths the way from A to B. numbers is 728. What is the smaller number?
A. (1, -7) B. (-4, 1) A. 22 B. 24 C. 26 D. 28
C. (1, 1) D. (-4, -7)

214. What time is 219 minutes past 6:40 AM?


206. CPCTC stands for “____________ parts of A. 8:59 AM B. 9:19 AM
congruent triangles are congruent”. C. 9:49 AM D. 10:19 AM
A. collinear B. complementary
C. corresponding D. conjugate

This is a free reviewer. All rights reserved.


1000 MMR Author: Victor A. Tondo Jr., LPT
215. Find the vertex of y = 3x2 – 2x + 11. 223. Which of the following is not a function?
A. ( , ) B. ( , ) A. y = x2 + 2017x – 2017
B. y = |2017x| - 2017
C. ( , ) D. ( , ) C. y = √2017 + 2017
D. y2 = x + 2017

216. After getting a 20% discount, Mr. Lopez


paid P4,000 for a gadget. How much was its 224. 12 + 17 + 22 + 27 + … + 117 = _____
original price? A. 1409 B. 1414
A. P4,800 B. P5,000 C. 1419 D. 1424
C. P8,000 D. P20,000

225. Mr. G sold 80% of his apples and still had


217. When a number is increased by 3, its square 213 apples left. How many apples did he have
increases by 111. By what does its square originally?
increase when the number is increased by 6? A. 1704 B. 1065
A. 222 B. 240 C. 444 D. 480 C. 852 D. 293

218. How many prime numbers are there from 1 226. When a number is increased by 4, its square
to 100? also increases by 168. What is this number?
A. 23 B. 24 C. 25 D. 26 A. 15 B. 19 C. 23 D. 27

219. Find the range of f(x) = 2x2 – 8x + 9. 227. Solve for k to make a perfect square
A. y 0 B. y 1 trinomial: 9x2 + kx + 25
C. y 9 D. y A. 10 B. 15 C. 20 D. 30

228. Find the y-intercept of 2x + 3y = 4.


220. Find the domain of y =
A. B. C. D. 2
A. x ±7, ±10 B. x ±7
C. x ±10 D. x 1

229. Which of the following points is on the line


221. Solve for x: (x+3)2 = (x-4)2. y = 2x + 5?
A. x = 0 B. x = ½ A. (1, 3) B. (2, 9)
C. x = 1 D. no solution C. (0, 10) D. (3, 10)

222. The diagonal of a rectangular prism is 13 230. Find the intersection of y = -2x + 1 and
cm long. If it is 3 cm thick and 12 cm long, how y = 3x + 16.
wide is it? A. (-3, 7) B. (-4, 9)
A. 3 cm B. 4 cm C. (3, -7) D. (4, -9)
C.4√3 cm D. 5 cm

This is a free reviewer. All rights reserved.


1000 MMR Author: Victor A. Tondo Jr., LPT
231. Find the slope of 3x + 5y = 7. 239. Find the slope of the line tangent to
A. B. y = x3 – 6x2 + 2x + 7 at x = 4.
A. -8 B. -2 C. 2 D. 8
C. D.

240. Find the average rate of change of


232. Which of the following is a polynomial? y = x3 – 2x + 3 from x = 0 to x = 3.
A. √3 + 4 + 2 B. 2x + 3√ A. 5 B. 6 C. 7 D. 8
C. +3 D. √3 x + 7

241. Find the radius of x2 + y2 + 2x – 4y = 44.


233. What is the degree of the polynomial A. √39 B. 2√11 C. 7 D. 3√6
9x4 + 5x3 – 2x2 + 3x – 17?
A. 4 B. 5 C. 9 D. 10
242. Gian has 8 more P5 coins than P1 coins. If
he has a total of P106, how many P5 coins does
he have?
234. log2 32√2 = __________.
A. 13 B. 15 C. 17 D. 19
A. 2.5 B. 3.5 C. 4.5 D. 5.5

243. After using half of her budget on bills, one-


235. If y = √3 + 6 , what is x in terms of y? third on groceries, and P270 on a shirt, Mrs. D
A. x = √ –1 B. x = √ +1 still had P130 left. How much was her budget?
A. P2400 B. P2700
C. x = √ +1 D. x = √ –1 C. P3000 D. P3300

244. x varies directly as y and inversely as z. If


236. Which of the following is a pair of parallel x = 24 when y = 32 and z = 4, what is x when
lines? y = 21 and z = 7?
A. y = 2 and x = 2 A. 3 B. 5 C. 7 D. 9
B. 12x + 13y = 14 and 13x + 14y = 15
C. y = 3x + 8 and 3y = x + 9
D. 4x + 5y = 6 and 8x + 10y = 21 245. Find the mode of the following scores:
78 78 78 78 79 79 79
79 80 80 80 80
237. Which of the following is a pair of A. 79 B. 78, 79, and 80
perpendicular lines? C. 80 D. no mode
A. x = 5 and y = 7
B. y = x and 2y = 4x + 5
C. x = 2y + 3 and 2x + 3y = 4 246. The average grade of 23 students in Section
D. y = 5x + 6 and y = 0.2x – 8 A is 86, while the average grade of 27 students in
Section B is 91. What is the average grade of all
50 students in both sections?
238. Find the altitude to the hypotenuse of a A. 88.5 B. 88.6 C. 88.7 D. 88.8
right triangle whose sides measure 5 cm, 12 cm,
and 13 cm.
A. B. C. D. 26

This is a free reviewer. All rights reserved.


1000 MMR Author: Victor A. Tondo Jr., LPT
247. Find the axis of symmetry of y = 3x2 – 5x. 254. Find the equation of the circle with center at
A. x = B. x = (2, 3), passing through (5, -1).
A. x2 + y2 + 4x + 6y = 0
C. x = D. x = B. x2 + y2 + 4x + 6y = 12
C. x2 + y2 – 4x – 6y = 0
D. x2 + y2 – 4x – 6y = 12
248. Find the range of the following scores:
19 25 24 31 23 29 33
A. 12 B. 13 C. 14 D. 15 255. Find the equation of the vertical line passing
through (-3, 4).
A. x = -3 B. x = 4
249. Mr. C travels for 2 hours at a speed of 38 C. y = -3 D. y = 4
kph and then north for 3 hours at a speed of 53
kph. What is his average speed?
A. 44 kph B. 45.5 kph 256. Find the equation of the horizontal line
C. 47 kph D. 48.5 kph passing through (-3, 4).
A. x = -3 B. x = 4
C. y = -3 D. y = 4
250. Victor, Chris, and Aira volunteered to teach
at a nearby daycare. Chris worked for 2 hours
less than Aira. Victor worked twice as many 257. Which of the following lines passes through
hours as Chris. Altogether, they worked for 58 the point (3, -2)?
hours. How many hours did Victor work? A. y = x + 5 B. y = 2x – 8
A. 14 B. 16 C. 28 D. 32 C. y = 5 – x D. y = 5 – 2x

251. What conic figure does the equation 258. Given that I(2, -3) is the midpoint of V(-4, 5)
x2 + y2 + 4x = -4 form? and C, find the coordinates of C.
A. Real circle B. Degenerate circle A. (-1, 1) B. (1, -1)
C. Imaginary circle D. Ellipse C. (8, -11) D. (-10, 16)

252. What conic figure does the equation 259. The endpoints of the diameter of a circle are
x2 + y2 + 8x – 6y = -100 form? A(9, -5) and B(-3, 11). What is the equation of
A. Real circle B. Degenerate circle the circle?
C. Imaginary circle D. Ellipse A. (x – 3)2 + (y – 3)2 = 100
B. (x – 3)2 + (y – 3)2 = 400
C. (x + 3)2 + (y + 3)2 = 100
253. Find the center of x2 + y2 + 6x – 10y = 2. D. (x + 3)2 + (y + 3)2 = 400
A. (6, -10) B. (-6, 10)
C. (-3, 5) D. (3, -5)
260. Find the distance between the line
3x + 4y – 5 = 0 and the point (8, -1).
A. 2 B. 3 C. 4 D. 5

This is a free reviewer. All rights reserved.


1000 MMR Author: Victor A. Tondo Jr., LPT
261. B is one-fourth of the way from A(-13,9) to 267. Which of the following equations pertain to
C(7,-7). Find the coordinates of B. a parabola that opens to the right?
A. (2, -3) B. (-8, 5) A. 4(y + 3) = (x – 2)2
C. (2.5, -3.5) D. (-7.5, 5.5) B. -3(y – 4) = (x + 5)2
C. (y – 6)2 = 5(x + 1)
D. (y + 8)2 = -2(x – 3)
261. Find the area of the triangle whose vertices
are X(-9, -3), Y(-2, 8), and Z(5, 1).
A. 61 B. 62 C. 63 D. 64 268. Which of the following equations pertain to
a parabola that opens downward?
A. 4(y + 3) = (x – 2)2
262. Which of the following is outside the circle B. -3(y – 4) = (x + 5)2
defined by the equation (x – 3)2 + y2 = 40? C. (y – 6)2 = 5(x + 1)
A. (5, 6) B. (7, 5) D. (y + 8)2 = -2(x – 3)
C. (0, -5) D. (-1, 4)

269. How long is the latus rectum of the parabola


263. Which of the following is parallel to the line defined by 12(y – 4) = (x + 3)2?
defined by the equation y = 3x – 4? A. 12 B. 6 C. 4 D. 3
A. y + 3x = 5
B. x + 3y = 6
C. y = x + 7 270. How far is the vertex from the directrix of
the parabola defined by 16y = x2?
D. y = 3x + 8
A. 16 B. 8 C. 4 D. 2

264. Which of the following is perpendicular to


271. Find the equation of the directrix of the
the line defined by the equation y = 3x – 4?
parabola defined by (y – 2)2 = -4(x + 3).
A. y + 3x = 9 B. -x + 3y = 10
A. x = -2 B. x = 2
C. y = x + 11 D. y = 3x – 12 C. y = -4 D. y = -2

265. Which of the following is coincidental to the 272. Find the coordinates of the focus of the
line defined by the equation y = 2x + 13? parabola defined by -12(y – 4) = (x + 5)2.
A. y + 2x = 13 B. 2x – y + 13 = 0 A. (-5, 7) B. (-5, 1)
C. y = x + 13 D. 2y = 2x + 13 C. (-8, 4) D. (-2, 4)

273. In which quadrant would G(3,-4) fall?


266. Which of the following equations pertain to
A. First quadrant B. Second quadrant
a parabola?
C. Third quadrant D. Fourth quadrant
A. y2 + 5y = x
B. x2 + y2 + 3x – 4y = 0
( ) ( )
C. + =1 274. Find the distance between the parallel lines
D.
( ) ( )
=1 y = 3x + 9 and y = 3x – 12.
√ √
A. 7 B. 21 C. D.

This is a free reviewer. All rights reserved.


1000 MMR Author: Victor A. Tondo Jr., LPT
275. Find the intersection of the lines y = 2x + 5 282. Which of the following is NOT a cofunction
and y = -4x + 23. identity?
A. (3, 10) B. (3, 11) A. cos θ = sin (90 – θ)
C. (4, 10) D. (4, 11) B. cot (90 – θ) = tan θ
C. sec θ = csc (90 – θ)
D. csc θ = sin (90 – θ)
276. Which of the following pertains to a circle
that is concentric with (x – 3)2 + y2 = 24?
A. x2 + y2 – 6x + 3 = 0 283. The hypotenuse of a 30-60-90 triangle is 48
B. x2 + y2 + 6y –15 = 0 cm long. How long is its shortest side?
C. x2 + y2 + 6x – 25 = 0 A. 24 cm B. 24 √2 cm
D. x2 + y2 + 6x + 6y – 24= 0 C. 24 √3 cm D. 16√3 cm

277. Which of the following is the equation of the 284. In a right triangle, the side opposite an
parabola that opens upward, whose latus rectum angle measuring 50o is 100 cm long. How long is
is 12 units long, directrix is y = -3, and line of the side adjacent to the 50o angle?
symmetry is x = 7? A. 93.45 cm B. 83.91 cm
A. 12y2 = x – 7 C. 149.14 cm D. 200 cm
B. 12y = (x – 7)2
C. 12(y + 3) = (x – 7)2
D. 12(y – 3) = (x – 7)2 285. A hundred cards are numbered 1 to 100.
What is the probability of drawing a card whose
number is divisible by seven?
278. Convert rad to degrees. A. B. C. D.
A. 1.309o B. 75o
C. 150o D. 216o
286. What is the probability of rolling a sum of
10 when rolling two dice?
279. Which of the following angles is coterminal A. B. C. D.
with 143o?
A. 217o B. -37o
C. 323o D. 503o 287. Factorize 3x2 + 5x – 2.
A. (3x – 1) (x + 2) B. (3x + 1) (x – 2)
C. (3x – 2) (x + 1) D. (3x + 2) (x – 1)
280. Which of the following is NOT a
trigonometric identity?
A. sin2 θ + cos2 θ = 1 288. Six-sevenths of a number is 6 less than nine-
B. tan θ = tenths of the same number. What is the number?
C. 1 + tan2 θ = sec2 θ A. 130 B. 140 C. 200 D. 210
D. 1 – cot2 θ = csc2 θ

281. Which of the following is false?


A. tan θ = B. csc θ =
C. cos θ = D. sec θ =

This is a free reviewer. All rights reserved.


1000 MMR Author: Victor A. Tondo Jr., LPT
289. A certain University has a dormitory. If 10 295. Determine the relation that matches the
students stay in a room, 24 students will not table of values.
have a room. If 12 students stay in a room, there x 1 2 3 4 5
will be 6 vacant beds. How many rooms are there y 13 11 9 7 5
in the dormitory? How many students are A. y = 21 – x B. y = 15 – 2x
staying in the dormitory? C. y = 3x + 7 D. y = 2x + 11
A. 116 B. 115 C. 114 D. 113

296. Determine which polynomial expression


290. Which of the following is not between matches the algebra tile model.
and ?
A. B. C. D. A. 2x2 + x + 4 B. 3x2 + x + 4
C. 3x2 – x + 4 D. 3x2 + x – 5

291. Which value describes the position of C?


297. Subtract: (–2x2 + 5x – 9) – (2x – 7)
A. 2x2 + 3x + 16 B. -2x2 + 3x + 2
A. -0.75 B. -0.6 C. -1.25 D. -1.4 C. -2x2 + 3x – 2 D. -2x2 + 7x + 2

292. Choose the correct value of (x + y)(x – y) 298. Evaluate the polynomial 4x2 – 6x – 3 if x = 2.
when x = 3.5 and y = –8.7 A. -1 B. 1 C. 3 D. 5
A. -63.44 B. 63.44
C. -148.84 D. 10.4
299. Determine the measure of Y and Z.

293. Which two triangles are similar?

A. 45o, 55o B. 40o, 50o


C. 48o, 48o D. 50o, 50o
A. A and B B. A and C
C. B and D D. B and C
300. Which of the following is equal to x?

294. Determine which equation is equivalent to


4x – 1 = 11.
A. 4x = 10 B. 3x = 11
C. 4 – 1x = 11x D. 4x = 12 A. 15 sin 72o B. 15 sin 18o
C. 72 sin 72o D. 72 sin 18o

This is a free reviewer. All rights reserved.


1000 MMR Author: Victor A. Tondo Jr., LPT
301. Find the points of intersection of the graphs
of y = x2 and y = 3x – 2.
A. (1, 1) and (1, 4) B. (1, 1) and (2, 4)
C. (1, -1) and (2, 4) D. (-2, 4) and (1, 1)
C. D.
302. An approximate value for
. .
is: 308. Which expression is equivalent to (9-2)8?
.
A. 2 B. 20 A. -8132 B. -818 C. D.
C. 200 D. 2000

309. What is 5 × 10–4 written in standard


303. The graph below represents the motion of a notation?
car. The graph shows us that the car is: A. 0.00005 B. 0.0005
C. 5,000 D. 50,000

310. What is the value of 54 × 5-6?


A. accelerating A. -25 B. C. D. 25
B. standing still
C. travelling north-east
D. travelling at a constant speed 311. . Which comparison is true?
A. 4 < 180.5 < 4.5 B. 4.5 < 180.5 < 5
C. 8.5 < 18 < 9.5
0.5 D. 17 < 180.5 < 19
304. The units digit of the number 543444 is:
A. 3 B. 9 C. 7 D. 1
312. A weather station recorded the amount of
rain that fell during an 8-hour time frame using a
305. 4n+1 (4n+2) equals rain gauge. The findings are recorded in the
A. 42n + 3 B. 82n + 3 graph below.
C. 162n + 3 D. 4n + 3

306. The greatest number of Fridays that can


occur in a 75 day period is:
A. 10 B. 11 C. 12 D. 13

307. Which model is not a function?

Between which hours was the rate at which the


rain fell greater than the rate at which the rain
fell between hours 0 and 1?
A. B. A. between hours 3 and 4
B. between hours 4 and 5
C. between hours 5 and 6
D. between hours 7 and 8

This is a free reviewer. All rights reserved.


1000 MMR Author: Victor A. Tondo Jr., LPT
312. Each day of the month, Carl earns an 316. Praetor jogged on a path that was 2 miles
allowance, in cents, equal to the square of that long, took a break, and then jogged back along
date of the month. Which is a number of cents the same path to where he started. He jogged at
Carl could earn in a single day? different speeds for different distances along the
A. 21 B. 31 C. 64 D. 111 path as shown in the graph.

313. Which set of ordered pairs models a


function?
A. {(2, 9), (7, 5), (3, 14), (2, 6)}
B. {(5, 10), (5, 15), (5, 20), (5, 25)}
C. {(3, 10), (4, 15), (5, 20), (3, 25)}
D. {(–10, 20), (–20, 30), (–30, 40), (–40, 10)}

314. Rayon has a piece of rectangular paper that Between which times did Praetor jog the fastest?
is 12 inches wide by 16 inches long. He drew a A. 0 minutes and 10 minutes
straight line along the diagonal of the paper. B. 10 minutes and 25 minutes
What is the length of the line Rayon drew? C. 25 minutes and 30 minutes
A. √28 inches B. √192 inches D. 30 minutes and 60 minutes
C. 20 inches D. 28 inches

317. Which expression has a value of -2?


315. Which equation has infinitely many A. |2| + |-4| B. |-2| – |4|
solutions? C. |4| – |-2| D. |-4| + |2|
A. 2x + 4 = 7x + 9
B. 3(2x + 5) = 6x + 15
C. 4x + 13 = 5x + (20 – x) 318. Reion is tossing a six-sided number cube
D. x + 3 = 5x – 21 labeled 1, 2, 3, 4, 5, and 6. What is the probability
of tossing 6 twice in a row?
A. B. C. D.

319. Which represents the value of x in


6 – 4x 26?
A. x -8 B. x -8
C. x -5 D. x -5

320. The table below shows the resting heart


rates in beats per minutes of six students. The
rate, 40 beats per minute, seems to be an outlier.
Which measure of central tendency changes the
least by dropping 40 from the data?
Heart
Rate
78 71 79 80 40 71
A. mean B. median
C. mode D. range

This is a free reviewer. All rights reserved.


1000 MMR Author: Victor A. Tondo Jr., LPT
321. The sum of a number, n, and 5 is subtracted 328. Which of the following is closest to the
from 8. Which expression represents this value of the expression below?
statement? (20.0143642359)2 x 8π
A. 8 – (n + 5) B. (n + 5) + 8 A. 1,000 B. 10,000
C. (n + 5) – 8 D. 8 + (n + 5) C. 100,000 D. 1,000,000

322. How is 0.5600 written in scientific notation? 329. Which of the following expressions has a
A. 5.6 × 10 B. 5.6 × 10-1 value of 0?
C. 5.6 × 10-2 D. 5.6 × 10-3 A. (2 – 3) – (2 – 3) B. (2 – 3) – |2 – 3|
C. (2 – 3) + (-3 + 2) D. |2 – 3| – (2 – 3)

323. What is the value of x in 3(x – 4) = –21?


A. x = –11 B. x = –3 330. What is the factorization of 10x2 – x – 21?
C. x = 3 D. x = 11 A. (5x – 7) (2x + 3) B. (5x + 7) (2x – 3)
C. (5x + 3) (2x – 7) D. (5x – 3) (2x – 7)

324. In the spinner, what is the probability of the


arrow NOT landing on the space with the ∆? 331. Evaluate: (√343)2
A. 7√7 B. 49 C. 49√7 D. 343

332. Find the length of the latus rectum of the


( ) ( )
ellipse defined by + = 1.
A. B. C. D.
A. B. C. D.

325. Which values of x and y make the system of


equations below true? 333. Let A be a set such that A = {v, w, x, y, z}.
2x - y = -1 How many subsets does set A have?
3x - y = -3 A. 5 B. 10 C. 25 D. 32

A. x = -4; y = -7 B. x =-2; y = -3
C. x = 2; y = 5 D. x = 4; y = 15 334. Solve for x:
2 (5x – 11) + 7 = 3 (x – 7) – 15
A. x = 3 B. x = 1
326. The lengths of two sides of a triangle are 8 C. x = -1 D. x = -3
inches and 13 inches. Which of the following
represents x, the possible length in inches of the
remaining side of the triangle? 335. What is the 4th term in the expansion of
A. 5 < x < 21 B. 5 x 21 (2x + 3y)7?
C. x < 5 or x > 21 D. x 5 or x 21 A. 15120 x4y3 B. 7560 x4y3
C. 3780 x3y4 D. 1890 x3y4

327. What is the value of the expression


below?
80 ÷ ( 6 + (3 – 5) x 2)
A. -8 B. 8 C. 10 D. 40

This is a free reviewer. All rights reserved.


1000 MMR Author: Victor A. Tondo Jr., LPT
336. The shell shape , as used in the Mayan 343. A ride in a Feak Taxi costs P25.00 for the
numeral system, is the symbol for which first km and P10.00 for each additional km.
number? Which of the following could be used to calculate
A. 100 B. 10 C. 1 D. 0 the total cost, y, of a ride that was x km?
A. y = 25x + 10
B. y = 10x + 25
337. Which of the following is irrational? C. y = 25(x 1) + 10
̅̅̅̅̅
A. 0.125 B. 43.29% D. y = 10(x 1) + 25
C. √200 D. √343

344. Which of the following points is in the


338. Aira is six years older than Zayne. Six years fourth quadrant?
ago, she was twice as old as he. How old is Aira A. (3, 4) B. (-3, 4)
now? C. (3, -4) D. (-3, -4)
A. 21 B. 18 C. 15 D. 12

345. The distance from the sun to the earth is


339. The two parallel sides of a trapezoidal lot approximately 9.3 × 107 miles. What is this
measure 100m and 70m. If these sides are 80m distance expressed in standard notation?
apart, what is the area of the lot? A. 9,300,000,000 B. 930,000,000
A. 13600 m2 B. 6800 m2 C. 93,000,000 D. 651
C. 3400 m2 D. 2400 m2

346. The square of a number added to 25 equals


340. If y = x and y = 2x + 2, find the value of x. 10 times the number. What is the number?
A. x = -2 B. x = -1 A. -10 B. -5 C. 5 D. 10
C. x = 0 D. x = 1

347. The sum of the square of a number and 12


341. If the difference between the squares of two times the number is 27. What is the smaller
consecutive counting numbers is 49, what is the possible value of this number?
larger number? A. -9 B. -3 C. 3 D. 9
A. 99 B. 49 C. 25 D. 7

348. Let x = 1. Find the corresponding y given


342. Rayon needed to find the perimeter of an that 2x 3y = 5.
equilateral triangle whose sides measure x + 4 A. y = -1 B. y = 1
cm each. Jake realized that he could multiply C. y = 3 D. y = -3
3 (x + 4) = 3x + 12 to find the total perimeter in
terms of x. Which property did he use to
multiply? 349. The sum of two consecutive even integers is
A. Associative Property of Addition 126. What is the smaller integer?
B. Distributive Property of Multiplication over A. 63 B. 62 B. 61 D. 60
Addition
C. Commutative Property of Multiplication
D. Inverse Property of Addition

This is a free reviewer. All rights reserved.


1000 MMR Author: Victor A. Tondo Jr., LPT
350. Factorize: a2 – a – 90 358. The cost of renting a bike at the local bike
shop can be represented by the equation
A. (a – 10) (a + 9)
y = 2x + 2, where y is the total cost and x is the
B. (a + 10) (a + 9)
number of hours the bike is rented. Which of the
C. (a + 10) (a – 9)
following ordered pairs would be possible
D. (a – 10) (a – 9)
number of hours rented and the corresponding
total cost?
A. (0, 2) B. (2, 6)
351. Evaluate 10P5.
C. (6, 2) D. ( 2, 6)
A. 2 B. 100,000
C. 1,024 D. 30,240
359. The distance from the earth to the moon is
approximately 240,000 miles.
352. Jay bought twenty-five P4.57 stamps. How
What is this distance expressed in scientific
much did he spend?
notation?
A. P 104.25 B. P 114.25
A. 24 × 104 B. 2.4 × 104
C. P 119.75 D. P124.25
C. 2.4 × 105 D. 2.4 × 10 5

353. Given f(x) = x3 + kx2 – 7, find k if f(2) = 41.


A. 5 B. 10 C. 15 D. 20 360. =

A. B. C. D.
354. If y = x and y = 2x + 2, find x + y.
A. -8 B. -4 C. 0 D. 4
361. The sum of two angles is 180°. The measure
of one angle is 34° greater than the measure of
355. Mulan and Lilo are competing to see who the other angle. What is the measure of the
can sell the most candy bars for a fundraiser. smaller angle?
Mulan sold 4 candy bars on the first day and 2
A. 74° B. 73° C. 72° D. 71°
each day after that. Lilo sold 7 on the first day
and 1 each day after that. On what day will they
have the same number of candy bars sold?
362. Solve the following system of linear
A. 7th B. 6th C. 4th D. 3rd
equations:
3x + y = -9
3x 2y = 12
356. Which of the following is not a polynomial?
A. ( 2, 3) B. (2, 3)
A. -3x2 + x – 9 B. √2x + π C. (3, 2) D. ( 3, 2)

C. D. 7 +9
363. Find the x-intercept of 3x + 2y = 24
A. x = 8 B. x = -8
357. Factorize: 3p2 – 2p – 5 C. y = 12 D. y = -12
A. (3p – 5) (p + 1) B. (3p + 5) (p – 1)
C. (3p + 1) (p – 5) D. (3p – 1) (p + 5)

This is a free reviewer. All rights reserved.


1000 MMR Author: Victor A. Tondo Jr., LPT
364. A circle is drawn such that ̅̅̅̅ is a diameter 370. Ten factorial is equal to _____.
and its midpoint is O. Given that C is a point on A. 100
the circle, what is the measure of ACB? B. e10
A. 180o B. 90o C. 10 x 9 x 8 x 7 x 6 x 5 x 4 x 3 x 2 x 1
C. 60o D. not enough info D. 10e

365. Samantha owns a rectangular field that has 371. How many 3-digit numbers can be made
an area of 3,280 square meters. The length of the using the digits 5, 6, 7, 8, 9, and 0 if repetition is
field is 2 more than twice the width. What is the not allowed?
width of the field? A. 80 B. 100 C. 120 D. 140
A. 40 m B. 41 m
C. 82 m D. 84 m
372. A researcher is curious about the IQ of
students at the Utrecht University. The entire
366. Rayon used the following mathematical group of students is an example of a:
statement to show he could change an A. parameter B. statistic
expression and still get the same answer on both C. population D. sample
sides:
10 × (6 × 5) = (10 × 6) × 5
Which mathematical property did Rayon use? 373. Jordan filled a bottle with grains until it was
1/4 full and weighed 8 kg. He added more grains
A. Identity Property of Multiplication
into the bottle until it was 7/8 full. It now
B. Commutative Property of Multiplication
weighed 18 kg. What is the mass of the empty
C. Distributive Property of Multiplication over
bottle?
Addition
A. 16 B. 8 C. 4 D. 2
D. Associative Property of Multiplication

374. If 37 – 4x < 17, then


367. Factorize x3 – 27y3.
A. x < 5 B. x > 5
A. (x – 3y) (x2 – 3x + 9y2)
C. x < -5 D. x > -5
B. (x – 3y) (x2 + 3x + 9y2)
C. (x + 3y) (x2 + 3x – 9 y2)
D. (x + 3y) (x2 – 3x + 9 y2)
375. How many solutions are there for the
following system of linear equations?
-3x + 5y = 6
368. What is the intersection of the lines
6x 10y = 0
x + 3y = 5 and -2x + 4y = 0?
A. (2, 4) B. (-2, 1) A. only one solution
C. (2, 1) D. (-2, 4) B. two solutions
C. infinitely many solutions
D. no solution
369. Given f(x) = 7x3 – 3x2 + 2x – 9, f(2) =
A. 56 B. 48 C. 44 D. 39

This is a free reviewer. All rights reserved.


1000 MMR Author: Victor A. Tondo Jr., LPT
376. Find a and b so that the system below has 382. A teacher asks students to identity their
the unique solution (-2, 3). favorite reality television show. What type of
ax + by = 17 measurement scale do the different television
2ax by = 11 shows make up?
A. Nominal B. Ordinal
A. a = 3, b = -1 B. a = 1, b = 5
C. Interval D. Ratio
C. a = 1, b = 3 D. a = 1, b = 5

383. What is the center of the circle defined by


377. What is the probability choosing only one
x2 + y2 – 8x + 6y – 10 = 0?
vowel when three letters are randomly selected
A. (-8, 6) B. (8, -6)
from the word NUMBERS?
C. (-4, 3) D. (4, -3)
A. B. C. D.

384. Find the equation of the line passing (1, 4)


378. If A > B, which is always true? with slope equal to 5.
A. B. A2 > B2 A. y = 5x + 3 B. y = 5x + 1
C. A < B + 2 D. A – B > 0 C. y = 5x – 1 D. y = 5x – 3

379. Statistical techniques that summarize and 385. The seminar rooms in the library are
organize the data are classified as what? identified by the letters A to H. A researcher
A. Population statistics records the number of classes held in each room
B. Sample statistics during the first semester. What kind of graph
C. Descriptive statistics would be appropriate to present the frequency
D. Inferential statistics distributions of these data?
A. Histogram B. Scatterplot
C. Bar chart D. Box plot
380. Five-point Likert scales (strongly disagree,
disagree, neutral, agree, strongly agree) are
frequently used to measure motivations and 386. Find the slope of the line passing the points
attitudes. A Likert scale is a: A(2, 3) and B(-7, -15).
A. Discrete variable. A. 1 B. -1 C. ½ D. 2
B. Ordinal variable.
C. Categorical variable.
D. All of the above 387. Factorize: 3n2 – 8n + 4
A. (3n – 2) (n – 2)
B. (3n – 2) (n + 2)
381. What is the radius of the circle defined by C. (3n + 2) (n + 2)
(x + 2)2 + (y – 3)2 = 16? D. (3n + 2) (n – 2)
A. 256 B. 16 C. 8 D. 4

388. In now many ways can the letters


AAABBCDEEE be arranged in a straight line?
A. 50,400 B. 25,200
C. 12,600 D. 6,300

This is a free reviewer. All rights reserved.


1000 MMR Author: Victor A. Tondo Jr., LPT
389. Find the smaller angle formed by the x-axis 395. Find the number of subsets having 4
and the line y = 5x. elements of the set {1,2,3,4,5,6,7,8,9,10,11}.
A. 78.69o B. 63.48o A. 165 B. 330
C. 54.15o D. 41.32o C. 660 D. 1320

390. Convert 48o to radians. 396. There are ten true - false questions in an
A. π rad B. π rad exam. How many responses are possible?
A. 1024 B. 256 C. 20 D. 10
C. π rad D. π rad

397. In a 500m speed skating race, time results


391. The median is always: would be considered an example of which level
A. The most frequently occurring score in set of of measurement?
data A. Nominal B. Ordinal
B. The middle score when results are ranked in C. Interval D. Ratio
order of magnitude
C. The same as the average
D. The difference between the maximum and 398. In how many ways can 4 girls and 5 boys be
minimum scores. arranged in a row so that all the four girls are
together?
A. 4,320 B. 8,640
392. A teacher gave a statistics test to a class of C. 17,280 D. 34,560
Geography students and computed the measures
of central tendency for the test scores. Which of
the following statements cannot be an accurate 399. A box contains 8 batteries, 5 of which are
description of the scores? good and the other 3 are defective. Two batteries
A. The majority of students had scores above the are selected at random and inserted into a toy. If
mean. the toy only functions with two good batteries,
B. The majority of students had scores above the what is the probability that the toy will function?
median. A. B. C. D.
C. The majority of students had scores above the
mode.
D. All of the above options (A, B and C) are false
400. IQ tests are standardized so that the mean
statements.
score is 100 for the entire group of people who
take the test. However, if you select a group of 50
who took the test, you probably would not get
393. Find the area of a semicircle whose radius
100. What statistical concept explains the
measures 28 cm.
difference between the two means?
A. 784 π cm2 B. 392 π cm2
A. Statistical error B. Inferential error
C. 28 π cm 2 D. 14 π cm2
C. Residual error D. Sampling error

394. Find the length of each side of an equilateral


401. Which Mathematician pioneered the study
triangle whose perimeter is 90 cm.
A. 45 cm B. 30 cm of conic sections?
A. Euclid B. Apollonius
C. 22.5 cm D. 10 cm
C. Archimedes D. Hipparchus

This is a free reviewer. All rights reserved.


1000 MMR Author: Victor A. Tondo Jr., LPT
402. A researcher studies the factors that 408. Which of the following sets of scores has the
determine the number of children future couples greatest variability or range?
decide to have. The variable ‘number of children’ A. 2, 5, 8, 11 B. 13, 13, 13, 13
is a: C. 20, 25, 26 ,27 D. 42, 43, 44, 45
A. Discrete variable
B. Continuous variable
C. Categorical variable 409. This Mathematician was the first to describe
D. Ordinal variable a pinwheel calculator in 1685 and invented the
wheel named in his honor, which was used in the
arithmometer, the first mass-produced
403. Surface area and volume, center of gravity, mechanical calculator. He also refined the binary
and hydrostatics are some of the studies of number system, which is the foundation of all
which Mathematician? digital computers. Which Mathematician is this,
A. Apollonius B. Euclid who is also crucial to the development of
C. Archimedes D. Hipparchus computers?
A. Gottfried Wilhelm Leibniz
B. Charles Babbage
404. The book Philosophiæ Naturalis Principia C. Ada Lovelace
Mathematica, more fondly known simply as D. Alexander Graham Bell
Principia, is the work of which Mathematician?
A. Euclid B. Newton
C. Einstein D. Archimedes 410. Solve for x, given 9x – 10 = 11x + 30
A. x = 40 B. x = 20
C. x = -20 D. x = -40
405. A researcher is interested in the travel time
of Rayon’s University students to college. A
group of 50 students is interviewed. Their mean
411. Using Calculus, this Mathematician
travel time is 16.7 minutes. For this study, the
explained why tides occur, why the shapes of
mean of 16.7 minutes is an example of a
planetary orbits are conic sections, and how to
A. parameter B. statistic
get the shape of a rotating body of fluid, among
C. population D. sample
many other things. Which Mathematician is this?
A. Kepler B. Euclid
C. Apollonius D. Newton
406. Who is considered by many Mathematicians
as “The Last Universalist”?
A. Jules Henri Poincare
412. Which of the following terms does NOT
B. Hendrik Lorentz
describe the number 9?
C. Georg Cantor
A. rational number B. integer
D. Gottfried Wilhelm Leibniz
C. real number D. prime number

407. In the theory he developed, there are


infinite sets of different sizes (called 413. Which expression below is equal to 5?
cardinalities). Which Mathematician formalized A. (1 + 2)2 B. 9 – 22
many ideas related to infinity and infinite sets C. 11 10 × 5 D. 45 ÷ 3 × 3
during the late 19th and early 20th centuries?
A. Jules Henri Poincare
B. Hendrik Lorentz
C. Georg Cantor
D. Gottfried Wilhelm Leibniz
This is a free reviewer. All rights reserved.
1000 MMR Author: Victor A. Tondo Jr., LPT
414. A bus picks up a group of tourists at a hotel. 421. Of the following Z-score values, which one
The sightseeing bus travels 2 blocks north, 2 represents the location closest to the mean?
blocks east, 1 block south, 2 blocks east, and 1 A. Z = +0.5 B. Z = +1.0
block south. Where is the bus in relation to the C. Z = -1.5 D. Z = -0.3
hotel?
A. 2 blocks north B. 1 block west
C. 3 blocks south D. 4 blocks east 422. The sum of five consecutive integers is 215.
What is the largest of these integers?
A. 43 B. 44 C. 45 D. 46
415. When five is added to three more than a
certain number, the result is 29. What is the
number? 423. You go to the cafeteria for lunch and have a
A. 24 B. 21 C. 8 D. 4 choice of 4 entrees, 5 sides, 5 drinks, and 4
desserts. Assuming you have one of each
category, how many different lunches could be
416. The math club is electing new officers. made?
There are 3 candidates for president, 4 A. 18 B. 81 C. 40 D. 400
candidates for vice-president, 4 candidates for
secretary, and 2 candidates for treasurer. How
many different combinations of officers are 424. What can be said about the following
possible? statements?
A. 13 B. 96 i. Any quadrilateral with four congruent sides is
C. 480 D. 17,160 a square.
ii. Any square has four congruent sides.
A. Only the first statement is true.
417. Twelve points lie on a circle. How many B. Only the second statement is true.
cyclic quadrilaterals can be drawn by using these C. Both statements are true.
points? [Note: Cyclic quadrilaterals are D. Both statements are fall.
quadrilaterals whose vertices are on a circle.]
A. 48 B. 495
C. 11,880 D. 1,663,200 425. Out of 6 boys and 4 girls, a committee of 5
has to be formed. In how many ways can this be
done if we take 2 girls and 3 boys?
418. What is the variance for the following set of A. 120 B. 186 C. 240 D. 256
scores? 143 143 143 143 143 143
A. 0 B. 2 C. 4 D. 25
426. In the figure, which of the following will
yield the value of the hypotenuse x?
419. When 18 is subtracted from six times a
certain number, the result is 42. What is the
number?
A. 10 B. 4 C. -4 D. -10
A. x = B. x =
420. Find the equation of the line passing (2, 3) C. x = D. x = 10 tan 35o
and (-7, -15)
A. y = 2x + 1 B. y = 2x – 1
C. y = x + 2 D. y = x – 2

This is a free reviewer. All rights reserved.


1000 MMR Author: Victor A. Tondo Jr., LPT
427. The shortest side of a 30-60-90 triangle is 434. Which triangle has the centroid, incenter,
20.19 cm long. How long is the hypotenuse? circumcenter, orthocenter, and nine-point-center
A. 40.38 cm B. 30.29 cm at the same location?
C. 34.97 cm D. 17.48 cm A. isosceles right triangle
B. 30-60-90 triangle
C. equilateral triangle
428. The hypotenuse of a 30-60-90 triangle is D. hyperbolic triangle
34.96 cm long. How long is the shortest side?
A. 40.38 cm B. 30.29 cm
C. 34.97 cm D. 17.48 cm 435. Given cos θ = and θ QIV, find tan θ.
A. B. C. D.
429. The second angle of a triangle is three times
as large as the first. The measure of the third
angle is 40 degrees greater than that of the first 436. Find the measure of T:
angle. How large is the first angle?
A. 28o B. 30o C. 35o D. 38o

430. Normally distributed data are normally


referred to as:
A. Bell-shaped B. Asymmetrical A. 59o B. 69o C. 79o D. 89o
C. Skewed D. Peaked
437. If the scores on a test have a mean of 26 and
431. A population has a mean of μ=35 and a a standard deviation of 4, what is the z-score for
standard deviation of σ=5. After 3 points are a score of 18?
added to every score of the population, what are A. -1.41 B. 11 C. -2 D. 2
the new values for the mean and standard
deviation?
A. μ=35 and σ=5 438. If a researcher sets a level of significance at
B. μ=35 and σ=8 0.05 (i.e. 5%), what does this mean?
C. μ=38 and σ=5 A. Five times out of 100, a significant result will
D. μ=38 and σ=8 be found that is due to chance alone and not to
true relationship.
B. Ninety-five times out of 100, a significant
result will be found that is due to chance alone
432. Given sin θ = , find cos θ.
and not to true relationship.
A. B. C. D. C. Five times out of 100, a significant result will
be found that is not due to chance, but to true
relationship.
433. In a triangle, what is located 2/3 of the D. None of the above
distance from each vertex to the midpoint of the
opposite side?
A. centroid B. incenter 439. When does a researcher risk a Type I error?
C. circumcenter D. orthocenter A. Anytime the decision is ‘fail to reject’.
B. Anytime H0 is rejected.
C. Anytime Ha is rejected.
D. All of the above options
This is a free reviewer. All rights reserved.
1000 MMR Author: Victor A. Tondo Jr., LPT
440. Solve for x: 446. How much water must be evaporated from
2000 mL of 30% acid solution to make a 50%
acid solution?
A. 800 mL B. 850 mL
C. 900 mL D. 950 mL

447. Which of the following is the intersection of


A. 4 B. 5 C. 6 D. 7 angle bisectors of a triangle?
A. circumcenter B. incenter
C. centroid D. orthocenter
441. Which of the following is equidistant from
the vertices of the triangle?
A. circumcenter B. orthocenter 448. In terms of a conditional statement, what is
C. incenter D. centroid the statement formed by exchanging and
negating the antecedent and the consequent?
A. inverse B. converse
442. Which of the following is equidistant from C. adverse D. contrapositive
the sides of the triangle?
A. circumcenter B. centroid
C. orthocenter D. incenter 449. What is formed when the hypothesis and
the conclusion of the conditional statement are
interchanged?
443. Although rarely used in proving, what is the A. converse B. inverse
extra line or line segment drawn in a figure to C. adverse D. contrapositive
help in a proof?
A. base line B. auxiliary line
C. converse line D. Euler’s line 450. What is formed when both the hypothesis
and the conclusion of the conditional statement
are negated?
444. What is the measure of V in the following A. converse B. inverse
figure? C. adverse D. contrapositive

451. Which of the following is the converse of the


following statement?
“If two angles are congruent,
then they have the same measure.”
A. If two angles are not congruent, then they do
A. 60o B. 65o C. 70o D. 75o not have the same measure.
B. If two angles have the same measure, then
they are congruent.
445. What is the intersection of all three C. If two angles do not have the same measure,
altitudes of a triangle? then they are not congruent.
A. incenter B. centroid D. If two angles are not congruent, then they
C. orthocenter D. circumcenter have the same measure.”

This is a free reviewer. All rights reserved.


1000 MMR Author: Victor A. Tondo Jr., LPT
452. In which geometry are there no parallel 458. Which lines are not in the same plane and
lines? do not intersect but are not parallel?
A. elliptic geometry A. asymptotes B. tangent lines
B. hyperbolic geometry C. skew lines D. directrices
C. spatial geometry
D. solid geometry
459. Two adjacent angles whose distinct sides lie
on the same line are called what?
453. What do we call the ratio of two numbers A. linear pair B. vertical pair
(larger number: smaller number) whose ratio to C. alternate D. corresponding
each other is equal to the ratio of their sum to
the larger number? [Note: This is applied in
Fibonacci sequences] 460. The point of concurrency of a triangle’s
A. pi B. golden ratio three altitudes is called _____.
C. 1.618 D. Euler’s ratio A. circumcenter B. incenter
C. orthocenter D. centroid

454. Which of the following pertains to the law of


cosines? 461. What do we call three positive integers with
A. c2 = a2 + b2 – 2 ab cos C the property that the sum of the squares of two
B. c2 = a2 + b2 + 2 ab cos C of the integers equals the square of the third?
C. c2 = a2 + b2 – ab cos C A. Euclid’s triple
D. c2 = a2 + b2 + ab cos C B. Pythagorean triple
C. Newton’s triple
D. Cartesian triple
455. Solve for x:

462. Which of the following expressions will give


the value of x?

A. x = 8 B. x = 9
C. x = 10 D. x = 11
A. 50 tan 37o B. 50 cos 37o
C. 50 sin 37o D. 50 cot 37o
456. What do we call an angle formed by two
chords of the circle with a common endpoint
(the vertex of the angle)? 463. Solve for x:
A. inscribed angle B. tangential angle
C. circumscribed angle D. interior angle

457. Find the inverse of y = .


A. y-1 = B. y-1 =
C. y-1 = D. y-1 =
A. 6 B. 7 C. 7.5 D. 8

This is a free reviewer. All rights reserved.


1000 MMR Author: Victor A. Tondo Jr., LPT
464. A bus travels 600 km in 7 hrs and another 470. Find the equation of the circle whose center
300 km in 5 hrs. What is its average speed? is at (7, -24) given that it passes the point of
A. 72.86 kph B. 75 kph origin.
C. 77.86 kph D. 80 kph A. (x + 7)2 – (y – 24)2 = 961
B. (x – 7)2 + (y + 24)2 = 961
C. (x + 7)2 + (y – 24)2 = 625
465. A sniper on a cliff observes that the angle of D. (x – 7)2 + (y + 24)2 = 625
depression to his target is 30o. If the cliff is 10
meters high, how far must the bullet travel to hit
the sniper’s target? 471. Ana left their house and jogged at a speed of
A. 20 meters B. 10√3 meters 60 meters per minute. Bea followed her two
C. 10√2 meters D. 10 meters minutes later and jogged at a speed of 70 meters
per minute. How many minutes after Bea left
would she catch up with Ana?
466. Rowena received a total of 25 bills. These A. 14 B. 13.5 C. 13 D. 12
bills are either P20 or P50 bills. If Rowena
received an amount of P800, how many P20 bills
did she receive? 472. How many mL of 40% acid must be added
A. 10 B. 13 C. 15 D. 17 to 1000 mL of 10% acid solution to make a 20%
acid solution?
A. 250 B. 500 C. 600 D. 750
467. How many ways can the word PILIPINAS be
rearranged?
A. 302 473. The hypotenuse of a 30-60-90 triangle is
B. 3,024 432 cm long. How long is the leg opposite the 30o
C. 30,240 angle?
D. 302,400 A. 216 cm B. 216 √2 cm
C. 216 √3 cm D. 432 cm

468. A coin is tossed 60 times. Head appeared 27


times. Find the experimental probability of 474. The shortest leg of a 30-60-90 triangle is
getting heads. 123 cm long. How long is the leg adjacent to the
A. B. C. D. 30o angle?
A. 123 cm B. 123√2 cm
C. 123√3 cm D. 246 cm
469. A parabola is defined by the equation
5x = -3y2 – 4y + 2. Which of the following is true
about the parabola? 475. How many odd 4digit numbers can be
A. It opens to the left. formed using the digits 7, 6, 5, 4, 3, 2, and 1 if
B. It opens to the right. repetition is not allowed?
C. It opens upward. A. 720 B. 480 C. 240 D. 120
D. It opens downward.

476. There are 70 dogs and geese in a farm. If


there are a total of 200 legs, how many dogs are
there?
A. 25 B. 30 C. 35 D. 40

This is a free reviewer. All rights reserved.


1000 MMR Author: Victor A. Tondo Jr., LPT
477. Find the maximum area of a rectangle if the 485. What is the remainder when 3x6+ 4x5 – 5x4
perimeter is set at 350 cm. + 6x3 + 7x2 – 8x + 3 is divided by (x – 1)?
A. 8656.25 cm2 B. 7656.25 cm2 A. 8 B. 9 C. 10 D. 11
C. 6656.25 cm2 D. 5656.25 cm2

486. Seven people have an average weight of 49


478. A rectangle is 60 cm long and 45 cm wide. kg. A child was added to the group and the
How long is its diagonal? average became 45 kg. How heavy is the child?
A. 75 cm B. 85 cm A. 15 kg B. 16 kg
C. 95 cm D. 105 cm C. 17 kg D. 18 kg

479. Find the length of the diagonal of a cube 487. Six numbers have an average of 71. If 85 is
given each side measures 17 cm. added to the group, what is the new average?
A. √290 cm B. 17√2 cm A. 72 B. 73 C. 74 D. 75
C. 17√3 cm D. 34 cm

488. In an arithmetic sequence, the 7th term is 25


480. Find the measure of each interior angle of a and the 10th term is 67. What is the common
regular 20-sided polygon. difference?
A. 162o B. 150o A. 42 B. 21 C. 14 D. 7
C. 144o D. 126o

489. Triangle ABC has sides measuring 20 cm, 20


481. How many diagonals does a regular 14- cm, and 29 cm. What kind of triangle is ABC?
sided polygon have? A. acute B. right
A. 77 B. 91 C. 96 D. 101 C. obtuse D. reflex

482. How many ways can 14 people be seated in 490. Find the surface area of a sphere given that
a Ferris wheel given that each cart can only the sphere sits perfectly inside a cube whose
contain one person? sides measure 20 cm each.
A. 15! B. 14! C. 13! D. 12! A. 400 cm2 B. 800 cm2
C. 1200 cm 2 D. 2400 cm2

483. There are 24 mangoes in a basket, of which


7 are rotten. What is the probability that when 491. Given f(x) = (25 x20 – 24x10)(x2 – 9x + 3),
randomly getting two mangoes at the same time, find f ’(x).
both are rotten? A. f ‘(x) = 550 x21 – 4725 x20 + 1500 x19 – 288 x11
+ 2376 x10 – 720 x9
A. B. C. D.
B. f ‘(x) = 450 x21 – 4725 x20 + 150 x19 + 288 x11
+ 2376 x10 – 720 x9
C. f ‘(x) = 550 x21 – 4725 x20 + 150 x19 – 288 x11
484. In a gathering of gamers and admins, there + 2376 x10 – 720 x9
are 24 gamers of which 6 are females, and 3 D. f ‘(x) = 450 x21 – 4725 x20 + 1500 x19 – 288 x11
admins of which one is female. If a female is + 2376 x10 – 720 x9
randomly called, what is the probability that she
is an admin?
A. B. C. D.
This is a free reviewer. All rights reserved.
1000 MMR Author: Victor A. Tondo Jr., LPT
492. Simplify: eln 2019 x 499. Mocha can finish a job in 24 hours, while
A. 2019x B. 2019 x her sister Tiramisu can do the same job in only
C. x D. 20 hours. How long will it take them to finish the
job by working together?
A. hrs B. hrs
493. If the roots of a quadratic equation are C. 11 hrs D. hrs
and , which of the following could be the
quadratic equation?
A. 63x2 + 22x – 21 = 0 500. What conic figure does the equation
B. 63x2 – 22x – 21 = 0 x2 + y2 + 10x – 16y = -100 form?
C. 63x2 + 22x + 21 = 0 A. Real circle B. Degenerate circle
D. 63x2 – 22x + 21 = 0 C. Imaginary circle D. Ellipse

494. If three more than twice a number is


seventeen less than seven times the number,
what is the number?
A. 2 B. 3 C. 4 D. 5 End of first 500 items.

495. A team is to be made from a group of seven


teachers and six scientists. If the team is to be Please, take a break.
composed two teachers and two scientists, how
many different ways can they form a team?
A. 325 B. 315 C. 300 D. 285
Remember:
Nasa Diyos ang awa,
496. Find the measure of the smaller angle Nasa tao ang gawa.
formed by the hands of the clock at 11:20.
A. 130o B. 135o
C. 140o D. 145o

497. How many even 3-digit even numbers can


be formed using the digits 7, 6, 5, 4, 3, 2, 1, and 0
if repetition is not allowed?
A. 150 B. 160 C. 170 D. 180

498. There are 50 students in a class. Twenty of


them have a laptop. Thirty-two of them have a
smartphone. Seven of them have both a laptop
and a smartphone. How many of them have
neither a laptop nor a smartphone?
A. 4 B. 5 C. 6 D. 7

This is a free reviewer. All rights reserved.


1000 MMR Author: Victor A. Tondo Jr., LPT
501. Victor, Praetor, and Rowena volunteered to 507. What is the sum of the first 2019 counting
teach at a nearby daycare. Praetor worked for numbers?
twice as long as Rowena did. Victor worked A. 2,021,019 B. 2,027,190
twice as many hours as Praetor. Altogether, they C. 2,039,190 D. 2,043,190
worked for 56 hours. For how many hours did
Victor work?
A. 14 B. 16 C. 28 D. 32 508. In a certain school, the ratio of boys to girls
is 4 is to 9. If there are 260 boys and girls in the
school, how many boys are there?
502. Which of the following is the factorization of A. 100 B. 90
the binomial x4 – 20194? C. 85 D. 80
A. (x + 2019)2 (x – 2019)2
B. (x – 2019)4
C. (x2 + 20192) (x + 2019) (x – 2019) 509. Solve for x: 8x + 9y = 17
D. (x – 4)(x + 4) 9x + 10y = 18
A. x = 9 B. x = 8
C. x = -9 D. x = -8
503. The average of 5 different counting
numbers is 143. What is the highest possible
value that one of the numbers can have?
510. Mr. Park is 20 years older than his son now.
A. 706 B. 705
Five years from now, the Mr. Park’s age will be 5
C. 704 D. 703
less than twice his son’s age. Find the son’s
present age.
A. 18 B. 20
504. What value of x will satisfy the equation:
C. 38 D. 40
0.25(20x – 2020) = x?
A. 125.25 B. 125.75
C. 126.25 D. 126.5
510. What is the minimum value of
f(x) = 3x2 + 6x + 11?
A. 1 B. 2 C. 4 D. 8
505. Three brothers inherited a cash amount of
P5,670,000 and they divided it among
themselves in the ratio of 2:3:4. How much more
511. What is the sum of the roots of
is the largest share than the smallest share?
5x2 + 10x – 17?
A. P1,260,000 B. P1,270,000
C. P630,000 D. P635,000 A. B. C. 2 D. -2

506. Rayon and Wena can do a job together in 512. If xy = 17 and x2 + y2 = 135, find x + y.
four hours. Working alone, Rayon does the job in A. 13 B. 12.8749
six hours. How long will it take Wena to do the C. 12.5249 D. 12.5
job alone?
A. 12 hours B. 11 hours
C. 10 hours D. 9 hours 513. How many mL of 50% acid solution must be
added to 200mL of 10% acid solution to make a
40% acid solution?
A. 400 mL B. 500 mL
C. 600 mL D. 700 mL

This is a free reviewer. All rights reserved.


1000 MMR Author: Victor A. Tondo Jr., LPT
514. Nine consecutive even numbers have a sum 523. If two numbers have a product of 267 and
of 180. What is the smallest of these even the sum of their squares is 250, what is their
numbers? sum?
A. 18 B. 16 A. -30√3 B. 28
C. 14 D. 12 C. 7√3 + 2√5 D. 30

515. If x = 13, which of the following is equal to 524. How many ml of 10% acid must be added to
200? 500 ml of 40% acid to make a 30% acid solution?
A. 15x + 2 B. x2 + 2x + 5 A. 1000 ml B. 750 ml
C. x – 4x – 2
3 D. x2 + x + 2 C. 500 ml D. 250 ml

516. For which value of k does 9x2 – kx + 25 525. 3log2 3 + 2 log2 5 – log2 7 = ______.
have only one root?
A. log2 B. log2
A. 3.75 B. 7.5 C. 15 D. 30
C. log2 D. log2

517. If A and B are the roots of x2 + 19x + 20,


what is AB? 526. Find the intersection of y = 3x + 4 and
A. 20 B. 2√5 + 5 y = 5x – 8.
C. 3√2 + 2√3 D. -19 A. (8/5, 0) B. (-4/3, 0)
C. (1, 7) D. (6, 22)

518. 0.25 + 0.5 + 1 + 2 + 4 + … + 1024 = ____


A. 2046.75 B. 2047.75 527. Find the remainder when
C. 2048.75 D. 2049.75 x5 – 3x3 + 5x2 – 7x + 9 is divided by (x – 1).
A. 11 B. 9 C. 7 D. 5

519. How many terms are there in the sequence


2, 9, 16, 23, …, 345? 528. Multiply: (3x – 7) (5x + 9)
A. 40 B. 45 A. 15x2 – 8x – 63 B. 15x2 – 8x + 63
C. 50 D. 70 C. 15x + 8x – 63
2 D. 15x2 + 8x + 63

520. If 2x + 3 = 25, then what is (2x + 3)2 – 25? 529. Multiply: (2x2 – 5x + 3) (3x + 4)
A. 650 B. 625 C. 600 D. 575 A. 6x3 – 7x2 – 11x + 12
B. 6x3 + 7x2 + 11x + 12
C. 6x3 – 7x2 + 11x + 12
521. What is 50% of 200% of 2019? D. 6x3 + 7x2 – 11x + 12
A. 1009.5 B. 2019
C. 4038 D. 8076
530. x varies directly as y and inversely as z. If
x = 40 when y = 8 and z = 2, what is x when
522. If 3x = 5 and 2y = 11, what is 12(x – y)? y = 24 and z = 4?
A. -46 B. -45 C. 45 D. 46 A. 120 B. 90 C. 60 D. 30

This is a free reviewer. All rights reserved.


1000 MMR Author: Victor A. Tondo Jr., LPT
531. Pedro has 7 more P10 coins than P5 coins. If 540. Rowena can do a job in 15 hours, while
he has a total of P475, how many P5 coins does Victor can do the same job in 25 hours. How long
he have? will it take them to finish the job by working
A. 23 B. 25 C. 27 D. 29 together?
A. 9.375 hours B. 9.25 hours
C. 9.125 hours D. 8.875 hours
532. When a number is increased by 5, its square
also increases by 255. What is this number?
A. 22 B. 23 C. 24 D. 25 541. Armel is twice as heavy as Kuku. Gabbi is
17kg heavier than Kuku. The sum of their masses
is 217kg. How heavy is Kuku in kg?
533. Find k such that 16x2 + kx + 81 is a perfect A. 40 B. 45 C. 50 D. 55
square trinomial.
A. 144 B. 72 C. 36 D. 18
542. Find + given x + y = 30 and xy = 219.

534. Solve for x: (x + 4)2 = (x – 6)2. A. B. C. D.


A. x = 0 B. x = ½
C. x = 1 D. no solution
543. The product of two consecutive odd
counting numbers is 1023. What is their sum?
535. When a number is increased by 3, its square A. 60 B. 64 C. 68 D. 72
increases by 135. By what does its square
increase when the number is increased by 5?
A. 225 B. 235 C. 445 D. 485 544. There are 250 pigs and chickens in a farm,
all of which are healthy. If there are 720 legs in
total, how many pigs are there?
536. Ten cans of soda and six hamburgers cost a A. 110 B. 100 C. 90 D. 80
total of P440. Five cans of soda and seven
hamburgers cost a total of P360. How much is a
hamburger? 545. Solve for x given 13x + 17 = 15x – 21.
A. P31 B. P33 C. P35 D. P37 A. x = 17 B. x = 18
C. x = 19 D. x = 20

537. The product of two consecutive odd


numbers is 1763. What is the smaller number? 546. Factorize: x3 + 3x2 – 4x – 12
A. 31 B. 37 C. 41 D. 47 A. (x + 2) (x + 3) (x – 2)
B. (x + 4) (x + 3) (x – 1)
C. (x + 2) (x – 3) (x + 2)
538. Find the remainder when the polynomial D. (x + 4) (x – 3) (x + 1)
x4 – 5x3 + 7x2 – 11x + 19 is divided by (x – 2).
A. 5 B. 3 C. 1 D. -1
547. Factorize (x4 – 16) completely.
A. (x – 2)4
539. If three-fourths of a number is 44 more than B. (x – 2)2 (x + 2)2
its one-fifth, what is that number? C. (x + 2)2 (x – 2)2
A. 120 B. 100 C.80 D. 60 D. (x + 2) (x – 2) (x2 + 4)

This is a free reviewer. All rights reserved.


1000 MMR Author: Victor A. Tondo Jr., LPT
548. √125 + √45 √245 = ____ 556. Find the remainder when x4 – 4x3 + 3x2 +
A. 6√5 B. 4√5 5x – 6 is divided by (x – 2).
C. 2√5 D. √5 A. 3 B. 2 C. 1 D. 0

549. The product of two consecutive even 557. 2 + 9 + 16 + 23 + … + 135 = ____


counting numbers is 3968. Find the smaller A. 1353 B. 1360
number. C. 1370 D. 1377
A. 42 B. 46 C. 54 D. 62
558. Factorize: (2x + y) (2x + y + 3) + 2.
550. A pipe can fill a pool in 5 hours while A. (2x + y + 1) (2x + y + 2)
another pipe can drain empty the pool in 10 B. (2x + y + 1) (2x + y + 6)
hours. How long will it take to fill the pool if both C. (2x + y + 2) (2x + y + 3)
pipes are open? D. (2x + y + 3) (2x + y + 4)
A. 8 hours B. 9.25 hours
C. 9.5 hours D. 10 hours
559. The area of a rectangle is (x2 – x – 30). If its
length is x + 5, what is its width?
551. Today, Rayon is 13 years old while his A. x + 4 B. x – 3
father is thrice his age. How many years from C. x + 2 D. x – 6
now will his father be twice as old as he?
A. 15 B. 13 C. 11 D. 10
560. Factorize 81x2 – 225y2 completely.
A. (9x + 15y)(9x – 15y)
552. Roy and Wena are on a seesaw. Roy weighs B. 9(9x2 – 25y2)
48 kg and sits 160 cm to the left of the fulcrum. If C. 9(3x – 5y)2
Wena weighs 60 kg, how far to the right of the D. 9(3x + 5y)(3x – 5y)
fulcrum must she sit to balance the seesaw?
A. 115 cm B. 123 cm
C. 128 cm D. 135 cm 561. What are the missing terms in the series
3, 6, 12, 24, ___, 96, ____?
A. 48; 192 B. 36, 120
553. Insert one term between 81 and 169 to C. 46, 196 D. 46, 192
make a geometric sequence.
A. 113 B. 117 C. 121 D. 125
562. In a parking lot, there are 65 tricycles and
motorcycles. If there are 150 wheels in all, how
554. Find the common difference of an many tricycles are there?
arithmetic sequence whose 21st term is 1987 and A. 18 B. 19 C. 20 D. 21
29th term is 2019.
A. 2.5 B. 3 C. 3.5 D. 4
563. Which fraction is equivalent to 0.425?
A. 21/50 B. 27/50
555. Find the general term An of the sequence C. 27/40 D. 17/40
7, 16, 25, 34, 43, 52, 61, …
A. An = 9n – 2 B. An = 8n – 1
C. An = 9n + 2 D. An = 8n + 1

This is a free reviewer. All rights reserved.


1000 MMR Author: Victor A. Tondo Jr., LPT
564. What are the zeroes of 6x2 – x – 35? 571. Factorize 27x3 + 125.
A. and B. and A. (3x + 5)(9x2 + 15x + 25)
B. (3x + 5)(9x2 – 15x + 25)
C. and D. and C. (3x – 5)(9x2 + 15x + 25)
D. (3x – 5)(9x2 – 15x + 25)

565. Seven more than four times a number is 79.


What is five less than three times the number? 572. What is the 5th term of (2x + 3)7?
A. 59 B. 49 C. 39 D. 29 A. 22680x3 B. 11340x3
C. 21600x3 D. 10800x3

566. Mr. Park is five times as old as his son. Eight


years from now, he will only be thrice as old as 573. In the equation 2x + 5y = 50, what is x
his son. How old is Mr. Park now? when y is 8?
A. 30 B. 35 C. 40 D. 45 A. 2 B. 3 C. 4 D. 5

567. Mr. Yu is four times as old as his daughter. 574. X varies directly as Y and inversely as Z. If X
Ten years from now, he will only be thrice as old is 24 when Y is 30 and Z is 5, find X when Y is 36
as her. How old is his daughter now? and Z is 4.
A. 10 B. 15 C. 20 D. 25 A. 36 B. 32 C. 24 D. 16

568. The speed of the current of a river is 9 kph. 575. What is the 4th term of (3x – 2)5?
Rayon rows his boat upstream for 8 hours, and A. 360x2 B. 240x2
then travels back to his original position by C. -240x2 D. -360x2
rowing downstream for 4 hours. What is Rayon’s
speed on calm water?
A. 9 kph B. 18 kph 576. Factorize 64x6 – y6.
C. 27 kph D. 36 kph A. (2x – y) (4x2 + 2xy + y2) (2x + y) (4x2 – 2xy + y2)
B. (2x – y) (4x2 + 2xy + y2) (2x + y) (4x2 + 2xy + y2)
C. (2x + y) (4x2 – 2xy – y2) (2x + y) (4x2 – 2xy + y2)
569. Thrice Rayon’s age 16 years ago is equal to D. (2x + y) (4x2 – 2xy – y2) (2x + y) (4x2 – 2xy + y2)
his age 16 years from now. How old is Rayon
now?
A. 16 B. 24 C. 32 D. 40 577. Factorize x2 + 6x – y2 + 2y +8.
A. (x + y + 2) (x + y + 4)
B. (x + y + 2) (x – y + 4)
570. Divide 8x5 – 4x3 + 2x2 – 3x + 4 by x + 2. C. (x + y + 2) (x + y – 4)
A. 8x4 – 16x3 + 28x2 – 54x + 105 r. -206 D. (x + y + 2) (x – y – 4)
B. 8x4 + 16x3 + 28x2 – 54x + 105 r. -206
C. 8x4 – 16x3 + 28x2 – 54x – 105 r. 206
D. 8x4 + 16x3 + 28x2 – 54x – 105 r. 206 578. What value of x will satisfy the equation:
2.5(4x – 504) = x?
A. 150 B. 145 C. 140 D. 135

This is a free reviewer. All rights reserved.


1000 MMR Author: Victor A. Tondo Jr., LPT
579. Factorize (2x + y) (2x + y + 6) + 5 588. Solve for x: 2x + 3y = -8, and 5x – 7y = 67
A. (2x + y + 5) (2x + y + 1) A. 5 B. 6 C. -5 D. -6
B. (2x + y – 5) (2x + y – 1)
C. (2x + y + 5) (2x – y + 1)
D. (2x – y + 5) (2x – y + 1) 589. A farmer has 15 goats, 23 pigs, and a few
chickens in his farm. If he counted a total of 200
legs in his farm (excluding his, of course), how
580. Simplify: (2x3y4z2)5 many chickens does he have?
A. 10x15y20z10 B. 25x15y20z10 A. 96 B. 72 C. 48 D. 24
C. 32x15y20z10 D. 100x15y20z10


590. Rationalize
581. Find k such that 5x2+ 30x + k has two √
√ √ √ √ √ √
unequal real roots. A. B.
A. k > 6 B. k < 6 √ √ √ √ √ √
C. k > 45 D. k < 45 C. D.

582. Which of the following is not a polynomial? 591. If 3x + 2 = 4y and 5y – 3 = z, express z in


A. π r2 + 2 π r B. m – n terms of x.
C. √2 x + √3 y – √5 z D. ab √ cd A. 15x + 9 B. 15x + 7
C. D.

583. Which of the following is irrational?


A. 0.193193193193… B. 592. If 72x + 3 = B, what is B ÷ 49x?
C. √1023 D. √2401 A. 343 B. 21 C. 7 D. 3

584. If x + y = 17 and x2 + y2 = 135, find xy. 593. Multiply: (7x + 4) (2x – 1) (3x + 5)
A. 78 B. 77 C. 76 D. 75 A. 42x3 + 73x2 + 7x – 20
B. 42x3 – 73x2 + 7x – 20
C. 42x3 + 73x2 – 7x – 20
585. If H and K are the roots of 5x2 – 8x + 9, find D. 42x3 – 73x2 – 7x – 20
H + K – HK.
A. -2/5 B. -1/5 C. 0 D. 1/5
594. Rayon invested P1,600,000.00 in a bank
that offers 2.5% interest per annum. How much
586. If H and K are the roots of 3x2 + 4x + 5, find will his account hold after 4 years?
H2 + K2. A. P160,000 B. P1,160,000
A. -5/9 B. -1 C. -14/9 D. -2 C. P1,460,000 D. P1,760,000

587. Factorize 9x2 + 24xy + 16y2 – 81z2. 595. Solve for x: 45x+3 = 82x+6
A. (3x + 4y + 9z) (3x + 4y – 9z) A. 2 B. 3 C. 4 D. 5
B. (3x – 4y + 9z) (3x + 4y – 9z)
C. (3x + 4y + 9z) (3x – 4y – 9z)
D. (3x – 4y + 9z) (3x – 4y – 9z)

This is a free reviewer. All rights reserved.


1000 MMR Author: Victor A. Tondo Jr., LPT
596. Given = , which of the following 605. Which of these has the longest perimeter?
A. A regular pentagon with sides 21 cm long
expressions is equal to x?
B. A rectangle 29 cm long and 24 cm wide
A. B. C. D. C. An equilateral triangle with sides 36 cm long
D. A square whose area is 1024 cm2

597. Add: (2x2 + 4x + 5) + (5 – 6x – x2)


A. x2 + 2x + 10 B. 3x2 + 2x + 10 606. The hypotenuse of a right triangle is 82 feet.
C. x – 2x + 10
2 D. 3x2 + 2x + 10 If one leg is 80 feet, what is the length of the
other leg in feet?
A. 15 B. 16 C. 17 D. 18
598. Solve for x: 272x–8 = 9x+10
A. 9 B. 10 C. 11 D. 12
607. Find the surface area of a rectangular box
whose dimensions are 25 cm x 35 cm x 45 cm.
599. If 25x – 49y = 2019, what is 49y – 25x? A. 7000 cm2 B. 7045 cm2
A. 2018 B. -2019 C. 7150 cm 2 D. 7745 cm2
C. D. cannot be determined

608. A and B form a linear pair. If m A = 3x


and m B = 5x + 40, what is the value of x?
600. If x + y = 90 and x2 + y2 = 2020, find xy.
A. 17.5 B. 18 C. 19 D. 19.75
A. 3025 B. 3030
C. 3035 D. 3040
609. 1 and 3 are opposite angles in a
parallelogram. If m 1 = 50o, what is m 3?
601. Which of the following is ALWAYS true?
A. 40o B. 50o C. 140o D. 130o
A. Vertical pairs of angles are supplementary.
B. Vertical pairs of angles are congruent.
C. Linear pairs of angles are congruent.
610. A and C are consecutive angles in a
D. Linear pairs of angles are complementary.
parallelogram. If m A = 60o, what is m C?
A. 40o B. 50o C. 150o D. 120o
602. How many line segments can be made from
28 non-collinear points?
611. Two parallel lines are cut by a transversal,
A. 378 B. 394
forming R and V. If the two angles are
C. 412 D. 422
corresponding angles, what is the measure of R
if the measure of V is 70o?
603. The vertex angle of an isosceles triangle is A. 35o B. 70o
70°. What is the measure of one of the base C. 160 o D. 110o
angles?
A. 45° B. 50° C. 55° D. 60°
612. If the sum of the supplement and the
complement of an angle is 100 degrees, what is
604. A shoebox measures 20 inches by 15 inches the angle?
by 9 inches. What is its volume in cubic inches? A. 65o B. 70o C. 75o D. 85o
A. 900 B. 1350
C. 1650 D. 2700

This is a free reviewer. All rights reserved.


1000 MMR Author: Victor A. Tondo Jr., LPT
613. It is the perpendicular bisector of a regular 620. In the figure, A is the center of the circle.
polygon’s side, passing through the center. What is the measure of BDC?
A. side B. apothem A. 160o B. 80o C. 40o D. 20o
C. asymptote D. diagonal

614. In the figure, // . If m AFE = 130o,


find the measure of CGH.

621. Find the measure of E using the figure of


parallelogram WENA below.

A. 50o B. 130o C. 65o D. 25o

615. The measure of each interior angle of a A. 15 B. 65 C. 115 D. 85


regular polygon is 170o. How many vertices does
it have?
A. 36 B. 24 C. 12 D. 10 622. In parallelogram
MATH, m A = 7x – 32 and
m T = 5x + 32. Find m A.
616. How many diagonals does a regular A. 15 B. 63
dodecahedron have? C. 73 D. 117
A. 45 B. 54 C. 60 D. 66

623. The diagonal of a rectangular prism is 17


617. Find the area of a square whose perimeter cm long. If it is 9 cm thick and 12 cm long, how
is 480 cm. wide is it?
A. 14400 cm2 B. 28800 cm2 A. 15 cm B. 10 cm
C. 57600 cm2 D. 270400 cm2 C.8√3 cm D. 8 cm

618. A circle has a diameter of 34 cm. What is its 624. Statement 1: A square is a rhombus.
area? Statement 2: A square is a rectangle.
A. 17 π cm2 B. 34 π cm2 A. Only the first statement is true.
C. 289 π cm2 D. 1156 π cm2 B. Only the second statement is true.
C. Both statements are true.
D. Both statements are false.
619. The perimeter of a rectangle is 84. If its
length is 29 cm, find its area.
A. 377 cm2 B. 1218 cm2 625. Which of the following has its incenter,
C. 2436 cm2 D. 4872 cm2 circumcenter, centroid, and orthocenter in just
one point?
A. Equilateral Triangle B. Right Triangle
C. Scalene Triangle D. Obtuse Triangle

This is a free reviewer. All rights reserved.


1000 MMR Author: Victor A. Tondo Jr., LPT
626. In triangles, this line segment is drawn from 634. Find the volume of a rectangular pyramid
midpoint of one side to its opposite vertex. given that its base has length 21 cm and width 10
A. Median B. Altitude cm, and its height is 15 cm.
C. Bisector D. Longitude A. 700 cm3 B. 1050 cm3
C. 1575 cm 3 D. 1400 cm3

627. The radius of a cone measures 12 cm and its


height is 15 cm. Find its volume. 635. The diameter of a cylinder is 24 cm. If its
A. 240 π cm3 B. 360 π cm3 height is 40 cm, find its surface area.
C. 480 π cm3 D. 720 π cm3 A. 312 π cm2 B. 624 π cm2
C. 936 π cm 2 D. 1248 π cm2

628. This is located at the intersection of the


perpendicular bisectors of a triangle. 636. In the figure, AC is a diagonal of rhombus
A. Centroid B. Circumcenter ABCD. If m B = 118o, what is m CAD?
C. Incenter D. Orthocenter A. 124o B. 118o C. 62o D. 31o

629. The radius of a cylinder measures 9 cm and


its height is 15 cm. Find its volume.
A. 1215 π cm3 B. 810 π cm3
C. 540 π cm3 D. 360 π cm3

637. A rhombus has diagonals measuring 20 cm


630. What is the measure of each exterior angle and 30 cm. What is its area?
of a regular decagon? A. 75 cm2 B. 150 cm2
A. 108o B. 72o C. 60o D. 36o C. 300 cm2 D. 600 cm2

631. Find the area of an isosceles trapezoid given 638. A rhombus as diagonals measuring 30 cm
bases measuring 20 cm and 30 cm, with each of and 16 cm. What is its perimeter?
the congruent slants measuring 13 cm. A. 136 cm B. 92 cm
A. 250 cm2 B. 275 cm2 C. 68 cm D. 46 cm
C. 300 cm 2 D. 325 cm2

639. Find the approximate distance around a


632. Which of the following side lengths belong semicircular park with radius 100m.
to an acute triangle? A. ~1028 m B. ~771 m
A. 5 cm, 7 cm, 9 cm B. 6 cm, 8 cm, 10 cm C. ~514 m D. ~257 m
C. 6 cm, 9 cm, 12 cm D. 7 cm, 9 cm, 11 cm

640. Find the equation of the line perpendicular


633. A prism has a right triangle as its base. The to 7x + 4y = 12, passing through (-2, 9).
two legs of the right triangular base are 6 cm and A. 4x – 7y = -71 B. 4x + 7y = 71
8 cm long. If the prism is 2 cm thick, find its C. 4x – 7y = 71 D. 4x + 7y = -71
lateral surface area.
A. 24 cm2 B. 48 cm2
C. 56 cm 2 D. 72 cm2

This is a free reviewer. All rights reserved.


1000 MMR Author: Victor A. Tondo Jr., LPT
641. Find the equation of the line perpendicular 646. In the figure below, m CAB = 2x + 4, while
to 9x –7y = -6, passing through (4, -3). m COB = 5x – 7. Find x.
A. 7x + 9y = 1 B. 7x + 9y = -1
C. 7x – 9y = 1 D. 7x – 9y = -1

642. A rectangle is drawn with dimensions 28 cm


by 42 cm. A larger rectangle is drawn by adding
5 cm margins from each side of the original
A. B. -2 C. D. 15
rectangle. What is the area of the larger
rectangle?
A. 1456 cm2 B. 1551 cm2
C. 1556 cm 2 D. 1976 cm2 647. In the figure below, // . If m AGE =
7x + 11 and m CHF = 5x + 1, what is m AGE?

643. Find the length of the diagonals of a


rectangular prism 6 cm thick, 15 cm long, and 10
cm wide.
A. 17√3 cm B. 18√2 cm
C. 19 cm D. 20√5 cm

A. 119o B. 109o C. 71o D. 61o


644. Find the volume of the following pool.

648. Find the sum of all interior angles of a


regular 15-sided polygon.
A. 5400o B. 2700o
C. 2340 o D. 1170o

649. If each square in the figure has an area of 36


A. 700 m3 B. 1050 m3 square cm, find the perimeter of the figure.
C. 1400 m3 D. 2800 m3

645. What is the slope of the line defined by the


equation 3x + 5y = 11?
A. B. C. 3 D. 5 A. 864 cm B. 792 cm
C. 144 cm D. 132 cm

This is a free reviewer. All rights reserved.


1000 MMR Author: Victor A. Tondo Jr., LPT
650. Find the area of the shaded part of the 656. Find the equation of the line passing
circle. through (2, -7) with a slope of -3.
A. y = -3x – 1 B. y = -3x + 1
C. y = -3x – 2 C. y = -3x + 2

657. A square has a diagonal measuring 28 cm.


Find its area.
A. 9720 π cm2 B. 972 π cm2 A. 784 cm2 B. 392 cm2
C. 540 π cm2 D. 270 π cm2 C. 196 cm2 D. 98 cm2

651. In the figure, ̂ = 120o and m E = 28o. 658. Symbol for line.
Find ̂ . A. B.
C. D.

659. Symbol for ray.


A. B.
A. 46o B. 54o C. 64o D. 72o C. D.

652. Find the volume of a cone whose radius is 660. Find the volume of the swimming pool
30 cm and height is 170 cm. drawn below.
A. 289,000 π B. 153,000 π A. 2000 cu. ft. B. 2100 cu. ft.
C. 51,000 π D. 17,000 π C. 2250 cu. ft. D. 2500 cu. ft.

653. Find the center of a circle given one of its


longest chords has endpoints at M(-11,8) and
N(23,-24).
A. (12, -16) B. (6, -8)
C. (-17,16) D. (17, -16)

654. Find the radius of a circle if its diameter has


endpoints at M(3,5) and N(-12,13).
A. 8 B. 8.5 C. 9 D. 9.5
661. Which geometric figure extends indefinitely
in two opposite directions?
655. Find the equation of the line passing A. angle B. ray
through (7, 5) and (5, 1). C. line D. line segment
A. y = 2x + 9 B. y = 2x – 9
C. y = ½ x + 9 D. y = ½ x – 9

This is a free reviewer. All rights reserved.


1000 MMR Author: Victor A. Tondo Jr., LPT
662. In the figure, ACB is formed by the 670. Find the equation of a circle if its diameter
tangents AC and BC. If m ACB = 34o, what is the has endpoints at M(3,5) and N(-12,13).
measure of the minor intercepted arc? A. (x + 4.5)2 + (y + 9)2 = 72.25
B. (x – 4.5)2 + (y + 9)2 = 72.25
C. (x + 4.5)2 + (y – 9)2 = 72.25
D. (x – 4.5)2 + (y – 9)2 = 72.25

A. 112o B. 146o C. 68o D. 34o 671. A bicycle’s wheel is 24 inches in diameter. If


it revolves 625 times, how far does the bicycle
travel?
663. X(7,9) is the midpoint of A(1,2) and B. Find A. 7,500 π B. 15,000 π
the coordinates of B. C. 30,000 π D. 60,000 π
A (-13, -16) B. (13, -16)
C. (-13, 16) D. (13, 16)
672. A right triangle is inscribed in a circle. If the
triangle’s legs are 16 cm and 30 cm, what is the
664. Point O is two-thirds the way from A(-4,5) area of the circle?
to B(8,-10). Find the coordinates of point O. A. 289 π cm2 B. 480 cm2
A. (0, 0) B. (4, -5) C. 480 π cm2 D. 1156 π cm2
C. (3, -6) D. (3, -4)

673. In the figure, a rectangle is


665. H and K form a vertical pair. If m H = inscribed in a circle. If the rectangle
4x + 20 and m K = 6x – 10, find x. is 20 cm by 48 cm, what is the area of
A. 13 B. 15 C. 17 D. 19 the shaded figure?
A. 676π – 960 cm2 B. 1352π – 960 cm2
C. 2304π – 960 cm 2 D. insufficient data
No item #666, as requested by many subscribers.

674. What is the area of the red trapezium in the


667. F and G form a linear pair. If m F = given figure?
4x + 20 and m G = 6x – 10, find x.
A. 13 B. 15 C. 17 D. 19

668. Point C is two-fifths the way from M(-3,4) to


N(7,-11). Find the coordinates of point C.
A. (0, 0) B. (2, -1)
C. (1, -2) D. (1, -1) A. 23 sq. units B. 24 sq. units
C. 25 sq. units D. 26 sq. units

669. Find the slope of the line 5x – 6y = 7.


A. B. C. D.

This is a free reviewer. All rights reserved.


1000 MMR Author: Victor A. Tondo Jr., LPT
675. Which of the following are not congruent? 682. Which of the following is false?
A. Corresponding angles formed by two parallel A. Radii of the same circle are always congruent.
lines cut by a transversal. B. Any two right angles are congruent.
B. Alternate exterior angles formed by two C. An inscribed right angle always intercepts a
parallel lines cut by a transversal. semicircle.
C. Pairs of vertical angles. D. There are infinitely many lines that may be
D. Interior angles on the same side of the drawn parallel to a given line passing through a
transversal cutting two parallel lines. given point.

676. A rectangle is 24 cm long. If its diagonal is 683. The volume of a cube is 343 cm3. What is its
25 cm, what is its perimeter in cm? surface area?
A. 49 B. 62 C. 64 D. 98 A. 24 cm2 B. 24√3 cm2
C. 48 √13 cm2 D. 294 cm2

677. Square FACE is drawn such that one of its


sides AC is the diagonal of a rectangle, ABCD. If 684. Find the volume of a rectangular plank of
AB = 20 and BC = 43, find the area of FACE. wood that is half-inch thick, six inches wide, and
6 feet long.
A. 2209 cm2 B. 2229 cm2 A. 18 cubic inches B. 36 cubic inches
C. 2249 cm2 D. 2269 cm2 C. 108 cubic inches D. 216 cubic inches

678. If the vertex angle of an isosceles triangle is 685. In the following figure, O is the center of the
50o, what is the measure of each base angle? circle. If m CAB = 36o, what is m DBA?
A. 130o B. 80o C. 65o D. 25o

679. Which of the following is a square?


A. VCTR whose sides measure 20 cm each
B. LUVS whose angles are 90o each
C. RWNA whose diagonals are 50 cm each A. 18o B. 36o C. 54o D. 72o
D. AYIE whose congruent diagonals
perpendicularly bisect each other
686. What is the measure of each interior angle
of a regular nonagon?
680. The following are measures of sides of four A. 135o B. 140o
triangles. Which of them are taken from an C. 144 o D. 145o
obtuse triangle?
A. 20, 20, 29 B. 20, 21, 29
C. 20, 22, 28 D. 20, 23, 30 687. Find the surface area of a ball whose radius
is 24 cm.
A. 576 π cm2 B. 1152 π cm2
681. Find the length of the intercepted arc of a C. 2304 π cm 2 D. 4608 π cm2
central angle measuring 60o given that the radius
of the circle is 12 cm.
A. 4 π cm B. 6 π cm
C. 8 π cm D. 12 π cm

This is a free reviewer. All rights reserved.


1000 MMR Author: Victor A. Tondo Jr., LPT
688. Give the converse of the conditional 693. How many diagonals does a regular
statement “If two angles are vertical angles, then decagon have?
they are congruent.” A. 30 B. 35 C. 40 D. 45
A. “If they are are congruent, then two angles are
vertical angles.”
B. “If two angles not vertical angles, then they are 694. Each side of a regular hexagon is 256 cm
not congruent.” long. Find the distance around it.
C. “If they are not congruent, then two angles are A. 1536 cm B. 1280 cm
not vertical angles.” C. 1024 cm D. 718 cm
D. “If two angles are linear angles, then they are
supplementary.”
695. What can be said about these statements?
i. Any rectangle has two congruent diagonals.
689. Give the inverse of the conditional ii. Any rhombus has two perpendicular
statement “If two angles are vertical angles, then diagonals.
they are congruent.” A. Only the first statement is true.
A. “If they are are congruent, then two angles are B. Only the second statement is true.
vertical angles.” C. Both statements are true.
B. “If two angles not vertical angles, then they are D. Both statements are false.
not congruent.”
C. “If they are not congruent, then two angles are
not vertical angles.” 696. In the figure, A is the midpoint of and
D. “If two angles are linear angles, then they are . By which triangle congruence postulate can
supplementary.” we prove that △CAB △DAE?

690. In which geometry do two distinct lines


intersect in two points?
A. Euclidean Geometry
B. Hyperbolic Geometry
C. Elliptic Geometry
D. Plane Geometry

691. In which geometry can the sum of the A. SSS Congruence Postulate
measures of the angles of a triangle be less than B. SAS Congruence Postulate
180 degrees? C. ASA Congruence Postulate
A. Euclidean Geometry D. ASS Congruence Postulate
B. Hyperbolic Geometry
C. Elliptic Geometry
D. Plane Geometry 697. Find the circumference of a coin whose
radius is 13 cm.
A. 13 π cm B. 26 π cm
692. In which geometry are the summit angles of C. 169 π cm D. 676 π cm
a Saccheri quadrilateral obtuse angles?
A. Euclidean Geometry
B. Hyperbolic Geometry
C. Elliptic Geometry
D. Plane Geometry
This is a free reviewer. All rights reserved.
1000 MMR Author: Victor A. Tondo Jr., LPT
698. Find the area of a square whose perimeter 707. The longest side in a 30-60-90 triangle is
is 280 cm. 480 cm. How long is the shortest side?
A. 1120 cm2 B. 4900 cm2 A. 120 cm B. 160 cm
C. 19600cm2 D. 78400 cm2 C. 240 cm D. 360 cm

699. Find the altitude to the hypotenuse of a 708. If the hypotenuse of a 45-45-90 triangle is
right triangle whose legs are 60 cm and 80 cm. 50 cm long, how long is a leg?
A. 40 cm B. 48 cm A. 25 cm B. 25√2 cm
C. 96 cm D. 100 cm C. 25√3 cm D. 50√2 cm

700. This is formed by two rays with a common 709. Which of the ff. is coterminal with 143o?
end point. A. 2403o B. 2303o
A. polygon B. angle C. 2103o D. 2003o
C. line D. line segment

710. What is the measure of the smaller angle


701. Convert rad to degrees. formed by the hands of the clock at 5:55?
A. 210o B. 420o A. 180o B. 174.75o
C. 630o D. 840o C. 163.625o D. 152.5o

702. What is the reference angle of 212o? 711. What is the measure of the smaller angle
A. 32o B. 68o C. 106o D. 12o formed by the hands of the clock at 3:50?
A. 185o B. 175o
C. 170o D. 165o
703. A negative sine and a positive cosine are
properties of angles in which quadrant?
A. QI B. QII C. QIII D. QIV 712. Which of the following is false?
A. tan θ =
B. csc θ =
704. Which of the following equal to sec 50o?
A. sin 40o B. cos 40o C. sec θ =
C. csc 40o D. cot 40o
D. sin θ =

705. Which of the ff. is coterminal with 2019o?


A. 219o B. 209o 714. If sin x = 0.936, which of the following could
C. 199o D. 189o be cos x?
A. 0.064 B. 0.8
C. 0.016 D. -0.352
706. The angle of elevation to the top of a
building is 30o. If the observer is 50√3 meters
away from the building, how tall is the building?
A. 25 m B. 25√3 m
C. 50 m D. 100 m

This is a free reviewer. All rights reserved.


1000 MMR Author: Victor A. Tondo Jr., LPT
715. A man stands 20 meters away from a 724. A sniper is on top of a 20-meter cliff. He
building. If the angle of elevation to the top of the spots a target at an angle of depression of 30o.
building is 60o, how tall is the building? How far must his bullet travel to hit the target?
A. 20√2 m B. 20√3 m A. 20 meters B. 20 √2 meters
C. 40 m D. 20√5 m C. 20 √3 meters D. 40 meters

716. If csc x = , what is sin x? 725. A meter stick leans to a wall and reaches a
point 50 cm away from the wall. What is the
A. B. C. D. measure of the angle formed by the meter stick
and the wall?
A. 30o B. 45o C. 60o D. 75o
717. If sec x = , which of the following could be
sin x?
726. A tight rope connects the top of a 500 cm
A. B. C. D.
pole to the ground. If the angle formed by the
ground and the tight rope is 45o, how long is the
tight rope?
718. Which of the following is false? A. 500 cm B. 500√2 cm
A. cos A (tan A) = sin A
C. 500√3 cm D. 500√5 cm
B. csc A (cos A) = cot A
C. sin A (sec A) = tan A
D. tan A (csc A) = cos A
727. Which of the
following can be used
to solve for x in the
719. Which of the ff. is coterminal with 63o?
figure?
A. 7503o B. 7303o
C. 7103o D. 6903o A. sin
B. cos
C. arcsin
720. Convert 160o to radians.
A. π B. π C. π D. π D. arccos

721. What is the reference angle of 156o? 728. Evaluate: 2 sin 30 – 4 cos 60 + 3 tan 45.
A. 14o B. 24o C. 56o D. 66o A. -10.2378 B. -12.1426
C. 2 D. 1

722. In which quadrants is the sine function


negative? 729. Which of the following is true?
A. QI and QII B. QII and QIII A. sin x = sin (-x) B. cos x = cos (90 – x)
C. QIII and QIV D. QIV and QI C. –sin x = sin (-x) D. tan x = (sin x)(cos x)

723. What is the reference angle if 2019o?


A. 19o B. 39o C. 51o D. 59o

This is a free reviewer. All rights reserved.


1000 MMR Author: Victor A. Tondo Jr., LPT
730. Which of the following is true? 740. Give the range of f(x) = 2020.
A. sin x = csc (90 – x) A. y | y B. y 2020
B. cos x = csc (90 – x) C. y = 2020 D. y = { }
C. tan x = cot (90 – x)
D. sec x = cos (90 – x)
741. Give the range of f(x) =

731. Evaluate: lim 2019 A. y | y B. y


A. 2019 B. -2019 C. y 2 D. y -2
C. 0 D. 1

742. Give the range of f(x) = 20x – 20.


732. Evaluate: lim (20 – 5x2) A. y | y B. y 1
A. 1 B. 15 C. -15 D. 10 C. y = 20 D. y = { }

733. Evaluate: lim ( 3 – 5x2) 743. Give the range of f(x) = x125.
A. 3 B. 18 C. -18 D. 72 A. y | y B. y 0
C. y 2019 D. y = { }

734. Evaluate: lim


744. Give the range of f(x) =
A. LDNE B. 2 C. 4 D. 0
A. y | y B. y
C. y D. y -2
735. Evaluate: lim
A. + B. - C. LDNE D. 2
745. Give the range of f(x) = x2019.
A. y | y B. y 0
736. Evaluate: lim C. y 2019 D. y = { }

A. + B. - C. LDNE D.
746. Give the range of f(x) = 2019x – 2019.
A. y | y B. y 1
737. Give the domain of f(x) = C. y = 2019 D. y = { }
A. x | x B. x
C. x 2 D. x -2
747. Give the range of f(x) =
A. y | y B. y
738. Give the domain of f(x) = √2018 2019. C. y D. y
A. x | x B. x
C. x > 0 D. x
748. Give the domain of f(x) =
A. x | x B. x
739. Give the domain of f(x) = x2 – 20x + 19.
C. x 2 D. x -3
A. x | x B. x 1, 9
C. x > -81 D. x -81
This is a free reviewer. All rights reserved.
1000 MMR Author: Victor A. Tondo Jr., LPT
749. What is 0! equal to? 758. Find the remainder when 1998! is divided
A. 0 B. 1 by 2000.
C. undefined D. + A. 1024 B. 256 C. 64 D. 0

750. Give the range of f(x) = 2019x + 2020. 759. In Mathematics, what does the delta ∆
A. y | y B. y 2020 symbol mean or refer to?
C. y 2019 D. y 2020 A. the change in B. the summation of
C. therefore D. belonging to

751. Find the first derivative: f(x) = (2x + 3)5.


A. f ’(x)= 5 (2x + 3)4 760. In Mathematics, what does the sigma
B. f ’(x)= 10 (2x + 3)4 symbol mean or refer to?
C. f ’(x)= 15 (2x + 3)4 A. the change in B. the summation of
D. f ’(x)= 20 (2x + 3)4 C. therefore D. belonging to

752. Find f ‘(x) given f(x) = (3x2 – 2x + 1 )7. 761. In Mathematics, what does the epsilon
A. f ’(x)= (42x – 14) (3x2 – 2x + 1)6 symbol mean or refer to?
B. f ’(x)= (42x – 28) (3x2 – 2x + 1)6 A. such that B. element of
C. f ’(x)= (42x + 14) (3x2 – 2x + 1)6 C. therefore D. ergo
D. f ’(x)= (42x + 28) (3x2 – 2x + 1)6

762. In set theory, what does the symbol mean


753. Find the maximum area that can be or refer to?
enclosed by a rectangle given its perimeter A. not O B. not zero
should only be 202 meters. C. does not D. empty
A. 2550 m2 B. 2550.25 m2
C. 2550.5 m2 D. 2550.75 m2
763. Which of the following is ALWAYS an odd
number?
754. Find the lowest possible value of the A. The difference of two odd numbers.
function f(x) = x2 – 8x + 7. B. The sum of two odd numbers.
A. -10 B. -9 C. -8 D. -7 C. The product of two odd numbers.
D. The product of two even numbers.

755. Find the instantaneous rate of change for


f(x) = 5x3 – 2x2 + 7x – 9 at x = 2. 764. Given 2x + 2x = 2y, express y in terms of x.
A. 37 B. 48 C. 59 D. 70 A. y = 2x B. y = x2
C. y = x + 2 D. y = x + 1

756. Find the instantaneous rate of change for


f(x) = 4x3 + 5x2 – 7x – 11 at x = 1. 765. In set theory, what does the symbol mean
A. -15 B. 0 C. 15 D. 30 or refer to?
A. no elements B. common elements
C. all elements D. half of the elements
757. Find the slope of the line tangent to
y = 3x5 – 4x3 + 5x2 – 10x + 9 at x = 2.
A. 50.5 B. 101 C. 151.5 D. 202
This is a free reviewer. All rights reserved.
1000 MMR Author: Victor A. Tondo Jr., LPT
766. In Mathematics, what does the symbol 774. Which of the
mean? following
A. since B. prove represents the
C. therefore D. previously shaded region in
the Venn diagram?
A. A’ B’
767. Which of the following is the Geometric B. A’ B’
symbol for congruent? C. (A B)’
A. B. C. D. // D. (A B)’

768. Which of the following is the obelus sign? 775. In a class of 50, there are 20 students who
A. ! B. ÷ C. ⊙ D. θ excel in Math, 24 who excel in Science, and 4 who
excel in both Math and Science. How many of
them excel in neither?
769. Given X = {1, 4, 16, 64} and Y = {1, 2, 3, 4}, A. 2 B. 4 C. 7 D. 10
find X Y.
A. {1, 4} B. {16, 64}
C. {2, 3} D. {1, 2, 3, 4, 16, 64} 776. There are 50 students in a class. 39 of them
have a cellphone, 17 of them have a laptop, and 5
of them have neither. How many have both a
770. Given X = {1, 2, 4, 8} and Y = {1, 2, 3, 4}, find cellphone and a laptop?
X – Y. A. 6 B. 7 C. 9 D. 11
A. {1, 4, 8} B. {8}
C. {3, 8} D. {1, 2, 3, 4, 8}
777. Rayon has 10 shirts, 5 pairs of pants, and 4
pairs of shoes. How many ways can he pick what
771. Given X = {1, 2, 4, 8} and Y = {1, 2, 3, 4}, find to wear for today?
Y – X. A. 19 B. 20 C. 190 D. 200
A. {1, 4, 8} B. {3}
C. {3, 8} D. {1, 2, 3, 4, 8}
778. Using the digits 1, 2, 3, 4, 5, and 6 without
repetition, how many 3-digit numbers can be
772. Given X = {6, 5, 4, 3} and Y = {1, 2, 3, 4}, find made?
X Y. A. 480 B. 240 C. 120 D. 60
A. {3, 4} B. {6, 5}
C. {1, 2} D. {1, 2, 3, 4, 5, 6}
779. Using the digits 1, 2, 3, 4, 5, 6, and 7 without
repetition, how many 4-digit numbers can be
made?
A. 1680 B. 840 C. 420 D. 210
773. Which of the
following
represents the 780. Using the digits 0, 1, 2, 3, 4, 5, and 6 without
shaded region in repetition, how many 3-digit numbers can be
the Venn diagram? made?
A. A B B. A B A. 360 B. 180 C. 90 D. 45
C. A – B D. B – A

This is a free reviewer. All rights reserved.


1000 MMR Author: Victor A. Tondo Jr., LPT
781. Using the digits 0, 1, 2, 3, 4, 5, 6, and 7 789. Evaluate: log6 8 + 3 log6 3.
without repetition, how many 4-digit numbers A. 24 B. 14 C. 9.5 D. 3
can be made?
A. 1680 B. 1470 C. 840 D. 735
790. How many ways can the word ABABA be
rearranged?
782. Using the digits 0, 1, 2, 3, 4, 5, 6, and 7 A. 120 B. 60 C. 20 D. 10
without repetition, how many 3-digit numbers
can be made?
A. 672 B. 588 C. 336 D. 294 791. There are 10 different beads and a locking
mechanism to be used in making a bracelet. How
many different bracelet patterns can be made?
783. Using the digits 0, 1, 2, 3, 4, 5, and 6 without A. 3,628,800 B. 362,880
repetition, how many odd 3-digit numbers can C. 1,814,400 D. 181,440
be made?
A. 180 B. 150 C. 90 D. 75
792. There are six different Math books, four
different Science books, and two different
784. Using the digits 0, 1, 2, 3, 4, 5, 6, and 7 English books to be arranged on a shelf. How
without repetition, how many odd 4-digit many ways can this be done?
numbers can be made? A. 12C3 B. 12! C. 48 D. 3!
A. 1470 B. 1440 C. 735 D. 720

793. There are six different Math books, four


785. Using the digits 0, 1, 2, 3, 4, 5, 6, and 7 different Science books, and two different
without repetition, how many odd 3-digit English books to be arranged on a shelf. If books
numbers can be made? of the same subject must be together, how many
A. 336 B. 288 C. 168 D. 144 ways can this be done?
A. 479,001,600 B. 207,360
C. 34560 D. 144
786. Using the digits 0, 1, 2, 3, 4, 5, 6, and 7
without repetition, how many even 3-digit
numbers can be made? 794. There are six people to be seated on a bench
A. 162 B. 156 C. 150 D. 144 for a picture. A certain couple, Vic and Rowena,
are to be seated next to each other. How many
ways can this be done?
787. Using the digits 0, 1, 2, 3, 4, 5, 6, and 7 A. 120 B. 240 C. 360 D. 720
without repetition, how many even 4-digit
numbers can be made?
A. 765 B. 750 C. 735 D. 720 795. There are six people to be seated on a bench
for a picture. A certain trio, Vic, Rayon, and Aira,
do not want to be separated. How many ways
788. Using the digits 0, 1, 2, 3, 4, 5, and 6 without can this be done?
repetition, how many even 3-digit numbers can A. 60 B. 72 C. 144 D. 180
be made?
A. 120 B. 105 C. 90 D. 75

This is a free reviewer. All rights reserved.


1000 MMR Author: Victor A. Tondo Jr., LPT
796. TNB Gaming has five “carry” players, four 804. There are ten red, nine blue, and six black
“offlane” players, and six “support” players. If a pens in a bag. If a pen is randomly drawn from
game calls for two carry, one offlane, and two the bag, what is the probability that it is red?
support players, how many possible lineups do A. B. C. D.
they have?
A. 1200 B. 600 C. 300 D. 150
805. There are ten red, nine blue, and six black
pens in a bag. If a pen is randomly drawn from
797. If today is a Sunday, what day is 125 days
the bag, what is the probability that it is black?
from now?
A. 0.06 B. 0.6 C. 0.24 D. 0.7
A. Friday B. Saturday
C. Sunday D. Monday
806. What does a probability of 0 pertain to?
A. an impossible event
798. If today is a Monday, what day is 200 days
B. a very unlikely event
from now?
C. a very likely event
A. Friday B. Saturday
D. a sure event
C. Sunday D. Monday

799. If three-fourths of a number is 33 more than 807. Simplify: √10 + 2√21.


one-fifth of itself, then what is the number? A. √2 + √5 B. √3 + √7
A. 80 B. 60 C. 40 D. 20 C. √3 √2 D. √5 √2

800. What time is 2019 hours after 3:00PM? 808. Simplify: √11 2√30.
A. 6:00 AM B. 12:00 NN
A. √6 √5 B. √6 + √5
C. 6:00 PM D. 12:00 MN
C. √3 2√2 D. 1

801. In a certain fastfood chain, soft drinks are


served in Small, Medium, and Large cups. 809. ∫(3 + 4)5 dx
What level of data is this? A. (3x + 4)6+ c B. (3x + 4)6+ c
A. Nominal B. Ordinal C. 5(3x + 4)4+ c D. (3x + 4)6+ c
C. Interval D. Ratio

810. ∫(6 + 6 6) dx
802. The prime factorization of a number is
given as 23 x 52 x 133. How many factors does it A. 2x + 3x + 6x + c
3 2 B. 2x3 – 3x2 + 6x + c
have? C. 2x3 + 3x2 – 6x + c D. 2x3 – 3x2 – 6x + c
A. 96 B. 48 C. 36 D. 18

811. ∫ dx
803. The prime factorization of a number is A. + c B. + c
given as 22 x 32 x 7 x 11. How many factors does
it have? C. +c D. +c
A. 96 B. 48 C. 36 D. 18

This is a free reviewer. All rights reserved.


1000 MMR Author: Victor A. Tondo Jr., LPT
812. ∫ dx 818. (x2 + 3x – 2)4 = _________
A. +c B. +c A. (x2 + 3x – 2)3
B. 4(x2 + 3x – 2)3
C. +c D. +c C. (8x + 12) (x2 + 3x – 2)3
D. (2x + 3) (x2 + 3x – 2)3

813. Mr. dela Cruz is going to present the growth


of a certain plant over a span of 10 weeks. Which 819. √ 2 + 3 = _________
graph would be best suited for this?
A. bar graph B. line graph A. B. √
C. pie graph D. pictograph √
C. √ D. √

814. Rayon is going to present the sales of six


different teams in the company from January to 820. (2x + 3)(4x – 5) = _________
June. Which graph would be best suited for this? A. 8x2 + 2x + 15 B. 8x2 + 2x – 15
A. bar graph B. line graph C. 8x – 2x – 15
2 D. 8x2 – 2x + 15
C. pie graph D. pictograph

821. ( ) = _________
815. Ms. Rowena is tasked to send 8 randomly
selected students from her class of 40 to the A. ( B. (
) )
clinic for drug test. She decides to let the
C. ( D. (
students count off from 1 to 5, after which she ) )
randomly picks a number from 1 to 5. All
students whose number is that which she picked
will be sent to the clinic. What sampling method 822. There are 17 red bags, 19 green bags, and
did Ms. Rowena use? 14 blue bags in a store. What percent of the bags
A. cluster B. systematic is blue?
C. stratified D. quota A. 38% B. 34%
C. 28% D. 27%

816. What is the 7th decile of the following data?


14 15 16 20 24 823. Which of the following is true?
25 26 26 27 29 A. (sin x) (cot x) = sec x
31 33 34 34 35 B. (cos x) (csc x) = tan x
C. (tan x) (csc x) = sec x
A. 31 B. 31.4 C. 31.5 D. 32 D. (cot x) (sec x) = cos x

817. Find the remainder when f(x) is divided by 824. Which of the following could be the value of
(x – 3) given f(x) = x3 – 3x2 + 5x – 13. x if x 4 (mod 10)?
A. 1 B. 2 C. 3 D. 4 A. 46 B. 49 C. 52 D. 54

825. Which of the following could be the value of


x if x 7 (mod 9)?
A. 146 B. 149 C. 151 D. 154

This is a free reviewer. All rights reserved.


1000 MMR Author: Victor A. Tondo Jr., LPT
826. Given ̅̅̅̅
BF bisects ABC and m ABF = 34o, 834. Fifteen guests shake hands with each other.
find m ABC. If each guest is to shake hands with all the other
A. 17o B. 34o C. 51o D. 68o guests, how many handshakes will be made?
A. 105 B. 150 C. 210 D. 225

827. If f(x) = x2 – 2x + 3 and g(x) = x + 1, find


fog(x). 835. The salary of 5 men for 6 days is P9,000.
A. x2 – 1 B. x2 – 2x + 4 How much is the salary of 7 men for 8 days?
C. x – 3x + 5
2 D. x2 + 2 A. P15,000 B. P15,600
C. P16,800 D. P17,400

828. How many 3-digit numbers can be formed


using the digits 0, 1, 2, 3, 4, 5, and 6 if repetition 836. The average grade of eleven students is 84.
is not allowed? If the average of seven of these students is 82,
A. 160 B. 180 C. 200 D. 210 what is the average of the other four students?
A. 88 B. 87.5 C. 86.5 D. 86

829. What is the longest side of ∆VAC if m V =


45o and m C = 65o? 837. A radius of a circle is 25 cm long. How long
A. ̅̅̅̅
VC B. ̅̅̅̅
AC C. ̅̅̅̅
VA D. ̅̅̅̅
CA is its longest chord?
A. 20√2 cm B. 40 cm
C. 50 cm D. It depends,
830. Which quadrilateral has two congruent
diagonals that are perpendicular to each other?
A. kite B. isosceles trapezoid 838. Find the largest area of a rectangular piece
C. rectangle D. rhombus of land that can be enclosed with 800 meters of
fencing material.
A. 40,000 m2 B. 56,250 m2
831. If the length of a rectangle is increased by C. 62,500 m 2 D. 80,000 m2
30% while the width is decreased by 30%, what
will happen to its area?
A. It stays the same. 839. Which statistical test is used for testing for
B. It is increased by 9%. relationship between two variables?
C. It is decreased by 9%. A. ANOVA B. t-test
D. It is decreased by 6%. C. Pearson R D. Chi Square

832. Rayon had an average of 93 on his first five 2 +4 5


Math tests. After taking the next test, his average 840. Given ( ) = { 5 = 5,
increased to 94. Find his most recent grade. 9 5
A. 96 B. 97 C. 98 D. 99 find lim ( ).
A. 5 B. 14
C. 16 D. limit does not exist
833. A bus drove for 7 hours at 73 kph and 3
hours at 88 kph. What was its average speed?
A. 75.5 kph B. 76.5 kph 841. Find the equation of the line passing
C. 77.5 kph D. 78.5 kph through (3, 7) and (-3, -5).
A. y = 2x + 1 B. y = 3x + 1
C. y = 2x – 1 D. y = 3x – 1
This is a free reviewer. All rights reserved.
1000 MMR Author: Victor A. Tondo Jr., LPT
842. In which non-Euclidean model for geometry A. 65 B. 61
can we have any given line ℓ and a point A which C. 57 D. 53
is not on ℓ, wherein all lines through A will
intersect ℓ?
A. hyperbolic B. elliptic 850. An item is sold for P7,280 after being
C. Saccheri D. Pythagorean marked down by 30%. What was its original
price?
A. P11,400 B. P10,900
843. If A is at (-9, 11) and B is at (6,-9), find C if C C. P10,400 D. P9,900
is three-fifths the way from A to B.
A. (1, -1) B. (0, 1)
C. (1, 1) D. (0, -1) 851. Which of the following lines passes through
the point (-4, 5)?
A. y = x + 1 B. y = 2x – 3
844. Find the vertex of y = 5x2 – 4x – 7. C. y = 1 – x D. y = 3 – 2x
A. ( , ) B. ( , )
C. ( , ) D. ( , )
852. Which of the following is outside the circle
defined by the equation (x + 2)2 + y2 = 50?
A. (-9, 1) B. (-7, 5)
845. Mr. G sold 70% of his chickens and still had C. (-5, 7) D. (2, 4)
129 chickens left. How many chickens did he
have originally?
A. 344 B. 430 853. Which of the following equations pertain to
C. 598 D. 860 a parabola?
A. y2 – 5y = x2 + 4
B. x2 + 5x – 4y = 13
846. Find the average rate of change of ( ) ( )
y = x3 – 6x2 + 5x – 20 from x = 1 to x = 6. C. + =1
A. 5 B. 6 ( ) ( )
D. =1
C. 7 D. 8

854. Which of the following equations pertain to


847. What is the remainder when
a parabola that opens to the left?
534,214,557,989,215 is divided by 4?
A. -2(y + 7) = (x – 3)2
A. 1 B. 2 C. 3 D. 0
B. 5(y – 9) = (x – 5)2
C. (y – 4)2 = 2(x + 7)
D. (y + 3)2 = -3(x + 4)
848. What do you call an equation in which only
integer solutions are allowed?
A. Diophantine equations
855. The lengths of two sides of a triangle are 7
B. Euclidean equations
inches and 15 inches. Which of the following
C. Fibonacci equations
represents x, the possible length in inches of the
D. Newtonian equations
remaining side of the triangle?
A. 8 < x < 22 B. 8 x 22
C. x < 8 or x > 22 D. x 8 or x 22
849. The sum of two numbers is 73 and their
difference is 41. What is the larger number?
This is a free reviewer. All rights reserved.
1000 MMR Author: Victor A. Tondo Jr., LPT
856. Rayon has 46 coins in P1 and P5 865. Which of the following Mathematicians had
denominations for a total worth of P186. How a method “Sieve” for identifying prime numbers?
many P5 coins does he have? A. Pythagoras B. Euclid
A. 25 B. 30 C. 35 D. 40 C. Eratosthenes D. Archimedes

857. If x = 2 and y = 3, what is x2 – y2 + 3xy? 866. Which Mathematician developed a formula


A. 31 B. 13 C. -5 D. -13 for finding the area of a triangle using its side
lengths?
A. Hipparchus B. Pythagoras
858. In an arithmetic sequence, the first term is C. Heron D. Euclid
200 and the common difference is -3. What is the
64th term?
A. 20 B. 17 C. 14 D. 11 867. Who refined and perfected the decimal
place value number system?
A. Greeks B. Romans
859. In an arithmetic sequence, the 23rd term is C. Chinese D. Indians
157 and the common difference is -4. What is the
34th term?
A. 113 B. 117 C. 121 D. 125 868. This Mathematician invented natural
logarithms.
A. Isaac Newton B. John Napier
860. Find the inverse of y = x2 – 6x + 5. C. Marin Mersenne D. Rene Descartes
A. √ + 4 + 3 B. √ + 4 – 3
C. √ 4+3 D. √ 4–3
869. Prime numbers that are 1 less than a power
of 2 are called ______.
861. ln e4 = _____ A. Pythagorean Primes B. Mersenne Primes
A. 24.1295 B. 12.06475 C. Fermat Primes D. Pascal Primes
C. 6.032375 D. 4

870. Which Mathematician developed the


862. eln 5 = _____ triangle of binomial coefficients?
A. 5 B. 6.25 C. 12.5 D. 25 A. Gottfried Leibniz B. Isaac Newton
C. Blaise Pascal D. Johann Bernoulli

863. What is ln e?
A. 0 B. 1 C. e D. 10 871. Two Mathematicians are known for the
theory of hyperbolic geometry although they
worked independently from each other. Who are
864. Who are the first to use papyrus? these Mathematicians?
A. Sumerians B. Egyptians A. Euclid and Pythagoras
C. Chinese D. Greeks B. Euclid and Descartes
C. Lobachevsky and Bolyai
D. Babbage and Bolyai

This is a free reviewer. All rights reserved.


1000 MMR Author: Victor A. Tondo Jr., LPT
872. Which Mathematician developed the use of 879. Convert rad to degrees.
AND, OR, and NOT operators in Algebra?
A. 70o B. 105o
A. George Boole C. 210o D. 420o
B. Janos Bolyai
C. Charles Babbage
D. August Mobius
880. Find the volume of a cylinder whose radius
is 50 cm and height is 24 cm.
A. 20,000 π cm3 B. 30,000 π cm3
873. Which Mathematician is known for his last
C. 45,000 π cm3 D. 60,000 π cm3
theorem?
A. Andrew Wiles
B. Pierre de Fermat 881. Find the altitude to the hypotenuse of a
C. John Wallis
right triangle whose legs measure 16 cm and 30
D. Rene Descartes
cm.
A. cm B. 240 cm
874. Who have the first fully-developed base-10 C. 34√2 cm D. 24√3 cm
number system in use?
A. Romans B. Egyptians
C. Indians D. Sumerians 882. How many line segments can be made from
25 non-collinear points?
A. 900 B. 600 C. 450 D. 300
875. Notched tally bones proved their use of
Mathematics around 35000 BCE.
A. Africans B. Asians 883. Dividing by 0.25 is the same as multiplying
C. Europeans D. Americans by which number?
A. 2 B. 4 C. 5 D. 16

876. Which sampling method is best used when


the population has subgroups? 884. Find the surface area of a sphere whose
A. systematic sampling radius is 24 cm.
B. stratified sampling A. 192 π cm2 B. 768 π cm2
C. quota sampling C. 1152 π cm2 D. 2304 π cm2
D. cluster sampling

885. Which of the following is the reference


877. Mr. Lazada drove for 2 hours at a speed of angle of 295o?
48 kph, 3 hours at 58 kph, and then 5 hours at 64 A. 25o B. 45o C. 65o D. 75o
kph. What was his average speed?
A. 57 kph B. 58 kph
C. 59 kph D. 60 kph 886. Which numerical system is sexagesimal
(base-60)?
A. Roman B. Babylonian
878. Find the domain of y = 3x. C. Mayan D. Hindu-Arabic
A. x | x B. x 0
C. x 3 D. x 0

This is a free reviewer. All rights reserved.


1000 MMR Author: Victor A. Tondo Jr., LPT
887. In solid geometry, what do you call a solid 894. The amount of money you have falls under
bound by polygons? what level of data?
A. tessellation B. multigon A. nominal B. ordinal
C. polyhedron D. soligon C. interval D. ratio

888. Two triangles have a pair of congruent 895. To study tooth decay a researcher takes a
angles. Which of the following is true about these sample at random but with the stipulation that
triangles? all age groups are represented proportionally.
A. They are isosceles triangles. What sampling method did the researcher use?
B. They are equilateral triangles. A. systematic B. cluster
C. They are congruent triangles. C. stratified D. convenience
D. They are similar triangles.

896. By what property do we say that when


889. Rayon deposited an amount of P500,000 in A = B and B = C, then A = C?
a bank that offers 4% interest compounded per A. reflexive B. symmetrical
annum. How much will he have in his account C. transitive D. closure
after 3 years?
A. P720,000 B. P600,000
C. P560,000 D. P562,432 897. In number theory, what do we call a
polynomial equation with integer coefficients
that also allows the variables and solutions to be
890. Find the range of the following scores: integers only?
37 40 44 45 41 39 48 A. Integral equations
B. Differential equations
A. 7 B. 9 C. 10 D. 11
C. Diophantine equations
D. Bolyai-Lobachevsky equations
891. What is the 9th decile of the following data?
13 15 17 19 21
898. How long is the latus rectum of the parabola
23 24 26 28 29
defined by (y + 3)2 = 20x – 196?
30 33 35 36 40
A. 3 B. 12 C. 20 D. 19
A. 34 B. 35.5 C. 37.6 D. 38

899. In the set of real numbers, what do we call


892. -20192 is NOT equal to ___. the set that includes only the counting numbers
A. (-2019)2 C. (-20192) and zero?
B. –(2019 x 2019) D. –(2019)2 A. rational numbers B. integers
C. whole numbers D. irrational numbers

893. The product of two numbers is 1425. If each


of the two numbers is doubled, the product of 900. Find the sum of the first 60 counting even
these larger numbers is _____. numbers.
A. 2,850 B. 5,700 A. 3660 B. 3600 C. 1830 D. 900
C. 8,550 D. 11.400

This is a free reviewer. All rights reserved.


1000 MMR Author: Victor A. Tondo Jr., LPT
901. If M and N are complementary angles, 908. Find the radius of x2 + y2 + 12x – 14y = 36.
which of the following is true? A. 6 B. 7 C. 11 D. 13
A. cos M = sec N B. sin M = cos N
C. tan M = csc N D. sec M = -cos N
909. Find the instantaneous rate of change for
f(x) = 4x3 + 7x2 – 27x – 49 at x = 1.
902. The top ten students of a graduating class A. -2 B. -1 C. 1 D. 2
got the following scores in their final
examination in Calculus:
89 84 83 89 89 910. log3 24 – 3 log3 2 = _____
85 88 93 83 98 A. 1 B. 0 C. -1 D. e
What is their mean score?
A. 87.4 B. 87.5 C. 88.1 D. 89.3
911. After using one-sixth of her budget on bills,
two-fifths on groceries, and P1700 on books and
903. The top ten students of a graduating class magazines, Mrs. Lazada still had P7400 left. How
got the following scores in their final much was her budget?
examination in Calculus: A. P20,000 B. P21,000
89 84 83 89 89 C. P22,500 D. P30,000
85 88 93 83 98
What is the modal score?
A. 83 B. 85 C. 89 D. 98 912. An assistant’s response times were
recorded on the table below. What is her average
response time in minutes?
904. The sum of five consecutive integers is 980. Response Time Frequency
What is the value of the greatest integer? 1 minute 3
A. 192 B. 194 C. 196 D. 198 2 minutes 7
3 minutes 11
4 minutes 9
905. x varies directly as y and inversely as z. If 5 minutes 5
x = 20 when y = 10 and z = 3, what is x when 6 minutes 3
y = 15 and z = 9? 7 minutes 2
A. 10 B. 15 C. 25 D. 30
A. 3.625 B. 3.575 C. 3.525 D. 3.5

906. Factorize 15x2 – 13x – 20.


A. (5x – 4)(3x – 5) B. (5x – 4)(3x + 5) 913. The average grade of 22 students in Section
C. (5x + 4)(3x – 5) D. (5x + 4)(3x + 5) Abaca is 95, while the average grade of 28
students in Section Acacia is 89. What is the
average grade of all 50 students in both sections?
907. Which of the following is the axis of A. 92.36 B. 91.64 C. 90.72 D. 89.8
symmetry of the parabola defined by the
equation y = x2 – 6x + 5?
A. x = 5 B. x = -5
C. x = 3 D. x = -3

This is a free reviewer. All rights reserved.


1000 MMR Author: Victor A. Tondo Jr., LPT
914. Find the slope of 2x – 6y = 13. 922. If x = 10, which of the following is equal to
A. 3 B. C. -3 D. 219?
A. 23x – 1 B. 2x2 + 5x + 5
C. 2x2 + 2x – 1 D. x2 + x + 2
915. Find the intersection of y = -3x – 2 and
y = 2x + 23.
923. In the equation 3x – 4y = 25, what is x when
A. (5, 13) B. (-5, -13)
y is 5?
C. (5, -13) D. (-5, 13)
A. 18 B. 15 C. 12 D. 9

916. A book was sold for P630 after a 10%


924. What is the measure of each interior angle
discount was given. How much was the book
of a regular 30-sided polygon?
originally?
A. 158o B. 162o
A. P800 B. P750
C. 165 o D. 168o
C. P700 D. P690

917. If the sum of the supplement and the 925. Simplify:


complement of an angle is 146, what is the A. csc x – tan x B. csc x + tan x
angle? C. csc x – cot x D. csc x + cot x
A. 61 B. 62 C. 63 D. 64

926. Simplify: x
918. A bag contains some marbles. When the
marbles are grouped by 2, 3, 4, 5, or 6, there is A. B. C. D.
always one marble left. Which of the following
could be the number of marbles in the bag?
A. 31 B. 41 C. 51 D. 61 927. Which of the following is a diagonal matrix?
1 6 9 0 2 4
A. [ 2 2 6] B. [1 0 5]
919. Amitaf, Bernard, and Chloe share a total of 3 4 3 2 3 0
P2,460 in the ratio 3:5:4 respectively. How much 4 0 0 3 2 3
is Amitaf’s share? C. [0 5 0 ] D. [2 3 6]
A. P600 B. P615 0 0 6 1 4 3
C. P630 D. P660

928. Which of the following is a scalar matrix?


920. Fifty-four kilometers per hour is equal to 5 7 9 0 2 3
how many meters per second? A. [ 2 8 6 ] B. [ 4 0 7]
A. 15 B. 30 C. 54 D. 60 3 4 3 3 8 0
7 0 0 7 0 0
C. [0 5 0] D. [0 7 0]
921. Simplify . 0 0 6 0 0 7
A. 1 B. C. D. 2

This is a free reviewer. All rights reserved.


1000 MMR Author: Victor A. Tondo Jr., LPT
929. Which of the following is an identity matrix? 936. Which of the following graphs have a
1 0 0 0 5 3 positive leading coefficient and an even degree?
A. [0 1 0] B. [ 4 0 9]
0 0 1 7 10 0
6 1 1 0 1 1
C. [1 12 1 ] D. [1 0 1]
1 1 18 1 1 0
A.

930. Which of the following is the transposition


1 4 3
of A = * +?
5 10 15
5 10 15 4 5 15 B.
A. AT = * + B. AT = * +
1 4 3 10 1 3
1 5 1 4 3
C. AT = [4 10] B. AT = [5 10 15]
3 15 1 1 1

931. When dealing with matrices A, B, and C,


which of the following is always true?
A. AB = BA
B. If AB = 0, then A = 0 or B = 0 C.
C. If AB = AC, then B = C
D. None of the above.

932. If the numbers x-1, x+2 and 2x+4 are


consecutive terms of a geometric sequence, what
is x?
A. 1 B. 2 C. 4 D. 8
D.
933. What are the zeros of 12x2 – x – 35?
A. and B. and
C. and D. and

934. What is the sum of the interior angles of a


regular dodecagon?
A. 1620o B. 1800o
C. 1980o D. 2160o

935. 45x – 20 = _____.


A. 45x – 20 B. 5(45x – 20)
C. 20(4 5x – 20) D. (5x – 20)(45x – 20)

This is a free reviewer. All rights reserved.


1000 MMR Author: Victor A. Tondo Jr., LPT
937. Which of the following graphs have a 939. What can be said about the equation f(x) of
negative leading coefficient and an odd degree? the following graph?

A.
A. The leading coefficient is positive.
B. The constant is positive.
C. The degree is 4.
D. The function is logarithmic.

B.
+1 4
940. If ( ) {2 + 1 = 4 , find lim ( ).
5 3 4
A. Limit does not exist. B. 9
C. 13 D. 17

2 +1 3
941. If ( ) { 7 = 3 , find lim ( ).
C.
3 3
A. 3 B. 6
C. 7 D. Limit does not exist.

942. Give the domain of f(x) = √8 + 24.


A. x | x B. x -3
C. x > 24 D. x -3

D.
943. What is the 3rd term in the expansion of
(A + B)5?
938. Given f(x) = (x + 2)(x – 1)(2x – 3)(x – 5), A. 5A4B B. 10A3B2
where can we find f(3)? C. 10A B
2 3 C. 5A3B2
A. On the x-axis.
B. Above the x-axis.
C. Below the x-axis. 944. Find the remainder when
D. On the intersection of the x- and y-axis. 2x4 – 5x3 + 3x2 – 5x + 7 is divided by (x – 1).
A. 1 B. 2 C. 4 D. 7

945. Give the range of f(x) =


A. y | y B. y
C. y D. y 0

This is a free reviewer. All rights reserved.


1000 MMR Author: Victor A. Tondo Jr., LPT
946. In the arithmetic sequence -2, 1, 4, 7, …, 954. The freezing temperature of water is 0o
which term is 100? Celsius. What level of data is temperature in
A. 34th B. 35th C. 36th D. 37th Celsius?
A. nominal B. ordinal
C. interval D. ratio
947. Find the length of the intercepted arc of a
central angle measuring 45o given the radius is
80 cm. 955. Factorize: x3 + 5x2 – 9x – 45
A. 20 π cm B. 30 π cm A. (x + 1)(x + 9)(x – 5)
C. 40 π cm D. 50 π cm B. (x + 1)(x – 9)(x + 5)
C. (x + 3)(x + 3)(x – 5)
D. (x + 3)(x – 3)(x + 5)
948. If f(x) = 5x2 – 4x + 9, find the average rate
of change from x = 1 to x = 4.
A. 20 B. 21 C. 22 D. 23 956. Which of these is equal to A-1 + B-1?
A. B. +
( )

949. If f(x) = 4x2 + x – 5, find the instantaneous C. D.


rate of change at x = 5.
A. 39 B. 40 C. 41 D. 42
957. Find the equation of the line with slope 3,
passing through (2, 5).
950. How is 67500 written in scientific notation?
A. y = 3x – 2 B. y = 3x – 1
A. 67.5 x 103 B. 6.75 x 103
C. y = 3x + 1 D. y = 3x + 2
C. 67.5 x 104 D. 6.75 x 104

958. Find the distance of the point (7, 8) from


951. Find the mean of the following data:
the line 2x – 3y + 4 = 0
24 24 25 26 28 29 √ √
A. B.
A. 24 B. 25.5 C. 26 D. 29
C. 4√73 D. 5√13

952. Rayon’s Social Security System (SSS) ID


number is 02-2525255-2. What level of data is 959. Find the distance between the parallel lines
the SSS ID? y = 2x – 1 and y = 2x + 4.
A. nominal B. ordinal A. 5 B. 2√5 C. √5 D. ½
C. interval D. ratio

960. There are 80 cows and ducks in a farm, all


953. A tree is 4 meters tall, while the flagpole is 8 of which are healthy. If there are 190 legs in
meters tall. What level of data is used? total, how many cows are there?
A. nominal B. ordinal A. 5 B. 10 C. 15 D. 20
C. interval D. ratio

This is a free reviewer. All rights reserved.


1000 MMR Author: Victor A. Tondo Jr., LPT
961. Which of the following angles in standard 970. The distance D of a
position is coterminal with 150o? projectile from the
A. 2570o B. 5490o ground t seconds
C. 7830o D. 9870o after launch is given as
D = 8t – t2. How
many seconds
962. Which of the following is false? after launch does
A. sin x = B. tan x = the projectile hit
the ground?
C. cos x = D. csc x = A. 4 B. 6 C. 7 D. 8

963. V and R form a vertical pair. If m V = 7x 971. The distance D of a projectile from the
and m R = 3x + 60, find m V. ground t seconds after launch is given as
A. 105o B. 84o C. 20o D. 15o D = 14t – t2. How many seconds after launch
does it attain its peak?
A. 4 B. 6 C. 7 D. 8
964. V and R form a linear pair. If m V = 13x
and m R = 5x + 72, find m R.
A. 112o B. 102o C. 78o D. 68o 972. The distance D of a projectile (in meters)
from the ground t seconds after launch is given
as D = 16t – 2t2. What is its maximum height?
965. Evaluate: tan 45o – sin 60o + cos 30o A. 16 m B. 24 m
A. -1 B. 0 C. 1 D. 2 C. 32 m D. 40 m

966. Evaluate: lim 973. Find the slope of the line tangent to the
graph of f(x) = x3 – 3x2 + 5x – 1 at x = 1.
A. LDNE B. undefined C. 0 D. 27
A. -1 B. 0 C. 1 D. 2

967. Which of the following circles is concentric


974. Which of the following is a diagonal matrix?
with x2 + y2 – 7x + 9y = 25?
9 0 0 0 1 4
A. (x – 3)2 + (y + 5)2 = 25 A. [ 0 3 0] B. [ 1 0 7]
B. (x – 7)2 + (y + 9)2 = 50 0 0 6 2 3 0
C. (x + 3.5)2 + (y – 4.5)2 = 100
D. (x – 3.5)2 + (y + 4.5)2 = 125 7 6 9 4 2 9
C. [4 3 8] D. [ 1 8 7]
5 4 5 2 4 6
968. Find k such that (x + 2) is a factor of the
polynomial 5x3 + 11x2 – kx + 12.
975. Which of the following is a scalar matrix?
A. 11 B. 7 C. -3 D. -8 5 1 2 5 0 0
A. [ 4 9 10 ] B. [ 0 5 0]
8 5 3 0 0 5
969. If P = 4S and A = S2, express A in terms of P.
7 1 1 0 2 5
A. A = B. A = C. [1 5 1] D. [ 9 0 1]
C. A = 16P2 D. A = 4P2 1 1 6 1 8 0

This is a free reviewer. All rights reserved.


1000 MMR Author: Victor A. Tondo Jr., LPT
976. Which of the following is an identity matrix? 983. A whole number is 3 more than another
4 1 1 0 6 3 number. The sum of their squares is 4145. Find
A. [1 8 1 ] B. [ 4 0 2 ] the larger number.
1 1 12 7 9 0 A. 47 B. 46 C. 45 D. 44
1 0 0 0 1 1
B. [0 1 0] D. [1 0 1]
0 0 1 1 1 0 984. The numbers x, y, z, and w have an average
of 30. If x, y and z have an average 35, what is w?
A. 5 B. 10 C. 15 D. 20
977. Find the quotient when the polynomial
7x5 – 3x3 + 2x2 – 9x + 3 is divided by x – 1.
A. 7x4 + 7x3 + 4x2 + 6x – 3 985. A is a constant. Find A such that the
B. 7x4 + 7x3 + 4x2 – 6x – 3 equation 2x + 1 = 2A + 3(x + A) has a solution
C. 7x4 + 7x3 – 4x2 – 6x – 3 at x = 2.
D. 7x4 – 7x3 – 4x2 – 6x – 3 A. -0.4 B. -0.2 C. 0 D. 0.2

978. Find k such that 9x2 + 36x + k = 0 has only 986. It takes 4 men 9 days to build 2 houses. How
one unique root. many days will it take 6 men to build 5 houses?
A. 4 B. 16 C. 18 D. 36 A. 12 B. 13 C. 14 D. 15

979. There are 210 candies in a jar. The ratio of 987. If 860 = 32x, what is x?
red candies to blue is 2:3, and the ratio of blue to A. 18 B. 24 C. 20 D. 36
yellow is 4:5. How many yellow candies are
there?
A. 35 B. 45 C. 60 D. 90 988. In a certain university, a student’s grade is
computed as the sum of 25% of his prelims
grade, 30% of his midterms grade, and 45% of
980. A certain bacteria doubles its population his finals grade. He knows that his prelims grade
after 3 minutes. If the bacteria in a petri dish is is 72 and his midterms grade is 70. What is his
512,000 at 8:45 AM, at what time was its grade for the finals if he got a grade of 75 in
population count 125? Calculus?
A. 6:30 AM B. 7:15 AM A. 75 B. 77 C. 79 D. 80
C. 8:09 AM D. 9:21 AM

989. Mr. Lazada bought 75 pieces of Elunium for


981. A square picture was framed and given 3 cm 8,000 per piece and sold them for a total of
margins. If the total area of the margin is 324 720,000. What is his mark-up rate?
cm2, what is the area of the picture? A. 20% B. 25%
A. 441 cm2 B. 576 cm2 C. 32.5% D. 40%
C. 729 cm 2 D. 900 cm2

990. Triangular numbers are numbers that can


982. Given f(x) = x2 + 10x + 25 and be shown by triangular arrangements of dots.
g(x) = 3x + 1, find f(g(x)). The triangular numbers are 1, 3, 6, 10, 15, 21, …
A. 3x2 + 30x + 26 B. 9x2 + 36x + 36 What is the 20th triangular number?
C. 9x + 30x + 26
2 D. 3x2 + 36x + 36 A. 200 B. 205 C. 210 D. 220

This is a free reviewer. All rights reserved.


1000 MMR Author: Victor A. Tondo Jr., LPT
991. What is a pattern of shapes that covers a 999. A certain Math challenge gives the
surface completely without overlaps or gaps? competitors a score of 4 for each correct answer,
A. translation B. tessellation and a deduction of 1 point for each wrong
C. constellation D. transposition answer. If a contestant answered all 100 items
and got a score of 200, how many items did the
contestant answer correctly?
992. Find the equation of the line perpendicular A. 45 B. 50 C. 60 D. 35
to 5x + 3y = 12, passing through (3, -7).
A. 3x – 5y = -44 B. 3x + 5y = 44
C. 3x – 5y = 44 D. 3x + 5y = -44 1000. What is 111011012 in decimal?
A. 257 B. 237
C. 217 D. 197
993. Point V is two-fifths the way from A(9, -7)
to B(-6, 13). Find the coordinates of point V.
A. (2, 2) B. (2, 0)
C. (3, 1) D. (3, 2)

994. Find the angle of inclination of the line


passing through V(5, -5) and T(2, 4)
A. 88.145o B. 92.429o
C. 97.593o D. 108.435o

995. The first term of an arithmetic sequence is 3


and the 20th term is 98. What is the 25th term?
A. 108 B. 115 C. 118 D. 123

End of 1000MMR.
996. An educational psychologist classifies
students as high, medium and low intelligence.
What kind of scale is being used?
A. nominal scale B. ordinal scale
C. interval scale D. ratio scale

997. The GCF of two numbers is 8 and their LCM


is 80. What is their product? Congratulations!!
A. 80 B. 160 D. 320 D. 640

998. If c1.5 – 4 = 7, what is c3?


A. 784 B. 121 C. 49 D. 16

This is a free reviewer. All rights reserved.


1000 MMR Author: Victor A. Tondo Jr., LPT

Solutions
and
Explanations

This is a free reviewer. All rights reserved.


1000 MMR Author: Victor A. Tondo Jr., LPT
1. How many line segments can be made from 30 5. Which of the following is ALWAYS true?
non-collinear points? A. Vertical pairs of angles are supplementary.
A. 900 B. 870 B. Vertical pairs of angles are complementary.
C. 450 D. 435 C. Linear pairs of angles are congruent.
D. Linear pairs of angles are supplementary.
Solution:
( ) Explanation:
30C2 = 435; or = 435
................................................ Linear pairs are supplementary, while vertical
pairs are congruent.
2. Calculate the mean absolute deviation of the ................................................
following numbers: 60, 80, 100, 75 and 95
A. 12.4 B. 14.2 6. The average of 5 different counting numbers is
C. 16.1 D. 18.9 20. What is the highest possible value that one of
the numbers can have?
Solution: A. 20 B. 40 C. 30 D. 90
Mean = (60 + 80 + 100 + 75 + 95)/5 = 82
Solution:
Mean absolute deviation daw, ibig sabihin, mean
The 5 different counting numbers will assume
or average ng absolute value ng x- ̅ .
the values of 1, 2, 3, 4, and N. Since the average is
MAD = (|60-82| + |80 – 82| + | 100 – 82| + 20, the sum is 5(20) or 100.
|75 – 82| + |95 – 82|) / 5 = 62/5 = 12.4
1+2+3+4+N = 100
................................................
10 + N = 100
N = 90
3. Which of the following is the factorization of
................................................
the binomial x2 - 42?
A. (x + 4)(x + 2)
7. Three brothers inherited a cash amount of
B. (x – 4)2
P62,000 and they divided it among themselves in
C. x(x + 2x + 2)
the ratio of 5:4:1. How much more is the largest
D. (x – 4)(x + 4)
share than the smallest share?
A. P75,000 B. P30,000
Explanation:
C. P24,800 D. P37,200
The factors of the difference of two squares are
the sum and difference of their roots. Solution:
................................................
Let the three numbers 5x, 4x, and x so that the
ratio will still be 5:4:1.
4. What value of x will satisfy the equation:
0.4(5x – 1470) = x? 5x + 4x + x = 62000
A. 490 B. 2,130 10x = 62000;
C. 1470 D. 588 x = 6200
Difference: 5x – x = 4x; 4x = 4(6200) = 24,800
Solution:
................................................
0.4(5x - 1470) = x
2x – 588 = x 8. What are the missing terms in the series
2x – x = 588 5, 20, 80, ___,1280, ___, 20480?
x = 588 A. 50; 210 B. 40; 160
C. 35; 135 D. 320; 5120

This is a free reviewer. All rights reserved.


1000 MMR Author: Victor A. Tondo Jr., LPT
Solution: 12. The vertex angle of an isosceles triangle is
20°. What is the measure of one of the base
Since the common ratio is 4, then next terms
angles?
should be 80(4) and 1280(4), or 320 and 5120.
A. 150° B. 60° C. 75° D. 80°
................................................
Solution:
9. At what rate per annum should P2400 be
invested so that it will earn an interest of P800 in (180-20)/2 = 160/2 = 80
8 years? ................................................
A. 6 ½ % B. 5 ½ %
C. 4.17 % D. 6 % 13. Ana and Beth do a job together in three
hours. Working alone, Ana does the job in 5
Solution: hours. How long will it take Beth to do the job
alone?
i=PRT
A. 3 and 1/3 hours B. 2 and 1/3 hours
800 = 2400 x R x 8
C. 3 hours D. 7 and 1/2 hours
800 = 19200 R
0.0416666 = R
Solution:
................................................
Time to finish a job by working together =
10. The area of a rectangle is (x2 + 2x - 8). If its
length is x + 4, what is its width? =3 where A = 5
A. x + 2 B. x - 2 =3
C. x + 1 D. x + 6 5B = 15 + 3B
2B = 15
Explanation: B = 7.5
Just factorize. ................................................
................................................
14. How much greater is the sum of the first 100
11. What is the value of 12⅙ - 3 ⅜ - 5 ⅔ + 20 ¾? counting numbers than the sum of the first 50
A. 21 B. 22 C. 23 D. 21 counting numbers?
A. 110 B. 3,775
C. 3,155 D. 1200
Solution:
LCD = 24 Solution:

12 –3 –5 + 20 Sum of the first N counting numbers =


Sum of the first 100 counting numbers:
= 12 + 20 –3 –5 (1002 + 100)/2 = 5050
= 32 –8 Sum of the first 50 counting numbers:
(502 + 50)/2 = 1275
= 24 or 24 5050 – 1275= 3775
................................................
=23

................................................ 15. Which of the following has the largest value?


A. 85 B. 39 C. 65 D. 94

This is a free reviewer. All rights reserved.


1000 MMR Author: Victor A. Tondo Jr., LPT
Explanation: (just use your calculator) 19. Ruben’s grades in 6 subjects are 88, 90, 97,
85 = 32,768 39 =19,683 90, 91 and 86. What is the grade that he should
6 = 7,776
5 94 = 6,561 aim for in the 7th subject if he has to have an
................................................ average of 91?
A. 97 B. 95 C. 92 D. 89
16. A water tank contains 18 liters when it is
20% full. How many liters does it contain when Solution:
50% full?
91(7) – (88+90+97+90+91+86) = N
A. 60 B. 30 C. 58 D. 45
637 – 542 = 95
................................................
Solution:
18:20 = n : 50 20. On a certain day, three computer technicians
18(50) = 20n took turns in manning a 24-hour internet shop.
900 = 20n The number of hours Cesar, Bert, and Danny
n = 45 were on duty was in the ratio 3:4:5, respectively.
................................................ The shop owner pays them P50 per hour. How
much would Danny receive for that day?
17. The edges of a rectangular solid have these A. P 230 B. P500
measures: 1.5 feet by 1½ feet by 3 inches. What C. P160 D. P480
is its volume in cubic inches?
A. 324 B. 225 C. 972 D. 27 Solution:
Let their respective times be 3x, 4x, and 5x for a
Solution:
total of 24 hours.
Convert the side measures from feet to inches 3x + 4x + 5x = 24
before proceeding with multiplication 12x = 24
1.5 ft = 1.5(12) or 18 in x=2
Vol = 18 (18) (3) = 972 .: Danny works for 10 hours at P50/hr, or P500
for that day.
................................................
................................................
18. In a certain school, the ratio of boys to girls is
21. A retailer buys candies for P90.25. The pack
5 is to 7. If there are 180 boys and girls in the
has 35 pieces of candies. If she sells each candy
school, how many boys are there?
for P3.25, how much profit does she make?
A. 105 B. 90
A. P11.50 B. P23.50
C. 45 D. 75
C. P37.50 D. P18.75
Solution:
Solution:
Let 5x = boys
Profit = 35(3.25) – 90.25 = 113.75 – 90.25
7x = girls
Profit = 23.50
5x + 7x = 180 ................................................
12x = 180
x = 15 22. Determine the midpoint of the line segment
joining the points (7, -3) and (-1, 6).
5x = 5(15) = 75
A. (2, 3/2) B. (2, -3/2)
................................................
C. (3, 3/2) D. (1, 5/2)

This is a free reviewer. All rights reserved.


1000 MMR Author: Victor A. Tondo Jr., LPT
Solution: 26. Which of these has the longest perimeter?
x = (7+ -1)/2 = 3 A. A square 21 cm on a side
y = (6 + -3)/2 = 3/2 B. A rectangle 19 cm long and 24 cm wide
................................................ C. An equilateral triangle whose side is 28 cm
D. A right triangle whose two legs are 24 and 32
23. One side of a 45° - 45° - 90° triangle cm
measures x cm. What is the length of its
hypotenuse? Solution:
A. x √3 cm B. x cm
A. P = 4S; 4(21) = 84
C. (x √3)/2 cm D. x √2 cm
B. P = 2(L+W) 2(24+19) = 86
C. P = 3S 3(28) = 84
Explanation:
D. P = L1 + L2 + H 24 + 32 + 40 = 96
In a 45-45-90 triangle, the hypotenuse is √2 ................................................
times of the leg.
................................................ 27. How many square inches are in 2 square
yards?
24. The legs of one right triangle are 9 and 12, A. 900 B. 144
while those of another right triangle are 12 and C. 1296 D. 2,592
16. How much longer is the perimeter of the
larger triangle than the perimeter of the smaller Solution:
triangle? 1 yard = 3 feet = 3(12) or 36 inches
A. 84 B. 7 C. 12 D. 14 1 square yard = 362 or 1296 square inches
.: 2 square yards = 2(1296) = 2592 sq in
Solution: ................................................
Solve for the hypotenuse of the two triangles.
28. In a playground for Kindergarten kids, 18
The first one will have 15, while the other will
children are riding tricycles or bicycles. If there
have 20.
are 43 wheels in all, how many tricycles are
Get their respective perimeters. The first triangle
there?
has a perimeter of 9+12+15 or 36. The other
A. 8 B. 9 C. 7 D. 11
triangle’s perimeter is 12+16+20 or 48.
48 – 36 = 12
Solution:
................................................
T + B = 18  2T + 2B = 36
25. An online shop sells a certain calculator for 3T + 2B = 43  3T + 2B = 43
P950 and charges P150 for shipping within T=7
Manila, regardless of the number of calculators ................................................
ordered. Which of the following equations shows
the total cost (y) of an order as a function of the 29. Aira takes ¾ hour to dress and get ready for
number of calculators ordered (x)? school. It takes 4/5 hour to reach the school. If
A. y = (950 + 150)x B. y = 150x +950 her class starts promptly at 8:00 am; what is the
C. x = 950y + 150 D. y = 950x + 150 latest time she can jump out of bed in order not
to be late for school?
Explanation: A. 6:42 am B. 6:27 am
C. 6:57 am D. 7:02 am
The cost of each calculator is P950, so x
calculators cost P950x. Add the constant
shipping cost which is P150 and that’s D.

This is a free reviewer. All rights reserved.


1000 MMR Author: Victor A. Tondo Jr., LPT
Solution: Then, solve algebraically.
¾ hr = 45 mins, while 4/5 hr = 48 mins (3x – 4) + (x – 4) = 36
45+48 = 93 mins, 93 mins = 1 hr 33 mins 4x – 8 = 36
8:00 7:60 4x = 44; x = 11;
- 1:33  1:33 3x = 3(11) = 33
6:27 ................................................
.: 1hr 33 mins before 8:00 AM is 6:27 AM
................................................ 33. What is the least common multiple of 12, 24
and 72?
30. Which common fraction is equivalent to A. 12 B. 72 C. 144 D. 36
0.215?
A. 43/200 B. 27/125 Explanation:
C. 21/50 D. 108/375
Use continuous division.
12 24 72
Explanation:
6 2 4 12
0.215 is read as 215 thousandths. In fraction 2 1 2 6
form, that’s . In simplest form, . 2 1 1 3
6 x 2 x 2 x 1 x 1 x 3 = 72
Alternative Method: ................................................
Just use your calculator.
34. The hypotenuse of a right triangle is 25 feet.
................................................
If one leg is 24 feet, what is the length of the
other leg?
31. What are the next three terms in the
A. 6 ft. B. 5 ft. C. 20 ft. D. 7 ft.
progression 1, 4, 16 …?
A. 64, 256, 1024 B. 67, 259, 1027
Explanation:
C. 48, 198, 1026 D. 65, 257, 1025
Use the Pythagorean Theorem.
Explanation: ................................................
Each term is 4 times its precedent.
35. If two variables X and Y are directly related,
................................................
which of these is NOT true?
A. When X is low, Y is also low.
32. A man is 3 times as old as his son now. Four
B. As X increases, Y also increases.
years ago, the sum of their ages was 36. Find the
C. When X increases, Y decreases.
man’s age now.
D. A high Y is associated with a high X.
A. 33 B. 11
C. 29 D. 36
Explanation:
Solution: C refers to an inverse or indirect relation.
................................................
First, create a table.
Age Now Age 4 Yrs Ago
Man 3x 3x-4 36. Find the domain of f(x) = .
Son x x-4 A. x B. x = 1
C. x = -1 D. x ,x -1

This is a free reviewer. All rights reserved.


1000 MMR Author: Victor A. Tondo Jr., LPT
Explanation: 40. Factorize (x4 – 81) completely.
A. (x-3)4
The given function is a rational algebraic
B. (x – 3)2 (x + 3)2
expression (RAE). When facing RAE, just look at
C. (x+3) (x-3) (x2+9)
the denominator and see if it can be equated to 0
D. (x+3)3 (x-3)
to make the RAE undefined.
The RAE will have an undefined value at
Solution:
x = -1. Otherwise, it will always be equal to a real
number. (x4 – 81) = (x2 – 9) (x2 + 9)
................................................ (x4 – 81) = (x+3) (x-3) (x2 + 9)
................................................
37. A car travels D km in H hours. Which of the
following expressions shows the distance 41. √8 + √18 √2 = ____
travelled by the car after M minutes? A. 4√2 B. 5√2 C. √24 D. 2√6
A. MD/H B. 60MD/H
C. MD/60H D. 60HD/M Solution:

Solution: √8 + √18 √2 = 2√2 + 3√2 √2 = 4√2


................................................
Distance = Speed x Time (the unit of time should
be consistent) 42. By which property can we state the
The car is traveling at a speed of D/H km per hr. following:
The time is M minutes or M/60 hrs (for “If ax + b = c, then ax + b - b = c – b.”
consistency). A. transposition B. transitive
Distance = (D/H) (M/60) = MD/60H C. additive inverse D. addition property
................................................
Explanation:
38. Find the surface area of a rectangular box
whose dimensions are 30 cm x 40 cm x 50 cm. We added –b to both sides of the equation, thus
A. 4700 cm2 B. 7050 cm2 we used APE (addition property of equality).
C. 9400 cm 2 D. 11750 cm2 ................................................

Solution: 43. The midpoint of P and (-7, 4) is (-3, 1). What


are the coordinates of P?
SA = 2 (LW + WH + LH) A. (-5, 5/2) B. (-11, 7)
SA = 2 (50x40 + 40x30 + 30x50) C. (1, -2) D. (-2, 3/2)
SA = 2 (2000 + 1200 + 1500) = 9400
................................................ Solution:
39. If x – y = 3, then (y-x)-3 = ___. Let P be at (x,y). By Midpoint formula:
A. 9 B. -9 (-7 + x)/2 = -3 (4 +y)/2 = 1
C. 1/27 D. -1/27 -7 + x = -6 4+y=2
x = -6 + 7 y=2–4
Solution: x=1 y = -2
................................................
Since x – y = 3, then y – x = -3.
(-3)-3 = 1/(-3)3 = 1/-27 or -1/27 44. What is the slope of the line 3x – y = 11?
................................................ A. -1/3 B. 1/3 C. -3 D. 3

This is a free reviewer. All rights reserved.


1000 MMR Author: Victor A. Tondo Jr., LPT
Solution: 48. A and B form a vertical pair. If m A = 3x
and m B = 5x – 44, what is the value of x?
Isolate y on one side of the equation to rewrite
A. 50.5 B. 28 C. 22 D. 16.75
the equation in the form y = mx + b.
3x – y = 11
Solution:
-y = -3x + 11
y = 3x – 11 Since the two angles form a vertical pair, then
................................................ they are congruent.
3x = 5x – 44
45. What is the minimum value of 44 = 5x – 3x
f(x) = 3x2 + 6x + 7? 44 = 2x; 22 = x
A. 1 B. -1 C. 4 D. -4 ................................................

Solution: 49. The angle of elevation from an observer to


the top of a building is 30o. If the building is 50
Min Value = c – b2/4a
meters high, how far is the observer from the
That’s 7 – 36/12 or 7-3=4
building?
................................................
A. 25 B. 25√3 C. 50√3 D. 100
46. If xy = 23 and x2 + y2 = 75, find x + y.
A. 10.7845 B. 11 Solution:
C. 11.2155 D. 11.7845 Use a 30-60-90 triangle. The side opposite of the
30o angle will represent the building.
Solution: ................................................
x2 + 2xy +y2 = x2 + y2 + 2xy
x2 + 2xy +y2 = 75 + 2(23) 50. 1 and 3 are opposite angles in a
x2 + 2xy +y2 = 121 parallelogram. If m 1 = 40o, what is m 3?
x + y = 11 A. 40o B. 50o C. 70o D. 140o
................................................
Explanation:
47. How much water must be evaporated from Opposite angles of a parallelogram are
90 ml of a 50% salt solution to increase its congruent.
concentration to 75%? ................................................
A. 40 ml B. 38 ml
C. 35 ml D. 30 ml 51. Two parallel lines are cut by a transversal,
forming H and K. If the two angles are
Solution: exterior angles on the same side of the
V1 C1 + V2 C2 = VR CR transversal, what is the measure of H if the
Since we are evaporating water, we will be measure of K is 50o?
adding a NEGATIVE volume of water (or simply A. 25o B. 50o
put, we are subtracting water, diba?) C. 100 o D. 130o
90(50) + (-X)(0) = (90-X)(75)
4500 + 0 = 6750 – 75X Explanation:
75X = 6750 – 4500 Exterior angles on the same side of the
75X = 2250; X = 30 transversal are supplementary.
................................................ *Mnemonic: SST (same side of transversal)
means supplementary. ALTERNATE (either
interior or exterior) means congruent. Also,
CORRESPONDING angles are congruent.
This is a free reviewer. All rights reserved.
1000 MMR Author: Victor A. Tondo Jr., LPT
52. There are 33 red bags, 25 green bags, and 17 The nearest multiple of 7 to 125 is 126. That
blue bags in a store. What percent of the bags is means 126 days after today is Saturday again,
red? and 125 days after today should be Friday.
A. 33% B. 44% ................................................
C. 66% D. 67%
56. Car A is traveling towards the east at a speed
Solution: of 35 kph, while car B is traveling towards the
west at 45 kph. If they left the same point at 1:00
33/(33+25+17) = 33/75 or 11/25
PM, how far apart are they at 3:45 PM?
11/25 in percent is 44%
A. 240 km B. 220 km
................................................
C. 200 km D. 180 km
53. Given sin θ = 0.28, which of the following
Solution:
could possibly be cos θ?
A. 0.72 B. -0.86 Time spent driving: 1:00 to 3:45 = 2.75 hrs
C. 0.96 D. 1.14 (45 mins in decimals is 45/60 since there are 60
mins in 1 hr)
Solution: Car A distance from mid: 2.75 (35) = 96.25
Car B distance from mid: 2.75 (45) = 123.75
sin2 θ + cos2 θ = 1
Total distance: 123.75 + 96.25 = 220 km
(0.28)2 + cos2 θ = 1
cos2 θ = 1 – 0.0784
Alternative Solution:
cos2 θ = 0.9216
cos θ = √0.9216 = ±0.96 Since the two cars are traveling in two opposite
................................................ directions, add their speeds and multiply by
elapsed time.
54. If the sum of the supplement and the 2.75 (45+35) = 2.75 (80) = 220 km
complement of an angle is 130 degrees, what is ................................................
the angle?
A. 65o B. 70o 57. Mr. Santos left the house at 1:00 PM and
C. 50o D. 25o traveled east at an average speed of 40 kph. His
wife Mrs. Santos left the at 2:00 PM and traveled
Solution: west at an average speed of 30 kph. How far
apart are they at 4:00 PM?
(90-x) + (180-x) = 130 A. 180 km B. 140 km
270 – 2x = 130 C. 100 km D. 60 km
270 – 130 = 2x
140 = 2x Solution:
70 = x
................................................ Mr. Santos’s data:
Speed: 40 kph
55. If today is a Saturday, what day is 125 days Elapsed time: 1PM to 4PM = 3 hrs
from now? Distance: 40kph (3hrs) = 120 km
A. Thursday B. Friday
C. Sunday D. Monday Mrs. Santos’s data:
Speed: 30 kph
Solution: Elapsed time: 2PM to 4PM = 2 hrs
Distance: 30 kph (2hrs) = 60 km
Every 7 days, it would be a Saturday again.
Total Distance: 60 + 120 = 180 km
This is a free reviewer. All rights reserved.
1000 MMR Author: Victor A. Tondo Jr., LPT
58. Five consecutive even numbers have a sum of 61. Given f(x) = ln , what is f ‘(x)?
120. What is the sum of the 2nd and 5th even
A. B.
numbers?
A. 46 B. 48 C. 50 D. 52 C. (2x+2) ln (x2+2x) D. 2x + 2

Solution: Solution:
You can rewrite ln as x2 + 2x since ln is
Let x = lowest even number the natural logarithm (the logarithm whose base
x + (x+2) + (x+4) + (x+6) + (x+8) = 120 is the natural number, e).
5x + 20 = 120
5x = 100 Remember: ln eu = u, wherein u is the exponent
x = 20 to which e is being raised.
.: numbers are 20, 22, 24, 26, 28 The derivative of x2 + 2x is, of course, 2x + 2.
................................................
22 + 28 = 50
62. Which of the following could be the value of x
Alternative Solution: if x 3(mod 11)?
The middle (3rd) even number is 120/5 or 24. A. 33 B. 47 C. 52 D. 2
That means the 2nd even number is 24 – 2 or 22,
and the 5th is 24 + 2(2) or 28. Solution:
................................................ Just divide the numbers by 11 and see which one
gives a remainder of 3.
59. If x = 3, which of the following is equal to 13? ................................................
A. 5x + 2 B. x2 + 2x + 1
C. x3 – 4x – 2 D. x2 + x + 2
63. If = 6x2 + 8x – 7, which could be u?
Explanation: A. 12x + 8 B. 3x3 + 4x2 – 7x + 11
C. 2x3 + 4x2 -7x +1 D. 12x2 + 8x - 10
Just substitute x with 3.
................................................ Explanation:
60. If f(x) = x2 + 4x + 3, which of the following is Anti-derivatives. If you already forgot how to do
equal to 99? that, simply check which choice has a derivative
A. f(11) B. f(-12) of 6x2 + 8x – 7.
C. f(12) D. f(-8) ................................................

64. What is the center of x2 + y2 – 8x + 6y = 0?


Solution: A. (-8.6) B. (8, -6)
x2 + 4x + 3 = 99 C. (-4, 3) D. (4, -3)
x2 + 4x + 3 + 1 = 99 + 1
x2 + 4x + 4 = 100 Solution:
√( + 2) = 100 The center, C(h,k) is given as h = -D/2 and
x + 2 = ± 10 k = -E/2 wherein D and E are from the equation
x = -2 ± 10 x2 + y2 + Dx + Ey + F = 0.
That’s -2+10 or 8, and -2-10 or -12

This is a free reviewer. All rights reserved.


1000 MMR Author: Victor A. Tondo Jr., LPT
65. Which of the following is a parabola that Explanation:
opens to the right?
Since A and B are the roots, then AB pertains to
A. 6y = (x+9)2 - 8 B. -4y = (x-6)2 + 3
the product of the roots which is given as c/a.
C. -5x + 3 = (y-2)2 D. 2x + 6 = (y+3)2
................................................
Explanation:
69. 1 + 2 + 4 + 8 + … + 2048 = ____
When x is the squared variable, the parabola A. 4095 B. 4096
opens upward when the coefficient of y is C. 4097 D. 4098
positive (example: A).
When x is the squared variable, the parabola Solution:
opens downward when the coefficient of y is
You may use the Geometric Series formula which
negative (example: B).
When y is the squared variable, the parabola is ∑ = ( ), where r is the common
opens to the left when the coefficient of x is ratio, n is the number of terms, and a1 is the first
negative (example: C). term.
When y is the squared variable, the parabola
opens to the right when the coefficient of x is Alternative Solution:
positive (example: D).
In this problem, however, you cannot easily use
................................................
the GS formula since you don’t know n, the
number of terms.
66. Factorize: 12x2 – 7x – 10.
I will personally use the shortcut for the sum of a
A. (6x + 5) (2x – 2) B. (6x – 2) (2x + 5)
geometric sequence wherein the ratio is 2 or ½.
C. (3x + 2) (4x – 5) D. (3x – 2) (4x + 5)
The shortcut is SUM = 2(largest) – smallest. In
................................................
this problem, that’s 2(2048)-1 = 4095.
You may also apply this in the next item, #70.
67. For which value of k does 4x2 + kx + 49 have
only one root?
70. 24 + 12 + 6 + 3 + 1.5 + … = ____
A. -28 B. -14 C. 7/2 D. -7/4
A. 48 B. 50 C. 54 D. 60
Explanation:
Solution:
You may use Completing Square Trinomials. The
You may use the Infinite Geometric Series
middle term is twice the product of the square
roots of the first and third terms. In the problem, formula which is ∑ = ( ), where r is the
the middle term is twice the product of √4 and common ratio and a1 is the first term.
√49. That’s 2(2x)(7) or 28x. Don’t forget that the
middle term could be positive or negative. Alternative Solution:
In this problem, I would still be using the
You may also use the discriminant to answer shortcut since the ratio is ½. Since this is an
this: infinite geometric sequence, then the last term
b2 – 4ac = 0 when there’s only one root, won’t have any significant value. Thus, the sum is
b2 – 4ac > 0 when there are two real roots simply twice the first term. That’s 2(24) = 48.
b2 – 4ac < 0 when there are no real roots

68. If A and B are the roots of x2 + 7x + 15, what


is AB?
A. 7√3 + 2 B. 2√3 + 7
C. 3√2 + 2√3 D. 15
This is a free reviewer. All rights reserved.
1000 MMR Author: Victor A. Tondo Jr., LPT
71. How many terms are there in the sequence 74. C is the midpoint of ̅̅̅̅
AB where A is at (-3,4)
5, 13, 21, 29, …, 357? and B is at (7,-10). Find the coordinates of C.
A. 40 B. 44 A. (5,-7) B. (-5,7) C. (2,-3) D. (-2,3)
C. 45 D. 70
Solution:
Solution:
Midpoint Formula: ( , )
An = A1 + (n – 1)d
357 = 5 + (n – 1)(8) Midpoint: ( , )
357-5 = 8(n – 1) ................................................
352 =8(n – 1)
44 = n – 1 75. It is a line segment formed by connecting two
45 = n non-consecutive vertices of a polygon.
A. side B. apothem
Alternative Solution: C. altitude D. diagonal
(this is the “y=mx+b” solution I taught my grade
3 student for Singapore. Yes, Grade 3.) Explanation:
Before anything else, since this might be “new” A side is formed by connecting two consecutive
to you, your d is our m, your An is our y, your n is vertices of a polygon.
our x, and b is your A1 – d. The apothem is only for regular polygons. It is
357 = 8x +(5 – 8) the perpendicular bisector of one of its sides,
357 = 8x – 3 passing through the center.
360 = 8x A diagonal is a line segment formed by
45 = x connecting two non-consecutive vertices of a
................................................ polygon.
................................................
72. How many ways can a group of 5 be selected
from 5 boys and 5 girls if the group must contain 76. Find the equation of the line perpendicular to
3 boys and 2 girls? 2x – 3y = 7, passing through (1,2).
A. 151,200 B. 1200 A. 2x + 3y = 8 B. 3x + 2y = 7
C. 252 D. 100 C. 2x – 3y = -4 D. 3x – 2y = -1

Solution: Solution:
A group, committee, or team (any set with no Simply interchange the numerical coefficients of
hierarchy of members) calls for Combinations. x and y in the original equation, then change the
To pick 3 boys from a total of 5 boys, use 5C3 operation between them. 2x – 3y becomes
and that’s 10. To pick 2 girls from a total of 5 3x + 2y.
girls, use 5C2 and that’s 10. Lastly, 10x10 = 100. For the constant, simply substitute the
................................................ x and y values of the point ((1 ,2) in this
problem) and solve for the constant.
73. What is the probability of getting a sum of 9 3(1) + 2(2)=7.
when rolling 2 dice? Thus, 3x+2y=7.
A. 1/9 B. 5/36
C. 1/6 D. 7/36

The only pairs with a sum of 9 are (3,6), (4,5),


(5,4), and (6,3). There are only 4 pairs out of 36.

This is a free reviewer. All rights reserved.


1000 MMR Author: Victor A. Tondo Jr., LPT
77. Two parallel lines are cut by a transversal to Solution:
form X, Y, and Z. Given that X and Y are
Usually, people would
alternate interior angles while Y and Z are
straight go for the Test
interior angles on the same side of the
Point Table method which
transversal, find m Z if m X = 40o.
we use in Calculus.
A. 40o B. 50o
However, since this is the
C. 130 o D. 140o
licensure exam, I’d prefer
that you use a simpler and
Explanation:
quicker approach to this
Alternate, corresponding, and vertical pairs problem.
automatically suggest that the two angles are First, identify the zeros of
congruent. Linear pairs and angles on the same the inequality by equating
side of transversal (SST) are supplementary. each factor to 0. Our zeros
m X = 40, .: m Y = 40 since alternate interior are -9 and 3.
angles Next, identify the opening
m Z = 180-40 = 140 since Y and Z are of the parabola. Since the
interior angles on the same side of the leading coefficient would
transversal. be positive, then the
................................................ parabola opens upwards.
Since the parabola opens upwards, then the
78. The measure of each interior angle of a parts less than 0 should be between the zeros of
regular polygon is 144o. How many vertices does the inequality. That means x should be between
it have? -9 and 3.
A. 36 B. 24 C. 12 D. 10 ................................................

Solution: 80. The product of two consecutive even


( ) counting numbers is 3248. Find the smaller
MIA = = number.
A. 42 B. 46 C. 52 D. 56
Alternative Solution:
Solution:
Personally, I always go for the exterior angle first
to get the number of sides or vertices. Since the x(x+2) = 3248
exterior and interior are supplementary, then x2 + 2x = 3248
each exterior measures 180-144 or 36. The x2 + 2x + 1 = 3249
formula for number of sides or vertices given the √ + 2 + 1 = √3249
measure of each exterior is 360÷MEA, so that’s x + 1 = 57
360÷36 or 10 vertices. x = 56
By the way, you may derive this solution by
( ) WAIS Solution:
manipulating the formula for MIA: . That
becomes 180 – . That means = 180 – MIA, Get your scientific calculator, extract √3248 and
then scrape the decimals or round down.
or = n.
#2EZ4U
................................................ ................................................
79. Solve: (x + 9) (x – 3) < 0 81. Solve for x: 2log2 3 – log2 18 = x
A. -9 < x < 3 B. x < -3 x > 9 A. ½ B. -1 C. -2 D. 1
C. x < -9 x > 3 D. x ; x -9, 3

This is a free reviewer. All rights reserved.


1000 MMR Author: Victor A. Tondo Jr., LPT
Solution: surface area. Just remember that the slant height
is always longer than the height. The slant height
Rewrite the logarithm as a single logarithm by
is the hypotenuse, while the height is one of the
applying the rules of logarithms.
legs with the radius as the other. Just use the
2log2 3 becomes log2 , or log2 ½ Pythagorean formula to solve for whichever is
log2 ½ = -1 missing.
................................................ The height is 12 cm (after using Pythagorean
formula).
82. Twinkle Bucks has four serving sizes for their Vol = π r2 h = π (92) (12) = 324 π cm2
milk tea: Small, Medium, Large, and Extra Large. ................................................
What level of data are they using for their
serving sizes? 86. If f(x) = x2 + 4x + 4 and g(x) = x-2, find
A. nominal B. ordinal f(g(x)).
C. interval D. ratio A. x2 B. x3 – 6x2 + 6x – 9
................................................ C. x2 + 8x + 16 D. x2 – 8x + 16
83. After receiving a 20% markup, a bag was sold Solution:
for P960. How much was it originally?
A. P1152 B. P4800 f(g(x)) = f(x-2)
C. P800 D. P1200 = (x-2)2 + 4(x-2) + 4
= (x2 – 4x + 4)+ (4x – 8) + 4 = x2
Solution: ................................................
Selling Price = Original Price (1 + Markup Rate) 87. A 10 ft ladder leans against a wall, forming a
960 = OP (1 + 0.20) 30o angle with it. How high on the wall does it
960/1.2 = OP reach?
800 = OP
A. 5 ft B. 5 √3 ft
................................................
C. 10 √3 ft D. 10 √6 ft
̅̅̅̅ bisects ABC and m ABT = 40o,
84. Given BT
Solution:
find m ABC.
A. 20o B. 40o C. 60o D. 80o Draw the problem first. The ladder and the wall
form a 30o angle with each other and the wall is
Explanation: of course perpendicular to the ground.
ABT is formed after the bisection of ABC. That That means the ladder forms a 60o angle with the
means ABT is half of ABC, or ABC is twice of ground. The ladder is the hypotenuse, while its
reach on the wall is adjacent to the 30o angle or
ABT.
simply put, the longer side. The smaller side
................................................
measures half of 10 or 5 ft, therefore the longer
85. A cone has a radius of 9 cm and a slant height side must be 5√3 ft.
of 15 cm. Find its volume. ................................................
A. 243 π cm3 B. 324 π cm3
C. 405 π cm 3 D. 486 π cm3 88. How many ways can a committee of 5 be
selected from 9 people?
Explanation: A. 126 B. 120
C. 3024 D. 15120
Be careful with cones. Tendency kasi sa LET that
they will give the slant height while looking for
volume and the height while looking for the
This is a free reviewer. All rights reserved.
1000 MMR Author: Victor A. Tondo Jr., LPT
Explanation: 93. How many 3-digit numbers can be formed
using the digits 0, 1, 2, 3, 4 and 5 if repetition is
Just use your scientific calculator: 9C5.
not allowed?
................................................
A. 60 B. 80 C. 100 D. 120
89. What is 60% of 80% of 500?
Solution:
A. 480 B. 240
C. 120 D. 60 Use FCP (Fundamental Counting Principle):
__ x __ x __
Solution: For the first digit, we cannot use 0. That means
we only have 5 choices for the first digit.
(0.6)(0.8)(500) = 240
For the second digit, we can now use 0. Since we
................................................
have already used one digit for the first, that
means we have 5 choices for the second digit.
90. If 3x = 7 and 2y = 5, what is 6(x-y)?
For the last digit, since we have already used two
A. -1 B. 1-√35 digits, we only have 4 choices.
C. √7 - √5 D. 5 x 5 x 4 = 100
................................................
Solution:
94. How many ml of 20% acid must be added to
6(x-y) = 6x – 6y = 2(3x) – 3(2y) 400 ml of 50% acid to make a 30% acid solution?
2(3x) – 3(2y) = 2(7) – 3(5) = 14-15 = -1
A. 1000 ml B. 900 ml
................................................
C. 800 ml D. 750 ml
91. If two numbers have a product of 71 and the Solution:
sum of their squares is 147, what is their sum?
A. -17 B. 5 C1 V1 + C2 V2 = CR VR
C. 12√3 + √5 D. 12 + √3 20 (V) + 50 (400) = 30 (V + 400)
20V + 20,000 = 30V + 12,000
Solution: 20,000 – 12,000 = 30V – 20 V
8,000 = 10V; 800 = V
Let A and B be our two numbers. ................................................
AB = 71; A2 + B2 = 147
.: A2 + B2 + 2AB = 147 + 2(71) = 289 95. How many ml each of 10% and 50% solution
(A + B)2 = 289; A + B = ±17 should be mixed to make 500 ml of 18%
................................................ solution?
A. 400 ml of 10% and 100 ml of 50%
92. Find the median: 7, 9, 11, 10, 9, 13, 17, 14 B. 350 ml of 10% and 150 ml of 50%
A. 10 and 11 B. 9 and 10 C. 300 ml of 10% and 200 ml of 50%
C. 10.5 D. 9.5 D. 200 ml of 10% and 300 ml of 50%
Solution: Solution:
Rearrange the numbers from least to greatest: Since our resultant volume is 500, then our two
7, 9, 9, 10, 11, 13, 14, 17  there are 8 nos. volumes will be x and (500-x).
The median is the th or 4.5th number. That C1 V1 + C2 V2 = CR VR
means we have to get half the sum of our 4th and 10(x) + 50(500-x) = 18(500)
5th numbers. (10+11)/2 = 10.5 10x + 25,000 – 50x = 9,000
25,000 – 9,000 = 50x – 10x
16,000 = 40x; 400 = x
This is a free reviewer. All rights reserved.
1000 MMR Author: Victor A. Tondo Jr., LPT
96. It takes 28 men a total of 24 days to build a A rectangle’s diagonals are congruent and they
house. How long would it take 32 men to build a bisect each other. However, they are not
similar house? perpendicular.
A. 28 days B. 27 days An isosceles trapezoid has congruent diagonals,
however, they do not bisect each other, nor are
C. 21 days D. 19 days they perpendicular.
A rhombus has diagonals that are perpendicular
Solution: and that bisect each other. However, they are not
This is an indirect or inverse proportion. congruent.
Let x = number of days it would take the 32 men PS: A square has diagonals that are congruent,
to build the house perpendicular, and that bisect each other.
28(24) = 32 x ................................................
672 = 32 x
21 = x 100. A pipe can fill a pool in 6 hours while
................................................ another pipe can drain empty the pool in 15
hours. How long will it take to fill the pool if both
pipes are open?
97. Evaluate: lim A. 9 hours B. 9.125 hours
A. undefined B. limit does not exist C. 9.45 hours D. 10 hours
C. 8 D. +
Solution:
Explanation:
This is similar to our “Working Together”
You may simplify the function first before problem, except instead of adding their times, we
substituting x with 4. will subtract (since the draining pipe is doing the
................................................ opposite of helping).
AB/(A-B) = 15(6)/(15-6) = 90/9 = 10 hrs
98. A box contains 7 red, 8 blue, and 9 white ................................................
balls. When taking two balls in succession, what
is the probability that both balls are white? 101. If log n – 1 = 2, find n.
A. 9/64 B. 9/69 A. 3 B. 1000 C. e3 D. 3e
C. 7/64 D. 7/69
Solution:
Solution:
log n – 1 = 2
First white ball: 9/24 log n = 2 + 1 = 3
Second white ball: 8/23 (note that the base of the log is 10)
9/24 x 8/23 = 9/69 log n = 3 translates to 103 = n
................................................ Therefore n = 1000
................................................
99. Which of the following has two diagonals that
are perpendicular bisectors of each other? 102. log2 3 + 2 log2 7 – log2 5 = ______.
A. kite B. rectangle A. log2 B. log2
C. rhombus D. isosceles trapezoid
C. log2 D. log2
Explanation:
Explanation:
A kite’s diagonals are perpendicular but only one
diagonal will bisect the other. Just apply the laws of logarithms.

This is a free reviewer. All rights reserved.


1000 MMR Author: Victor A. Tondo Jr., LPT
103. The surface areas of two spheres are 12 π 106. Find the volume of a steel cylinder of radius
cm2 and 108 π cm2. What is the ratio of their 5 cm and height 12 cm.
volumes? A. 300 π cm3 B. 250 π cm3
A. 1:3√3 B. 1:9 C. 200 π cm3 D. 100 π cm3
C. 1:27 D. 2:3√3
Solution:
Solution: Vol = π r2 h = 52 (12) π = 300 π cm3
Ratio of surface areas: 12:108 or 1:9 ................................................
Ratio of radii: √1: √9 or 1:3
Ratio of volumes: 13:33 or 1:27 107. A cube sits perfectly inside a sphere of
................................................ volume 108 √3 π cm3. Find the volume of the
cube.
104. The volume of a regular hexahedron is 64 A. 27 cm3 B. 54 cm3
in3. How long is each side? C. 108 cm 3 D. 216 cm3
A. 2 in B. 4 in C. 6 in D. 8 in
Solution:
Explanation: Volume of sphere = 108√3 π cm3
A regular hexahedron is simply a cube. π r3 = 108√3 π
................................................
r3 = (108)√3
105. Which of the following statements is r3 = 81√3; r = 3√3; d = 6√3
ALWAYS true? Diagonal of cube = s√3 = 6√3
A. The square of a prime number is odd. .: s = 6; volume = s3 = 63 = 216
B. The sum of two consecutive even numbers is
divisible by 4. Alternative Solution:
C. Any even number is composite.
Ratio of volume of cube to sphere (cube is inside
D. The product of two consecutive even numbers
is divisible by 8. sphere) = 2 : √3 π
N : 108 √3 π = 2 : √3 π
( √ )( )
Explanation: N= = 216

A. Counterexample: The prime number, 2. The ................................................
square of 2 is 4 which is even.
B. Always false. One example is 2 and 4. Their 108. Find the distance in cm of an 80 cm chord
sum, 6, is not divisible by 4. from the center of a circle whose radius is 41 cm.
C. Counterexample: The prime even number, 2. A. 41 - 2√10 B. 41 - 4√10
D. Proof by Algebra: Let the first even number be C. 9√2 D. 9
2x. The second even number will be 2x + 2.
Their product will be 4x2 + 4x. Solution:
If x is an odd number, x = 2y + 1 where y is a
counting number. 4x2 + 4x = 4(4y2 + 4y + 1) + The chord is perpendicularly bisected by a
4(2y + 1) = 16y2 + 16y + 4 + 8y + 4 = 16y2 + segment connected to the center of the circle,
24y + 8, which is divisible by 8. whose length is the distance we are looking for.
If x is even, x = 2y where y is any counting If the radius is drawn connected to one endpoint
number. 4(4y2) + 4(2y) = 16y2 + 8y, which is of the chord, we can form a right triangle whose
also divisible by 8. hypotenuse is the radius and one leg is half of the
Either ways, the statement holds true. chord. Using the Pythagorean theorem, the
distance is √41 40 or simply, 9.
This is a free reviewer. All rights reserved.
1000 MMR Author: Victor A. Tondo Jr., LPT
109. Which quadrilateral has two congruent 113. Find the intersection of y = 2x + 3 and
diagonals that bisect each other? y = 4x – 11.
A. kite B. isosceles trapezoid A. (-4/3, 0) B. (4/3, 0)
C. rectangle D. rhombus C. (7, 17) D. (-7,-17)
................................................
Solution:
110. What is the longest side of ∆MTC if m M =
y = 2x + 3
40o and m C = 60o?
- y = 4x – 11
A. ̅̅̅̅
MC B. ̅̅̅̅
TC ̅̅̅̅
C. MT D. ̅̅̅̅
CT
0 = -2x + 14
2x = 14; x=7
Explanation:
................................................
m T = 180-(40+60) = 80
The longest side is opposite the largest angle, T. 114. Find the area of the triangle whose vertices
................................................ are (1,4), (2,3), and (3,0).
A. 0 B. 1 C. 5/3 D. 3/4
111. Find the altitude to the hypotenuse of a
right triangle whose legs measure 10 cm and 24 Solution:
cm. 1 2 3 1
A. 120 cm B. cm | | | = ( 2) = 1
4 3 0 4
C. 120√2 cm D. 24√5 cm Note: If the result is negative, that means your
points are simply arranged clockwise. Just get
Solution: the absolute value of the answer.
Find the hypotenuse first. That would be 26 cm. ................................................
Altitude to the Hyp = (L1 L2)/Hyp = 24(10)/26
Altitude to the Hyp = 120/13 115. Find the tenth term: 3, 10, 17, 24, …
................................................ A. 66 B. 67 C. 68 D. 69

112. Find the inverse of y = x2 + 10x. Solution:


A. y-1 = √ 25 + 5 A10 = 3 + (10-1) (7) = 3 + 63 = 66
B. y-1 = √ 25 – 5 ................................................
C. y-1 = √ + 25 + 5
D. y-1 = √ + 25 – 5 116. Find the remainder when
x4 – 5x3 + 6x2 + 2x + 1 is divided by (x – 2).
Solution: A. 17 B. 13 C. 9 D. 5

y = x2 + 10x Solution:
y + 25 = x2 + 10x + 25
y + 25 = (x+5)2 24 – 5(23) + 6(22) + 2(2) + 1
= 16 – 40 + 24 + 4 + 1 = 5
√ + 25 = x + 5 ................................................
√ + 25 – 5 = x
√ + 25 – 5 = y-1 117. The sum of Fe’s age and Sita’s age is 60.
................................................ Twelve years ago, Fe was twice as old as Sita.
How old is Sita now?
A. 18 B. 24 C. 30 D. 36

This is a free reviewer. All rights reserved.


1000 MMR Author: Victor A. Tondo Jr., LPT
Solution: By transitive property of equality,
Age Now Age 12 yrs ago 8x = 15z, or x = z
Fe x x – 12 ................................................
Sita 60-x (60 – x) – 12 or 48 – x
121. Victor had an average of 94 on his first four
x – 12 = (2)(48 – x) Math tests. After taking the next test, his average
x – 12 = 96 – 2x dropped to 93. Find his most recent grade.
x + 2x = 96 + 12 A. 88 B. 89 C. 90 D. 91
x + 2x = 108
x = 36 Solution:
60 – x = 60 – 36 = 24 New Score =(New Number)(New Average) –
................................................ (Old Number)(Old Average)
New Score = 5(93) – 4(94) = 465 – 376 = 89
118. If the length of a rectangle is increased by ................................................
20% while the width is decreased by 10%, what
will happen to its area?
122. X is of Y and Y is of Z. What part of Z is X?
A. decreased by 10%
B. increased by 10% A. X = Z B. X = Z
C. increased by 8%
C. X = Z D. X = Z
D. decreased by 2%

Solution: Solution:

(L x 1.2) (W x 0.9) = (1.08 x LW) X= Y


................................................ X = ( Z) = Z or Z
119. The 19th term of an arithmetic sequence is ................................................
85 and the 12th term is 43. Find the common
difference. 123. Two buses leave the same station at 8:00
A. 5 B. 6 C. 7 D. 8 pm. One bus travels north at the rate of 30 kph
and the other travels east at 40 kph. How many
Solution: kilometers apart are the buses at 10 pm?
A. 140 km B. 100 km
d= = = =6 C. 70 km D. 50 km

Solution:
120. If 2x = 3y and 4y = 5z, what is z in terms of From 8 to 10 PM is 2 hours. After two hours, one
x? bus will have travelled 60 km while the other, 80
A. z = x B. z = x km. Since the two buses are traveling on
C. z = x D. z = x perpendicular directions, we can use the
Pythagorean Theorem to find their distance.
Solution: D = √60 + 80 = 100 km
................................................
Make two equations wherein y will have the
same numerical coefficients. 124. A bus drove for 6 hours at 75 kph and 4
2x = 3y  8x = 12y hours at 80 kph. What was its average speed?
4y = 5z  12y = 15z A. 76 kph B. 77 kph
C. 77.5 kph D. 78 kph
This is a free reviewer. All rights reserved.
1000 MMR Author: Victor A. Tondo Jr., LPT
Solution: 128. Mr. Tondo has P100,000 to invest, from
which he wants to earn P5600 per year. Bank A
Get the total distance and the total time first.
offers 5% per annum while Bank B offers 6%.
6 hrs x 75 kph = 450 km
How much should he invest at Bank B?
4 hrs x 80 kph = 320 km
A. P45,000 B. P50,000
Total distance = 770 km, total time = 10 hrs
C. P55,000 D. P60,000
Average spd = = = 77 kph
................................................ Solution:
Let x = investment in Bank B
125. 18 students failed a quiz. They represent
.: 100,000 – x = investment in Bank A
30% of the class. How many students passed the
0.05(100,000 – x) + 0.06x = 5,600
quiz?
(5,000 – 0.05x) + 0.06x = 5,600
A. 60 B. 42 C. 36 D. 24
0.01x = 600; x = 60,000
................................................
Solution:
18:30% = N:70% 129. Evaluate when x = ¾ and y = .
18(70)/30 = N
42 = N A. -38 B. -19 C. 19 D. 38
................................................
Solution:
126. Rationalize: + = = ; = =


A. +1 B. 2√5 – 4 ÷ = x = 19
C.

D.
√ ................................................

130. Today, Vic is 11 years old while his father is


Solution: 37. How many years from now will his father be
To rationalize this, multiply both numerator and twice as old as he?
denominator by the conjugate of the A. 15 B. 13 C. 11 D. 10
denominator. By doing this, we are sure to have
a rational denominator. Solution:
√ √ √
x = = Let x = number of years from now
√ √ √
................................................ 2(11+x) = 37+x
22 + 2x = 37 + x
2x – x = 37 – 22; 15 = x
127. RNHS has 130 quizzers. 67 of them are
................................................
Math, 60 are Science, and 20 are quizzers for
both Math and Science. How many quizzers are
neither Math nor Science? 131. Carla and Diana are on a seesaw. Carla
weighs 50 kg and sits 168 cm to the left of the
A. 0 B. 13 C. 17 D. 23
fulcrum. If Diana weighs 60 kg, how far to the
Solution: right of the fulcrum must she sit to balance the
seesaw?
(A B)’ = U - (A B) A. 140 cm B. 170.8 cm
(A B)’ = 130 – [A + B – (A B)] C. 201.6 cm D. 210 cm
(A B)’ = 130 – (67 + 60 – 20) = 130 – 107 = 23

This is a free reviewer. All rights reserved.


1000 MMR Author: Victor A. Tondo Jr., LPT
Solution: 136. Find the largest area of a rectangle whose
perimeter is 100 cm.
Seesaw problems call for inverse or indirect
A. 2500 cm2 B. 2499 cm2
proportion.
C. 625 cm2 D. 624 cm2
50(168) = 60N
8400 = 60N
Solution:
140 = N
................................................ Instead of jumping to differential calculus
(minima and maxima) to solve this, simply make
132. Twenty guests shake hands with each other. it a square. That’s the shortcut for this kind of
If each guest is to shake hands with all the other question.
guests, how many handshakes will be made? ................................................
A. 400 B. 380 C. 200 D. 190
137. What time is 200 minutes past 10:30 PM?
Solution: A. 12:30 AM B. 12:30 PM
C. 1:50 AM D. 1:50 PM
20C2 = 190
................................................
Solution:
133. How many line segments can be made from 200 minutes = 3 hrs 20 mins
30 non-collinear points? 10:30 PM
A. 900 B. 870 C. 450 D. 435 + 3:20
13:50 PM  1:50 AM
Solution: ................................................
30C2 = 435
138. Find the product of two numbers whose
................................................
GCF is 24 and LCM is 120.
A. 2880 B. 1440 C. 720 D. 360
134. The longest chord of a circle is 80 cm. How
long is its radius?
Explanation:
A. 20 cm B. 30 cm
C. 20√2 cm D. 40 cm The product of two numbers is equal to the
product of their LCM and GCF.
Explanation: 24 x 120 = 2880
................................................
The longest chord is the diameter, and the radius
is half the diameter. 139. The salary of 4 men for 5 days is P9,000.
................................................ How much is the salary of 5 men for 6 days?
A. P12,000 B. P12,600
135. Find k such that 34k67 is divisible by 9. C. P13,500 D. P14,400
A. 5 B. 6 C. 7 D. 8
Solution:
Solution:
First, find the cost of each “man-day”.
Remember that for a number to be divisible by 9, 4 men x 5 days = 20 man-days
the sum of its digits must be equal to 9.
P9,000 ÷ 20 man-days = P450 per man-day
3+4+k+6+7 = 20+k
2+0+k = 2 + k = 9; k=7 You may now solve the problem.
................................................ 5 men x 6 days = 30 man-days
30 man-days x P450 per man-day = P13,500

This is a free reviewer. All rights reserved.


1000 MMR Author: Victor A. Tondo Jr., LPT
140. The average grade of eleven students is 83. 144. The average of x+5, 2x-4, and x+7 is 20.
If the average of six of these students is 88, what Find x.
is the average of the other 5 students? A. 18 B. 13 C. 9 D. 8
A. 77 B. 78 C. 79 D. 80
Solution:
Solution: ( ) ( ) ( )
= 20
Sum of grades of 11 students: 11 x 83 = 913
(x + 5) + (2x – 4) + (x + 7) = 60
Sum of grades of 6 students: 6 x 88 = 528
4x + 8 = 60
Sum of grades of other five: 913 – 528 = 385
4x = 52
Average of grades of other five: 385 ÷ 5 = 77 x = 13
................................................ ................................................

141. If x is 80% of y, what percent of y is x? 145. Mia is 16 years younger than Kia. 13 years
A. 120% B. 125% ago, Kia was thrice as old as Mia. What is Kia’s
C. 130% D. 135% present age?
A. 43 B. 40 C. 37 D. 34
Solution:
x = 0.8 y Solution:
=y  1÷0.8 = 1.25 Age today Age 13 years ago
.
1.25x = y Kia x x – 13
................................................ Mia x – 16 x – 16 – 13 or x – 29
(x – 13) = 3(x – 29)
142. Bus X left the terminal at 1 PM and traveled x – 13 = 3x – 87
at a speed of 60 kph. Bus Y left the same terminal -13 + 87= 3x - x
2 hours later and traveled 80 kph on the same 74 = 2x
route. What time will Bus B catch up with Bus A? 37 = x
A. 6 PM B. 9 PM ................................................
C. 11 PM D. 1 AM
146. Insert one term between 18 and 32 to make
Solution: a geometric sequence.
Let x = running time for Bus X A. 20 B. 24 C. 25 D. 27
60x = 80(x-2)  Bus Y left 2 hrs later Solution:
60x = 80x – 160
160 = 20x; Shortcut for inserting one term is √AB. This is
8=x also the formula for the geometric mean.
8 hours after Bus X left the terminal is 9AM. √18(32) = √576 = 24
................................................ ................................................

143. What is the degree of the polynomial 147. There are 100 pigs and chickens in a farm,
-3 x2y3 + 21 x3y4 – 7 x5y6 – 15? all of which are healthy. If there are 340 legs in
A. 4 B. 5 C. 11 D. 21 total, how many pigs are there?
A. 70 B. 65
Explanation: C. 60 D. 55

The degree of a polynomial is the highest sum of


exponents in a term.
This is a free reviewer. All rights reserved.
1000 MMR Author: Victor A. Tondo Jr., LPT
Solution: 150. Solve for x: 49x = 343
A. 1.142857 B. 7
Let P = number of pigs;
C = number of chickens C. 1.5 D. √7

P + C = 100 Solution:
4P + 2C = 340  since pigs have four legs
and chickens have two First, express both numbers as powers of the
2(P + C = 100)  2P + 2C = 200 same base.
4P + 2C = 340 - 4P + 2C = 340 49x = 343  (72)x = 73
-2P = - 140 Next, apply the laws of exponents.
P = 70 (72)x = 73
................................................ 72x = 73
2x = 3;
148. Adam can do a job alone in 8 hours, while x = 3/2 or 1.5
Bam can do the same job in 12 hours. One day, ................................................
they worked together for 1 hour before Bam left
Adam to finish the job. How long will it take 151. What is the highest possible product of two
Adam to finish the remaining job? numbers if their sum is 45?
A. 6 hrs 50 mins B. 6 hrs 40 mins A. 506 B. 506.25
C. 6 hrs 30 mins D. 6 hrs 20 mins C. 506.5 D. 506.725

Solution: Solution:
( )
Instead of jumping straight to minima and
,
( ) ( ) maxima under differential calculus, simply make
= = or 6 hrs your numbers equal to maximize their product.
That’s 6 hrs. and 20 mins. 45/2 = 22.5
Both numbers will be 22.5, so their product is
Mnemonic: 22.5 x 22.5 = 506.25
................................................
For questions like this (about working together
and then someone leaves), use PuTS U. When
152. Which statistical test is used for comparing
someone leaves you, “PuTS U” !!
observed frequencies to expected frequencies?
PuTS U stands for P(u)roduct, Time, Sum, Umalis
A. ANOVA B. t-test
................................................
C. Pearson R D. Chi Square
149. Find x if 2748 = 9x.
Explanation:
A. 144 B. 81 C. 72 D. 60
Observed vs Expected: Chi Square
Solution: Relationship: Pearson R (R for relationship)
Group differences: ANOVA (variance =
Express both sides as a power of 3.
differences)
(33)48 = (32)x
Comparing sets of normal distributions: T-test
3144 = 32x
................................................
144 = 2x
72 = x
153. The product of two consecutive odd
................................................
counting numbers is 1443. What is their sum?
A. 76 B. 78 C. 80 D. 82

This is a free reviewer. All rights reserved.


1000 MMR Author: Victor A. Tondo Jr., LPT
Solution: 157. Find + given x + y = 20 and xy = 81.
Let x = first number; x+2 = next number A. B. C. D.
x(x+2) = 1443
x2 + 2x = 1443
Solution:
x2 + 2x + 1 = 1443 + 1
√ + 2x + 1 = √1444 + = + = =
x+1 = 38; x = 37 x+2 = 39 ................................................
................................................
158. What is the remainder when
2 +1 4 534,214,557,989,215 is divided by 4?
154. Given ( ) = { 4 = 4, A. 0 B. 1 C. 2 D. 3
7 4
find lim ( ).
Explanation:
A. 4 B. 9
C. 0 D. limit does not exist The divisibility rule for 4 tells us that our
concern would only be the last 2 digits.
Solution: 15 ÷ 4 = 3 r. 3
Limit from the left: 2(4) + 1 = 9 ................................................
Limit from the right: 42 – 7 = 9
Since both limits are equal, then the limit is 9. 159. Dividing by 0.125 is the same as multiplying
................................................ by which number?
A. 5 B. 8 C. 10 D. 16
155. If today is a Saturday, what day is 125 days
from now? Explanation:
A. Friday B. Sunday Just use 1 as your test number.
C. Monday D. Tuesday
1÷ 0.125 = 8
Solution: ................................................

This is an application of modulo. 160. Find the surface area of a sphere whose
125 6 (mod 7) or 125÷7 = 17 r. 6 radius is 6 cm.
6 days after Saturday is Friday A. 72 π cm2 B. 108 π cm2
................................................ C. 144 π cm2 D. 192 π cm2

156. If the sum of the supplement and the Solution:


complement of an angle is 124, what is the
angle? Surface Area = 4 π r2 = 4 (62) π = 144 π
A. 71 B. 72 C. 73 D. 74 161. Which of the following is the reference
angle of 216o?
Solution: A. 84o B. 66o C. 54o D. 36o

(180 – x) + (90 – x) = 124 Explanation:


270 – 2x = 124
270 – 124 = 2x The reference angle for angles from the different
146 = 2x; 73 = x quadrants are as follows:
................................................ QI: the angle θ itself
QII. 180 – θ
QIII. θ – 180
QIV. 360 – θ
This is a free reviewer. All rights reserved.
1000 MMR Author: Victor A. Tondo Jr., LPT
162. Which of the following angles in standard 165. Find the equation of the line passing
position is coterminal with 40o? through the point of origin and (3,4).
A. 2200o B. 1760o A. y = x B. y= x
C. 1520o D. 1360o
C. y = x + D. y = x + 1
Explanation:
Solution:
Coterminal angles are congruent, modulo 360.
That means they will leave the same remainder y – y1 = (x x ); (0,0) and (3,4)
when divided by 360.
y–0= (x 0)
In textbooks, θ is coterminal with any angle
expressed as 360N + θ wherein N is an integer. y= x
To easily tackle this question, simply subtract 40 ................................................
from each of the choices, then see if any of those
is divisible by 360 (or leaves a remainder of 0 166. Find the range of f(x) = -2x2 + 4x.
when divided by 360) using your calculator. A. y 2 B. y 2
2200 – 40 = 2160 C. y -2 D. y -2
2160 ÷ 360 = 6
.: 2200o is coterminal with 40o Explanation:
................................................ Since this is a quadratic function, you need to
know two things to determine its range: its
163. Find the equation of the line passing
opening and k of its vertex (h,k).
through (2,7) and (-3,-3).
A. y = 4x -1 B. y = 3x + 1 The parabola opens downward since a = -2.
C. y = 3x + 6 D. y = 2x + 3 k = c – (b2/4a) = -16/(-8) = 2
Since the parabola opens downward, the graph
Solution: starts from - , going to k which is 2.
Two-point form of linear equations: Thus, y 2.
y – y1 = (x x ) ................................................

y–7= (x 2) 167. If a3/2 – 1 = 7, what is a?


y – 7 = 2(x – 2) A. 4 B. 8 C. 9 D. 18
y – 7 = 2x – 4
y = 2x + 3 Solution:
................................................
a3/2 – 1 = 7
a3/2 = 8
164. In which quadrant can we find θ if tan θ < 0
(a3/2 = 23)2/3
and sin θ > 0?
a = 22 = 4
A. First Quadrant
................................................
B. Second Quadrant
C. Third Quadrant
168. Which of the following is true?
D. Fourth Quadrant
A. A rectangle is a square.
B. A rhombus is a rectangle.
Explanation:
C. A trapezoid is a rhombus.
Use the CAST D. A square is a rhombus.
mnemonic.

This is a free reviewer. All rights reserved.


1000 MMR Author: Victor A. Tondo Jr., LPT
169. What is the measure of each exterior angle Solution:
of a pentagon?
P250 = discount
A. 108o B. 72o
10% = discount rate
C. 60o D. 36o
Original Price (OP) = ???
Selling Price = ???
Solution:
DC = OP x DC Rate
MEA = 360/N = 360/5 = 72o
................................................ 250 = OP x (0.1)
250/0.1 = OP
170. How many diagonals does a nonagon have? 2500 = OP
A. 27 B. 36 C. 45 D. 54
Selling Price = OP – DC
Selling Price = 2500 – 250 = P2250
Solution:
................................................
Diagonals = N(N-3)/2 = 9(6)/2 = 27
................................................ 173. A book was sold for P270 after a 10%
discount was given. How much was the book
171. What is the fractional equivalent of originally?
0.123123123123…? A. P330 B. P300
A. B. C. D. C. P297 D. P280

Solution:
Algebraic Solution:
SP = OP (1-DC Rate)
Let x = 0.123123123123…
270 = OP (0.9)
1000x = 123.123123123123… 270/0.9 = OP
1000x – x = 123 300 = OP
999x = 123 ................................................
x = 123/999
x = 41/333 174. Find the area of an equilateral triangle
whose sides measure 12 cm each.
Alternative Solution: A. 36√3 cm2 B. 48√3 cm2
Write a fraction whose numerator is the C. 60√3 cm2 D. 72√3 cm2
repeated digits (123) and whose denominator
has the same number of digits but is made of 9s Solution:
(123 is 3-digit, so use 999). √ √
Thus, 123/999 or 41/333. AreaEqTri = = = 36√3
................................................ ................................................

172. Mrs. Pasay saved P250 after buying a phone 175. This is located at the intersection of the
with a 10% discount. How much did she pay for angle bisectors of a triangle.
the phone? A. Incenter
A. P2500 B. P2250 B. Circumcenter
C. P2000 D. P1750 C. Centroid
D. Orthocenter

This is a free reviewer. All rights reserved.


1000 MMR Author: Victor A. Tondo Jr., LPT
Explanation: Solution:
Incenter: intersection of angle bisectors D = 2P M=P+4
Circumcenter: intersection of perpendicular
D + P + M = 164
bisectors
2P + P + (P+4) = 164
Centroid: intersection of medians
4P + 4 = 164
Orthocenter: intersection of altitudes
4P = 160;
................................................
P = 40,
176. ∆ABC is similar to ∆DEF. ̅̅̅̅AB is 9 cm long D = 2(40) = 80
̅̅̅̅ is 12 cm long. If the area of ∆ABC is
while DE ................................................
27 cm2, what is the area of ∆DEF?
A. 36 cm2 B. 48 cm2 180. A circle is drawn inside a triangle such that
C. 60 cm 2 D. 72 cm2 it is tangent to the sides of the triangle. Its center
will be the triangle’s ___________________.
Solution: A. Incenter B. Circumcenter
C. Centroid D. Orthocenter
Ratio of sides = 9:12 or 3:4
Ratio of areas = 32:42 or 9:16
Explanation:
9:16 = 27:N
16(27) = 9N; 48 = N If the circle is inside the triangle, its center is the
................................................ INcenter. If the circle circumscribes the triangle,
its center is the CIRCUMcenter.
177. Find the remainder when x4 – 3x3 + 2x2 + ................................................
3x – 9 is divided by (x-3).
A. -18 B. -9 C. 9 D. 18 181. Rayon can do a job in 3 hours, while Carlyn
can do the same job in 7 hours. How long will it
Solution: take them to finish the job by working together?
A. 2.1 hours B. 2.5 hours
Use the remainder theorem.
C. 5 hours D. 10 hours
x-a = x-3  a=3
Remainder = f(3) Solution:
f(3) = 34 – 3(33) + 2(32) + 3(3) – 9 = = 2.1
f(3) = 81 – 81 + 18 + 9 – 9 = 18 ................................................
................................................
182. This line is perpendicular to one side of the
178. Which of the following has its incenter, triangle passing through the opposite vertex.
circumcenter, centroid, and orthocenter in just A. Longitude B. Median
one point? C. Altitude D. Bisector
A. Right Triangles B. Equilateral Triangles
C. Isosceles Triangles D. Scalene Triangles
................................................ Explanation:
179. Dexter is twice as heavy as Pablo. Ming is Altitude: perpendicular to one side, passing
4kg heavier than Pablo. The sum of their masses through the opposite vertex.
is 164kg. How heavy is Dexter? Median: line segment from midpoint of one side
A. 40 kg B. 44 kg to opposite vertex
C. 80 kg D. 88 kg Bisector: line segment that bisects an angle of a
triangle

This is a free reviewer. All rights reserved.


1000 MMR Author: Victor A. Tondo Jr., LPT
183. How many ways can Lola Leonor arrange 186. How many ways can the letters of the word
her six meals on the Lazy Susan (the rotating BANANA be rearranged?
circular wooden server on top of the table)? A. 720 B. 240 C. 120 D. 60
A. 720 B. 120 C. 36 D. 30
Solution:
Explanation:
This is a permutation with repeated elements:
This problem is about Circular Permutations or !
P = ! ! !… where n is the total number of
arrangements on a circle. The formula is (N-1)!.
elements (or letters) and a!, b!, c!, … are the
................................................
number of times the different elements (or
letters) were repeated.
184. In parallelogram
BANANA has 6 letters: 1 B, 3 A, and 2 N
MATH, m M = 7x – 12 and !
m T = 5x + 32. Find m A. P = ! ! ! = ( )( ) = 60
A. 22 B. 38 ................................................
C. 44 D. 142
187. “The temperature in Baguio City is 20o
Solution: while the temperature in Tuguegarao City is
40o”. What level of data is temperature in
M and T are opposite angles,
degrees Celsius?
therefore m M = m T.
A. Nominal B. Ordinal
7x – 12 = 5x + 32 C. Interval D. Ratio
7x – 5x = 32 + 12
2x = 44 Explanation:
x = 22
Since you cannot infer that Tuguegarao City is
.: m M = 7(22)-12 = 154-12 = 142 TWICE AS HOT as Baguio City, then the data is
M and A are consecutive angles, not ratio.
therefore m M + m A = 180. Remember: Temperature in degrees Celsius or
142 + m A = 180 Fahrenheit is interval, but temperature in Kelvin
m A = 180 – 142 is ratio.
m A = 38 ................................................
................................................
188. What is formed by the intersection of two
185. Find the equation of the line perpendicular planes?
to 2x + 5y = 7, passing through (1, 2). A. a point B. a line
A. 2x + 5y = 12 B. 2x – 5y = -8 C. a plane D. space
C. 5x + 2y = 9 D. 5x – 2y = 1 ................................................

Solution: 189. What is formed when a plane intersects a


cone parallel to its circular base?
Just like what we did in item #76, simply
A. ellipse
interchange the numerical coefficients of x and y
B. hyperbola
in the original equation, then change the
C. circle
operation between them. For the constant,
D. parabola
simply substitute the x and y values of the point
and solve.

This is a free reviewer. All rights reserved.


1000 MMR Author: Victor A. Tondo Jr., LPT
Explanation: 194. If three-fourths of a number is 33 more than
its one-fifth, what is that number?
The conic sections are formed by intersecting a
A. 240 B. 120
cone with a plane.
C.90 D. 60
Parallel to its base: circle
Perpendicular to its base: parabola
Solution:
Slanted relative to the base: ellipse
................................................ x= x + 33

190. In which non-Euclidean model for geometry 20 ( x = x + 33)  15x = 4x + 660


can we have any given line ℓ and a point A which 11x = 660
is not on ℓ, wherein all lines through A will x = 60
intersect ℓ? ................................................
A. hyperbolic B. elliptic
C. Saccheri D. Pythagorean 195. Which of the following has the greatest
value:
Explanation: A. 3 + 32 + (3 + 3)2 B. 33
C. [(3 + 3) ]
2 2 D. (3 + 3 + 3)2
In Euclidean geometry, only one line will pass
through A. In elliptic geometry, all lines will pass
Solution:
through A and intersect ℓ.
................................................ A. 3 + 32 + (3 + 3)2 = 3 + 9 + 36 = 48
B. 33 = 27
191. Which numerical system is sexagesimal C. [(3 + 3)2]2 = 362 = 1296
(base-60)? D. (3 + 3 + 3)2 = 92 = 81
A. Mayan B. Roman ................................................
C. Babylonian D. Hindu-Arabic
196. Which of the following has an undefined
Explanation: slope?
A. a vertical line
Mayans: base-20 (vigesimal)
B. a horizontal line
Babylonians: base-60 (sexagesimal)
C. a line parallel to the x-axis
Romans and Hindu-Arabic: base-10 (decimal)
D. a diagonal line
................................................
Horizontal line (parallel to x-axis): m = 0
192. Which numerical system makes use of dots
Vertical line (parallel to y-axis): m is undefined
and horizontal lines, and shell shapes for zero?
Slanted downwards to the right: m is negative
A. Egyptian B. Roman
Slanted upwards to the right: m is positive
C. Greek D. Mayan
................................................
................................................
197. In solid geometry, what do you call a solid
193. Which of the following is false?
bound by polygons?
A. sin2 θ + cos2 θ = 1
A. multigon B. tessellation
B. sin θ (csc θ) = 1
C. porygon D. polyhedron
C. sin θ ÷ cos θ = tan θ
D. sin θ (tan θ) = cos θ

Explanation:

= tan ; or sin θ = tan θ (cos θ)

This is a free reviewer. All rights reserved.


1000 MMR Author: Victor A. Tondo Jr., LPT
198. Tchr. Victor needs to randomly get 10 out of Explanation:
his 50 students for drug testing. He proceeds by A. ABCD is a rhombus
making the students count off from 1 to 5. He B. MATH is a rectangle
then randomly picks a number from 1 to 5. D. FROG is a rectangle
Which sampling method did he use? ................................................
A. stratified B. cluster
C. systematic D. convenience 203. Which of the following is the set of points
whose sum of distance to two fixed points is
Explanation: constant?
A. parabola B. circle
Stratified: /strata/ population has hierarchy or
C. ellipse D. hyperbola
sub-classifications
Cluster: for homogenous population in a large
Explanation:
area
Systematic: counting off Parabola: set of points equidistant to a fixed
................................................ point (focus) and a fixed line (directrix)
Circle: set of points equidistant to a fixed point
199. Which statistical test must be used in (center)
testing the significance of group differences Ellipse: set of points equidistant to two fixed
between 2 or more groups? points (foci)
A. Chi Square B. t-test Hyperbola: set of points whose difference of
C. ANOVA D. Pearson R distances to two fixed points is constant
................................................ ................................................

200. Which Mathematician is famous for the 204. Which of the following is not a triangle
Fibonacci sequence? congruence postulate?
A. Ptolemy A. SAS B. ASA C. SAA D. AAA
B. Leonardo Pisano Bigollo
C. Pierre de Fermat Explanation:
D. Luca Pacioli
AAA is a triangle similarity postulate.
................................................
................................................
201. Which Mathematician is famous for his last
205. If A is at (-8,5) and B is at (4,-11), find C if C
theorem?
is three-fourths the way from A to B.
A. Pythagoras B. Isaac Newton
A. (1, -7) B. (-4, 1)
C. Daniel Bernoulli D. Pierre de Fermat
C. (1, 1) D. (-4, -7)
................................................
Solution:
202. Which of the following is a square?
A. Polygon ABCD which has 4 congruent sides. Since C is three-fourths the way from A to B, then
B. Polygon MATH which has 4 perpendicular its coordinates are:
sides. x = -8 + [4 – (-8)]
C. Quadrilateral HEAD which has one pair of
x = -8 + 9
congruent perpendicular bisecting diagonals.
x=1
D. Quadrilateral FROG which has 4 right angles.
y = 5 + (-11 – 5)
y = 5 + (-12)
y=7

This is a free reviewer. All rights reserved.


1000 MMR Author: Victor A. Tondo Jr., LPT
206. CPCTC stands for “____________ parts of Rate comes with the word “percent” or the
congruent triangles are congruent”. percent symbol (%).
A. collinear B. complementary
The formulas are:
C. corresponding D. conjugate
................................................ Rate: R = x 100%
Part: P = B x R (convert rate to decimal first)
207. Victor deposited an amount of P200,000 in Base: B = (convert rate to decimal first)
a bank that offers 5% interest compounded per We are looking for the Rate in this problem, so
annum. How much will he have in his account
after 3 years? R = x 100% = 0.6875 x 100% = 68.75%
A. P230,000 B. P231,525 ................................................
C. P233,050 D. P234,575
211. The hypotenuse of a right triangle measures
Solution: 40 cm. Find its area if one angle measures 30o.
A. 100√3 cm2 B. 200√2 cm2
Since the interest is compounded annually,
Acct = Principal x (1 + rate)time C. 200√3 cm2 D. 400√2 cm2
Acct = 200,000 x (1.053)
Acct = 200,000 x 1.157625 = 231,525 Solution:
................................................ Since the hypotenuse is 40 cm, then the leg
opposite 30o is 20 cm (half the hypotenuse), and
208. Find the remainder when the polynomial the leg opposite 60o is 20√3 cm (√3 times the
x4 – 3x3 + 2x2 – 5x + 8 is divided by (x – 3). short leg). The area of a right triangle is given by
A. 5 B. 8 C. 11 D. 14
= .

Solution: = = 200√3 cm2
Use the Remainder Theorem. ................................................
Our divisor is (x – 3), so a = 3.
f(3) = 34 – 3 (33) + 2 (32) – 5(3) + 8 212. Nine cans of soda and four hamburgers cost
f(3) = 81 – 81 + 18 – 15 + 8 = 11 a total of P257. Five cans of soda and seven
................................................ hamburgers cost a total of P224. How much is a
can of soda?
209. What is 60% of 120? A. P17 B. P19 C. P21 D. P23
A. 50 B. 72 C. 180 D. 200
Solution:
Solution: Let C = price of a can of soda,
60% of 120 translates to (0.60) x 120, or 72. B = price of a hamburger
................................................ 9C + 4B = 257 x7  63C + 28B = 1799
5C + 7B = 224 x4  20C + 28B = 896
210. What percent of 80 is 55? 63C + 28B = 1799
A. 145.45% B. 135% 20C + 28B = 896
C. 68.75% D. 44% 43C = 903
43 43
Solution: C = 21
Identify the Part (in textbooks, they use the word
“Percentage”), the Base, and the Rate when you
face questions like this. Part comes with the
word “is”, Base comes with the word “of”, while
This is a free reviewer. All rights reserved.
1000 MMR Author: Victor A. Tondo Jr., LPT
213. The product of two consecutive even Solution:
numbers is 728. What is the smaller number?
Sell Price = Orig Price x (1 - Disct Rate)
A. 22 B. 24 C. 26 D. 28
4000 = OP x (1 – 0.2)
4000 = OP x (0.8)
Solution:
4000 ÷ 0.8 = OP
Let x = smaller number; 5000 = OP
x + 2 = larger number ................................................
x (x+2) = 728
x2 + 2x = 728; 217. When a number is increased by 3, its square
x + 2x + 1 = 729
2 increases by 111. By what does its square
√x + 2x + 1 = √729 increase when the number is increased by 6?
x + 1 = 27 A. 222 B. 240 C. 444 D. 480
x = 26
................................................ Solution:
Let’s find the original number first.
214. What time is 219 minutes past 6:40 AM? (x+3)2 – x2 = 111
A. 8:59 AM B. 9:19 AM x2 + 6x + 9 – x2 = 111
C. 9:49 AM D. 10:19 AM 6x + 9 = 111
6x = 102
Solution: x = 17
219 minutes = 3 hrs 39 mins 172 = 289; (17+6)2 = 232 = 529
6:40 529 – 289 = 240
+ 3:39 ................................................
9:79 or 10:19
................................................ 218. How many prime numbers are there from 1
to 100?
215. Find the vertex of y = 3x2 – 2x + 11. A. 23 B. 24 C. 25 D. 26
A. ( , ) B. ( , ) ................................................
C. ( , ) D. ( , ) 219. Find the range of f(x) = 2x2 – 8x + 9.
A. y 0 B. y 1
Solution: C. y 9 D. y
The vertex is at (h,k) where
Solution:
h= ; k=c-
( ) ( )
This is a quadratic function so the graph is a
h= k = 11 – parabola opening upwards (since A = 2). Solve
( ) ( )
h= k= for k to find its minimum value.
k = c – b2/4a = 9 – 64/8
................................................ k=9–8=1
Therefore, y 1.
216. After getting a 20% discount, Mr. Lopez
paid P4,000 for a gadget. How much was its
original price? 220. Find the domain of y =
A. P4,800 B. P5,000 A. x ±7, ±10 B. x ±7
C. P8,000 D. P20,000 C. x ±10 D. x 1

This is a free reviewer. All rights reserved.


1000 MMR Author: Victor A. Tondo Jr., LPT
Explanation: 224. 12 + 17 + 22 + 27 + … + 117 = _____
A. 1409 B. 1414
The denominator should not be equal to 0.
C. 1419 D. 1424
x2 – 49 0
x2 49 Solution:
x ±7
................................................ n= +1

221. Solve for x: (x+3)2 = (x-4)2. n= +1 = 22


A. x = 0 B. x = ½ Sum = (n)
C. x = 1 D. no solution
Sum = (22) = 1419
Solution: ................................................
(x+3)2 = (x-4)2 225. Mr. G sold 80% of his apples and still had
x2 + 6x + 9 = x2 – 8x + 16 213 apples left. How many apples did he have
6x + 8x = 16 – 9 originally?
14x = 7 A. 1704 B. 1065
x=½ C. 852 D. 293
................................................
Solution:
222. The diagonal of a rectangular prism is 13
cm long. If it is 3 cm thick and 12 cm long, how 213 = 20% x Original number of apples
wide is it? 213 ÷ 0.2 = Original number of apples
A. 3 cm B. 4 cm 1065 = Original number of apples
C.4√3 cm D. 5 cm ................................................

Solution: 226. When a number is increased by 4, its square


also increases by 168. What is this number?
Diagonal2 = Length2 + Width2 + Height2
132 = 122 + W2 + 32 A. 15 B. 19 C. 23 D. 27
169 = 144 + W2 + 9
Solution:
169 – 144 – 9 = W2
16 = W2; (x+4)2 – x2 = 168
4=W x2 + 8x + 16 – x2 = 168
................................................ 8x + 16 = 168
8x = 152
223. Which of the following is not a function? x = 19
A. y = x2 + 2017x – 2017 ................................................
B. y = |2017x| - 2017
C. y = √2017 + 2017 227. Solve for k to make a perfect square
D. y2 = x + 2017 trinomial: 9x2 + kx + 25
A. 10 B. 15 C. 20 D. 30
Explanation:
Solution:
When y is raised to an even number, it
automatically becomes not a function. 9x2 + kx + 25 = (3x)2 + 2(3x)(5) + 52
= (3x)2 + 30x + 52

This is a free reviewer. All rights reserved.


1000 MMR Author: Victor A. Tondo Jr., LPT
228. Find the y-intercept of 2x + 3y = 4. Solution:
A. B. C. D. 2 Convert it to its slope-intercept form.
3x + 5y = 7  5y = -3x + 7
Solution: y= x+
y-intercept is taken when x = 0. .: m =
2(0) + 3y = 4
................................................
3y = 4
y= 232. Which of the following is a polynomial?
................................................ A. √3 + 4 + 2 B. 2x + 3√
C. +3 D. √3 x + 7
229. Which of the following points is on the line
y = 2x + 5?
Explanation:
A. (1, 3) B. (2, 9)
C. (0, 10) D. (3, 10) A polynomial accepts only WHOLE numbers as
exponents of the variable/s. Only D has whole
Explanation: numbers as exponents of x.
Just substitute the x and y-values of each point ................................................
and see which one makes a true equation.
A. (1, 3)  3 = 2(1) + 5 False 233. What is the degree of the polynomial
B. (2, 9)  9 = 2(2) + 5 True 9x4 + 5x3 – 2x2 + 3x – 17?
C. (0, 10)  10 = 2(0) + 5 False A. 4 B. 5 C. 9 D. 10
D. (3, 10)  10 = 2(3) + 5 False
Explanation:
................................................
Degree refers to the highest exponent or sum of
230. Find the intersection of y = -2x + 1 and exponents of the variables in any term of a
y = 3x + 16. polynomial.
A. (-3, 7) B. (-4, 9) ................................................
C. (3, -7) D. (4, -9)
234. log2 32√2 = __________.
Solution: A. 2.5 B. 3.5 C. 4.5 D. 5.5
y = -2x + 1
(-) y = 3x + 16 Solution:
0 = -5x – 15 32 = 25 and √2 = 21/2
5x = -15 5 + ½ = 5.5
x = -3; y = -2(-3) + 1
................................................
y = 6+1 = 7
................................................
235. If y = √3 + 6 , what is x in terms of y?
231. Find the slope of 3x + 5y = 7. A. x = √ –1 B. x = √ +1
A. B.
C. D. C. x = √ +1 D. x = √ –1

This is a free reviewer. All rights reserved.


1000 MMR Author: Victor A. Tondo Jr., LPT
Solution: 239. Find the slope of the line tangent to
y = √3 + 6 y = x3 – 6x2 + 2x + 7 at x = 4.
Square the equation y2 = 3x2 + 6x A. -8 B. -2 C. 2 D. 8
Divide by 3 = x2 + 2x
Solution:
Complete the square + 1= x2 + 2x + 1
Get the first derivative of y.
= x2 + 2x + 1 y' = 3x2 – 12x + 2
Substitute x = 4 for every x in y’.
Extract the root √ =x+1
3(4)2 – 12(4) + 2 = 48 – 48 + 2 = 2
................................................
Finally, √ - 1= x
................................................ 240. Find the average rate of change of
y = x3 – 2x + 3 from x = 0 to x = 3.
236. Which of the following is a pair of parallel A. 5 B. 6 C. 7 D. 8
lines?
A. y = 2 and x = 2 Solution:
B. 12x + 13y = 14 and 13x + 14y = 15 ( ) ( )
C. y = 3x + 8 and 3y = x + 9 Average Rate of Change = = =7
D. 4x + 5y = 6 and 8x + 10y = 21 ................................................

Explanation: 241. Find the radius of x2 + y2 + 2x – 4y = 44.


Parallel lines have the same slope. Both lines A. √39 B. 2√11 C. 7 D. 3√6
have a slope of -4/5 in D.
................................................ Solution:
Complete the squares on the left side of the
237. Which of the following is a pair of equation to return it to its center-radius form.
perpendicular lines? x2 + y2 + 2x – 4y = 44
A. x = 5 and y = 7 x2 + 2x + y2– 4y = 44
B. y = x and 2y = 4x + 5 (x2 + 2x + 1)+ (y2– 4y +4)= 44 + 1 + 4
C. x = 2y + 3 and 2x + 3y = 4 (x2 + 2x + 1)+ (y2– 4y +4)= 49
D. y = 5x + 6 and y = 0.2x – 8 ................................................

Explanation: 242. Gian has 8 more P5 coins than P1 coins. If


Perpendicular lines have negative reciprocal he has a total of P106, how many P5 coins does
slopes. For choice A, x=5 is horizontal and y=7 is he have?
vertical, therefore they are perpendicular. A. 13 B. 15 C. 17 D. 19
................................................
Solution:
238. Find the altitude to the hypotenuse of a Let x = number of P1 coins
right triangle whose sides measure 5 cm, 12 cm, .: x+8 = number of P5 coins
and 13 cm. 1(x) + 5 (x+8) = 106
A. B. C. D. 26 x + 5x + 40 = 106
6x + 40 = 106
Solution: 6x = 66
x = 11
Altitude to the Hyp = (L1 L2)/Hyp = 5(12)/13 .: He has 11 P1 coins and (11+8) or 19 P5 coins.
Altitude to the Hyp = 60/13
This is a free reviewer. All rights reserved.
1000 MMR Author: Victor A. Tondo Jr., LPT
243. After using half of her budget on bills, one- 246. The average grade of 23 students in Section
third on groceries, and P270 on a shirt, Mrs. D A is 86, while the average grade of 27 students in
still had P130 left. How much was her budget? Section B is 91. What is the average grade of all
A. P2400 B. P2700 50 students in both sections?
C. P3000 D. P3300 A. 88.5 B. 88.6 C. 88.7 D. 88.8

Solution: Solution:
Let x = budget ( ) ( )
Average = = = 88.7
x + x + 270 + 130 = x ................................................
x + 400 = x
247. Find the axis of symmetry of y = 3x2 – 5x.
400 = x – x
A. x = B. x =
400 = x
C. x = D. x =
2400 = x
................................................
Explanation:
244. x varies directly as y and inversely as z. If The axis of symmetry is located at x = .
x = 24 when y = 32 and z = 4, what is x when
y = 21 and z = 7? ................................................
A. 3 B. 5 C. 7 D. 9
248. Find the range of the following scores:
Solution: 19 25 24 31 23 29 33
A. 12 B. 13 C. 14 D. 15
x = ky/z 24 = k(32)/4
24 = 8k Explanation:
3=k
x = 3y/z Range = Highest Score – Lowest Score
x = 3 (21) / 7 ................................................
x = 63/7
x=9 249. Mr. C travels for 2 hours at a speed of 38
................................................ kph and then north for 3 hours at a speed of 53
kph. What is his average speed?
245. Find the mode of the following scores: A. 44 kph B. 45.5 kph
78 78 78 78 79 79 79 C. 47 kph D. 48.5 kph
79 80 80 80 80 Solution:
A. 79 B. 78, 79, and 80 Average Speed =
C. 80 D. no mode
Average Speed = = = 47 kph
Explanation:
250. Victor, Chris, and Aira volunteered to teach
Mode, by definition, is the score with the highest
at a nearby daycare. Chris worked for 2 hours
frequency. Since each score has a frequency of 4,
less than Aira. Victor worked twice as many
then there is no mode.
hours as Chris. Altogether, they worked for 58
................................................
hours. How many hours did Victor work?
A. 14 B. 16 C. 28 D. 32

This is a free reviewer. All rights reserved.


1000 MMR Author: Victor A. Tondo Jr., LPT
Solution: Solution:
The center of x2 + y2 + Dx + Ey + F = 0
Aira = n hours
.: Chris = (n – 2) hours is at ( , ).
.: Victor = 2(n-2) hours ................................................
n + (n-2) + 2(n-2) = 58
n + n - 2 + 2n - 4 = 58 254. Find the equation of the circle with center at
4n – 6 = 58 (2, 3), passing through (5, -1).
4n = 64 A. x2 + y2 + 4x + 6y = 0
n = 16 B. x2 + y2 + 4x + 6y = 12
C. x2 + y2 – 4x – 6y = 0
.: Victor worked for 2(16-2) or 28 hours.
D. x2 + y2 – 4x – 6y = 12
................................................
Solution:
251. What conic figure does the equation
x2 + y2 + 4x = -4 form? Find the radius first by using the center-radius
A. Real circle B. Degenerate circle form.
C. Imaginary circle D. Ellipse (x – 2)2 + (y – 3)2 = r2
(5 – 2)2 + (-1 – 3)2 = r2
Solution: 9 + 16 = r2
25 = r2
Use CTS (completing trinomial squares) to
5=r
convert the equation to its center-radius form.
x2 + y2 + 4x = -4 Substitute this to your center-radius form and
(x + 4x + 4) + y2 = -4 + 4
2 simplify.
(x + 2)2 + y2 = 0 (x – 2)2 + (y – 3)2 = 52
x2 – 4x + 4 + y2 – 6y + 9 = 25
Since r2 = 0, then the radius is also 0, making it a x2 + y2 – 4x – 6y + 13 = 25
degenerate circle. x2 + y2 – 4x – 6y = 12
................................................ ................................................

252. What conic figure does the equation 255. Find the equation of the vertical line passing
x2 + y2 + 8x – 6y = -100 form? through (-3, 4).
A. Real circle B. Degenerate circle A. x = -3 B. x = 4
C. Imaginary circle D. Ellipse C. y = -3 D. y = 4

Explanation:
Solution:
Equation of vertical lines: x = abscissa of your
Use CTS (completing trinomial squares) to
point
convert the equation to its center-radius form.
................................................
x2 + y2 + 8x – 6y = -100
(x2 + 8x + 16) + (y2 – 6y + 9) = -100 + 16 + 9
256. Find the equation of the horizontal line
(x + 4)2 + (y – 3)2 = -75
passing through (-3, 4).
Since r2 = -75, then the radius is imaginary, A. x = -3 B. x = 4
making it an imaginary circle. C. y = -3 D. y = 4
................................................
Explanation:
253. Find the center of + + 6x – 10y = 2.
x2 y2 Equation of horizontal lines: y = ordinate of your
A. (6, -10) B. (-6, 10) point
C. (-3, 5) D. (3, -5)
This is a free reviewer. All rights reserved.
1000 MMR Author: Victor A. Tondo Jr., LPT
257. Which of the following lines passes through of the circle. The second option would be more
the point (3, -2)? preferable as it already gives us the equation of
A. y = x + 5 B. y = 2x – 8 the circle.
C. y = 5 – x D. y = 5 – 2x
(x – 3)2 + (y – 3)2 = r2
Solution: Using the point A(9, -5), we get
(9 – 3)2 + (-5 – 3)2 = r2
Simply substitute x and y from your point. If the
36 + 64 = r2
equation holds true, then the given line passes
100 = r2
through your point.
.: (x – 3)2 + (y – 3)2 = 100
y=x+5  (-2) 3 + 5 ................................................
y = 2x – 8  (-2) = 2(3) – 8
y=5–x  (-2) 5 - 3 260. Find the distance between the line
y = 5 – 2x  (-2) 5 – 2(3) 3x + 4y – 5 = 0 and the point (8, -1).
................................................ A. 2 B. 3 C. 4 D. 5

258. Given that I(2, -3) is the midpoint of V(-4, 5) Solution:


and C, find the coordinates of C.
A. (-1, 1) B. (1, -1) Use the formula for distance of point (x1, y1)
C. (8, -11) D. (-10, 16) from line Ax + By + C = 0: D = √
( ) ( )
D=
Solution: √
D= =3
Use the midpoint formula: xm = , ym =
................................................
( )
2= -3 =
261. B is one-fourth of the way from A(-13,9) to
4 = (-4) + x2 -6 = 5 + y2 C(7,-7). Find the coordinates of B.
8 = x2 -11 = y2 A. (2, -3) B. (-8, 5)
................................................ C. (2.5, -3.5) D. (-7.5, 5.5)

259. The endpoints of the diameter of a circle are Solution:


A(9, -5) and B(-3, 11). What is the equation of
the circle? In this question, it is important to note the
A. (x – 3)2 + (y – 3)2 = 100 “from” point and the “to” point.
B. (x – 3)2 + (y – 3)2 = 400 Abscissa of point B = abscissa of “from” point +
C. (x + 3)2 + (y + 3)2 = 100 ¼ (abscissa of “to” point minus abscissa of
D. (x + 3)2 + (y + 3)2 = 400 “from” point)
Abscissa of point B = -13 + ¼ (7 – (-13))
Solution: = -13 + ¼ (20) = -8

Find the midpoint of the diameter to get the Ordinate of point B = ordinate of “from” point +
center, C(h,k). ¼ (ordinate of “to” point minus ordinate of
( ) “from” point)
h= k=
Ordinate of point B = 9 + ¼ (-7 – 9)
h = 3, k = 3 = 9 + ¼ (-16) = 5
................................................
Next, find the length of the radius. You can do
this by either A) getting half of the length of the
diameter AB, or B) using the center-radius form

This is a free reviewer. All rights reserved.


1000 MMR Author: Victor A. Tondo Jr., LPT
261. Find the area of the triangle whose vertices Solution:
are X(-9, -3), Y(-2, 8), and Z(5, 1).
Two lines are parallel when their slopes are
A. 61 B. 62 C. 63 D. 64
equal. Check the slopes of the lines by converting
them to their slope-intercept form (y = mx + b).
Solution:
A) y + 3x = 5  y = -3x + 5 m = -3
The biggest mistake you could make in solving B) x + 3y = 6  y= x+2 m=
this problem is by getting the equation of one the
line formed by one pair of vertices, getting the C) y = x + 7 m=
length of the said side, and getting the distance of D) y = 3x + 8  m=3
the third point from the line you first obtained, ................................................
then multiplying the obtained distance by the
length of the line segment and then dividing by 264. Which of the following is perpendicular to
two. That’s a very long solution. the line defined by the equation y = 3x – 4?
A. y + 3x = 9 B. -x + 3y = 10
Simply use the formula A∆ = ½ * + C. y = x + 11 D. y = 3x – 12
9 2 5
A∆ = ½ * + Solution:
3 8 1
A∆ = ½ [(-72 + -2 + -15) – (6 + 40 + -9)] Two lines are perpendicular when their slopes
A∆ = ½ |(-89 – 37)| are negative reciprocals of each other, or when
A∆ = ½ (126) = 63 m1 x m2 = -1. The slope of the given line is 3, so
................................................
the slope of the other line is . Check the slopes
262. Which of the following is outside the circle of the lines by using their slope-intercept form
defined by the equation (x – 3)2 + y2 = 40? (y = mx + b).
A. (5, 6) B. (7, 5) A) y + 3x = 9  y = -3x + 9 m = -3
C. (0, -5) D. (-1, 4)
B) -x + 3y = 10 y= x+ m=
Solution: C) y = x + 11 m=
A point is outside the circle when its distance D) y = 3x – 12  m=3
from the center is greater than the radius. ................................................
A: (5, 6)  (5 – 3)2 + 62 = 40
This point is ON the circle. 265. Which of the following is coincidental to the
B: (7, 5)  (7 – 3)2 + 52 > 40 line defined by the equation y = 2x + 13?
This point is OUTSIDE the circle. A. y + 2x = 13 B. 2x – y + 13 = 0
C: (0, -5)  (0 – 3)2 + (-5)2 < 40 C. y = x + 13 D. 2y = 2x + 13
This point is IN the circle.
D: (-1, 4)  (-1 – 3)2 + 42 < 40 Solution:
This point is IN the circle.
................................................ Two lines are coincidental when their equations
are equivalent to each other, or when their
263. Which of the following is parallel to the line slopes and y-intercepts are equal.
defined by the equation y = 3x – 4? A) y + 2x = 13  y = -2x + 13
A. y + 3x = 5 B) 2x – y + 13 = 0  y = 2x + 13
B. x + 3y = 6 C) y = x + 13  y = x + 13
C. y = x + 7 D) 2y = 2x + 13  y=x+
D. y = 3x + 8

This is a free reviewer. All rights reserved.


1000 MMR Author: Victor A. Tondo Jr., LPT
266. Which of the following equations pertain to Explanation:
a parabola?
In the form 4A(y – k) = (x – h)2, the length of the
A. y2 + 5y = x
latus rectum is 4A.
B. x2 + y2 + 3x – 4y = 0
( ) ( )
................................................
C. + =1
( ) ( )
D. =1
270. How far is the vertex from the directrix of
the parabola defined by 16y = x2?
Explanation: A. 16 B. 8 C. 4 D. 2
A parabola’s equation includes two variables,
one of which is squared. Explanation:
................................................ The distance of the vertex from the directrix is A
(from 4A, which is the length of the latus
267. Which of the following equations pertain to rectum). The distance of the vertex from the
a parabola that opens to the right? focus is also A. The vertex is between the focus
A. 4(y + 3) = (x – 2)2 and the directrix, which means from focus to
B. -3(y – 4) = (x + 5)2 directrix is 2A. The distance from focus to any
C. (y – 6)2 = 5(x + 1) endpoint of the latus rectum is also 2A.
D. (y + 8)2 = -2(x – 3) ................................................
Explanation: 271. Find the equation of the directrix of the
The parabola opens to the right when the parabola defined by (y – 2)2 = -4(x + 3).
squared variable is y and the coefficient of x is A. x = -2 B. x = 2
positive. It opens to the left when the squared C. y = -4 D. y = -2
variable is y and the coefficient of x is negative.
................................................ Explanation:
To find the directrix,
268. Which of the following equations pertain to first get the vertex
a parabola that opens downward? and the opening of
A. 4(y + 3) = (x – 2)2 the parabola. The
B. -3(y – 4) = (x + 5)2 vertex is at (-3, 2)
C. (y – 6)2 = 5(x + 1) and the parabola
D. (y + 8)2 = -2(x – 3) opens to the left,
which means the
Explanation: directrix is vertical.
The parabola opens upward when the squared The directrix is A units from the vertex. Since
variable is x and the coefficient of y is positive. It 4A = -4, then A = -1. The directrix is 1 unit to the
opens to the downward when the squared right of the vertex (-3, 2), meaning it passes
variable is x and the coefficient of y is negative. through (-2, 2). The vertical line passing through
................................................ (-2, 2) is x = -2. See attached image.
................................................
269. How long is the latus rectum of the parabola
defined by 12(y – 4) = (x + 3)2? 272. Find the coordinates of the focus of the
A. 12 B. 6 C. 4 D. 3 parabola defined by -12(y – 4) = (x + 5)2.
A. (-5, 7) B. (-5, 1)
C. (-8, 4) D. (-2, 4)

This is a free reviewer. All rights reserved.


1000 MMR Author: Victor A. Tondo Jr., LPT
Explanation: 276. Which of the following pertains to a circle
that is concentric with (x – 3)2 + y2 = 24?
The parabola opens downward, with vertex at
A. x2 + y2 – 6x + 3 = 0
(-5, 4). From the equation, 4A = -12, or A = -3.
B. x2 + y2 + 6y –15 = 0
This means the focus is 3 units downward from
C. x2 + y2 + 6x – 25 = 0
the vertex. Therefore, the vertex is at (-5, 1).
D. x2 + y2 + 6x + 6y – 24= 0
................................................

273. In which quadrant would G(3,-4) fall? Solution:


A. First quadrant B. Second quadrant
Concentric circles have the same center. That
C. Third quadrant D. Fourth quadrant
means the center we are looking for is (3, 0).
A. x2 + y2 – 6x + 3 = 0 Center: (3, 0)
Explanation: B. x2 + y2 + 6y –15 = 0 Center: (0, -3)
C. x + y + 6x – 25 = 0
2 2 Center: (-3, 0)
Quadrant Abscissa Ordinate
D. x2 + y2 + 6x + 6y – 24= 0Center: (-3, -3)
First Positive Positive
................................................
Second Negative Positive
Third Negative Negative 277. Which of the following is the equation of the
Fourth Positive Negative parabola that opens upward, whose latus rectum
................................................ is 12 units long, directrix is y = -3, and line of
symmetry is x = 7?
274. Find the distance between the parallel lines A. 12y2 = x – 7
y = 3x + 9 and y = 3x – 12. B. 12y = (x – 7)2
√ √
A. 7 B. 21 C. D. C. 12(y + 3) = (x – 7)2
D. 12(y – 3) = (x – 7)2
Solution:
Solution:
Use the distance between two parallel lines
Since the latus rectum is 12 units long, then
D=√
4A = 12, or A = 3. The parabola opens upward,
( ) √ which means the vertex must be 3 units above
D= √
= =

the directrix y = -3. Therefore, the ordinate of
................................................
the vertex (k) is equal to (-3) + 3 or 0. The line
of symmetry is x = 7, which means that the
275. Find the intersection of the lines y = 2x + 5
abscissa of the vertex (h) is also 7. The parabola
and y = -4x + 23.
opens upward, therefore the equation should be
A. (3, 10) B. (3, 11)
4A(y – k) = (x – h)2.
C. (4, 10) D. (4, 11)
................................................
Solution:
278. Convert rad to degrees.
Here we have a system of linear equations in two
A. 1.309o B. 75o
variables, namely x and y. Solve for x and y by
C. 150o D. 216o
using either substitution or elimination.
y = 2x + 5 y = -4x + 23 Solution:
2x + 5 = -4x + 23
π rad = 180o
6x = 18
x=3 Therefore rad = (180o) = 75o
................................................
y = 2x + 5 = 2(3) + 5 = 11
This is a free reviewer. All rights reserved.
1000 MMR Author: Victor A. Tondo Jr., LPT
279. Which of the following angles is coterminal Explanation:
with 143o?
The value of a trigonometric function of an angle
A. 217o B. -37o
equals the value of the cofunction of the
C. 323o D. 503o
complement of the angle. Cofunctions are sine
and COsine, tangent and COtangent, and secant
Explanation: and COsecant.
Coterminal angles share the same initial side and cos θ = sin (90 – θ) sec θ = csc (90 – θ)
terminal sides. Finding coterminal angles is done tan θ = cot (90 – θ) sin θ = cos (90 – θ)
by adding or subtracting multiples of 360 or 2π csc θ = sec (90 – θ) cot θ = tan (90 – θ)
rad to each angle, depending on whether the ................................................
given angle is in degrees or radians.
503o = 143o + (1)(360o). 283. The hypotenuse of a 30-60-90 triangle is 48
................................................ cm long. How long is its shortest side?
A. 24 cm B. 24 √2 cm
280. Which of the following is NOT a C. 24 √3 cm D. 16√3 cm
trigonometric identity?
A. sin2 θ + cos2 θ = 1 Explanation:
B. tan θ =
For any 30-60-90 triangle, the ratio of the sides
C. 1 + tan2 θ = sec2 θ are as follows:
D. 1 – cot2 θ = csc2 θ Short leg (opposite of the 30o angle) = n
Long leg (opposite of the 60o angle) = n√3
Explanation: Hypotenuse = 2n
The correct identity goes 1 + cot2 θ = csc2 θ. Since the hypotenuse 2n is equal to 48, then
................................................ n equals 24.
................................................
281. Which of the following is false?
A. tan θ = B. csc θ = 284. In a right triangle, the side opposite an
C. cos θ = D. sec θ = angle measuring 50o is 100 cm long. How long is
the side adjacent to the 50o angle?
A. 93.45 cm B. 83.91 cm
Explanation:
C. 149.14 cm D. 200 cm
Just use the SohCahToa – ChoShaCao mnemonic.
The reciprocal of sine is cosecant (csc), the Solution:
reciprocal of cosine is secant (sec), and the
The given are an angle and the sides opposite
reciprocal of tangent is cotangent (cot).
and adjacent to it. Use the tangent function.
................................................

282. Which of the following is NOT a cofunction tan 50o =


identity? tan 50o =
A. cos θ = sin (90 – θ)
B. cot (90 – θ) = tan θ x= = 83.91 cm
C. sec θ = csc (90 – θ)
D. csc θ = sin (90 – θ)

This is a free reviewer. All rights reserved.


1000 MMR Author: Victor A. Tondo Jr., LPT
285. A hundred cards are numbered 1 to 100. 289. A certain University has a dormitory. If 10
What is the probability of drawing a card whose students stay in a room, 24 students will not
number is divisible by seven? have a room. If 12 students stay in a room, there
A. B. C. D. will be 6 vacant beds. How many rooms are there
in the dormitory? How many students are
staying in the dormitory?
Solution:
A. 116 B. 115 C. 114 D. 113
There are 14 multiples of 7 from 1 to 100 since
100 ÷ 7 = 14 r 2. Solution:
Therefore the probability is or . Let n = the number of students
................................................ and x = the number of rooms

286. What is the probability of rolling a sum of n = 10x + 24 and n = 12x + 6


10 when rolling two dice? 10x + 24 = 12x + 6
A. B. C. D. 24 – 6 = 12x – 10x
18 = 2x
Solution: 9=x

The preferred outcomes are (4,6), (5,5), and Next, n = 10x + 24 = 10(9) + 24 = 114
(6,4). There are 36 possible outcomes. Therefore Therefore, there are 114 students.
the probability is 3/36 or 1/12. ................................................
................................................
290. Which of the following is not between
287. Factorize 3x2 + 5x – 2.
A. (3x – 1) (x + 2) B. (3x + 1) (x – 2) and ?
C. (3x – 2) (x + 1) D. (3x + 2) (x – 1) A. B. C. D.
................................................
Solution:
288. Six-sevenths of a number is 6 less than nine-
tenths of the same number. What is the number? = -0.4 and = -0.75.
A. 130 B. 140 C. 200 D. 210
A. = -0.8
Solution:
B. = -0.65
= 6 C. = -0.5
= D. = -0.6
10(6x) = 7(9x – 60)
60x = 63x – 420 Only A is beyond -0.4 to -0.75.
420 = 63x – 60x ................................................
420 = 3x
140 = x 291. Which value describes the position of C?

................................................
A. -0.75 B. -0.6 C. -1.25 D. -1.4

This is a free reviewer. All rights reserved.


1000 MMR Author: Victor A. Tondo Jr., LPT
292. Choose the correct value of (x + y)(x – y) 297. Subtract: (–2x2 + 5x – 9) – (2x – 7)
when x = 3.5 and y = –8.7 A. 2x2 + 3x + 16 B. -2x2 + 3x + 2
A. -63.44 B. 63.44 C. -2x + 3x – 2
2 D. -2x2 + 7x + 2
C. -148.84 D. 10.4 ................................................

Solution: 298. Evaluate the polynomial 4x2 – 6x – 3 if x = 2.


A. -1 B. 1 C. 3 D. 5
(x + y)(x – y) = (3.5 + (-8.7))(3.5 – (-8.7))
= (-5.2)(12.2)
Solution:
= -63.44
................................................ 4x2 – 6x – 3 when x = 2
= 4(2)2 – 6(2) – 3
293. Which two triangles are similar? =1
................................................

299. Determine the measure of Y and Z.

A. A and B B. A and C
C. B and D D. B and C

Explanation:
A. 45o, 55o B. 40o, 50o
B and D are isosceles right triangles, or 45-45-90 C. 48o, 48o D. 50o, 50o
triangles. Since they have congruent
corresponding angles, then they are similar. Since Y and Z intercept the same arc as X,
................................................ then they are congruent to X.
................................................
294. Determine which equation is equivalent to
4x – 1 = 11. 300. Which of the following is equal to x?
A. 4x = 10 B. 3x = 11
C. 4 – 1x = 11x D. 4x = 12
................................................

295. Determine the relation that matches the A. 15 sin 72o B. 15 sin 18o
table of values. C. 72 sin 72o D. 72 sin 18o
x 1 2 3 4 5
y 13 11 9 7 5 Solution:
A. y = 21 – x B. y = 15 – 2x
x is the adjacent side to the 72o angle and 15 is
C. y = 3x + 7 D. y = 2x + 11
the hypotenuse. The trigonometric function for
................................................
adjacent and hypotenuse is cosine. However,
none of the choices made use of cos 72o.
296. Determine which polynomial expression
Therefore we must use the other angle which
matches the algebra tile model.
measures 18o. Its opposite side is x and the
hypotenuse is still 15. Therefore,
A. 2x2 + x + 4 B. 3x2 + x + 4 sin 18o =
C. 3x2 – x + 4 D. 3x2 + x – 5 15 sin 18o = x

This is a free reviewer. All rights reserved.


1000 MMR Author: Victor A. Tondo Jr., LPT
301. Find the points of intersection of the graphs Solution:
of y = x2 and y = 3x – 2.
The last digit of the powers of 3 (the last digit of
A. (1, 1) and (1, 4) B. (1, 1) and (2, 4)
543) are 3, 9, 7, 1, 3, 9, 7, 1, 3, 9, 7, 1, …
C. (1, -1) and (2, 4) D. (-2, 4) and (1, 1)
The last digit repeats after four powers.
The last digit of 31, 35, 39, … is 3.
Solution:
The last digit of 32, 36, 310, … is 9.
By transitive property, we can say x2 = 3x – 2. The last digit of 33, 37, 311,… is 7.
The last digit of 34, 38, 312, … is 1.
x2 – 3x + 2 = 0
(x – 2)(x – 1) = 0 Therefore the units digit of 543444 is 1.
x = 1, 2 ................................................

When x = 1, then y = 12 = 1. 305. 4n+1 (4n+2) equals


When x = 2, then y = 22 = 4. A. 42n + 3 B. 82n + 3
C. 162n + 3 D. 4n + 3
Therefore, the points of intersection of the
graphs are (1,1) and (2,4).
Explanation:
................................................
Apply the laws of exponents.
302. An approximate value for ................................................
. .
is:
.
306. The greatest number of Fridays that can
A. 2 B. 20
occur in a 75 day period is:
C. 200 D. 2000
A. 10 B. 11 C. 12 D. 13
Explanation:
Explanation:
Ignore the decimals of the large numbers and use
Let the first day be Friday. The last Friday will be
only the first few decimals of 0.0403289925.
Using your calculator, 302476 x 0.04 ÷ 5962 2. on the 71st day. That’s a total of 11 Fridays.
................................................ ................................................

303. The graph below represents the motion of a 307. Which model is not a function?
car. The graph shows us that the car is:

A. B.
A. accelerating
B. standing still
C. travelling north-east
D. travelling at a constant speed
................................................ C. D.

304. The units digit of the number 543444 is: Explanation:


A. 3 B. 9 C. 7 D. 1 In a function, no x-value can ever have two or
more corresponding y-values.

This is a free reviewer. All rights reserved.


1000 MMR Author: Victor A. Tondo Jr., LPT
308. Which expression is equivalent to (9-2)8? A. between hours 3 and 4
A. -8132 B. -818 C. D. B. between hours 4 and 5
C. between hours 5 and 6
D. between hours 7 and 8
Solution:
................................................
(9-2)8 = ( ) = ( ( )) =
312. Each day of the month, Carl earns an
................................................
allowance, in cents, equal to the square of that
date of the month. Which is a number of cents
309. What is 5 × 10–4 written in standard
Carl could earn in a single day?
notation?
A. 21 B. 31 C. 64 D. 111
A. 0.00005 B. 0.0005
C. 5,000 D. 50,000
Explanation:
................................................
In the choices, only 64 is a perfect square.
310. What is the value of 54 × 5-6? ................................................
A. -25 B. C. D. 25
313. Which set of ordered pairs models a
function?
Solution:
A. {(2, 9), (7, 5), (3, 14), (2, 6)}
54 × 5-6 = 5-2 = B. {(5, 10), (5, 15), (5, 20), (5, 25)}
................................................ C. {(3, 10), (4, 15), (5, 20), (3, 25)}
D. {(–10, 20), (–20, 30), (–30, 40), (–40, 10)}
311. . Which comparison is true?
Explanation:
A. 4 < 180.5 < 4.5 B. 4.5 < 180.5 < 5
C. 8.5 < 180.5 < 9.5 D. 17 < 180.5 < 19 Only D has no repeated x-value.
................................................
Explanation:
180.5 = √18 4.24 314. Rayon has a piece of rectangular paper that
................................................ is 12 inches wide by 16 inches long. He drew a
straight line along the diagonal of the paper.
312. A weather station recorded the amount of What is the length of the line Rayon drew?
rain that fell during an 8-hour time frame using a A. √28 inches B. √192 inches
rain gauge. The findings are recorded in the C. 20 inches D. 28 inches
graph below.
Explanation:
Use the Pythagorean theorem.
................................................

315. Which equation has infinitely many


solutions?
A. 2x + 4 = 7x + 9
B. 3(2x + 5) = 6x + 15
C. 4x + 13 = 5x + (20 – x)
D. x + 3 = 5x – 21
Between which hours was the rate at which the
rain fell greater than the rate at which the rain
fell between hours 0 and 1?
This is a free reviewer. All rights reserved.
1000 MMR Author: Victor A. Tondo Jr., LPT
Explanation: 317. Which expression has a value of -2?
A. |2| + |-4| B. |-2| – |4|
A. 2x + 4 = 7x + 9
C. |4| – |-2| D. |-4| + |2|
4 – 9 = 5x – 2x
-5 = 3x
Explanation:
-5/3 = x  One unique solution
A. |2| + |-4| = 2 + 4 = 6
B. 3(2x + 5) = 6x + 15
B. |-2| – |4| = 2 – 4 = -2
6x + 15 = 6x + 15
C. |4| – |-2| = 4 – 2 = 2
0=0  True equation. This
D. |-4| + |2| = 4 + 2 = 6
means there are infinitely
................................................
many solutions.
C. 4x + 13 = 5x + (20 – x) 318. Reion is tossing a six-sided number cube
4x + 13 = 4x – 20 labeled 1, 2, 3, 4, 5, and 6. What is the probability
4x – 4x = -20 – 13 of tossing 6 twice in a row?
0 = -33  False equation. There is A. B. C. D.
no solution.
................................................
D. x + 3 = 5x – 21
3 + 21 = 5x – x 319. Which represents the value of x in
24 = 4x 6 – 4x 26?
6=x  One unique solution A. x -8 B. x -8
................................................ C. x -5 D. x -5
................................................
316. Praetor jogged on a path that was 2 miles
long, took a break, and then jogged back along 320. The table below shows the resting heart
the same path to where he started. He jogged at rates in beats per minutes of six students. The
different speeds for different distances along the rate, 40 beats per minute, seems to be an outlier.
path as shown in the graph. Which measure of central tendency changes the
least by dropping 40 from the data?
Heart
Rate
78 71 79 80 40 71
A. mean B. median
C. mode D. range

Explanation:
The original mean is 69.833. When 40 is
removed from the data, it becomes 75.8. The
mean decreases by 5.967.
Between which times did Praetor jog the fastest? The original median is 74.5. When 40 is removed
A. 0 minutes and 10 minutes from the data, it becomes 78. The median
B. 10 minutes and 25 minutes increases by -3.5.
C. 25 minutes and 30 minutes The original mode is 71. When 40 is removed
D. 30 minutes and 60 minutes from the data, it stays the same.
The original range is 40. When 40 is removed
Explanation: from the data, it becomes 9. The range decreases
by 31.
The steepest part of the graph is between 0 and
10 minutes.

This is a free reviewer. All rights reserved.


1000 MMR Author: Victor A. Tondo Jr., LPT
321. The sum of a number, n, and 5 is subtracted 2x - y = -1
from 8. Which expression represents this 2(-2) - y = -1
statement? -4 - y = -1
A. 8 – (n + 5) B. (n + 5) + 8 -4 + 1 = y
C. (n + 5) – 8 D. 8 + (n + 5) -3 = y
................................................ ................................................

322. How is 0.5600 written in scientific notation? 326. The lengths of two sides of a triangle are 8
A. 5.6 × 10 B. 5.6 × 10-1 inches and 13 inches. Which of the following
C. 5.6 × 10-2 D. 5.6 × 10-3 represents x, the possible length in inches of the
................................................ remaining side of the triangle?
A. 5 < x < 21
323. What is the value of x in 3(x – 4) = –21? B. 5 x 21
A. x = –11 B. x = –3 C. x < 5 or x > 21
C. x = 3 D. x = 11 D. x 5 or x 21

Solution: Explanation:
3(x – 4) = –21 The third side of any triangle must be between
x – 4 = -7 the difference and the sum of the two other
x = -7 + 4 = -3 sides.
................................................ ................................................

324. In the spinner, what is the probability of the 327. What is the value of the expression
arrow NOT landing on the space with the ∆? below?
80 ÷ ( 6 + (3 – 5) x 2)
A. -8 B. 8 C. 10 D. 40
................................................

328. Which of the following is closest to the


A. B. C. D. value of the expression below?
................................................ (20.0143642359)2 x 8π
A. 1,000 B. 10,000
325. Which values of x and y make the system of C. 100,000 D. 1,000,000
equations below true?
2x - y = -1 Explanation:
3x - y = -3 Round off and solve.
A. x = -4; y = -7 B. x =-2; y = -3 202 x 8 x 3.14 = 10,048
C. x = 2; y = 5 D. x = 4; y = 15 ................................................

Solution: 329. Which of the following expressions has a


value of 0?
2x - y = -1 A. (2 – 3) – (2 – 3) B. (2 – 3) – |2 – 3|
(-) 3x - y = -3 C. (2 – 3) + (-3 + 2) D. |2 – 3| – (2 – 3)
-x = 2 ................................................
x = -2
330. What is the factorization of 10x2 – x – 21?
A. (5x – 7) (2x + 3) B. (5x + 7) (2x – 3)
C. (5x + 3) (2x – 7) D. (5x – 3) (2x – 7)
This is a free reviewer. All rights reserved.
1000 MMR Author: Victor A. Tondo Jr., LPT
331. Evaluate: (√343)2 Solution:
A. 7√7 B. 49 C. 49√7 D. 343 The mth term of the expansion of (a + b)n is given
................................................ as nC(m-1) an-m+1bm-1.
7C4 (2x)4(3y)3 = 35 (16x4) (27y3)
332. Find the length of the latus rectum of the = 15120 x4y3
( ) ( )
ellipse defined by + = 1. ................................................
A. B. C. D.
336. The shell shape , as used in the Mayan
numeral system, is the symbol for which
Solution: number?
A. 100 B. 10 C. 1 D. 0
Length of latus rectum of ellipse =
Since the lower denominator is 16, then b = 4. Explanation:
Since the higher denominator is 25, then a = 5.
2 2(4 ) 32 The Mayan numerals consisted of only three
= = symbols: zero, represented as a shell shape; one,
5 5
................................................ a dot; and five, a bar.
................................................
333. Let A be a set such that A = {v, w, x, y, z}.
How many subsets does set A have? 337. Which of the following is irrational?
̅̅̅̅̅
A. 0.125 B. 43.29%
A. 5 B. 10 C. 25 D. 32
C. √200 D. √343
Explanation:
Explanation:
Let n = number of elements ̅̅̅̅̅ in fraction is
A. 0.125
Number of subsets = 2n
................................................ B. 43.29% in fraction is
C. √200 cannot be rewritten as a fraction with
334. Solve for x: whole numbers as numerator and denominator
2 (5x – 11) + 7 = 3 (x – 7) – 15 D. √343 is 7.
A. x = 3 B. x = 1 ................................................
C. x = -1 D. x = -3
338. Aira is six years older than Zayne. Six years
Solution: ago, she was twice as old as he. How old is Aira
2 (5x – 11) + 7 = 3 (x – 7) – 15 now?
10x – 22 + 7 = 3x – 21 – 15 A. 21 B. 18 C. 15 D. 12
10x –15 = 3x – 36
7x = -21 Solution:
x = -3 Let A = Aira’s present age
................................................ A – 6 = Zayne’s present age
335. What is the 4th term in the expansion of .: A – 6 = Aira’s age 6 years ago
(2x + 3y)7? A – 12 = Zayne’s age 6 years ago
A. 15120 x4y3 B. 7560 x4y3 A – 6 = 2 (A – 12)
C. 3780 x y
3 4 D. 1890 x3y4 A – 6 = 2A – 24
-6 + 24 = 2A – A
18 = A

This is a free reviewer. All rights reserved.


1000 MMR Author: Victor A. Tondo Jr., LPT
339. The two parallel sides of a trapezoidal lot 342. Rayon needed to find the perimeter of an
measure 100m and 70m. If these sides are 80m equilateral triangle whose sides measure x + 4
apart, what is the area of the lot? cm each. Jake realized that he could multiply
A. 13600 m2 B. 6800 m2 3 (x + 4) = 3x + 12 to find the total perimeter in
C. 3400 m 2 D. 2400 m2 terms of x. Which property did he use to
multiply?
Solution: A. Associative Property of Addition
( ) B. Distributive Property of Multiplication over
Area of Trapezoid = Addition
( ) ( ) C. Commutative Property of Multiplication
= (80)= 6800 D. Inverse Property of Addition
................................................ ................................................

340. If y = x and y = 2x + 2, find the value of x. 343. A ride in a Feak Taxi costs P25.00 for the
A. x = -2 B. x = -1 first km and P10.00 for each additional km.
C. x = 0 D. x = 1 Which of the following could be used to calculate
the total cost, y, of a ride that was x km?
Solution: A. y = 25x + 10
B. y = 10x + 25
Since y = x and y = 2x + 2, then by transitive
C. y = 25(x 1) + 10
property of equality,
D. y = 10(x 1) + 25
x = 2x + 2
................................................
-2 = 2x - x
-2 = x
344. Which of the following points is in the
................................................
fourth quadrant?
A. (3, 4) B. (-3, 4)
341. If the difference between the squares of two
C. (3, -4) D. (-3, -4)
consecutive counting numbers is 49, what is the
larger number?
Explanation:
A. 99 B. 49 C. 25 D. 7
A point in the fourth quadrant has a positive
Solution: abscissa (x-value) and a negative ordinate
(y-value).
Let the consecutive counting numbers
................................................
be x and x + 1.
(x + 1)2 – x2 = 49
345. The distance from the sun to the earth is
(x + 2x + 1) – x2 = 49
2
approximately 9.3 × 107 miles. What is this
2x + 1 = 49
distance expressed in standard notation?
2x = 48
A. 9,300,000,000 B. 930,000,000
x =24
C. 93,000,000 D. 651
.: The larger number is 24 + 1 = 25. ................................................
................................................
346. The square of a number added to 25 equals
10 times the number. What is the number?
A. -10 B. -5 C. 5 D. 10

This is a free reviewer. All rights reserved.


1000 MMR Author: Victor A. Tondo Jr., LPT
Solution: 350. Factorize: a2 – a – 90
x2 + 25 = 10x A. (a – 10) (a + 9)
x2 – 10x + 25 = 0 B. (a + 10) (a + 9)
√ 10 + 25 =√0 C. (a + 10) (a – 9)
x–5=0 D. (a – 10) (a – 9)
x=5 ................................................
................................................
351. Evaluate 10P5.
347. The sum of the square of a number and 12 A. 2 B. 100,000
times the number is 27. What is the smaller C. 1,024 D. 30,240
possible value of this number?
A. -9 B. -3 C. 3 D. 9 Solution:
! !
10P5 = ( = = 30,240
Solution: )! !

Let x = the number Or simply use your calculator to evaluate 10P5


x2 + 12x = -27 by using the nPr button.
x + 12x + 27 = 0
2 ................................................
(x + 9) (x + 3) = 0
x = -9 or -3 352. Jay bought twenty-five P4.57 stamps. How
................................................ much did he spend?
A. P 104.25 B. P 114.25
348. Let x = 1. Find the corresponding y given C. P 119.75 D. P124.25
that 2x 3y = 5.
A. y = -1 B. y = 1 Solution:
C. y = 3 D. y = -3 25 x 4.57 = 114.25
Solution: If you’re using a scientific calculator and it shows
2x 3y = 5 you , simply press then S D button.
2(1) – 3y = 5 ................................................
2 – 3y = 5
2 – 5 = 3y
353. Given f(x) = x3 + kx2 – 7, find k if f(2) = 41.
-3 = 3y
A. 5 B. 10 C. 15 D. 20
-1 = y
................................................
Solution:
349. The sum of two consecutive even integers is Just substitute x with 2.
126. What is the smaller integer?
f(2) = 23 + k(22) – 7
A. 63 B. 62 B. 61 D. 60
41 = 8 + 4k – 7
41 = 4k + 1
Solution:
40 = 4k
Let x = smaller even integer 10 = k
.: x + 2 = next even integer ................................................
x + (x + 2) = 126
354. If y = x and y = 2x + 2, find x + y.
2x + 2 = 126
A. -8 B. -4 C. 0 D. 4
2x = 124
x = 62
This is a free reviewer. All rights reserved.
1000 MMR Author: Victor A. Tondo Jr., LPT
Solution: 358. The cost of renting a bike at the local bike
shop can be represented by the equation
Since y = x and y = 2x + 2, then by transitive
y = 2x + 2, where y is the total cost and x is the
property of equality,
number of hours the bike is rented. Which of the
x = 2x + 2 following ordered pairs would be possible
-2 = 2x - x number of hours rented and the corresponding
-2 = x .: y = -2 as well total cost?
A. (0, 2) B. (2, 6)
.: x + y = -2 + (-2) = -4
C. (6, 2) D. ( 2, 6)
................................................
Explanation:
355. Mulan and Lilo are competing to see who
can sell the most candy bars for a fundraiser. Simply substitute the x- and y-values of the point
Mulan sold 4 candy bars on the first day and 2 into the equation y = 2x + 2. Whichever point
each day after that. Lilo sold 7 on the first day holds a true equation is the correct answer.
and 1 each day after that. On what day will they ................................................
have the same number of candy bars sold?
A. 7th B. 6th C. 4th D. 3rd 359. The distance from the earth to the moon is
approximately 240,000 miles.
Solution: What is this distance expressed in scientific
notation?
Let x = number of days to pass after the 1st day
A. 24 × 104 B. 2.4 × 104
4 + 2x = 7 + x C. 2.4 × 105 D. 2.4 × 10 5
2x – x = 7 – 4 ................................................
x=3
They will have the same number of candy bars 360. =
sold 3 days after the first day, or on the 4th day.
................................................ A. B. C. D.

356. Which of the following is not a polynomial?


Solution:
A. -3x2 + x–9 B. √2x + π ( )( )
=
√ ( )( )
C. D. 7 +9
................................................

Explanation: 361. The sum of two angles is 180°. The measure


In a polynomial, x cannot be raised to exponents of one angle is 34° greater than the measure of
that are fractions or irrational numbers. the other angle. What is the measure of the
................................................ smaller angle?
A. 74° B. 73° C. 72° D. 71°
357. Factorize: 3p2 – 2p – 5
A. (3p – 5) (p + 1) B. (3p + 5) (p – 1) Solution:
C. (3p + 1) (p – 5) D. (3p – 1) (p + 5)
................................................ Let x = measure of the smaller angle
.: x + 34 = measure of the larger angle
x + (x + 34) = 180
2x + 34 = 180
2x = 146
x = 73
This is a free reviewer. All rights reserved.
1000 MMR Author: Victor A. Tondo Jr., LPT
362. Solve the following system of linear 365. Samantha owns a rectangular field that has
equations: an area of 3,280 square meters. The length of the
3x + y = -9 field is 2 more than twice the width. What is the
3x 2y = 12 width of the field?
A. 40 m B. 41 m
A. ( 2, 3) B. (2, 3)
C. 82 m D. 84 m
C. (3, 2) D. ( 3, 2)
Solution:
Solution:
Let w = width of the field
3x + y = -9
.: 2w + 2 = length of the field
(+) 3x 2y = 12
–y = 3
w (2w + 2) = 3280
y = -3
2w2 + 2w = 3280  divide equation by 2
w2 + w = 1640  complete the square
Using the first equation 3x + y = 9,
w + w + 0.25 = 1640 + 0.25
2

3x + (-3) = -9 w2 + w + 0.25 = 1640.25


3x = -6 √ + + 0.25 = √1640.25
x = -2 w + 0.5 = 40.5
................................................ w = 40
................................................
363. Find the x-intercept of 3x + 2y = 24
A. x = 8 B. x = -8 366. Rayon used the following mathematical
C. y = 12 D. y = -12 statement to show he could change an
expression and still get the same answer on both
Solution: sides:
To find the x-intercept, let y = 0. 10 × (6 × 5) = (10 × 6) × 5
Which mathematical property did Rayon use?
3x + 2y = 24  3x + 2(0) = 24
3x = 24 A. Identity Property of Multiplication
x=8 B. Commutative Property of Multiplication
................................................ C. Distributive Property of Multiplication over
Addition
364. A circle is drawn such that ̅̅̅̅ is a diameter D. Associative Property of Multiplication
and its midpoint is O. Given that C is a point on ................................................
the circle, what is the measure of ACB?
A. 180o B. 90o 367. Factorize x3 – 27y3.
C. 60o D. not enough info A. (x – 3y) (x2 – 3x + 9y2)
B. (x – 3y) (x2 + 3x + 9y2)
Explanation: C. (x + 3y) (x2 + 3x – 9 y2)
D. (x + 3y) (x2 – 3x + 9 y2)
................................................
Any inscribed angle that opens
to a diameter or a semicircle is 368. What is the intersection of the lines
a right angle, or measures 90o. x + 3y = 5 and -2x + 4y = 0?
A. (2, 4) B. (-2, 1)
C. (2, 1) D. (-2, 4)

This is a free reviewer. All rights reserved.


1000 MMR Author: Victor A. Tondo Jr., LPT
Solution: 372. A researcher is curious about the IQ of
students at the Utrecht University. The entire
2(x + 3y = 5)  2x + 6y = 10
group of students is an example of a:
-2x + 4y = 0  (+) -2x + 4y = 0
A. parameter B. statistic
10y = 10
C. population D. sample
y=1
x + 3y = 5  x + 3(1) = 5 Explanation:
x+3=5 Entirety is population, part of it is sample.
x=2 ................................................
................................................
373. Jordan filled a bottle with grains until it was
369. Given f(x) = 7x3 – 3x2 + 2x – 9, f(2) = 1/4 full and weighed 8 kg. He added more grains
A. 56 B. 48 C. 44 D. 39 into the bottle until it was 7/8 full. It now
weighed 18 kg. What is the mass of the empty
Solution: bottle?
Simply substitute x with 2. A. 16 B. 8 C. 4 D. 2
f(x) = 7x3 – 3x2 + 2x – 9 Solution:
f(2) = 7(23) – 3(22) + 2(2) – 9
f(2) = 56 – 12 + 4 – 9 Let B = weight of empty bottle,
f(2) = 39 G = weight of capacity of bottle
................................................
Eqn 1: B + G = 8
370. Ten factorial is equal to _____. Eqn 2: B + G = 18
A. 100
B. e10 Subtract Eqn 1 from Eqn 2:
C. 10 x 9 x 8 x 7 x 6 x 5 x 4 x 3 x 2 x 1
D. 10e B + G = 18
................................................ (–) B+ G=8
___________________________________________________

371. How many 3-digit numbers can be made G = 10


using the digits 5, 6, 7, 8, 9, and 0 if repetition is
5G = 80
not allowed?
G = 16
A. 80 B. 100 C. 120 D. 140
Solving for B,
Solution:
B + ¼ (16) = 8
There are only five possible choices for the
B + 4 = 8; B=4
hundreds digit since we cannot start with 0.
................................................
There are five choices for the tens since out of
the six, we have used one for the hundreds, and
374. If 37 – 4x < 17, then
we can use 0 for the tens digit.
A. x < 5 B. x > 5
We have used two for the hundreds and tens, so
C. x < -5 D. x > -5
we only have four choices for the units digit.
5 x 5 x 4 = 100

This is a free reviewer. All rights reserved.


1000 MMR Author: Victor A. Tondo Jr., LPT
Solution: Solution:
Substitute x and y with -2 and 3 respectively.
37 – 4x < 17
–4x < 17 – 37 ax + by = 17  -2a + 3b = 17
–4x < –20 2ax by = 11  -4a – 3b = -11
-¼ (–4x) < -¼ (–20) *
Solve for a and b by elimination.
x>5
-2(-2a + 3b = 17)  4a – 6b = -34
*Remember to change your inequality sign after
-4a – 3b = -11  -4a – 3b = -11
multiplying or dividing the inequality by a
negative number. 4a – 6b = -34
................................................ (+) -4a – 3b = -11
-9b = -45
375. How many solutions are there for the b=5
following system of linear equations?
Using -2a + 3b = 17 and substituting b = 5,
-3x + 5y = 6
6x 10y = 0 -2a + 3(5) = 17
-2a + 15 = 17
A. only one solution
-2a = 17 – 15
B. two solutions
-2a = 2
C. infinitely many solutions
a = -1
D. no solution
................................................
Explanation:
377. What is the probability choosing only one
Try to solve for x and y first. vowel when three letters are randomly selected
from the word NUMBERS?
-2 (-3x + 5y = 6)  6x 10y = -12
6x 10y = 0  6x 10y = 0 A. B. C. D.

6x 10y = -12
Solution:
(–) 6x 10y = 0
0 = -12 which is false. NUMBERS has two vowels and five consonants.
The number of ways we can get only one vowel
When you get a false equation, it means there is
out of three is taken as 2C1 x 5C2, or 20.
no solution for the given system of linear
The number of ways we can get three letters
equations.
from seven is 7C3 or 35.
Also, when there’s no solution for a system of Therefore, the answer is or .
linear equations, the lines defined by the two ................................................
equations are parallel – they will never intersect.
................................................ 378. If A > B, which is always true?
376. Find a and b so that the system below has A.
the unique solution (-2, 3). B. A2 > B2
ax + by = 17 C. A < B + 2
2ax by = 11 D. A – B > 0
A. a = 3, b = -1 B. a = 1, b = 5
C. a = 1, b = 3 D. a = 1, b = 5

This is a free reviewer. All rights reserved.


1000 MMR Author: Victor A. Tondo Jr., LPT
Explanation: 382. A teacher asks students to identity their
favorite reality television show. What type of
Use counter-examples to disprove the choices.
measurement scale do the different television
shows make up?
A. A. Nominal B. Ordinal
When A = 2 and B = 1, is false. C. Interval D. Ratio
................................................
B. A2 > B2
When A = 2 and B = -3, then 22 > (-3)2 is false. 383. What is the center of the circle defined by
C. A < B + 2 x2 + y2 – 8x + 6y – 10 = 0?
When A = 1 and B = 0, then 1 < 0 + 2 is false. A. (-8, 6) B. (8, -6)
C. (-4, 3) D. (4, -3)
D. A – B > 0
This will always be true for any A > B. Explanation:
................................................
Since the equation of the circle is already in the
379. Statistical techniques that summarize and form x2 + y2 + Dx + Ey + F = 0, then the center
( )
organize the data are classified as what? (h, k) is at ( , ) or ( , ) or (4, -3).
A. Population statistics ................................................
B. Sample statistics
C. Descriptive statistics 384. Find the equation of the line passing (1, 4)
D. Inferential statistics with slope equal to 5.
................................................ A. y = 5x + 3 B. y = 5x + 1
C. y = 5x – 1 D. y = 5x – 3
380. Five-point Likert scales (strongly disagree,
disagree, neutral, agree, strongly agree) are Solution:
frequently used to measure motivations and
attitudes. A Likert scale is a: Use the form y = mx + b and substitute the x-
A. Discrete variable. and y- values of the point.
B. Ordinal variable. x = 1, y = 4, m = 5
C. Categorical variable. y = mx + b
D. All of the above 4 = 5(1) + b
................................................ 4–5=b
-1 = b
381. What is the radius of the circle defined by y = mx + b
(x + 2)2 + (y – 3)2 = 16? y = 5x + (-1) or y = 5x – 1
A. 256 B. 16 C. 8 D. 4 ................................................

Explanation: 385. The seminar rooms in the library are


identified by the letters A to H. A researcher
The equation of the circle is already in the records the number of classes held in each room
center-radius form. during the first semester. What kind of graph
Since r2 = 16, then r = 4. would be appropriate to present the frequency
................................................ distributions of these data?
A. Histogram B. Scatterplot
C. Bar chart D. Box plot

This is a free reviewer. All rights reserved.


1000 MMR Author: Victor A. Tondo Jr., LPT
386. Find the slope of the line passing the points Solution:
A(2, 3) and B(-7, -15).
To convert from degrees to radians, simply
A. 1 B. -1 C. ½ D. 2
multiply the given degree measure by .
Solution: 48o x = =
Use the formula m = ................................................

m= = 391. The median is always:


A. The most frequently occurring score in set of
m=2 data
................................................ B. The middle score when results are ranked in
order of magnitude
387. Factorize: 3n2 – 8n + 4 C. The same as the average
A. (3n – 2) (n – 2) D. The difference between the maximum and
B. (3n – 2) (n + 2) minimum scores.
C. (3n + 2) (n + 2) ................................................
D. (3n + 2) (n – 2)
................................................ 392. A teacher gave a statistics test to a class of
Geography students and computed the measures
388. In now many ways can the letters of central tendency for the test scores. Which of
AAABBCDEEE be arranged in a straight line? the following statements cannot be an accurate
A. 50,400 B. 25,200 description of the scores?
C. 12,600 D. 6,300 A. The majority of students had scores above the
mean.
Solution: B. The majority of students had scores above the
! 10! median.
= = 50,400
! ! ! ! 3! 2! 3! C. The majority of students had scores above the
................................................ mode.
D. All of the above options (A, B and C) are false
389. Find the smaller angle formed by the x-axis statements.
and the line y = 5x.
A. 78.69o B. 63.48o Explanation:
C. 54.15o D. 41.32o
A. If majority of the students had scores above
Solution: the mean, then this is an example of a negatively-
skewed data. This can happen.
tan =5 B. 50% of the class is always above the median,
= tan-1 5 and the other 50% of the class is below the
= 78.69o median.
................................................ C. The mode can pop up anywhere. That means
the students may have a low modal score.
390. Convert 48o to radians. ................................................
A. π rad B. π rad
C. π rad D. π rad 393. Find the area of a semicircle whose radius
measures 28 cm.
A. 784 π cm2 B. 392 π cm2
C. 28 π cm 2 D. 14 π cm2

This is a free reviewer. All rights reserved.


1000 MMR Author: Victor A. Tondo Jr., LPT
Solution: 398. In how many ways can 4 girls and 5 boys be
arranged in a row so that all the four girls are
A = ½ π r2
together?
A = ½ π (28)2
A. 4,320 B. 8,640
A = 392 π cm2
C. 17,280 D. 34,560
................................................
Solution:
394. Find the length of each side of an equilateral
triangle whose perimeter is 90 cm. Let 4 girls be one unit. So now, there are 6 units
A. 45 cm B. 30 cm in all: 5 boys and the solo unit made by the four
C. 22.5 cm D. 10 cm girls. They can be arranged in 6! ways.
In each of these arrangements 4 girls can be
Solution: arranged in 4! ways.
Total number of arrangements in which girls
An equilateral triangle has three congruent sides.
are always together
Its perimeter is given as P = 3s.
= 6! × 4!
3s = 90 = 720 × 24
s = 30 = 17,280
................................................ ................................................

395. Find the number of subsets having 4 399. A box contains 8 batteries, 5 of which are
elements of the set {1,2,3,4,5,6,7,8,9,10,11}. good and the other 3 are defective. Two batteries
A. 165 B. 330 are selected at random and inserted into a toy. If
C. 660 D. 1320 the toy only functions with two good batteries,
what is the probability that the toy will function?
Solution: A. B. C. D.
11C4 = 330
................................................ Solution:

396. There are ten true - false questions in an =


exam. How many responses are possible? ................................................
A. 1024 B. 256 C. 20 D. 10
400. IQ tests are standardized so that the mean
Solution: score is 100 for the entire group of people who
take the test. However, if you select a group of 50
There are two possible responses per item, and who took the test, you probably would not get
there are ten items. 210 = 1024. 100. What statistical concept explains the
................................................ difference between the two means?
A. Statistical error B. Inferential error
397. In a 500m speed skating race, time results C. Residual error D. Sampling error
would be considered an example of which level ................................................
of measurement?
A. Nominal B. Ordinal 401. Which Mathematician pioneered the study
C. Interval D. Ratio of conic sections?
................................................ A. Euclid B. Apollonius
C. Archimedes D. Hipparchus

This is a free reviewer. All rights reserved.


1000 MMR Author: Victor A. Tondo Jr., LPT
Explanation: 406. Who is considered by many Mathematicians
as “The Last Universalist”?
Apollonius of Perga was a Greek geometer and
A. Jules Henri Poincare
astronomer known for his theories on the topic
B. Hendrik Lorentz
of conic sections. His definitions of the terms
C. Georg Cantor
ellipse, parabola, and hyperbola are the ones in
D. Gottfried Wilhelm Leibniz
use today.
................................................
Explanation:
402. A researcher studies the factors that Both Poincare and Leibniz are polymaths, but
determine the number of children future couples Jules Henri Poincare is fondly referred to by
decide to have. The variable ‘number of children’ fellow Mathematicians as the Last Universalist.
is a: ................................................
A. Discrete variable
B. Continuous variable 407. In the theory he developed, there are
C. Categorical variable infinite sets of different sizes (called
D. Ordinal variable cardinalities). Which Mathematician formalized
................................................ many ideas related to infinity and infinite sets
during the late 19th and early 20th centuries?
403. Surface area and volume, center of gravity, A. Jules Henri Poincare
and hydrostatics are some of the studies of B. Hendrik Lorentz
which Mathematician? C. Georg Cantor
A. Apollonius B. Euclid D. Gottfried Wilhelm Leibniz
C. Archimedes D. Hipparchus ................................................
................................................
408. Which of the following sets of scores has the
404. The book Philosophiæ Naturalis Principia greatest variability or range?
Mathematica, more fondly known simply as A. 2, 5, 8, 11 B. 13, 13, 13, 13
Principia, is the work of which Mathematician? C. 20, 25, 26 ,27 D. 42, 43, 44, 45
A. Euclid B. Newton
C. Einstein D. Archimedes Explanation:
................................................
Range = highest score – lowest score
................................................
405. A researcher is interested in the travel time
of Rayon’s University students to college. A
409. This Mathematician was the first to describe
group of 50 students is interviewed. Their mean
a pinwheel calculator in 1685 and invented the
travel time is 16.7 minutes. For this study, the
wheel named in his honor, which was used in the
mean of 16.7 minutes is an example of a
arithmometer, the first mass-produced
A. parameter B. statistic
mechanical calculator. He also refined the binary
C. population D. sample
number system, which is the foundation of all
digital computers. Which Mathematician is this,
Explanation:
who is also crucial to the development of
The 50 students in the above case is only part of computers?
the population of students of Rayon’s University, A. Gottfried Wilhelm Leibniz
thus, they are only a sample. Data taken from a B. Charles Babbage
sample is called statistic. C. Ada Lovelace
D. Alexander Graham Bell

This is a free reviewer. All rights reserved.


1000 MMR Author: Victor A. Tondo Jr., LPT
410. Solve for x, given 9x – 10 = 11x + 30 Solution:
A. x = 40 B. x = 20
C. x = -20 D. x = -40

Solution:
9x – 10 = 11x + 30
-10 – 30 = 11x – 9x
-40 = 2x
................................................

411. Using Calculus, this Mathematician


explained why tides occur, why the shapes of 415. When five is added to three more than a
planetary orbits are conic sections, and how to certain number, the result is 29. What is the
get the shape of a rotating body of fluid, among number?
many other things. Which Mathematician is this? A. 24 B. 21 C. 8 D. 4
A. Kepler B. Euclid
C. Apollonius D. Newton Solution:

Explanation: (n + 3) + 5 = 29
n + 8 = 29
Newton explained the above-mentioned topics in n = 21
his work Principia. ................................................
................................................
416. The math club is electing new officers.
412. Which of the following terms does NOT There are 3 candidates for president, 4
describe the number 9? candidates for vice-president, 4 candidates for
A. rational number B. integer secretary, and 2 candidates for treasurer. How
C. real number D. prime number many different combinations of officers are
possible?
Explanation: A. 13 B. 96
Its factors are 1, 3, and 9. Therefore, 9 is a C. 480 D. 17,160
composite number.
................................................ Solution:
Apply the fundamental counting principle.
413. Which expression below is equal to 5? 3 x 4 x 4 x 2 = 96
A. (1 + 2)2 B. 9 – 22 ................................................
C. 11 10 × 5 D. 45 ÷ 3 × 3
................................................ 417. Twelve points lie on a circle. How many
cyclic quadrilaterals can be drawn by using these
414. A bus picks up a group of tourists at a hotel. points? [Note: Cyclic quadrilaterals are
The sightseeing bus travels 2 blocks north, 2 quadrilaterals whose vertices are on a circle.]
blocks east, 1 block south, 2 blocks east, and 1 A. 48 B. 495
block south. Where is the bus in relation to the C. 11,880 D. 1,663,200
hotel?
A. 2 blocks north B. 1 block west Solution:
C. 3 blocks south D. 4 blocks east
12C4 = 495

This is a free reviewer. All rights reserved.


1000 MMR Author: Victor A. Tondo Jr., LPT
418. What is the variance for the following set of 422. The sum of five consecutive integers is 215.
scores? 143 143 143 143 143 143 What is the largest of these integers?
A. 0 B. 2 C. 4 D. 25 A. 43 B. 44 C. 45 D. 46

Explanation: Solution:
The scores are all the same, so the variance is 0. Let x = smallest number
................................................ .: The numbers will be expressed as x, x + 1,
x + 2, x + 3, and x + 4.
419. When 18 is subtracted from six times a
x + (x + 1) + (x + 2) + (x + 3) + (x + 4) = 215
certain number, the result is 42. What is the
5x + 10 = 215
number?
5x = 205
A. 10 B. 4 C. -4 D. -10
x = 41
Solution: .: The numbers are 41, 42, 43, 44, and 45.
................................................
6x – 18 = -42
6x = -42 + 18
423. You go to the cafeteria for lunch and have a
6x = -24
choice of 4 entrees, 5 sides, 5 drinks, and 4
x = -4
desserts. Assuming you have one of each
................................................
category, how many different lunches could be
made?
420. Find the equation of the line passing (2, 3)
A. 18 B. 81 C. 40 D. 400
and (-7, -15)
A. y = 2x + 1 B. y = 2x – 1
Solution:
C. y = x + 2 D. y = x – 2
Apply the fundamental counting principle.
Solution: 4 x 5 x 5 x 4 = 400
................................................
Use the two-point form.
y – y1 = (x x ) 424. What can be said about the following
y–3= (x 2) statements?
y – 3 = 2(x – 2) i. Any quadrilateral with four congruent sides is
y – 3 = 2x – 4 a square.
y = 2x – 1 ii. Any square has four congruent sides.
................................................ A. Only the first statement is true.
B. Only the second statement is true.
421. Of the following Z-score values, which one C. Both statements are true.
represents the location closest to the mean? D. Both statements are fall.
A. Z = +0.5 B. Z = +1.0
C. Z = -1.5 D. Z = -0.3 Explanation:
i. Any quadrilateral with four congruent sides is
Explanation: a RHOMBUS.
ii. Any square has four congruent sides. TRUE.
A lower absolute value of Z-score means the item ................................................
is closer to the mean. The Z-score with the
lowest absolute value is D, Z = -0.3.
................................................

This is a free reviewer. All rights reserved.


1000 MMR Author: Victor A. Tondo Jr., LPT
425. Out of 6 boys and 4 girls, a committee of 5 429. The second angle of a triangle is three times
has to be formed. In how many ways can this be as large as the first. The measure of the third
done if we take 2 girls and 3 boys? angle is 40 degrees greater than that of the first
A. 120 B. 186 C. 240 D. 256 angle. How large is the first angle?
A. 28o B. 30o C. 35o D. 38o
Solution:
Solution:
6C3 x 4C2 = 20 x 6 = 120
................................................ Let x = measure of the first angle
.: 3x = measure of the second angle
426. In the figure, which of the following will x + 40 = measure of the third angle
yield the value of the hypotenuse x?
x + 3x + x + 40 = 180
5x + 40 = 180
5x = 180 – 40 = 140
x = 28
................................................
A. x = B. x =
C. x = D. x = 10 tan 35o 430. Normally distributed data are normally
referred to as:
A. Bell-shaped B. Asymmetrical
Solution:
C. Skewed D. Peaked
The side measuring 10 units is opposite the ................................................
given angle, while x is the hypotenuse. The
trigonometric ratio that involves the opposite 431. A population has a mean of μ=35 and a
side and the hypotenuse is the sine function. standard deviation of σ=5. After 3 points are
From the mnemonic SohCahToa, we can recall added to every score of the population, what are
that sin = . the new values for the mean and standard
deviation?
sin 35o = A. μ=35 and σ=5
x sin 35o = 10 B. μ=35 and σ=8
x= C. μ=38 and σ=5
D. μ=38 and σ=8
................................................
Explanation:
427. The shortest side of a 30-60-90 triangle is
20.19 cm long. How long is the hypotenuse? When a constant is added or subtracted to every
A. 40.38 cm B. 30.29 cm score of a population, the mean increases or
C. 34.97 cm D. 17.48 cm decreases by the same amount. The standard
deviation, however, stays the same.
Explanation: ................................................
In a 30-60-90 triangle, the hypotenuse is always
twice the shortest side. 432. Given sin θ = , find cos θ.
................................................ A. B. C. D.
428. The hypotenuse of a 30-60-90 triangle is
34.96 cm long. How long is the shortest side?
A. 40.38 cm B. 30.29 cm
C. 34.97 cm D. 17.48 cm

This is a free reviewer. All rights reserved.


1000 MMR Author: Victor A. Tondo Jr., LPT
Solution: 435. Given cos θ = and θ QIV, find tan θ.
Since sin θ = , then θ = sin-1 . A. B. C. D.
Therefore, cos θ = cos (sin-1 )= .
Solution:
Alternative Solution: Since cos θ = and cos θ = , then
Draw a right triangle containing the angle θ. opposite = √13 5 = √144 = 12.
Mark the side opposite θ as 7 and the
hypotenuse as 25 since sin θ = . Since tan θ = , adjacent = 5, and
opposite = 12, by using the CAST mnemonic,
then tan θ = since θ QIV.
................................................

436. Find the measure of T:


Solve for the missing leg by using the
Pythagorean theorem. You will get the measure
of the adjacent side as √25 7 , or √625 49
which is equal to 24.

A. 59o B. 69o C. 79o D. 89o


................................................

437. If the scores on a test have a mean of 26 and


a standard deviation of 4, what is the z-score for
Now that we know the measure of the adjacent a score of 18?
A. -1.41 B. 11 C. -2 D. 2
side, we can substitute cos θ = = .
................................................ Solution:

433. In a triangle, what is located 2/3 of the Z-score =


distance from each vertex to the midpoint of the
opposite side? Z-score = = = 2
A. centroid B. incenter ................................................
C. circumcenter D. orthocenter 438. If a researcher sets a level of significance at
................................................ 0.05 (i.e. 5%), what does this mean?
A. Five times out of 100, a significant result will
434. Which triangle has the centroid, incenter, be found that is due to chance alone and not to
circumcenter, orthocenter, and nine-point-center true relationship.
at the same location? B. Ninety-five times out of 100, a significant
A. isosceles right triangle result will be found that is due to chance alone
B. 30-60-90 triangle and not to true relationship.
C. equilateral triangle C. Five times out of 100, a significant result will
D. hyperbolic triangle be found that is not due to chance, but to true
relationship.
D. None of the above

This is a free reviewer. All rights reserved.


1000 MMR Author: Victor A. Tondo Jr., LPT
Explanation: 443. Although rarely used in proving, what is the
The level of significance is also known as margin extra line or line segment drawn in a figure to
for error. help in a proof?
................................................ A. base line B. auxiliary line
C. converse line D. Euler’s line
439. When does a researcher risk a Type I error? ................................................
A. Anytime the decision is ‘fail to reject’.
B. Anytime H0 is rejected. 444. What is the measure of V in the following
C. Anytime Ha is rejected. figure?
D. All of the above options

Explanation:
Type I errors happen when we reject a true null
hypothesis.
Type II errors happen when we fail to reject a
false null hypothesis A. 60o B. 65o C. 70o D. 75o
................................................
Solution:
440. Solve for x:
90 + 86 + 114 + x = 360
290 + x = 360
x = 70
................................................

445. What is the intersection of all three


altitudes of a triangle?
A. incenter B. centroid
A. 4 B. 5 C. 6 D. 7
C. orthocenter D. circumcenter
................................................
Solution:
90 + 90 + 98 + 17x – 3 = 360 446. How much water must be evaporated from
275 + 17x = 360 2000 mL of 30% acid solution to make a 50%
17x = 85 acid solution?
x=5 A. 800 mL B. 850 mL
................................................ C. 900 mL D. 950 mL

441. Which of the following is equidistant from Solution:


the vertices of the triangle? Let x = amount of water to be evaporated
A. circumcenter B. orthocenter
C. incenter D. centroid C1V1 + C2V2 = CrVr
................................................
30%(2000) + 0% (-x) = 50%(2000 – x)
600 + 0 = 1000 – 0.5 x
442. Which of the following is equidistant from 0.5 x = 1000 – 600
the sides of the triangle?
0.5 x = 400
A. circumcenter B. centroid x = 800
C. orthocenter D. incenter
Note: Water is 0% acid, and evaporation means
removal of volume thus –x.

This is a free reviewer. All rights reserved.


1000 MMR Author: Victor A. Tondo Jr., LPT
447. Which of the following is the intersection of 452. In which geometry are there no parallel
angle bisectors of a triangle? lines?
A. circumcenter B. incenter A. elliptic geometry
C. centroid D. orthocenter B. hyperbolic geometry
................................................ C. spatial geometry
D. solid geometry
448. In terms of a conditional statement, what is ................................................
the statement formed by exchanging and
negating the antecedent and the consequent? 453. What do we call the ratio of two numbers
A. inverse B. converse (larger number: smaller number) whose ratio to
C. adverse D. contrapositive each other is equal to the ratio of their sum to
the larger number? [Note: This is applied in
Explanation: Fibonacci sequences]
A. pi B. golden ratio
Statement If p, then q.
C. 1.618 D. Euler’s ratio
Converse If q, then p.
................................................
Inverse If not p, then not q.
Contrapositive If not q, then not p. 454. Which of the following pertains to the law of
................................................ cosines?
A. c2 = a2 + b2 – 2 ab cos C
449. What is formed when the hypothesis and B. c2 = a2 + b2 + 2 ab cos C
the conclusion of the conditional statement are C. c2 = a2 + b2 – ab cos C
interchanged? D. c2 = a2 + b2 + ab cos C
A. converse B. inverse ................................................
C. adverse D. contrapositive
................................................ 455. Solve for x:
450. What is formed when both the hypothesis
and the conclusion of the conditional statement
are negated?
A. converse B. inverse
C. adverse D. contrapositive
................................................
A. x = 8 B. x = 9
C. x = 10 D. x = 11
451. Which of the following is the converse of the
................................................
following statement?
“If two angles are congruent,
456. What do we call an angle formed by two
then they have the same measure.”
chords of the circle with a common endpoint
A. If two angles are not congruent, then they do
(the vertex of the angle)?
not have the same measure.
A. inscribed angle
B. If two angles have the same measure, then
B. tangential angle
they are congruent.
C. circumscribed angle
C. If two angles do not have the same measure,
D. interior angle
then they are not congruent.
................................................
D. If two angles are not congruent, then they
have the same measure.”

This is a free reviewer. All rights reserved.


1000 MMR Author: Victor A. Tondo Jr., LPT
457. Find the inverse of y = . 462. Which of the following expressions will give
the value of x?
A. y-1 = B. y-1 =
C. y-1 = D. y-1 =

Solution:
First, solve for x in terms of y. A. 50 tan 37o B. 50 cos 37o
y= C. 50 sin 37o D. 50 cot 37o
2020 y = 2018 x + 2019 Solution:
2020 y – 2019 = 2018 x
=x tan 37o =
50 tan 37o = x
Then switch y and x. ................................................
=y
463. Solve for x:
Lastly, replace y with y-1.
= y-1.
................................................

458. Which lines are not in the same plane and


do not intersect but are not parallel?
A. asymptotes B. tangent lines
C. skew lines D. directrices A. 6 B. 7 C. 7.5 D. 8
................................................
Solution:
459. Two adjacent angles whose distinct sides lie 94 + 86 + 94 + 11x – 2 = 360
on the same line are called what? 272 + 11x = 360
A. linear pair B. vertical pair 11x = 360 – 72 = 88
C. alternate D. corresponding x=8
................................................ ................................................

460. The point of concurrency of a triangle’s 464. A bus travels 600 km in 7 hrs and another
three altitudes is called _____. 300 km in 5 hrs. What is its average speed?
A. circumcenter B. incenter A. 72.86 kph B. 75 kph
C. orthocenter D. centroid C. 77.86 kph D. 80 kph
................................................
Solution:
461. What do we call three positive integers with Average speed = total distance / total time
the property that the sum of the squares of two Average speed = (600 + 300) / (7 + 5)
of the integers equals the square of the third? Average speed = 900/12 = 75 kph
A. Euclid’s triple ................................................
B. Pythagorean triple
C. Newton’s triple
D. Cartesian triple

This is a free reviewer. All rights reserved.


1000 MMR Author: Victor A. Tondo Jr., LPT
465. A sniper on a cliff observes that the angle of Solution:
depression to his target is 30o. If the cliff is 10
This problem is on permutation with repeated
meters high, how far must the bullet travel to hit
elements. PILIPINAS has nine letters, of which
the sniper’s target?
two are P, three are I, and the other letters are
A. 20 meters B. 10√3 meters singular.
C. 10√2 meters D. 10 meters 9!
= 30,240
2! 3!
Solution: ................................................

468. A coin is tossed 60 times. Head appeared 27


times. Find the experimental probability of
getting heads.
A. B. C. D.

Let trajectory = x Explanation:


sin 30o = The experimental probability of an event is the
0.5 = ratio of the number of times the event occurs to
the total number of trials.
x = . = 20
................................................
................................................
469. A parabola is defined by the equation
466. Rowena received a total of 25 bills. These 5x = -3y2 – 4y + 2. Which of the following is true
bills are either P20 or P50 bills. If Rowena about the parabola?
received an amount of P800, how many P20 bills A. It opens to the left.
did she receive? B. It opens to the right.
A. 10 B. 13 C. 15 D. 17 C. It opens upward.
D. It opens downward.
Solution:
Let x = number of P20 bills Explanation:
25 – x = number of P50 bills The squared variable is y and the coefficient of y2
20(x) + 50(25 – x) = 800 is negative. Therefore it opens to the left.
20x + 1250 – 50x = 800 ................................................
1250 – 800 = 50x – 20x
450 = 30x 470. Find the equation of the circle whose center
15 = x is at (7, -24) given that it passes the point of
................................................ origin.
A. (x + 7)2 – (y – 24)2 = 961
467. How many ways can the word PILIPINAS be B. (x – 7)2 + (y + 24)2 = 961
rearranged? C. (x + 7)2 + (y – 24)2 = 625
A. 302 D. (x – 7)2 + (y + 24)2 = 625
B. 3,024
C. 30,240 Explanation:
D. 302,400 Use the center-radius form. The radius is the
distance from the center (7, -24) and the point of
origin (0, 0).

This is a free reviewer. All rights reserved.


1000 MMR Author: Victor A. Tondo Jr., LPT
471. Ana left their house and jogged at a speed of 474. The shortest leg of a 30-60-90 triangle is
60 meters per minute. Bea followed her two 123 cm long. How long is the leg adjacent to the
minutes later and jogged at a speed of 70 meters 30o angle?
per minute. How many minutes after Bea left A. 123 cm B. 123√2 cm
would she catch up with Ana? C. 123√3 cm D. 246 cm
A. 14 B. 13.5 C. 13 D. 12
Explanation:
Solution:
The leg adjacent to the 30o angle in a 30-60-90
Let x = number of minutes after Bea left to catch triangle is always √3 times the length of the
up with Ana shortest leg.
60 (x + 2) = 70 (x) ................................................
60x + 120 = 70x
120 = 70x – 60x 475. How many odd 4digit numbers can be
120 = 10x formed using the digits 7, 6, 5, 4, 3, 2, and 1 if
12 = x repetition is not allowed?
................................................ A. 720 B. 480 C. 240 D. 120
472. How many mL of 40% acid must be added Solution:
to 1000 mL of 10% acid solution to make a 20%
acid solution? 6 x 5 x 4 x 4 = 480
A. 250 B. 500 C. 600 D. 750 ................................................

Solution: 476. There are 70 dogs and geese in a farm. If


there are a total of 200 legs, how many dogs are
Let x = amount to be added in mL there?
40 (x) + 10 (1000) = 20 (x + 1000) A. 25 B. 30 C. 35 D. 40
40x + 10,000 = 20x + 20,000
40x – 20x = 20,000 – 10,000 Solution:
20x = 10,000 Leg D = number of dogs
x = 500 G = number of geese
................................................
D + G = 70  2D + 2G = 140
473. The hypotenuse of a 30-60-90 triangle is 4D + 2G = 200  4D + 2G = 200
432 cm long. How long is the leg opposite the 30o -2D = -60
angle? D = 30
A. 216 cm B. 216 √2 cm ................................................
C. 216 √3 cm D. 432 cm
477. Find the maximum area of a rectangle if the
Explanation: perimeter is set at 350 cm.
A. 8656.25 cm2 B. 7656.25 cm2
The shorter leg in a 30-60-90 triangle is always C. 6656.25 cm 2 D. 5656.25 cm2
half the length of the hypotenuse.
................................................ Shortcut:
Make it a square.
P = 4S = 350
S = 87.5 .: A = S2 = 87.52
A = 7656.25

This is a free reviewer. All rights reserved.


1000 MMR Author: Victor A. Tondo Jr., LPT
478. A rectangle is 60 cm long and 45 cm wide. Explanation:
How long is its diagonal?
This problem is on circular permutations so use
A. 75 cm B. 85 cm
(n – 1)!.
C. 95 cm D. 105 cm
................................................
Solution:
483. There are 24 mangoes in a basket, of which
The diagonal D is given as √ + . 7 are rotten. What is the probability that when
randomly getting two mangoes at the same time,
= √45 + 60 = √2025 + 3600 both are rotten?
D = √5625 = 75 A. B. C. D.
................................................
Solution:
479. Find the length of the diagonal of a cube
given each side measures 17 cm. =
A. √290 cm B. 17√2 cm ................................................
C. 17√3 cm D. 34 cm
484. In a gathering of gamers and admins, there
Solution: are 24 gamers of which 6 are females, and 3
admins of which one is female. If a female is
In a cube, the diagonal D is √3 times the measure
randomly called, what is the probability that she
of each side.
is an admin?
................................................
A. B. C. D.
480. Find the measure of each interior angle of a
regular 20-sided polygon. Explanation:
A. 162o B. 150o
There are only seven females, of which one is an
C. 144o D. 126o
admin.
................................................
Explanation:
MIA = 180 - where n = number of sides of 485. What is the remainder when 3x6+ 4x5 – 5x4
+ 6x3 + 7x2 – 8x + 3 is divided by (x – 1)?
regular polygon
A. 8 B. 9 C. 10 D. 11
................................................
Solution:
481. How many diagonals does a regular 14-
sided polygon have? Use the Remainder Theorem:
A. 77 B. 91 C. 96 D. 101
3(1)6+ 4(1)5 – 5(1)4 + 6(1)3 + 7(1)2 – 8(1) + 3
= 3 + 4 – 5 + 6 + 7 – 8 + 3 = 10
Solution:
................................................
( ) ( )
D= = = 77
486. Seven people have an average weight of 49
................................................
kg. A child was added to the group and the
average became 45 kg. How heavy is the child?
482. How many ways can 14 people be seated in
A. 15 kg B. 16 kg
a Ferris wheel given that each cart can only
C. 17 kg D. 18 kg
contain one person?
A. 15! B. 14! C. 13! D. 12!

This is a free reviewer. All rights reserved.


1000 MMR Author: Victor A. Tondo Jr., LPT
Solution: Solution:
Let N = child’s weight The diameter of the sphere is of the same length
( ) as each side of the cube. That means the
45 =
diameter of the sphere is 20 cm, and
45 = subsequently, the radius is 10 cm.
360 = 343 + N SA = 4 r2
17 = N SA = 4 (102) = 400 cm2
................................................ ................................................

487. Six numbers have an average of 71. If 85 is 491. Given f(x) = (25 x20 – 24x10)(x2 – 9x + 3),
added to the group, what is the new average? find f ’(x).
A. 72 B. 73 C. 74 D. 75 A. f ‘(x) = 550 x21 – 4725 x20 + 1500 x19 – 288 x11
+ 2376 x10 – 720 x9
Solution: B. f ‘(x) = 450 x21 – 4725 x20 + 150 x19 + 288 x11
+ 2376 x10 – 720 x9
Sum of the first six numbers: 6(71) = 426
C. f ‘(x) = 550 x21 – 4725 x20 + 150 x19 – 288 x11
New average = = = 73 + 2376 x10 – 720 x9
................................................ D. f ‘(x) = 450 x21 – 4725 x20 + 1500 x19 – 288 x11
+ 2376 x10 – 720 x9
488. In an arithmetic sequence, the 7th term is 25
and the 10th term is 67. What is the common Solution:
difference?
A. 42 B. 21 C. 14 D. 7 Use the product rule.
Let u = (25 x20 – 24x10)
Solution: and v = (x2 – 9x + 3)
D= = = = 14 .: du = 500 x19 – 240 x9
................................................ dv = 2x – 9
f ‘(x) = u dv + v du
489. Triangle ABC has sides measuring 20 cm, 20 = (25 x20 – 24x10) (2x – 9) + (x2 – 9x + 3)
cm, and 29 cm. What kind of triangle is ABC? (500 x19 – 240 x9)
A. acute B. right = 50 x21 – 48 x11 – 225 x20 + 216 x10 +
C. obtuse D. reflex 500 x21 – 4500 x20 + 1500 x19 – 240 x11 +
2160 x10 – 720 x9
Explanation: = 550 x21 – 4725 x20 + 1500 x19 – 288 x11
For any triangle with sides X, Y, and Z, given + 2376 x10 – 720 x9
that X Y Z, if X2 + Y2 > Z2, then the triangle is
obtuse. Alternative Solution:
................................................ f(x) = (25 x20 – 24x10)(x2 – 9x + 3)
= 25x22 – 225x21 + 75x20 – 24x12 + 216 x11
490. Find the surface area of a sphere given that – 72 x10
the sphere sits perfectly inside a cube whose f ‘(x) = 550 x21 – 4725 x20 + 1500 x19 – 288 x11
sides measure 20 cm each. + 2376 x10 – 720 x9
A. 400 cm2 B. 800 cm2 ................................................
C. 1200 cm2 D. 2400 cm2

This is a free reviewer. All rights reserved.


1000 MMR Author: Victor A. Tondo Jr., LPT
492. Simplify: eln 2019 x Solution:
A. 2019x B. 2019 x
7C2 x 6C2 = 21 x 15 = 315
C. x D. ................................................

Explanation: 496. Find the measure of the smaller angle


formed by the hands of the clock at 11:20.
Remember that eln u = u.
A. 130o B. 135o
In the question, u = 2019 x.
C. 140o D. 145o
Therefore, eln 2019 x = 2019 x
................................................
Solution:
493. If the roots of a quadratic equation are A = 30h - m = 30(11) - (20)
and , which of the following could be the = 330 – 110
= 220
quadratic equation?
A. 63x2 + 22x – 21 = 0 Since the angle taken from our formula is a reflex
B. 63x2 – 22x – 21 = 0 angle and we are looking for the smaller angle,
C. 63x2 + 22x + 21 = 0 then 360 – 220 = 140.
D. 63x2 – 22x + 21 = 0 ................................................

Solution: 497. How many even 3-digit even numbers can


be formed using the digits 7, 6, 5, 4, 3, 2, 1, and 0
x= x= if repetition is not allowed?
9x = -7 7x = 3 A. 150 B. 160 C. 170 D. 180
9x + 7 = 0 7x – 3 = 0
(9x + 7) (7x – 3) = 0 Solution:
63x2 + 49x – 27x – 21 = 0 Number of 3-digit numbers:
63x2 + 22x – 21 = 0 7 x 7 x 6 = 294
................................................ Number of 3-digit odd numbers:
6 x 6 x 4 = 144
494. If three more than twice a number is .: Number of 3-digit even numbers:
seventeen less than seven times the number, 294 – 144 = 150
what is the number? ................................................
A. 2 B. 3 C. 4 D. 5
498. There are 50 students in a class. Twenty of
Solution: them have a laptop. Thirty-two of them have a
2x + 3 = 7x – 17 smartphone. Seven of them have both a laptop
3 + 17 = 7x – 2x and a smartphone. How many of them have
20 = 5x neither a laptop nor a smartphone?
4=x A. 4 B. 5 C. 6 D. 7
................................................
Solution:
495. A team is to be made from a group of seven 50 – (20 + 32 – 7) = 50 – 45 = 5
teachers and six scientists. If the team is to be
composed two teachers and two scientists, how
many different ways can they form a team?
A. 325 B. 315 C. 300 D. 285

This is a free reviewer. All rights reserved.


1000 MMR Author: Victor A. Tondo Jr., LPT
499. Mocha can finish a job in 24 hours, while
her sister Tiramisu can do the same job in only What is the difference between
20 hours. How long will it take them to finish the a function and a relation?
job by working together? “Sa function, loyal si x.
A. hrs B. hrs In a relation, kiber
kung ilang beses nagamit si x.”
C. 11 hrs D. hrs

Solution:
( )
= = = hrs What are gametes?
................................................ Gametes are sex cells.
Ako nga, ilang beses na akong
500. What conic figure does the equation na-gamete eh.
x2 + y2 + 10x – 16y = -100 form?
A. Real circle B. Degenerate circle
C. Imaginary circle D. Ellipse
There are only 10 kinds of people:
Solution: Those who know binary
And those who don’t.
Use CTS (completing trinomial squares) to
convert the equation to its center-radius form.
x2 + y2 + 10x – 16y = -100
x2 + 10x + y2 – 16y = -100 Bakit daw nagalit ang mga Pinoy
(x2 + 10x + 25) + (y2 – 16y + 64) = -100 + 25 + nung namatay si Rizal?
64 Kasi wala siyang pang-buyback.
(x + 5)2 + (y – 8)2 = -11

Since r2 = -11, then the radius is imaginary,


making it an imaginary circle. Mabilis ako kumuha ng square root
ng malalaking numbers.
Ayaw mo maniwala?
Akin na cellphone number mo,
kukunin ko square root.
End of first 500 items.

Mr. Lazada bought 200 cars.


Please, take a break. OMG ang yaman ni Mr. Lazada!

Remember: Nabutas ang isang water tank.


Nasa Diyos ang awa, Normies: OMG! Sayang ang tubig!
Nasa tao ang gawa. Math major: *computes the rate
at which the tank is being emptied*

This is a free reviewer. All rights reserved.


1000 MMR Author: Victor A. Tondo Jr., LPT
501. Victor, Praetor, and Rowena volunteered to 504. What value of x will satisfy the equation:
teach at a nearby daycare. Praetor worked for 0.25(20x – 2020) = x?
twice as long as Rowena did. Victor worked A. 125.25 B. 125.75
twice as many hours as Praetor. Altogether, they C. 126.25 D. 126.5
worked for 56 hours. For how many hours did
Victor work? Solution:
A. 14 B. 16 C. 28 D. 32
0.25(20x – 2020) = x
5x – 505 = x
Solution:
5x – x = 505
Rowena = n hours 4x = 505
.: Praetor = 2n hours x = 156.25
.: Victor = 2(2n) hours ................................................
n + 2n + 4n = 56
505. Three brothers inherited a cash amount of
7n = 56
P5,670,000 and they divided it among
n=8
themselves in the ratio of 2:3:4. How much more
.: Victor worked for 2(2(8)) or 32 hours. is the largest share than the smallest share?
................................................ A. P1,260,000 B. P1,270,000
C. P630,000 D. P635,000
502. Which of the following is the factorization of
the binomial x4 – 20194? Solution:
A. (x + 2019)2 (x – 2019)2
Let the three numbers 2x, 3x, and 4x so that the
B. (x – 2019)4
ratio will still be 2:3:4.
C. (x2 + 20192) (x + 2019) (x – 2019)
D. (x – 4)(x + 4)
2x + 3x + 4x = 5,670,000
9x = 5,670,000
Solution:
x = 630,000
x4 – 20194 as a difference of squares
Difference: 4x – 2x = 2x
= (x2)2 – (20192)2
2x = 2(630,000) = 1,260,000
= (x2 + 20192) (x2 – 20192)
................................................
= (x2 + 20192) (x + 2019) (x – 2019)
................................................
506. Rayon and Wena can do a job together in
four hours. Working alone, Rayon does the job in
503. The average of 5 different counting
six hours. How long will it take Wena to do the
numbers is 143. What is the highest possible
job alone?
value that one of the numbers can have?
A. 12 hours B. 11 hours
A. 706 B. 705
C. 10 hours D. 9 hours
C. 704 D. 703
Solution:
Solution:
Formula for “working together”: AB/(A+B)
The 5 different counting numbers will assume
the values of 1, 2, 3, 4, and N. Since the average is AB/(A+B) = 4; but A = 6
143, the sum is 5(143) or 715. 6B/(6+B) = 4
1+2+3+4+N = 715 6B = 24 + 4B
10 + N = 715 2B = 24
N = 705 B = 12

This is a free reviewer. All rights reserved.


1000 MMR Author: Victor A. Tondo Jr., LPT
507. What is the sum of the first 2019 counting Solution:
numbers? Age 5 years
A. 2,021,019 B. 2,027,190 Age Now
from now
C. 2,039,190 D. 2,043,190 Mr. Park x + 20 x + 20 + 5
Son x x+5
Solution:
x + 20 + 5 = 2(x + 5) – 5
Sum of the first N counting numbers = x + 25 = 2x + 5
Sum of the first 2019 counting numbers: 25 – 5 = 2x – x
20 = x
= = 2,039,190 ................................................
................................................
510. What is the minimum value of
508. In a certain school, the ratio of boys to girls f(x) = 3x2 + 6x + 11?
is 4 is to 9. If there are 260 boys and girls in the A. 1 B. 2 C. 4 D. 8
school, how many boys are there?
A. 100 B. 90 Solution:
C. 85 D. 80
Minimum Value = c – b2/4a
Solution: 11 – 36/12 or 11 – 3 = 8
................................................
Let 4x = boys, 9x = girls
4x + 9x = 13x = 260 511. What is the sum of the roots of
.: x = 20 5x2 + 10x – 17?
Number of boys = 4x = 4(20) = 80 A. B. C. 2 D. -2
................................................
Solution:
509. Solve for x: 8x + 9y = 17
9x + 10y = 18 Sum of roots = = = -2
................................................
A. x = 9 B. x = 8
C. x = -9 D. x = -8
512. If xy = 17 and x2 + y2 = 135, find x + y.
A. 13 B. 12.8749
Solution:
C. 12.5249 D. 12.5
10 (8x + 9y = 17)  80x + 90y = 170
9 (9x + 10y = 18)  (-) 81x + 90y = 162 Solution:
-x =8
x2 + 2xy +y2 = x2 + y2 + 2xy
x = -8
x2 + 2xy +y2 = 133 + 2(17)
................................................
x2 + 2xy +y2 = 169
x + y = 13
510. Mr. Park is 20 years older than his son now.
................................................
Five years from now, the Mr. Park’s age will be 5
less than twice his son’s age. Find the son’s
513. How many mL of 50% acid solution must be
present age.
added to 200mL of 10% acid solution to make a
A. 18 B. 20
40% acid solution?
C. 38 D. 40
A. 400 mL B. 500 mL
C. 600 mL D. 700 mL

This is a free reviewer. All rights reserved.


1000 MMR Author: Victor A. Tondo Jr., LPT
Solution: 517. If A and B are the roots of x2 + 19x + 20,
what is AB?
C1 V1 + C2 V2 = CR VR
50(x) + 10(200) = 40(x + 200) A. 20 B. 2√5 + 5
50x + 2000 = 40x + 8000 C. 3√2 + 2√3 D. -19
50x – 40x = 8000 – 2000
10x = 6000 Explanation:
x = 600 Since A and B are the roots, then AB pertains to
................................................ the product of the roots which is given as c/a.
................................................
514. Nine consecutive even numbers have a sum
of 180. What is the smallest of these even 518. 0.25 + 0.5 + 1 + 2 + 4 + … + 1024 = ____
numbers? A. 2046.75 B. 2047.75
A. 18 B. 16 C. 2048.75 D. 2049.75
C. 14 D. 12
Solution:
Solution:
You may use the Geometric Series formula which
Let x = lowest even number
is ∑ = ( ), where r is the common
x + (x+2) + (x+4) + (x+6) + (x+8) + (x+10) ratio, n is the number of terms, and a1 is the first
+ (x+12) + (x+14) + (x+16) = 180 term.
9x + 72 = 180
9x = 108 Alternative Solution:
x = 12
................................................ Use the shortcut for the sum of a geometric
sequence wherein the ratio is 2 or ½, which is
515. If x = 13, which of the following is equal to SUM = 2(largest) – smallest.
200? In this problem, that’s 2(1024) – 0.25 = 2047.75.
A. 15x + 2 B. x2 + 2x + 5 ................................................
C. x – 4x – 2
3 D. x2 + x + 2
519. How many terms are there in the sequence
Explanation: 2, 9, 16, 23, …, 345?
A. 40 B. 45
Just substitute x with 13. C. 50 D. 70
A. 15(13) + 2 = 197
B. (13)2 + 2(13) + 5 = 200 Solution:
C. (13)3 – 4(13) – 2 = 2143 An = A1 + (n – 1)d
D. (13)2 + (13) + 2 = 184 345 = 2 + (n – 1)(7)
................................................ 345 – 2 = 7(n – 1)
343 = 7n – 7
516. For which value of k does 9x2 – kx + 25 350 = 7n
have only one root? 50 = n
A. 3.75 B. 7.5 C. 15 D. 30 ................................................
Explanation: 520. If 2x + 3 = 25, then what is (2x + 3)2 – 25?
b2 – 4ac = 0 when there’s only one root. A. 650 B. 625 C. 600 D. 575
k2 – 4(9)(25) = 0
k2 = 900
k = 30
This is a free reviewer. All rights reserved.
1000 MMR Author: Victor A. Tondo Jr., LPT
Solution: Solution:
(2x + 3)2 – 25 = 252 – 25 = 625 – 25 = 600 C1 V1 + C2 V2 = CR VR
................................................ 10 (V) + 40 (500) = 30 (V + 500)
10V + 20,000 = 30V + 15,000
521. What is 50% of 200% of 2019? 20,000 – 15,000 = 30V – 10V
A. 1009.5 B. 2019 5,000 = 20V
C. 4038 D. 8076 250 = V
................................................
Solution:
525. 3log2 3 + 2 log2 5 – log2 7 = ______.
(0.5)(2)(2019) = 2019
................................................ A. log2 B. log2
C. log2 D. log2
522. If 3x = 5 and 2y = 11, what is 12(x – y)?
A. -46 B. -45 C. 45 D. 46 Explanation:
Solution: Apply the laws of logarithms.
( )( )
12(x – y) = 12x – 12y 3log2 3 + 2 log2 5 – log2 7 = log2 = log2
= 4(3x) – 6(2y) ................................................
= 4(5) – 6(11)
= 20 – 66 526. Find the intersection of y = 3x + 4 and
= -46 y = 5x – 8.
................................................ A. (8/5, 0) B. (-4/3, 0)
C. (1, 7) D. (6, 22)
523. If two numbers have a product of 267 and
the sum of their squares is 250, what is their Solution:
sum? y = 3x + 4
A. -30√3 B. 28 - y = 5x – 8
C. 7√3 + 2√5 D. 30 0 = -2x + 12
2x = 12
Solution: x=6

Let A and B be our two numbers. y = 3x + 4


AB = 267; A2 + B2 = 250 = 3(6) + 4
= 22
.: A2 + B2 + 2AB = 250 + 2(267) ................................................
= 784
(A + B)2 = 784; 527. Find the remainder when
A + B = ±28 x5 – 3x3 + 5x2 – 7x + 9 is divided by (x – 1).
................................................ A. 11 B. 9 C. 7 D. 5

524. How many ml of 10% acid must be added to Solution:


500 ml of 40% acid to make a 30% acid solution?
A. 1000 ml B. 750 ml Use the Remainder Theorem.
C. 500 ml D. 250 ml 14 – 3(13) + 5(12) – 7(1) + 9 = r
1–3+5–7+9=r
5=r

This is a free reviewer. All rights reserved.


1000 MMR Author: Victor A. Tondo Jr., LPT
528. Multiply: (3x – 7) (5x + 9) Solution:
A. 15x2 – 8x – 63 B. 15x2 – 8x + 63
Let x = number of P5 coins
C. 15x + 8x – 63
2 D. 15x2 + 8x + 63
.: x+7 = number of P10 coins
5(x) + 10(x+7) = 475
Solution:
5x + 10x + 70 = 475
3x -7
15x + 70 = 475
times 5x +9
15x = 405
27x -63
x = 27
15x2 -35x
15x2 -8x -63 .: He has twenty-seven P5 coins.
................................................ ................................................

529. Multiply: (2x2 – 5x + 3) (3x + 4) 532. When a number is increased by 5, its square
A. 6x3 – 7x2 – 11x + 12 also increases by 255. What is this number?
B. 6x3 + 7x2 + 11x + 12 A. 22 B. 23 C. 24 D. 25
C. 6x3 – 7x2 + 11x + 12
D. 6x3 + 7x2 – 11x + 12 Solution:
(x+)2 – x2 = 255
Solution:
x2 + 10x + 25 – x2 = 255
2x2 -5x +3 10x + 25 = 255
times 3x +4 10x = 230
8x 2 -20x +12 x = 23
6x3 -15x2 +9x ................................................
6x3 -7x2 -11x +12
................................................ 533. Find k such that 16x2 + kx + 81 is a perfect
square trinomial.
530. x varies directly as y and inversely as z. If A. 144 B. 72 C. 36 D. 18
x = 40 when y = 8 and z = 2, what is x when
y = 24 and z = 4? Solution:
A. 120 B. 90 C. 60 D. 30 16x2 + kx + 81 = (4x)2 + 2(4x)(9) + 92
= (4x)2 + 72x + 92
Solution: ................................................
x = ky/z 40 = k(8)/2
40 = 4k 534. Solve for x: (x + 4)2 = (x – 6)2.
10 = k A. x = 0 B. x = ½
x = 10y/z C. x = 1 D. no solution
x = 10(24)/4
x = 240/4 Solution:
x = 60 (x + 4)2 = (x – 6)2
................................................ x2 + 8x + 16 = x2 – 12x + 36
8x + 12x = 36 - 16
531. Pedro has 7 more P10 coins than P5 coins. If 20x = 20
he has a total of P475, how many P5 coins does x=1
he have? ................................................
A. 23 B. 25 C. 27 D. 29

This is a free reviewer. All rights reserved.


1000 MMR Author: Victor A. Tondo Jr., LPT
535. When a number is increased by 3, its square 538. Find the remainder when the polynomial
increases by 135. By what does its square x4 – 5x3 + 7x2 – 11x + 19 is divided by (x – 2).
increase when the number is increased by 5? A. 5 B. 3 C. 1 D. -1
A. 225 B. 235 C. 445 D. 485
Solution:
Solution:
Use the Remainder Theorem.
Let’s find the original number first. Our divisor is (x – 2), so a = 2.
(x+3)2 – x2 = 135
f(2) = 24 – 5(23) + 7(22) – 11(2) + 19
x + 6x + 9 – x2 = 135
2
f(2) = 16 – 40 + 28 – 22 + 19 = 1
6x + 9 = 135
................................................
6x = 126
x = 21
539. If three-fourths of a number is 44 more than
212 = 441; (21+5)2 = 262= 676 its one-fifth, what is that number?
676 – 441 = 235 A. 120 B. 100 C.80 D. 60
................................................
Solution:
536. Ten cans of soda and six hamburgers cost a
total of P440. Five cans of soda and seven x= x + 44
hamburgers cost a total of P360. How much is a 20 ( x = x + 44)  15x = 4x + 880
hamburger?
11x = 880
A. P31 B. P33 C. P35 D. P37
x = 80
................................................
Solution:
Let C = price of a can of soda 540. Rowena can do a job in 15 hours, while
B = price of a hamburger Victor can do the same job in 25 hours. How long
will it take them to finish the job by working
10C + 6B = 440  10C + 6B = 440
together?
2(5C + 7B) = 360  (-) 10C + 14B = 720
A. 9.375 hours B. 9.25 hours
-8B = -280
C. 9.125 hours D. 8.875 hours
B = 35
................................................
Solution:
( )
537. The product of two consecutive odd = = = 9.375
numbers is 1763. What is the smaller number? ................................................
A. 31 B. 37 C. 41 D. 47
541. Armel is twice as heavy as Kuku. Gabbi is
Solution: 17kg heavier than Kuku. The sum of their masses
is 217kg. How heavy is Kuku in kg?
Let x = smaller number;
A. 40 B. 45 C. 50 D. 55
x + 2 = larger number
Solution:
x (x+2) = 1763
x2 + 2x = 1763 A = 2K G = K + 17
x + 2x + 1 = 1764
2
A + G + K = 217
√x + 2x + 1 = √1764 2K + (K + 17) + K = 217
x+1 = 42 4K + 17 = 217
x = 41 4K = 200
K = 50
This is a free reviewer. All rights reserved.
1000 MMR Author: Victor A. Tondo Jr., LPT
542. Find + given x + y = 30 and xy = 219. Solution:

A. B. C. D. 13x + 17 = 15x – 21
17 + 21 = 15x – 13x
38 = 2x
Solution:
19 = x
+ = + = = = ................................................
................................................
546. Factorize: x3 + 3x2 – 4x – 12
A. (x + 2) (x + 3) (x – 2)
543. The product of two consecutive odd
B. (x + 4) (x + 3) (x – 1)
counting numbers is 1023. What is their sum?
C. (x + 2) (x – 3) (x + 2)
A. 60 B. 64 C. 68 D. 72
D. (x + 4) (x – 3) (x + 1)
Solution:
Solution:
Let x = first number; x+2 = next number
Factorize by grouping.
x(x+2) = 1023
x3 + 3x2 – 4x – 12
x2 + 2x = 1023
= (x3 + 3x2) – (4x + 12)
x2 + 2x + 1 = 1023 + 1
= x2 (x + 3) – 4 (x + 3)
√ + 2x + 1 = √1024 = (x2 – 4) (x + 3)
x+1 = 32 = (x + 2) (x – 2) (x + 3)
Therefore x = 31 and x+2 = 33 ................................................
The numbers are 31 and 33, so their sum is 64.
................................................ 547. Factorize (x4 – 16) completely.
A. (x – 2)4
544. There are 250 pigs and chickens in a farm, B. (x – 2)2 (x + 2)2
all of which are healthy. If there are 720 legs in C. (x + 2)2 (x – 2)2
total, how many pigs are there? D. (x + 2) (x – 2) (x2 + 4)
A. 110 B. 100 C. 90 D. 80
Solution:
Solution: (x4 – 16) = (x2 – 4) (x2 + 4)
Let P = number of pigs = (x + 2) (x – 2) (x2 + 4)
C = number of chickens ................................................
P + C = 250
4P + 2C = 720  since pigs have four legs 548. √125 + √45 √245 = ____
and chickens have two A. 6√5 B. 4√5
2(P + C = 250)  2P + 2C = 500 C. 2√5 D. √5
4P + 2C = 720 - 4P + 2C = 720
-2P = -220 Solution:
P = 110
................................................ √125 + √45 √245 = 5√5 + 3√5 7√5 = √5
................................................
545. Solve for x given 13x + 17 = 15x – 21.
A. x = 17 B. x = 18 549. The product of two consecutive even
C. x = 19 D. x = 20 counting numbers is 3968. Find the smaller
number.
A. 42 B. 46 C. 54 D. 62

This is a free reviewer. All rights reserved.


1000 MMR Author: Victor A. Tondo Jr., LPT
Solution: Solution:
x(x+2) = 3968 Seesaw problems call for inverse or indirect
x2 + 2x = 3968 proportion.
x + 2x + 1 = 3968
2 48(160) = 60N
√ + 2 + 1 = √3969 7680 = 60N
x + 1 = 63 128 = N
x = 62 ................................................
................................................
553. Insert one term between 81 and 169 to
550. A pipe can fill a pool in 5 hours while make a geometric sequence.
another pipe can drain empty the pool in 10 A. 113 B. 117 C. 121 D. 125
hours. How long will it take to fill the pool if both
pipes are open? Solution:
A. 8 hours B. 9.25 hours
Shortcut for inserting one term is √AB. This is
C. 9.5 hours D. 10 hours
also the formula for the geometric mean.
Solution: √81(169) = 9(13) = 117
................................................
This is similar to our “Working Together”
problem, except instead of adding their times, we 554. Find the common difference of an
will subtract (since the draining pipe is doing the arithmetic sequence whose 21st term is 1987 and
opposite of helping). 29th term is 2019.
AB/(A-B) = 10(5)/(10-5) = 50/5 = 10 hrs A. 2.5 B. 3 C. 3.5 D. 4
................................................
Solution:
551. Today, Rayon is 13 years old while his
father is thrice his age. How many years from d= = = =4
now will his father be twice as old as he? ................................................
A. 15 B. 13 C. 11 D. 10
555. Find the general term An of the sequence
Solution: 7, 16, 25, 34, 43, 52, 61, …
Let x = number of years from now A. An = 9n – 2 B. An = 8n – 1
C. An = 9n + 2 D. An = 8n + 1
2(13+x) = 39+x
26 + 2x = 39 + x Solution:
2x – x = 39 – 26
x = 13 An = dn – (d – A1)
................................................ = 9n – (9 – 7)
= 9n – 2
552. Roy and Wena are on a seesaw. Roy weighs ................................................
48 kg and sits 160 cm to the left of the fulcrum. If
Wena weighs 60 kg, how far to the right of the 556. Find the remainder when x4 – 4x3 + 3x2 +
fulcrum must she sit to balance the seesaw? 5x – 6 is divided by (x – 2).
A. 115 cm B. 123 cm A. 3 B. 2 C. 1 D. 0
C. 128 cm D. 135 cm

This is a free reviewer. All rights reserved.


1000 MMR Author: Victor A. Tondo Jr., LPT
Solution: 560. Factorize 81x2 – 225y2 completely.
A. (9x + 15y)(9x – 15y)
Use the remainder theorem.
B. 9(9x2 – 25y2)
x – a = x – 2  That means a = 2
C. 9(3x – 5y)2
Remainder = f(a) = f(2) D. 9(3x + 5y)(3x – 5y)
= 24 – 4(23) + 3(22) + 5(2) – 6
= 16 – 32 + 12 + 10 – 6 Solution:
................................................
81x2 – 225y2 = 9(9x2 – 25y2)
= 9(3x + 5y)(3x – 5y)
557. 2 + 9 + 16 + 23 + … + 135 = ____
................................................
A. 1353 B. 1360
C. 1370 D. 1377
561. What are the missing terms in the series
3, 6, 12, 24, ___, 96, ____?
Solution:
A. 48; 192 B. 36, 120
n= +1 C. 46, 196 D. 46, 192

n= + 1 = 20 Solution:

Sum = (n) Since the common ratio is 2, then next terms


should be 24(2) and 96(2), or 48 and 192.
Sum = (20) = 1370 ................................................
................................................
562. In a parking lot, there are 65 tricycles and
558. Factorize: (2x + y) (2x + y + 3) + 2. motorcycles. If there are 150 wheels in all, how
A. (2x + y + 1) (2x + y + 2) many tricycles are there?
B. (2x + y + 1) (2x + y + 6) A. 18 B. 19 C. 20 D. 21
C. (2x + y + 2) (2x + y + 3)
D. (2x + y + 3) (2x + y + 4) Solution:
T + M = 65  2T + 2M = 130
Solution:
3T + 2M = 150  3T + 2M = 150
Let 2x + y = A T = 20
(2x + y) (2x + y + 3) + 2 = A (A + 3) + 2 ................................................
= A2 + 3A + 2
= (A + 2)(A + 1) 563. Which fraction is equivalent to 0.425?
A. 21/50 B. 27/50
Substitute A with 2x + y and you’ll get
C. 27/40 D. 17/40
(2x + y + 2) (2x + y + 1).
................................................
Explanation:
559. The area of a rectangle is (x2 – x – 30). If its 0.425 is read as 425 thousandths. In fraction
length is x + 5, what is its width? form, that’s or in its simplest form, .
A. x + 4 B. x – 3
C. x + 2 D. x – 6
Alternative Solution:
Solution: Just input 0.425 and press your scientific
calculator’s S D button.
x2 – x – 30 = (x + 5) W
(x + 5) (x – 6) = (x + 5) W
x–6=W
This is a free reviewer. All rights reserved.
1000 MMR Author: Victor A. Tondo Jr., LPT
564. What are the zeroes of 6x2 – x – 35? Solution:
A. and B. and Age 10 yrs
Person Age Now
C. and D. and from now
Mr. Yu 4x 4x + 10
Solution: Daughter x x + 10

6x2 – x – 35 = 0 4x + 10 = 3(x + 10)


(3x + 7) (2x – 5) = 0 4x + 10 = 3x + 30
3x + 7 = 0 2x – 5 = 0 4x – 3x = 30 – 10
x= x= x = 20
................................................ ................................................

565. Seven more than four times a number is 79. 568. The speed of the current of a river is 9 kph.
What is five less than three times the number? Rayon rows his boat upstream for 8 hours, and
A. 59 B. 49 C. 39 D. 29 then travels back to his original position by
rowing downstream for 4 hours. What is Rayon’s
Solution: speed on calm water?
A. 9 kph B. 18 kph
4x + 7 = 79 C. 27 kph D. 36 kph
4x = 72
x = 18 3x – 5 = 3(18) – 5 = 49 Solution:
................................................
8(x – 9) = 4(x + 9)
566. Mr. Park is five times as old as his son. Eight 8x – 72 = 4x + 36
years from now, he will only be thrice as old as 8x – 4x = 36 + 72
his son. How old is Mr. Park now? 4x = 108
A. 30 B. 35 C. 40 D. 45 x = 27
................................................
Solution:
569. Thrice Rayon’s age 16 years ago is equal to
Age 8 yrs his age 16 years from now. How old is Rayon
Person Age Now
from now now?
Mr. Park 5x 5x + 8 A. 16 B. 24 C. 32 D. 40
Son x x+8
Solution:
5x + 8 = 3(x + 8)
5x + 8 = 3x + 24 3(x – 16) = x + 16
5x – 3x = 24 – 8 3x – 48 = x + 16
2x = 16 3x – x = 16 + 48
x=8 5x = 40 2x = 64
................................................ x = 32
................................................
567. Mr. Yu is four times as old as his daughter.
Ten years from now, he will only be thrice as old 570. Divide 8x5 – 4x3 + 2x2 – 3x + 4 by x + 2.
as her. How old is his daughter now? A. 8x4 – 16x3 + 28x2 – 54x + 105 r. -206
A. 10 B. 15 C. 20 D. 25 B. 8x4 + 16x3 + 28x2 – 54x + 105 r. -206
C. 8x4 – 16x3 + 28x2 – 54x – 105 r. 206
D. 8x4 + 16x3 + 28x2 – 54x – 105 r. 206

This is a free reviewer. All rights reserved.


1000 MMR Author: Victor A. Tondo Jr., LPT
Solution: 573. In the equation 2x + 5y = 50, what is x
when y is 8?
8x -16x3 +28x -54x +10
A. 2 B. 3 C. 4 D. 5
4 2 5
x+ 8x -4x3 +2x2 -3x +4
Solution:
2 5

8x +16x 2x + 5(8) = 50
5 4 2x + 40 = 50
-16x4 -4x3 2x = 50 – 40
-16x4 -32x3 2x = 10
28x3 +2x2 x=5
28x3 +56x ................................................
2

-54x2 -3x 574. X varies directly as Y and inversely as Z. If X


-54x2 - is 24 when Y is 30 and Z is 5, find X when Y is 36
108x and Z is 4.
105x +4 A. 36 B. 32 C. 24 D. 16
105x +21
0 Solution:
-206 X = kY/Z X = 4Y/Z
Alternative: Use Synthetic division. 24 = 30k/5 X = 4(36)/9
-2 8 0 -4 2 -3 4 24 = 6k X = 144/9
- 4=k X= 16
-16 32 -56 108 ................................................
210
-
8 -16 28 -54 105 575. What is the 4th term of (3x – 2)5?
206
................................................ A. 360x2 B. 240x2
C. -240x2 D. -360x2
571. Factorize 27x3 + 125.
A. (3x + 5)(9x2 + 15x + 25) Solution:
B. (3x + 5)(9x2 – 15x + 25) 5C3 (3x)2 (-2)3 = 5 (9x2) (-8)
C. (3x – 5)(9x2 + 15x + 25) = -360x2
D. (3x – 5)(9x2 – 15x + 25) ................................................

Explanation: 576. Factorize 64x6 – y6.


A3 + B3 = (A + B)(A2 – AB + B2) A. (2x – y) (4x2 + 2xy + y2) (2x + y) (4x2 – 2xy + y2)
B. (2x – y) (4x2 + 2xy + y2) (2x + y) (4x2 + 2xy + y2)
................................................
C. (2x + y) (4x2 – 2xy – y2) (2x + y) (4x2 – 2xy + y2)
D. (2x + y) (4x2 – 2xy – y2) (2x + y) (4x2 – 2xy + y2)
572. What is the 5th term of (2x + 3)7?
A. 22680x3 B. 11340x3 Solution:
C. 21600x3 D. 10800x3
64x6 – y6 = (8x3 – y3)(8x3 + y3)
Solution: = (2x – y) (4x2 + 2xy + y2) (2x + y) (4x2 – 2xy + y2)

(7C4)(2x)3(3)4 = 35 (8x3) (81)


= 22680x3

This is a free reviewer. All rights reserved.


1000 MMR Author: Victor A. Tondo Jr., LPT
577. Factorize x2 + 6x – y2 + 2y +8. Solution:
A. (x + y + 2) (x + y + 4)
25 = 32 (x3y4z2)5 = x15y20z10
B. (x + y + 2) (x – y + 4)
.: (2x3y4z2)5 = 32 x15y20z10
C. (x + y + 2) (x + y – 4)
................................................
D. (x + y + 2) (x – y – 4)
581. Find k such that 5x2 + 30x + k has two
Solution:
unequal real roots.
x2 + 6x – y2 + 2y +8 A. k > 6 B. k < 6
= x2 + 6x – (y2 – 2y) +8 C. k > 45 D. k < 45
= x2 + 6x +9 – (y2 – 2y + 1)
= (x + 3)2 – (y – 1)2 Solution:
= (x + 3 + y – 1) (x + 3 – y + 1)
A=5 B = 30 C=k
= (x + y + 2) (x – y + 4)
................................................ B2 – 4AC > 0
302 – 4(5)k > 0
578. What value of x will satisfy the equation: 900 – 20k > 0
2.5(4x – 504) = x? -20k > -900
A. 150 B. 145 C. 140 D. 135 k < 45
................................................
Solution:
582. Which of the following is not a polynomial?
2.5(4x – 504) = x
10x – 1260 = x A. π r2 + 2 π r B. m – n
10x – x = 1260 C. √2 x + √3 y – √5 z D. ab √ cd
9x = 1260
x = 140 Explanation:
................................................
Variables cannot bear an irrational exponent.
579. Factorize (2x + y) (2x + y + 6) + 5 ................................................
A. (2x + y + 5) (2x + y + 1)
B. (2x + y – 5) (2x + y – 1) 583. Which of the following is irrational?
C. (2x + y + 5) (2x – y + 1) A. 0.193193193193… B.
D. (2x – y + 5) (2x – y + 1) C. √1023 D. √2401

Solution: Let 2x + y = A. Explanation:


(2x + y) (2x + y + 6) + 5 A. 0.193193… is in fraction form
= A (A + 6) + 5
= A2 + 6A + 5 B. is already a fraction
= (A + 5) (A + 1) Substitute A with 2x + y. C. √1023 cannot be expressed as a fraction
= (2x + y + 5) (2x + y + 1) D. √2401 is equal to 49
................................................ ................................................
580. Simplify: (2x3y4z2)5 584. If x + y = 17 and x2 + y2 = 135, find xy.
A. 10x15y20z10 B. 25x15y20z10 A. 78 B. 77 C. 76 D. 75
C. 32x15y20z10 D. 100x15y20z10

This is a free reviewer. All rights reserved.


1000 MMR Author: Victor A. Tondo Jr., LPT
Solution: 588. Solve for x: 2x + 3y = -8, and 5x – 7y = 67
A. 5 B. 6 C. -5 D. -6
(x + y)2 = x2 + 2xy + y2
172 = x2 + y2 + 2xy
Solution:
289 = 135 + 2xy
289 – 135 = 2xy 7(2x + 3y = -8) 14x + 21y = -56

154 = 2xy 3(5x – 7y = 67) 15x – 21y = 201

77 = xy 29x = 145
................................................ x=5
................................................
585. If H and K are the roots of 5x2 – 8x + 9, find
H + K – HK. 589. A farmer has 15 goats, 23 pigs, and a few
A. -2/5 B. -1/5 C. 0 D. 1/5 chickens in his farm. If he counted a total of 200
legs in his farm (excluding his, of course), how
many chickens does he have?
Solution:
A. 96 B. 72 C. 48 D. 24
H + K = sum of roots = 8/5
HK = product of roots = 9/5 Solution:
H + K – HK = 8/5 – 9/5 = -1/5
15(4) + 23(4) + n(2) = 200
................................................
60 + 92 + 2n = 200
2n = 200 – 152
586. If H and K are the roots of 3x2 + 4x + 5, find
2n = 48
H2 + K2.
n = 24
A. -5/9 B. -1 C. -14/9 D. -2
................................................
Solution: √
590. Rationalize
H + K = -4/3 √
√ √ √ √ √ √
HK = 5/3 A. B.
(H + K)2 = 16/9 √ √ √ √ √ √
H2 + K2 + 2HK = 16/9 C. D.
H2 + K2 + 2(5/3) = 16/9
H2 + K2 = 16/9 – 10/3 Solution:
H2 + K2 = -14/9
................................................ √ √
=
√ √ √
√ √
√ √ √
587. Factorize 9x2 + 24xy + 16y2– 81z2. =
A. (3x + 4y + 9z) (3x + 4y – 9z) ................................................
B. (3x – 4y + 9z) (3x + 4y – 9z)
C. (3x + 4y + 9z) (3x – 4y – 9z) 591. If 3x + 2 = 4y and 5y – 3 = z, express z in
D. (3x – 4y + 9z) (3x – 4y – 9z) terms of x.
A. 15x + 9 B. 15x + 7
Solution: C. D.
9x2 + 24xy + 16y2 – 81z2
= (3x + 4y)2 – (9z)2
= (3x + 4y + 9z) (3x + 4y – 9z)
................................................

This is a free reviewer. All rights reserved.


1000 MMR Author: Victor A. Tondo Jr., LPT
Solution: Solution:
3x + 2 = 4y A = P (1 + tr)
=y A = 1,600,000 [1 + 4(0.025)]
A = 1,600,000 (1 + 0.1)
5y – 3 = z
A = 1,600,000 (1.1)
5( )–3=z A = 1,760,000
=z ................................................
=z
595. Solve for x: 45x+3 = 82x+6
................................................ A. 2 B. 3 C. 4 D. 5

592. If 72x + 3 = B, what is B ÷ 49x? Solution:


A. 343 B. 21 C. 7 D. 3
Express each side as a power of a common
Solution: number before solving for x.

72x + 3 = B 45x+3 = 82x+6


(7 )x (73) = B
2 (22)5x+3 = (23)2x+6  Apply laws of exponents.
49x (343) = B 210x+6 = 26x+18
(343) 49x = B 10x + 6 = 6x + 18
B÷ 49x = (343) 49x
÷ 49x
= 343 10x – 6x = 18 – 6
................................................ 4x = 12
x=3
593. Multiply: (7x + 4) (2x – 1) (3x + 5) ................................................
A. 42x3 + 73x2 + 7x – 20
B. 42x3 – 73x2 + 7x – 20 596. Given = , which of the following
C. 42x3 + 73x2 – 7x – 20 expressions is equal to x?
D. 42x3 – 73x2 – 7x – 20 A. B. C. D.
Solution:
Solution:
7x +4
times 2x -1 =  Cross-multiply.
-7x -4 2cxy = 3abz  Isolate x.
14x2 8x x=
14x2 +x -4
times 3x +5 ................................................
70x2 +5x -20
42x3 +3x2 -12x 597. Add: (2x2 + 4x + 5) + (5 – 6x – x2)
42x3 +73x2 -7x -20 A. x2 + 2x + 10 B. 3x2 + 2x + 10
................................................ C. x – 2x + 10
2 D. 3x2 + 2x + 10
................................................
594. Rayon invested P1,600,000.00 in a bank
that offers 2.5% interest per annum. How much 598. Solve for x: 272x–8 = 9x+10
will his account hold after 4 years? A. 9 B. 10 C. 11 D. 12
A. P160,000 B. P1,160,000
C. P1,460,000 D. P1,760,000

This is a free reviewer. All rights reserved.


1000 MMR Author: Victor A. Tondo Jr., LPT
Solution: 602. How many line segments can be made from
28 non-collinear points?
Express each side as a power of a common
A. 378 B. 394
number before solving for x.
C. 412 D. 422
272x–8 = 9x+10
(33)2x–8 = (32)x+10  Apply laws of exponents. Solution:
26x–24 = 22x+20
28C2 = 378
6x – 24 = 2x + 20 ................................................
6x – 2x = 20 + 24
4x = 44 603. The vertex angle of an isosceles triangle is
x = 11 70°. What is the measure of one of the base
................................................ angles?
A. 45° B. 50° C. 55° D. 60°
599. If 25x – 49y = 2019, what is 49y – 25x?
A. 2018 B. -2019 Solution:
C. D. cannot be determined (180 – 70)/2 = 110/2 = 55
................................................
Explanation:
49y – 25y = (-1)(25x – 49y) 604. A shoebox measures 20 inches by 15 inches
= (-1)(2019) by 9 inches. What is its volume in cubic inches?
= -2019 A. 900 B. 1350
................................................ C. 1650 D. 2700

600. If x + y = 90 and x2 + y2 = 2020, find xy. Solution:


A. 3025 B. 3030
V=LxWxH
C. 3035 D. 3040
= 20 x 15 x 9 = 2700
................................................
Solution:
(x + y)2 = x2 + 2xy + y2 605. Which of these has the longest perimeter?
902 = x2 + y2 + 2xy A. A regular pentagon with sides 21 cm long
8100 = 2020 + 2xy B. A rectangle 29 cm long and 24 cm wide
8100 – 2020 = 2xy C. An equilateral triangle with sides 36 cm long
6080 = 2xy D. A square whose area is 1024 cm2
3040 = xy
................................................ Solution:
A. P = 5 (21) = 105
601. Which of the following is ALWAYS true?
B. P = 2 (29 + 24) = 106
A. Vertical pairs of angles are supplementary.
C. P = 3 (36) = 108
B. Vertical pairs of angles are congruent.
D. S = 32, so P = 4 (32) = 128
C. Linear pairs of angles are congruent.
................................................
D. Linear pairs of angles are complementary.
606. The hypotenuse of a right triangle is 82 feet.
Explanation:
If one leg is 80 feet, what is the length of the
Linear pairs are supplementary, while vertical other leg in feet?
pairs are congruent. A. 15 B. 16 C. 17 D. 18

This is a free reviewer. All rights reserved.


1000 MMR Author: Victor A. Tondo Jr., LPT
Explanation: Explanation:
Use the Pythagorean Theorem: A2 + B2 = C2 Consecutive angles of a parallelogram are
supplementary.
A2 + 802 = 822
................................................
A2 = 6724 – 6400
A2 = 324
611. Two parallel lines are cut by a transversal,
A = 18
forming R and V. If the two angles are
................................................
corresponding angles, what is the measure of R
if the measure of V is 70o?
607. Find the surface area of a rectangular box
A. 35o B. 70o
whose dimensions are 25 cm x 35 cm x 45 cm.
C. 160 o D. 110o
A. 7000 cm2 B. 7045 cm2
C. 7150 cm 2 D. 7745 cm2
Explanation:
Solution: Corresponding angles formed by two parallel
lines cut by a transversal are congruent.
SA = 2 (LW + WH + LH)
................................................
SA = 2 (45x35 + 35x25 + 25x45)
SA = 2 (1575 + 875 + 1125) = 7150
612. If the sum of the supplement and the
................................................
complement of an angle is 100 degrees, what is
the angle?
608. A and B form a linear pair. If m A = 3x
A. 65o B. 70o C. 75o D. 85o
and m B = 5x + 40, what is the value of x?
A. 17.5 B. 18 C. 19 D. 19.75
Solution:
Solution: (90 – x) + (180 – x) = 100
270 – 2x = 100
Since the two angles form a linear pair, then they
270 – 100 = 2x
are supplementary.
170 = 2x
3x + 5x + 40 = 180
85 = x
8x + 40 = 180
................................................
8x = 140
x = 17.5
613. It is the perpendicular bisector of a regular
................................................
polygon’s side, passing through the center.
609. 1 and 3 are opposite angles in a
A. side B. apothem
parallelogram. If m 1 = 50o, what is m 3?
C. asymptote D. diagonal
A. 40o B. 50o C. 140o D. 130o
Explanation:
Explanation:
The apothem is only for regular polygons. It is
Opposite angles of a parallelogram are
the perpendicular bisector of one of its sides,
congruent.
passing through the center.
A diagonal is a line segment formed by
connecting two non-consecutive vertices of a
610. A and C are consecutive angles in a
polygon.
parallelogram. If m A = 60o, what is m C?
A side is formed by connecting two consecutive
A. 40o B. 50o C. 150o D. 120o
vertices of a polygon.

This is a free reviewer. All rights reserved.


1000 MMR Author: Victor A. Tondo Jr., LPT
614. In the figure, // . If m AFE = 130o, 618. A circle has a diameter of 34 cm. What is its
find the measure of CGH. area?
A. 17 π cm2 B. 34 π cm2
C. 289 π cm2 D. 1156 π cm2

Solution:
D = 2r
34 = 2r A = π r2
17 = r A = π (172)
A. 50o B. 130o C. 65o D. 25o A = 289 π
................................................
Explanation:
AFE and CGH are exterior angles on the same 619. The perimeter of a rectangle is 84. If its
side of the transversal. Therefore, they are length is 29 cm, find its area.
supplementary. A. 377 cm2 B. 1218 cm2
................................................ C. 2436 cm2 D. 4872 cm2

615. The measure of each interior angle of a Solution:


regular polygon is 170o. How many vertices does P = 2(L + W)
it have? 84 = 2(L + W)
A. 36 B. 24 C. 12 D. 10 42 = (L + W) A=LxW
42 = 29 + W A = 29 x 13
Solution: 13 = W A = 377
................................................
N=

= = 36 620. In the figure, A is the center of the circle.


What is the measure of BDC?
................................................ A. 160o B. 80o C. 40o D. 20o
616. How many diagonals does a regular
dodecahedron have?
A. 45 B. 54 C. 60 D. 66

Solution:
( ) ( )
D= = = 54
................................................ Explanation:

617. Find the area of a square whose perimeter BAC is a central angle measuring 40o. That
is 480 cm. means ̂ = 40o. Since BDC is an inscribed
A. 14400 cm2 B. 28800 cm2 angle intercepting ̂ , then its measure is half of
C. 57600 cm2 D. 270400 cm2 ̂ , or 20o.

Solution:
P = 4S
480 = 4S A = S2
120 = S A = 1202
A = 14400
This is a free reviewer. All rights reserved.
1000 MMR Author: Victor A. Tondo Jr., LPT
621. Find the measure of E using the figure of Solution:
parallelogram WENA below.
Diagonal2 = Length2 + Width2 + Height2
172 = 92 + W2 + 122
289 = 81 + W2 + 144
289 – 144 – 81 = W2
64 = W2
A. 15 B. 65 C. 115 D. 85 8=W
................................................
Solution:
7x – 40 = 4x + 5 624. Statement 1: A square is a rhombus.
7x – 4x = 40 + 5 Statement 2: A square is a rectangle.
3x = 45 A. Only the first statement is true.
x = 15 B. Only the second statement is true.
C. Both statements are true.
So m W = 4x + 5 = 65, which means D. Both statements are false.
m E = 180 – 65 = 115 ................................................
................................................
625. Which of the following has its incenter,
622. In parallelogram circumcenter, centroid, and orthocenter in just
MATH, m A = 7x – 32 and one point?
m T = 5x + 32. Find m A. A. Equilateral Triangle B. Right Triangle
A. 15 B. 63 C. Scalene Triangle D. Obtuse Triangle
C. 73 D. 117 ................................................
Solution: 626. In triangles, this line segment is drawn from
A and T are consecutive angles, midpoint of one side to its opposite vertex.
therefore m A + m T = 180 A. Median B. Altitude
C. Bisector D. Longitude
7x – 32 + 5x + 32 = 180
7x + 5x = 180 Explanation:
12x = 180
x = 15 Altitude: perpendicular to one side, passing
through the opposite vertex.
m A = 7x – 32 Median: line segment from midpoint of one side
= 7(15) – 32 to opposite vertex
= 105 – 32 = 73 Bisector: line segment that bisects an angle of a
................................................ triangle
................................................
623. The diagonal of a rectangular prism is 17
cm long. If it is 9 cm thick and 12 cm long, how 627. The radius of a cone measures 12 cm and its
wide is it? height is 15 cm. Find its volume.
A. 15 cm B. 10 cm A. 240 π cm3 B. 360 π cm3
C.8√3 cm D. 8 cm C. 480 π cm3 D. 720 π cm3

This is a free reviewer. All rights reserved.


1000 MMR Author: Victor A. Tondo Jr., LPT
Solution: Solution:
V = πr2h Find the height of the trapezoid using the
Pythagorean Theorem first.
= π (144) (15)
= 720 π
................................................

628. This is located at the intersection of the


perpendicular bisectors of a triangle.
A. Centroid B. Circumcenter
C. Incenter D. Orthocenter
52 + h2 = 132
Explanation: 25 + h2 = 169
h2 = 144
Incenter: intersection of angle bisectors
h = 12
Circumcenter: intersection of perpendicular
bisectors
Centroid: intersection of medians Area = ( )
Orthocenter: intersection of altitudes Area = (12)
................................................
Area = 25(12) = 300
................................................
629. The radius of a cylinder measures 9 cm and
its height is 15 cm. Find its volume.
632. Which of the following side lengths belong
A. 1215 π cm3 B. 810 π cm3
to an acute triangle?
C. 540 π cm 3 D. 360 π cm3
A. 5 cm, 7 cm, 9 cm B. 6 cm, 8 cm, 10 cm
C. 6 cm, 9 cm, 12 cm D. 7 cm, 9 cm, 11 cm
Solution:
V = πr2h Solution:
= π (81) (15)
A. 52 + 72 < 92 Obtuse
= 1215 π
B. 6 + 8 = 10
2 2 2 Right
................................................
C. 62 + 92 < 122 Obtuse
D. 7 + 9 > 11
2 2 2 Acute
630. What is the measure of each exterior angle
................................................
of a regular decagon?
A. 108o B. 72o C. 60o D. 36o
633. A prism has a right triangle as its base. The
two legs of the right triangular base are 6 cm and
Solution:
8 cm long. If the prism is 2 cm thick, find its
MEA = 360/N lateral surface area.
= 360/10 = 36o A. 24 cm2 B. 48 cm2
................................................ C. 56 cm 2 D. 72 cm2

631. Find the area of an isosceles trapezoid given


Solution:
bases measuring 20 cm and 30 cm, with each of
the congruent slants measuring 13 cm. The hypotenuse of the base is 10 cm.
A. 250 cm2 B. 275 cm2 Lateral surface area = 2(6 + 8 + 10) = 48 cm2
C. 300 cm2 D. 325 cm2

This is a free reviewer. All rights reserved.


1000 MMR Author: Victor A. Tondo Jr., LPT
634. Find the volume of a rectangular pyramid 637. A rhombus has diagonals measuring 20 cm
given that its base has length 21 cm and width 10 and 30 cm. What is its area?
cm, and its height is 15 cm. A. 75 cm2 B. 150 cm2
A. 700 cm3 B. 1050 cm3 C. 300 cm2 D. 600 cm2
C. 1575 cm 3 D. 1400 cm3
Solution:
Solution:
A = ½ D1 D2
V= LxWxH A = ½ (600) = 300
................................................
V = (21)(10)(15)
V = 1050 638. A rhombus as diagonals measuring 30 cm
................................................ and 16 cm. What is its perimeter?
A. 136 cm B. 92 cm
635. The diameter of a cylinder is 24 cm. If its C. 68 cm D. 46 cm
height is 40 cm, find its surface area.
A. 312 π cm2 B. 624 π cm2 Solution:
C. 936 π cm2 D. 1248 π cm2

Solution:
SA = 2πr2 + 2πrh
SA = 2π (144) + 2π(12)(40) By Pythagorean Theorem, the hypotenuse of
SA = 288π + 960π each right triangle is 17 cm. That means each
SA = 1248π side of the rhombus is 17 cm, and its perimeter is
................................................ 4 x 17 or 68 cm.
................................................
636. In the figure, AC is a diagonal of rhombus
ABCD. If m B = 118o, what is m CAD? 639. Find the approximate distance around a
A. 124o B. 118o C. 62o D. 31o semicircular park with radius 100m.
A. ~1028 m B. ~771 m
C. ~514 m D. ~257 m

Solution:
Diameter = 2r = 200
Semicircle = r = 100π ~ 314
Solution:
Perimeter = ~514
Since ABCD is a rhombus, then AC bisects BAD, ................................................
and BAD and B are supplementary.
640. Find the equation of the line perpendicular
m B + m BAD = 180 to 7x + 4y = 12, passing through (-2, 9).
118 + m BAD = 180 A. 4x – 7y = -71 B. 4x + 7y = 71
m BAD = 62 C. 4x – 7y = 71 D. 4x + 7y = -71
m CAD = ½ (62) = 31
................................................ Solution:
The perpendicular line for Ax + By = C is given
as Bx – Ay = B(xp) – A(yp).
4x – 7y = 4(-2) – 7(-9)
4x – 7y = -71
This is a free reviewer. All rights reserved.
1000 MMR Author: Victor A. Tondo Jr., LPT
641. Find the equation of the line perpendicular 644. Find the volume of the following pool.
to 9x –7y = -6, passing through (4, -3).
A. 7x + 9y = 1 B. 7x + 9y = -1
C. 7x – 9y = 1 D. 7x – 9y = -1

Solution:
The perpendicular line for Ax + By = C is given
as Bx – Ay = B(xp) – A(yp).
-7x – 9y = -7(4) – 9(-3) A. 700 m3 B. 1050 m3
-7x – 9y = -7(4) – 9(-3) C. 1400 m3 D. 2800 m3
-7x – 9y = -1
Solution:
But since the new A is negative, then we will
The pool is a trapezoid prism. Its area is given as
change all signs of the new equation, giving us
7x + 9y = 1. A=
................................................ A= (8)(10)
A = 1400 m3
642. A rectangle is drawn with dimensions 28 cm
................................................
by 42 cm. A larger rectangle is drawn by adding
5 cm margins from each side of the original
645. What is the slope of the line defined by the
rectangle. What is the area of the larger
equation 3x + 5y = 11?
rectangle?
A. 1456 cm2 B. 1551 cm2 A. B. C. 3 D. 5
C. 1556 cm2 D. 1976 cm2
Solution:
Explanation:
3x + 5y = 11  Ax + By = C
The dimensions of the larger rectangle would be 5y = -3x + 11
(28 + 5 + 5) by (42 + 5 + 5), or 38 by 52. That y= x+  y = mx + b
means its area is 38 x 52 or 1976. ................................................
................................................
646. In the figure below, m CAB = 2x + 4, while
643. Find the length of the diagonals of a m COB = 5x – 7. Find x.
rectangular prism 6 cm thick, 15 cm long, and 10
cm wide.
A. 17√3 cm B. 18√2 cm
C. 19 cm D. 20√5 cm

Solution:
D=√ + + A. B. -2 C. D. 15
D = √15 + 10 + 6
D = √225 + 100 + 36 Solution:
D = √361 2(2x + 4) = 5x – 7
D = 19 4x + 8 = 5x – 7
8 + 7 = 5x – 4x
15 = x

This is a free reviewer. All rights reserved.


1000 MMR Author: Victor A. Tondo Jr., LPT
647. In the figure below, // . If m AGE = Solution:
7x + 11 and m CHF = 5x + 1, what is m AGE? Each square has a side of 6 cm. The perimeter of
the figure has 22 segments measuring 6 cm each.
Therefore the perimeter is 22 x 6 or 132 cm.
................................................

650. Find the area of the shaded part of the


circle.

A. 119o B. 109o C. 71o D. 61o

Solution:
Since AGE and CHF are exterior angles on the
same side of the transversal, then they are
supplementary. A. 9720 π cm2 B. 972 π cm2
C. 540 π cm2 D. 270 π cm2
m AGE + m CHF = 180
7x + 11 + 5x + 1 = 180
12x + 12 = 180 Solution:
12x = 168 A= π r2
x = 14
A = (900 π)
m AGE = 7x + 11
A = 270 π
m AGE = 7(14) + 11
................................................
m AGE = 109
................................................
651. In the figure, ̂ = 120o and m E = 28o.
648. Find the sum of all interior angles of a Find ̂ .
regular 15-sided polygon.
A. 5400o B. 2700o
C. 2340 o D. 1170o

Solution:
A. 46o B. 54o C. 64o D. 72o
180 (n – 2) = SIA
180 (15 – 2) = SIA Solution:
2340 = SIA ̂ ̂
................................................ m E=
̂
28 =
649. If each square in the figure has an area of 36
square cm, find the perimeter of the figure. 56 = 120 ̂
̂ = 120 – 56
̂ = 64
................................................

652. Find the volume of a cone whose radius is


A. 864 cm B. 792 cm 30 cm and height is 170 cm.
C. 144 cm D. 132 cm A. 289,000 π B. 153,000 π
C. 51,000 π D. 17,000 π

This is a free reviewer. All rights reserved.


1000 MMR Author: Victor A. Tondo Jr., LPT
Solution: 655. Find the equation of the line passing
through (7, 5) and (5, 1).
V = π r2 h A. y = 2x + 9 B. y = 2x – 9
V = π (30)2 170 C. y = ½ x + 9 D. y = ½ x – 9
V = (153,000 π) Solution:
V = 51,000 π
................................................ Use the two-point form.
y – y1 = (x – x1)
653. Find the center of a circle given one of its y–5= (x – 7)
longest chords has endpoints at M(-11,8) and
N(23,-24). y – 5 = (x – 7)
A. (12, -16) B. (6, -8) y – 5 = 2 (x – 7)
C. (-17,16) D. (17, -16) y – 5 = 2x – 14
y = 2x – 9
Solution: ................................................
The longest chord of a circle is the diameter and
656. Find the equation of the line passing
its midpoint is the center.
through (2, -7) with a slope of -3.
Midpoint Formula: ( , ) A. y = -3x – 1 B. y = -3x + 1
( ) C. y = -3x – 2 C. y = -3x + 2
Midpoint: ( , )

Midpoint: (6, 8) Solution:


................................................
y = mx + b where x = 2, y = -7, m = -3
-7 = -3(2) + b
654. Find the radius of a circle if its diameter has
-7 = -6 + b
endpoints at M(3,5) and N(-12,13).
-1 = b
A. 8 B. 8.5 C. 9 D. 9.5
.: y = -3x – 1
Solution: ................................................
The diameter’s midpoint is the center.
657. A square has a diagonal measuring 28 cm.
Center: ( , ) Find its area.
Center: ( 4.5,9) A. 784 cm2 B. 392 cm2
C. 196 cm 2 D. 98 cm2
Radius is the distance between the center and
any of its endpoints.
Solution:
R = √( 4.5 3) + (9 5)
R = √( 7.5) + 4 A=
R = √56.25 + 16 A=
R = √72.25
A = 392
R = 8.5
................................................
Better Solution:
658. Symbol for line.
R=½D where D = length of diameter A. B.
D = √( 12 3) + (13 5) C. D.
D = √225 + 64
D = 17 Therefore, R = ½ (17) or 8.5
This is a free reviewer. All rights reserved.
1000 MMR Author: Victor A. Tondo Jr., LPT
659. Symbol for ray. 662. In the figure, ACB is formed by the
A. B. tangents AC and BC. If m ACB = 34o, what is the
C. D. measure of the minor intercepted arc?
................................................

660. Find the volume of the swimming pool


drawn below.
A. 2000 cu. ft. B. 2100 cu. ft.
C. 2250 cu. ft. D. 2500 cu. ft. A. 112o B. 146o C. 68o D. 34o

Explanation:
When two tangents to a circle form an angle, the
intercepted arcs are 180 ± θ, where θ is the
measure of the angle formed by the tangents.
................................................

663. X(7,9) is the midpoint of A(1,2) and B. Find


the coordinates of B.
A (-13, -16) B. (13, -16)
C. (-13, 16) D. (13, 16)
Solution:
Solution:
7= 9=
14 = x + 1 18 = y + 2
13 = x 16 = y
................................................

664. Point O is two-thirds the way from A(-4,5)


to B(8,-10). Find the coordinates of point O.
A. (0, 0) B. (4, -5)
Let’s rebuild the pool as a figure that is left by C. (3, -6) D. (3, -4)
removing a triangular prism (gray figure) from a
rectangular prism (whole figure). Solution:
The rectangular prism is 25 ft by 8 ft by 15 ft. Its x = -4 + (8 – (-4))
volume is 3000 cubic feet.
The triangular prism’s base is 6 ft by 20 ft. Its x = -4 + (12)
base area is 60 square feet. Since its length is 15 x = -4 + 8
feet, then its volume is 900 cubic feet. x=4
Remove 900 from 3000 and you’ll get 2100 cubic
y = 5+ (-10 – 5)
feet.
................................................ y = 5+ (-15)
y = 5+ (-15)
661. Which geometric figure extends indefinitely
in two opposite directions? y = 5 + (-10)
A. angle B. ray y = -5
C. line D. line segment

This is a free reviewer. All rights reserved.


1000 MMR Author: Victor A. Tondo Jr., LPT
665. H and K form a vertical pair. If m H = Solution:
4x + 20 and m K = 6x – 10, find x.
5x – 6y = 7
A. 13 B. 15 C. 17 D. 19
-6y = -5x + 7
Solution: y= x–
................................................
Vertical pairs of angles are congruent.
4x + 20 = 6x – 10 670. Find the equation of a circle if its diameter
20 + 10 = 6x – 4x has endpoints at M(3,5) and N(-12,13).
30 = 2x A. (x + 4.5)2 + (y + 9)2 = 72.25
15 = x B. (x – 4.5)2 + (y + 9)2 = 72.25
................................................ C. (x + 4.5)2 + (y – 9)2 = 72.25
D. (x – 4.5)2 + (y – 9)2 = 72.25
No item #666, as requested by many subscribers.
................................................ Solution:
The diameter’s midpoint is the center.
667. F and G form a linear pair. If m F =
4x + 20 and m G = 6x – 10, find x. Center: ( , )
A. 13 B. 15 C. 17 D. 19 Center: ( 4.5,9)
Radius is the distance between the center and
Solution:
any of its endpoints.
Linear pairs of angles are supplementary. R = √( 4.5 3) + (9 5)
4x + 20 + 6x – 10 = 180 R = √( 7.5) + 4
10x + 10 = 180 R = √56.25 + 16
10x = 170 R = √72.25
x = 17 R = 8.5
................................................ ................................................

668. Point C is two-fifths the way from M(-3,4) to 671. A bicycle’s wheel is 24 inches in diameter. If
N(7,-11). Find the coordinates of point C. it revolves 625 times, how far does the bicycle
A. (0, 0) B. (2, -1) travel?
C. (1, -2) D. (1, -1) A. 7,500 π B. 15,000 π
C. 30,000 π D. 60,000 π
Solution:
Solution:
x = -3+ (7 – (-3)) y = 4 + (-11 – 4)
x = -3+ (10) y = 4 + (-15) Circumference = π D
Circumference = 24 π
x = -3 + 4 y = 4 + (-6) That means the bicycle travels 24 π inches per
x=1 y = -2 revolution.
................................................ 625 x 24 = 15,000 π
................................................
669. Find the slope of the line 5x – 6y = 7.
A. B. C. D. 672. A right triangle is inscribed in a circle. If the
triangle’s legs are 16 cm and 30 cm, what is the
area of the circle?
A. 289 π cm2 B. 480 cm2
C. 480 π cm2 D. 1156 π cm2
This is a free reviewer. All rights reserved.
1000 MMR Author: Victor A. Tondo Jr., LPT
Solution: The white triangle in the lower right area is 3 x 3
units, giving it an area of 4.5 sq. units.
The hypotenuse of the right
48 – (6 + 7.5 + 5 + 4.5) = 25
triangle is the diameter of the
................................................
circle since it is inscribed in the
circle. By Pythagorean Theorem,
675. Which of the following are not congruent?
the diameter is 34 cm, meaning the radius is 17
A. Corresponding angles formed by two parallel
cm. The area is given as π r2, meaning the area is
lines cut by a transversal.
289 π cm2.
B. Alternate exterior angles formed by two
................................................
parallel lines cut by a transversal.
C. Pairs of vertical angles.
673. In the figure, a rectangle is
D. Interior angles on the same side of the
inscribed in a circle. If the rectangle
transversal cutting two parallel lines.
is 20 cm by 48 cm, what is the area of
the shaded figure?
Explanation:
A. 676π – 960 cm2 B. 1352π – 960 cm2
C. 2304π – 960 cm2 D. insufficient data When two parallel lines are cut by a transversal,
corresponding and alternate interior/exterior
Solution: angles are congruent. Vertical pairs of angles are
also congruent. However, interior/exterior
The rectangle’s diagonal is the diameter of the
angles on the same side of the transversal are
circle. By Pythagorean Theorem, the diameter is
supplementary.
52 cm, meaning the radius is 26 cm. Therefore,
................................................
the area of the entire circle is 676 π cm2.
The area of the rectangle is 20 x 48 or 960 cm2. 676. A rectangle is 24 cm long. If its diagonal is
25 cm, what is its perimeter in cm?
That means the area of the shaded figure is equal
A. 49 B. 62 C. 64 D. 98
to 676π – 960 cm2.
................................................
Solution:
674. What is the area of the red trapezium in the Solve for the width first using the Pythagorean
given figure? Theorem.
W2 + 242 = 252
W2 + 576 = 625
W2 = 625 – 576
W2 = 49
W=7
P = 2(L + W)
A. 23 sq. units B. 24 sq. units P = 2(24 + 7)
C. 25 sq. units D. 26 sq. units P = 2(31) = 62
................................................
Solution:
677. Square FACE is drawn such that one of its
The entire figure is 8 x 6, or 48 sq. units. sides AC is the diagonal of a rectangle, ABCD. If
The white triangle in the upper left area is 3 x 4 AB = 20 and BC = 43, find the area of FACE.
units, giving it an area of 6 sq. units.
The white triangle in the upper right area is 5 x 3 A. 2209 cm2 B. 2229 cm2
units, giving it an area of 7.5 sq. units. C. 2249 cm2 D. 2269 cm2
The white triangle in the lower left area is 5 x 2
units, giving it an area of 5 sq. units.
This is a free reviewer. All rights reserved.
1000 MMR Author: Victor A. Tondo Jr., LPT
Solution: 681. Find the length of the intercepted arc of a
central angle measuring 60o given that the radius
(AC)2 = 202 + 432
of the circle is 12 cm.
(AC)2 = 400 + 1849
A. 4 π cm B. 6 π cm
(AC)2 = 2249
C. 8 π cm D. 12 π cm
................................................
Solution:
678. If the vertex angle of an isosceles triangle is
50o, what is the measure of each base angle? (2 π r) = (24 π) = 4 π
A. 130o B. 80o C. 65o D. 25o
................................................
Solution:
682. Which of the following is false?
50 + x + x = 180 A. Radii of the same circle are always congruent.
50 + 2x = 180 B. Any two right angles are congruent.
2x = 130 C. An inscribed right angle always intercepts a
x = 65 semicircle.
................................................ D. There are infinitely many lines that may be
drawn parallel to a given line passing through a
679. Which of the following is a square? given point.
A. VCTR whose sides measure 20 cm each
B. LUVS whose angles are 90o each Explanation:
C. RWNA whose diagonals are 50 cm each
There can only be one line that may be drawn
D. AYIE whose congruent diagonals
parallel to a given line passing through a given
perpendicularly bisect each other
point.
................................................
Explanation:
A. VCTR is a rhombus 683. The volume of a cube is 343 cm3. What is its
B. LUVS is a rectangle surface area?
C. RWNA is a rectangle A. 24 cm2 B. 24√3 cm2
D. Only squares have congruent diagonals that C. 48 √13 cm2 D. 294 cm2
are perpendicular bisectors of each other.
................................................ Solution:

680. The following are measures of sides of four S3 = 343


triangles. Which of them are taken from an S=7
obtuse triangle?
A. 20, 20, 29 B. 20, 21, 29 Surface Area = 6s2
C. 20, 22, 28 D. 20, 23, 30 = 6 (72)
= 294
Explanation: ................................................

If A < B < C, then A2 + B2 < C2 if ABC is an 684. Find the volume of a rectangular plank of
obtuse triangle. wood that is half-inch thick, six inches wide, and
A. 202 + 202 < 292  Obtuse 6 feet long.
B. 202 + 212 = 292  Right A. 18 cubic inches B. 36 cubic inches
C. 202 + 222 > 282  Acute C. 108 cubic inches D. 216 cubic inches
D. 202 + 232 > 302  Acute

This is a free reviewer. All rights reserved.


1000 MMR Author: Victor A. Tondo Jr., LPT
Solution: Solution:
V=LxWxH MIA = 180 –
V = 72 x 6 x ½  6 feet is 72 inches
V = 216 MIA = 180 –
................................................ MIA = 180 – 40
MIA = 140
685. In the following figure, O is the center of the ................................................
circle. If m CAB = 36o, what is m DBA?
687. Find the surface area of a ball whose radius
is 24 cm.
A. 576 π cm2 B. 1152 π cm2
C. 2304 π cm 2 D. 4608 π cm2

Solution:
A. 18o B. 36o C. 54o D. 72o
SA = 4 π r2
Solution: SA = 4 π (242)
SA = 2304 π
Most people would solve for m DBA by doing ................................................
this long solution:
Measure of arc BC = 2 x 36 = 72 688. Give the converse of the conditional
Since chord AC passes through center O, then AC statement “If two angles are vertical angles, then
is a diameter, and arc CBA is a semicircle. That they are congruent.”
means arc AB is 180 – 72 = 108. A. “If they are are congruent, then two angles are
Since chord BD passes through center O, then BD vertical angles.”
is a diameter too, making arc BAD a semicircle. B. “If two angles not vertical angles, then they are
That means arc AD = 180 – 108 = 72. not congruent.”
Finally, since DBA intercepts arc AD, then C. “If they are not congruent, then two angles are
m DBA = ½ x 72 = 36. not vertical angles.”
D. “If two angles are linear angles, then they are
That’s a long solution. The best way to deal with supplementary.”
this is to remember that OA and OB are both
radii of the same circle, making them congruent. Explanation:
That means △OAB is an isosceles triangle, with a
base angle at OAB (same angle as CAB). Since The converse of a conditional is formed by
the base angles of an isosceles triangle are interchanging the if-statement (hypothesis) and
congruent, then OBA (same angle as DBA) is the then-statement (conclusion).
the same measure as OAB, or 36o. ................................................
................................................

686. What is the measure of each interior angle


of a regular nonagon?
A. 135o B. 140o
C. 144 o D. 145o

This is a free reviewer. All rights reserved.


1000 MMR Author: Victor A. Tondo Jr., LPT
689. Give the inverse of the conditional 692. In which geometry are the summit angles of
statement “If two angles are vertical angles, then a Saccheri quadrilateral obtuse angles?
they are congruent.” A. Euclidean Geometry
A. “If they are are congruent, then two angles are B. Hyperbolic Geometry
vertical angles.” C. Elliptic Geometry
B. “If two angles not vertical angles, then they are D. Plane Geometry
not congruent.”
C. “If they are not congruent, then two angles are Explanation:
not vertical angles.”
Euclidean: right angles
D. “If two angles are linear angles, then they are
Hyperbolic: acute angles
supplementary.”
Elliptic: obtuse angles
................................................
Explanation:
The inverse of a conditional is formed by 693. How many diagonals does a regular
negating both the if-statement (hypothesis) and decagon have?
the then-statement (conclusion). A. 30 B. 35 C. 40 D. 45
................................................
Solution:
690. In which geometry do two distinct lines
D = n(n – 3)/2
intersect in two points?
................................................
A. Euclidean Geometry
B. Hyperbolic Geometry
694. Each side of a regular hexagon is 256 cm
C. Elliptic Geometry
long. Find the distance around it.
D. Plane Geometry
A. 1536 cm B. 1280 cm
C. 1024 cm D. 718 cm
Explanation:
In Elliptic Geometry, two distinct lines intersect Solution:
in two points. In both Euclidean and Hyperbolic P=nS
Geometry, two distinct lines will intersect at only P = 6 (256)
one point. P = 1536
................................................ ................................................

691. In which geometry can the sum of the 695. What can be said about these statements?
measures of the angles of a triangle be less than i. Any rectangle has two congruent diagonals.
180 degrees? ii. Any rhombus has two perpendicular
A. Euclidean Geometry diagonals.
B. Hyperbolic Geometry A. Only the first statement is true.
C. Elliptic Geometry B. Only the second statement is true.
D. Plane Geometry C. Both statements are true.
D. Both statements are false.
Explanation: ................................................
Euclidean: sum is exactly 180 degrees
Hyperbolic: sum is less than 180 degrees
Elliptic: sum is more than 180 degrees

This is a free reviewer. All rights reserved.


1000 MMR Author: Victor A. Tondo Jr., LPT
696. In the figure, A is the midpoint of and Solution:
. By which triangle congruence postulate can P = 4S
we prove that △CAB △DAE? 280 = 4S A = S2
70 = S A = 702 = 4900
................................................

699. Find the altitude to the hypotenuse of a


right triangle whose legs are 60 cm and 80 cm.
A. 40 cm B. 48 cm
C. 96 cm D. 100 cm

Solution:
A. SSS Congruence Postulate Solve for the hypotenuse first using Pythagorean
B. SAS Congruence Postulate Theorem.
C. ASA Congruence Postulate 602 + 802 = H2
D. ASS Congruence Postulate 3600 + 6400 = H2
10000 = H2
Explanation: 100 = H
AB = AE and AD = AC since A is the midpoint of Then use the following formula:
the segments and . Their included angles, Alt to Hyp =
( )
DAE and CAB, are also congruent because
( )
vertical angles are congruent. Alt to Hyp = = 48
When two sides and an included angle of a ................................................
triangle are congruent to two sides and an
included angle of another triangle, then the two 700. This is formed by two rays with a common
triangles are congruent by SAS Congruence end point.
Postulate. A. polygon B. angle
................................................ C. line D. line segment
................................................
697. Find the circumference of a coin whose
radius is 13 cm.
701. Convert rad to degrees.
A. 13 π cm B. 26 π cm
C. 169 π cm D. 676 π cm A. 210o B. 420o
C. 630o D. 840o
Solution:
Solution:
C=2πr
C = 2 π (13) = 26 π rad x = 210o
................................................ ................................................

698. Find the area of a square whose perimeter 702. What is the reference angle of 212o?
is 280 cm. A. 32o B. 68o C. 106o D. 12o
A. 1120 cm2 B. 4900 cm2
C. 19600cm 2 D. 78400 cm2 Solution:
Since 212o is in QIII, then 212o – 180o = 32o

This is a free reviewer. All rights reserved.


1000 MMR Author: Victor A. Tondo Jr., LPT
703. A negative sine and a positive cosine are
properties of angles in which quadrant?
A. QI B. QII C. QIII D. QIV

Explanation:
Use the CAST mnemonic. See item #164.
................................................ 50√3 = x√3
50 = x
704. Which of the following equal to sec 50o? ................................................
A. sin 40o B. cos 40o
C. csc 40o D. cot 40o 707. The longest side in a 30-60-90 triangle is
480 cm. How long is the shortest side?
Explanation: A. 120 cm B. 160 cm
C. 240 cm D. 360 cm
Apply the cofunctions. The COfunction for secant
is COsecant. The COmplement of 50o is 40o. Solution:
................................................
2x = 480
705. Which of the ff. is coterminal with 2019o? x = 240
A. 219o B. 209o ................................................
C. 199o D. 189o
708. If the hypotenuse of a 45-45-90 triangle is
Solution: 50 cm long, how long is a leg?
A. 25 cm B. 25√2 cm
2019/360 = 5.608333…  Grab 5 only C. 25√3 cm D. 50√2 cm
2019 – 5(360) = 219
................................................ Solution:
706. The angle of elevation to the top of a x√2 = 50
building is 30o. If the observer is 50√3 meters x = 25√2
away from the building, how tall is the building? ................................................
A. 25 m B. 25√3 m
C. 50 m D. 100 m 709. Which of the ff. is coterminal with 143o?
A. 2403o B. 2303o
Solution: C. 2103o D. 2003o
tan 30 = Solution:


= A. 2403 – 143 = 2260 (not divisible by 360)

50√3 (√3) = 3x B. 2303 – 143 = 2160 (divisible by 360)
150 = 3x C. 2103 – 143 = 2060 (not divisible by 360)
50 = x D. 2003 – 143 = 1960 (not divisible by 360)

Alternative Solution: Remember:

Use the 30-60-90 map. (You have to know this by Two angle measures are coterminal if and only if
heart.) their difference a multiple of 360.
................................................

This is a free reviewer. All rights reserved.


1000 MMR Author: Victor A. Tondo Jr., LPT
710. What is the measure of the smaller angle Explanation:
formed by the hands of the clock at 5:55?
Refer to this mnemonic hexagon. Any
A. 180o B. 174.75o
trigonometric function is equal to (katabi)
C. 163.625o D. 152.5o
divided by (susunod sa katabi).
Solution: That means tan = sin ÷ cos or tan = sec ÷ csc,
sin = cos ÷ cot or sin = tan ÷ sec, etc.
A = |30H – 5.5M|
A = |30(5) – 5.5(55)|
A = |150 – 302.5|
A = 152.5
................................................

711. What is the measure of the smaller angle


formed by the hands of the clock at 3:50?
A. 185o B. 175o ................................................
C. 170o D. 165o
714. If sin x = 0.936, which of the following could
Solution: be cos x?
A. 0.064 B. 0.8
A = |30H – 5.5M|
C. 0.016 D. -0.352
A = |30(3) – 5.5(50)|
A = |90 – 275|
Solution:
A = 185
sin2 x + cos2 x = 1  Trig. Identity
However, 185o is a reflex angle. The smaller
(0.936)2 + cos2 x = 1
angle is 360 – 185 or 175o.
cos2 x = 1 – 0.876096
cos2 x = 0.123904
cos x = ±0.352

Alternative Solution:
Input cos (sin-1 (0.936)) in your calculator.
................................................
................................................
715. A man stands 20 meters away from a
712. Which of the following is false? building. If the angle of elevation to the top of the
A. tan θ = building is 60o, how tall is the building?
B. csc θ = A. 20√2 m B. 20√3 m
C. 40 m D. 20√5 m
C. sec θ =
D. sin θ = Explanation:
Plot your entries in the 30-60-90 triangle and
solve. See item #706.
................................................

716. If csc x = , what is sin x?


A. B. C. D.

This is a free reviewer. All rights reserved.


1000 MMR Author: Victor A. Tondo Jr., LPT
Explanation: 720. Convert 160o to radians.
sin x is the reciprocal of csc x. A. π B. π C. π D. π
................................................
Solution:
717. If sec x = , which of the following could be
160o x = π
sin x?
................................................
A. B. C. D.
721. What is the reference angle of 156o?
Solution: A. 14o B. 24o C. 56o D. 66o

Since sec x = , then cos x = . Explanation:


sin2 x + cos2 x = 1
Since 156o is in QII, then the reference angle is
sin2 x + ( )2 = 1 180 – 156 or 24o.
sin2 x + =1 ................................................

sin2 x = 1 – 722. In which quadrants is the sine function


negative?
sin2 x =
A. QI and QII B. QII and QIII
sin x = C. QIII and QIV D. QIV and QI
................................................
Explanation:
718. Which of the following is false? Use the CAST mnemonic. See item #164.
A. cos A (tan A) = sin A ................................................
B. csc A (cos A) = cot A
C. sin A (sec A) = tan A 723. What is the reference angle if 2019o?
D. tan A (csc A) = cos A A. 19o B. 39o C. 51o D. 59o
Explanation: Explanation:
Refer to the mnemonic hexagon in #712. Kahit The coterminal angle for 2019o is 219o (see item
aling trig function ay product ng dalawang trig #705). Since 219o is in QIII, then its reference
functions na katabi niya. angle is 219 – 180 or 39o.
................................................ ................................................
719. Which of the ff. is coterminal with 63o? 724. A sniper is on top of a 20-meter cliff. He
A. 7503o B. 7303o spots a target at an angle of depression of 30o.
C. 7103o D. 6903o How far must his bullet travel to hit the target?
A. 20 meters B. 20 √2 meters
Solution:
C. 20 √3 meters D. 40 meters
A. 7503 – 63 = 7440 (not divisible by 360)
B. 7303 – 63 = 7240 (not divisible by 360) Solution:
C. 7103 – 63 = 7040 (not divisible by 360)
D. 6903 – 63 = 6840 (divisible by 360)
................................................
x = 20 m
.: 2x = 40 m
This is a free reviewer. All rights reserved.
1000 MMR Author: Victor A. Tondo Jr., LPT
725. A meter stick leans to a wall and reaches a 728. Evaluate: 2 sin 30 – 4 cos 60 + 3 tan 45.
point 50 cm away from the wall. What is the A. -10.2378 B. -12.1426
measure of the angle formed by the meter stick C. 2 D. 1
and the wall?
A. 30o B. 45o C. 60o D. 75o Solution:
2 sin 30 – 4 cos 60 + 3 tan 45
Solution:
= 2 (0.5) – 4 (0.5) + 3 (1)
= =1–2+3
=2
sin x = 0.5 ................................................
x = arcsin 0.5
x = 30o 729. Which of the following is true?
A. sin x = sin (-x) B. cos x = cos (90 – x)
................................................ C. –sin x = sin (-x) D. tan x = (sin x)(cos x)
726. A tight rope connects the top of a 500 cm Explanation:
pole to the ground. If the angle formed by the
ground and the tight rope is 45o, how long is the A/C: –sin x = sin (-x)
tight rope? B: cos x = sin (90 – x)
A. 500 cm B. 500√2 cm D. tan x = (sin x) ÷ (cos x)
C. 500√3 cm D. 500√5 cm ................................................

Solution: 730. Which of the following is true?


A. sin x = csc (90 – x)
= B. cos x = csc (90 – x)
C. tan x = cot (90 – x)
sin 45 = D. sec x = cos (90 – x)
x sin 45 = 500
x= Explanation:
x = 500√2 Definition of cofunctions. See #704.
................................................ ................................................
727. Which of the
following can be used 731. Evaluate: lim 2019
to solve for x in the A. 2019 B. -2019
figure? C. 0 D. 1
A. sin
Explanation:
B. cos
The limit of a constant function is the constant
C. arcsin itself.
D. arccos ................................................

732. Evaluate: lim (20 – 5x2)


Explanation:
A. 1 B. 15 C. -15 D. 10
is the ratio of the adjacent side to the
hypotenuse. That’s the cosine value for x. Since Solution:
the angle x is missing, then we will use arccos . Substitute x with 1.
20(1) – 5(12) = 20 – 5 = 15
This is a free reviewer. All rights reserved.
1000 MMR Author: Victor A. Tondo Jr., LPT
733. Evaluate: lim ( 3 – 5x2) 737. Give the domain of f(x) =
A. 3 B. 18 C. -18 D. 72
A. x | x B. x
Solution: C. x 2 D. x -2
Substitute x with 3. Explanation:
33 – 5(32) = 27 – 45 = -18
................................................ The given function is a rational function. It will
be undefined when the denominator is 0.
Therefore, 3x – 6 0, or x 2.
734. Evaluate: lim
................................................
A. LDNE B. 2 C. 4 D. 0
738. Give the domain of f(x) = √2018 2019.
Solution:
A. x | x B. x
lim C. x > 0 D. x
( )( )
= lim
= lim ( + 2) Explanation:
=4 The given function is a radical function, more
................................................ specifically a square root function. If the
radicand 2018x – 2019 is negative, f(x) becomes
735. Evaluate: lim imaginary. That means 2018x – 2019 must be
A. + B. - C. LDNE D. 2 greater than or equal to 0.
2018x – 2019 0
Explanation: 2018x 2019
x
Retain only the highest term of the numerator
and the denominator. ................................................

lim  lim 739. Give the domain of f(x) = x2 – 20x + 19.


lim = lim 2 A. x | x B. x 1, 9
C. x > -81 D. x -81
Since x approaches + , and the function is 2x2,
then the limit is + . Explanation:
................................................
The given function has no denominator and no
radicand. Therefore f(x) exists for all real values
736. Evaluate: lim of x.
A. + B. - C. LDNE D. ................................................

Explanation: 740. Give the range of f(x) = 2020.


A. y | y B. y 2020
Retain only the highest term of the numerator C. y = 2020 D. y = { }
and the denominator.
lim  lim Explanation:

lim The given function is a constant function. That


means y = 2020 for all values of x.
................................................ ................................................

This is a free reviewer. All rights reserved.


1000 MMR Author: Victor A. Tondo Jr., LPT
741. Give the range of f(x) = 744. Give the range of f(x) =
A. y | y B. y A. y | y B. y
C. y 2 D. y -2 C. y D. y -2

Solution:
Solution:
First, express y in terms of x.
First, express y in terms of x.
y=
y=
3xy – 6y = 2x + 3
6xy + 7y = 5x - 4
3xy – 2x = 6y + 3
6xy – 5x = -7y – 4
x(3y – 2) = 6y + 3
x (6y – 5) = -7y - 4
x=
x=
The denominator 3y – 2 cannot be equal to 0.
The denominator 6y – 5 cannot be equal to 0.
3y – 2 0
6y – 5 0
3y 2
6y 5
y
y

Shortcut:
Shortcut:
Since the numerator and denominator are of the
Since the numerator and denominator are of the
same degree, just take the leading coefficients of
same degree, just take the leading coefficients of
the numerator (2) and denominator (3).
the numerator (5) and denominator (6).
Proof:
Proof:
 Cross-multiply.
 Cross-multiply.
6x + 9 6x – 12
5(6x + 7) 6(5x – 4)
9 + 12 6x – 6x
30x + 35 30x – 24
21 0
30x – 30x -24 – 35
................................................
0 -59
................................................
742. Give the range of f(x) = 20x – 20.
A. y | y B. y 1
745. Give the range of f(x) = x2019.
C. y = 20 D. y = { }
A. y | y B. y 0
C. y 2019 D. y = { }
Explanation:
All linear functions have a range of y | y . Explanation:
................................................
(See item #743.)
................................................
743. Give the range of f(x) = x125.
A. y | y B. y 0
C. y 2019 D. y = { }

Explanation:
The function has an odd degree (125). All odd-
degree functions have a range of y | y .
This is a free reviewer. All rights reserved.
1000 MMR Author: Victor A. Tondo Jr., LPT
746. Give the range of f(x) = 2019x – 2019. 751. Find the first derivative: f(x) = (2x + 3)5.
A. y | y B. y 1 A. f ’(x)= 5 (2x + 3)4
C. y = 2019 D. y = { } B. f ’(x)= 10 (2x + 3)4
C. f ’(x)= 15 (2x + 3)4
Explanation: D. f ’(x)= 20 (2x + 3)4
All linear functions have a range of y | y .
Solution:
................................................
Let u = 2x + 3 so we can rewrite f(x) = u5.
747. Give the range of f(x) = Since du = 2, and f ’(x)= 5u4 du, then by
substitution,
A. y | y B. y f ’(x)= 5u4 du
C. y D. y f ’(x)= 5(2x + 3)4 (2)
f ’(x)= 10 (2x + 3)4
Explanation: ................................................

(See item #741.) 752. Find f ‘(x) given f(x) = (3x2 – 2x + 1 )7.
................................................ A. f ’(x)= (42x – 14) (3x2 – 2x + 1)6
B. f ’(x)= (42x – 28) (3x2 – 2x + 1)6
748. Give the domain of f(x) = C. f ’(x)= (42x + 14) (3x2 – 2x + 1)6
D. f ’(x)= (42x + 28) (3x2 – 2x + 1)6
A. x | x B. x
C. x 2 D. x -3 Solution:

Explanation: Let u = 3x2 – 2x + 1 so we can rewrite f(x) = u7.


Since du = 6x – 2, and f ’(x)= 7u6 du, then by
The given function is a rational function. It will substitution,
be undefined when the denominator is 0. f ’(x)= 7u6 du
Therefore, 3x + 9 0, or x -3. f ’(x)= 7(3x2 – 2x + 1)6 (6x – 2)
................................................ f ’(x)= (42x – 14) (3x2 – 2x + 1)6
................................................
749. What is 0! equal to?
A. 0 B. 1 753. Find the maximum area that can be
C. undefined D. + enclosed by a rectangle given its perimeter
................................................ should only be 202 meters.
A. 2550 m2 B. 2550.25 m2
750. Give the range of f(x) = 2019x + 2020. C. 2550.5 m 2 D. 2550.75 m2
A. y | y B. y 2020
C. y 2019 D. y 2020 Solution:

Explanation: Max area for such a question is taken by making


the rectangle a square.
The function is an exponential function P = 4S
increased by a constant (2020). The exponential 4S = 202
part 2019x is asymptotic to 0 (it will come super S = 50.5
close to 0 but will never be equal to 0) and can
only be positive. Add this to the constant 2020 A = S2
and you’ll get y 2020. S2 = 50.52
50.52 = 2550.25

This is a free reviewer. All rights reserved.


1000 MMR Author: Victor A. Tondo Jr., LPT
754. Find the lowest possible value of the 757. Find the slope of the line tangent to
function f(x) = x2 – 8x + 7. y = 3x5 – 4x3 + 5x2 – 10x + 9 at x = 2.
A. -10 B. -9 C. -8 D. -7 A. 50.5 B. 101 C. 151.5 D. 202

Solution: Solution:
Get the critical value of x first by getting the first Find f ’(2).
derivative and equating it to 0. f ’(x) = 15x4 – 12x2 + 10x – 10
f ’(2) = 15(2)4 – 12(2)2 + 10(2) – 10
f ‘(x) = 2x – 8
f ’(2) = 240 – 48 + 20 – 10
2x – 8 = 0
f ’(2) = 202
2x = 8
................................................
x=4  Critical Point
Get the minimum by get f(crit. value). 758. Find the remainder when 1998! is divided
f(4) = 42 – 8(4) + 7 by 2000.
f(4) = 16 – 32 + 7 A. 1024 B. 256 C. 64 D. 0
f(4) = -9
Explanation:
Shortcut:
1998! = 1 x 2 x 3 x … x 1000 x …
The minimum (for A>0) or maximum (for A<0) Therefore 1998 is divisible by 2000.
of f(x) = Ax2 + Bx + C is given by C – .
Also Note:
C – = 7 – = 7 – 16 = -9
(n – 2)! is divisible by n if and only if n is a
................................................ composite number greater than 4.
................................................
755. Find the instantaneous rate of change for
f(x) = 5x3 – 2x2 + 7x – 9 at x = 2. 759. In Mathematics, what does the delta ∆
A. 37 B. 48 C. 59 D. 70 symbol mean or refer to?
A. the change in B. the summation of
Solution: C. therefore D. belonging to
Find f ’(2). ................................................
f ’(x) = 15x2 – 4x + 7
f ’(2) = 15(22) – 4(2) + 7 760. In Mathematics, what does the sigma
f ’(2) = 59 symbol mean or refer to?
................................................ A. the change in B. the summation of
C. therefore D. belonging to
756. Find the instantaneous rate of change for ................................................
f(x) = 4x3 + 5x2 – 7x – 11 at x = 1.
A. -15 B. 0 C. 15 D. 30 761. In Mathematics, what does the epsilon
symbol mean or refer to?
Solution: A. such that B. element of
C. therefore D. ergo
Find f ’(1). ................................................
f ’(x) = 12x2 + 10x – 7
f ’(1) = 12(12) + 10(1) – 7 762. In set theory, what does the symbol mean
f ’(1) = 15 or refer to?
A. not O B. not zero
C. does not D. empty
This is a free reviewer. All rights reserved.
1000 MMR Author: Victor A. Tondo Jr., LPT
763. Which of the following is ALWAYS an odd 770. Given X = {1, 2, 4, 8} and Y = {1, 2, 3, 4}, find
number? X – Y.
A. The difference of two odd numbers. A. {1, 4, 8} B. {8}
B. The sum of two odd numbers. C. {3, 8} D. {1, 2, 3, 4, 8}
C. The product of two odd numbers.
D. The product of two even numbers. Explanation:
................................................
X – Y means tanggalin mo sa X lahat ng elements
na meron din sa Y.
764. Given 2x + 2x = 2y, express y in terms of x.
................................................
A. y = 2x B. y = x2
C. y = x + 2 D. y = x + 1
771. Given X = {1, 2, 4, 8} and Y = {1, 2, 3, 4}, find
Y – X.
Explanation:
A. {1, 4, 8} B. {3}
2x + 2x = 2y C. {3, 8} D. {1, 2, 3, 4, 8}
2(2x) = 2y ................................................
2x+1 = 2y
................................................ 772. Given X = {6, 5, 4, 3} and Y = {1, 2, 3, 4}, find
X Y.
A. {3, 4} B. {6, 5}
765. In set theory, what does the symbol mean C. {1, 2} D. {1, 2, 3, 4, 5, 6}
or refer to?
A. no elements B. common elements Explanation:
C. all elements D. half of the elements
X Y means the union of all the elements of X
................................................
and Y.
766. In Mathematics, what does the symbol
mean?
773. Which of the
A. since B. prove
following
C. therefore D. previously
represents the
................................................
shaded region in
the Venn diagram?
767. Which of the following is the Geometric
A. A B B. A B
symbol for congruent?
C. A – B D. B – A
A. B. C. D. //
................................................

768. Which of the following is the obelus sign?


774. Which of the
A. ! B. ÷ C. ⊙ D. θ following
................................................ represents the
shaded region in
769. Given X = {1, 4, 16, 64} and Y = {1, 2, 3, 4}, the Venn diagram?
find X Y. A. A’ B’ B. A’ B’
A. {1, 4} B. {16, 64} C. (A B)’ D. (A B)’
C. {2, 3} D. {1, 2, 3, 4, 16, 64}
Explanation:
Explanation:
The area that is not shaded is A B. The shaded
X Y refers to the common elements of X and Y. region is its complement, (A B)’.
This is a free reviewer. All rights reserved.
1000 MMR Author: Victor A. Tondo Jr., LPT
775. In a class of 50, there are 20 students who Solution:
excel in Math, 24 who excel in Science, and 4 who
7 x 6 x 5 x 4 = 840
excel in both Math and Science. How many of
................................................
them excel in neither?
A. 2 B. 4 C. 7 D. 10
780. Using the digits 0, 1, 2, 3, 4, 5, and 6 without
repetition, how many 3-digit numbers can be
Solution A:
made?
50 – (20 – 4) – (24 – 4) – 4 = 10 A. 360 B. 180 C. 90 D. 45

Solution B: Solution:
50 – (20 + 24 – 4) = 10 6 x 6 x 5 = 180
................................................ ................................................

776. There are 50 students in a class. 39 of them 781. Using the digits 0, 1, 2, 3, 4, 5, 6, and 7
have a cellphone, 17 of them have a laptop, and 5 without repetition, how many 4-digit numbers
of them have neither. How many have both a can be made?
cellphone and a laptop? A. 1680 B. 1470 C. 840 D. 735
A. 6 B. 7 C. 9 D. 11
Solution:
Solution:
7 x 7 x 6 x 5 = 1470
(39 + 17) – (50 – 5) = 56 – 45 = 11 ................................................
................................................
782. Using the digits 0, 1, 2, 3, 4, 5, 6, and 7
777. Rayon has 10 shirts, 5 pairs of pants, and 4 without repetition, how many 3-digit numbers
pairs of shoes. How many ways can he pick what can be made?
to wear for today? A. 672 B. 588 C. 336 D. 294
A. 19 B. 20 C. 190 D. 200
Solution:
Solution:
7 x 7 x 6 = 294
10 x 5 x 4 = 200 ................................................
................................................
783. Using the digits 0, 1, 2, 3, 4, 5, and 6 without
778. Using the digits 1, 2, 3, 4, 5, and 6 without repetition, how many odd 3-digit numbers can
repetition, how many 3-digit numbers can be be made?
made? A. 180 B. 150 C. 90 D. 75
A. 480 B. 240 C. 120 D. 60
Solution:
Solution:
5 x 5 x 3 = 75
6 x 5 x 4 = 120 ................................................
................................................
784. Using the digits 0, 1, 2, 3, 4, 5, 6, and 7
779. Using the digits 1, 2, 3, 4, 5, 6, and 7 without without repetition, how many odd 4-digit
repetition, how many 4-digit numbers can be numbers can be made?
made? A. 1470 B. 1440 C. 735 D. 720
A. 1680 B. 840 C. 420 D. 210

This is a free reviewer. All rights reserved.


1000 MMR Author: Victor A. Tondo Jr., LPT
Solution: 789. Evaluate: log6 8 + 3 log6 3.
A. 24 B. 14 C. 9.5 D. 3
6 x 6 x 5 x 4 = 720
................................................
Solution:
785. Using the digits 0, 1, 2, 3, 4, 5, 6, and 7 log6 8 + 3 log6 3
without repetition, how many odd 3-digit = log6 8 + log6 33
numbers can be made? = log6 8 + log6 27
A. 336 B. 288 C. 168 D. 144 = log6 (8x27)
= log6 216
Solution: =3
................................................
6 x 6 x 4 = 144
................................................
790. How many ways can the word ABABA be
rearranged?
786. Using the digits 0, 1, 2, 3, 4, 5, 6, and 7
A. 120 B. 60 C. 20 D. 10
without repetition, how many even 3-digit
numbers can be made?
Solution:
A. 162 B. 156 C. 150 D. 144
!
= 10 (See item #186.)
! !
Solution:
................................................
When asked about even n-digit numbers tapos
may 0 sa usable digits, do this: 791. There are 10 different beads and a locking
3-digit #s: 7 x 7 x 6 = 294 mechanism to be used in making a bracelet. How
odd 3-digit #s: 6 x 6 x 4 = 144 many different bracelet patterns can be made?
even 3-digit #s: 150 A. 3,628,800 B. 362,880
................................................ C. 1,814,400 D. 181,440

787. Using the digits 0, 1, 2, 3, 4, 5, 6, and 7 Solution:


without repetition, how many even 4-digit
n = 11 here since the locking mechanism is
numbers can be made?
considered as another unit
A. 765 B. 750 C. 735 D. 720 ( )! !
The formula is . Thus, there are or
Solution: 1,814,400 different patterns.
................................................
4-digit #s: 7 x 7 x 6 x 5 = 1470
odd 4-digit #s: 6 x 6 x 5 x 4 = 720
792. There are six different Math books, four
even 4-digit #s: 750
different Science books, and two different
................................................
English books to be arranged on a shelf. How
many ways can this be done?
788. Using the digits 0, 1, 2, 3, 4, 5, and 6 without
A. 12C3 B. 12! C. 48 D. 3!
repetition, how many even 3-digit numbers can
be made?
Explanation:
A. 120 B. 105 C. 90 D. 75
There are 12 different books to be arranged in a
Solution: linear fashion. There are 12! ways to do this.
3-digit #s: 6 x 6 x 5 = 180
odd 3-digit #s: 5 x 5 x 3 = 75
even 3-digit #s: 105
This is a free reviewer. All rights reserved.
1000 MMR Author: Victor A. Tondo Jr., LPT
793. There are six different Math books, four 796. TNB Gaming has five “carry” players, four
different Science books, and two different “offlane” players, and six “support” players. If a
English books to be arranged on a shelf. If books game calls for two carry, one offlane, and two
of the same subject must be together, how many support players, how many possible lineups do
ways can this be done? they have?
A. 479,001,600 B. 207,360 A. 1200 B. 600 C. 300 D. 150
C. 34560 D. 144
Solution:
Explanation:
5C2 x 4C1 x 6C2 = 10 x 4 x 15 = 600
There are 3 x 2 x 1 or 3! ways to arrange the ................................................
subjects Math, Science, and English. There are 6!
ways to arrange the Math books, 4! ways for 797. If today is a Sunday, what day is 125 days
Science, and 2! ways for English. from now?
3! x 6! x 4! x 2! = 6 x 720 x 24 x 2 = 207360 A. Friday B. Saturday
................................................ C. Sunday D. Monday

794. There are six people to be seated on a bench Explanation:


for a picture. A certain couple, Vic and Rowena,
Exactly 18 weeks or 126 days from now would
are to be seated next to each other. How many
be Sunday again, therefore, 125 days from now
ways can this be done?
would be Saturday.
A. 120 B. 240 C. 360 D. 720
................................................
Explanation:
798. If today is a Monday, what day is 200 days
Let Vic and Rowena take one unit, and then add from now?
the other four people to make five units. Since A. Friday B. Saturday
Vic and Rowena can be arranged in 2! ways and C. Sunday D. Monday
the five units can be arranged in 5! ways, then
the total number of ways this can be done is the Explanation:
product of 2! and 5!, or 240.
Exactly 28 weeks or 196 days from now would
................................................
be Monday again, therefore, 200 days from now
would be Friday.
795. There are six people to be seated on a bench
................................................
for a picture. A certain trio, Vic, Rayon, and Aira,
do not want to be separated. How many ways
799. If three-fourths of a number is 33 more than
can this be done?
one-fifth of itself, then what is the number?
A. 60 B. 72 C. 144 D. 180
A. 80 B. 60 C. 40 D. 20
Explanation:
Solution:
Let the trio take one unit, and then add the other
three people to make four units. The trio can be = + 33  Multiply equation by 20.
arranged in 3! ways and the four units can be 15x = 4x + 660
arranged in 4! ways. 3! x 4! = 144. 11x = 660
................................................ x = 60

This is a free reviewer. All rights reserved.


1000 MMR Author: Victor A. Tondo Jr., LPT
800. What time is 2019 hours after 3:00PM? 805. There are ten red, nine blue, and six black
A. 6:00 AM B. 12:00 NN pens in a bag. If a pen is randomly drawn from
C. 6:00 PM D. 12:00 MN the bag, what is the probability that it is black?
A. 0.06 B. 0.6 C. 0.24 D. 0.7
Explanation: ................................................
2019 ÷ 24 = 84 r. 3 806. What does a probability of 0 pertain to?
3 hours after 3 PM is 6 PM A. an impossible event
................................................ B. a very unlikely event
C. a very likely event
801. In a certain fastfood chain, soft drinks are D. a sure event
served in Small, Medium, and Large cups. ................................................
What level of data is this?
A. Nominal B. Ordinal
807. Simplify: √10 + 2√21.
C. Interval D. Ratio
A. √2 + √5 B. √3 + √7
Explanation: C. √3 √2 D. √5 √2

The data is presented according to the order of Explanation:


sizes, thus, ordinal.
................................................ (√a + √b)2 = a + b + 2√
Therefore,
802. The prime factorization of a number is
given as 23 x 52 x 133. How many factors does it √a + b + 2√ab = √a + √b
have? In the problem, a + b = 10 and ab = 21. The only
A. 96 B. 48 C. 36 D. 18 pair that has a sum of 10 and a product of 21 is 3
and 7.
Solution: ................................................
Add 1 to the exponent of each prime factor, then
multiply to get the number of factors. 808. Simplify: √11 2√30.
(3 + 1)(2 + 1)(3 + 1) = 4 x 3 x 4 = 48 A. √6 √5 B. √6 + √5
................................................ C. √3 2√2 D. 1

803. The prime factorization of a number is Explanation:


given as 22 x 32 x 7 x 11. How many factors does
( √a √b)2 = a + b – 2√
it have?
A. 96 B. 48 C. 36 D. 18 Therefore,
√a + b 2√ab = √a √b
Solution:
In the problem, a + b = 11 and ab = 30. The only
(2 + 1)(2 + 1)(1 + 1)(1 + 1) = 36 pair that has a sum of 11 and a product of 30 is 6
................................................ and 5.
................................................
804. There are ten red, nine blue, and six black
pens in a bag. If a pen is randomly drawn from 809. ∫(3 + 4)5 dx
the bag, what is the probability that it is red?
A. (3x + 4)6+ c B. (3x + 4)6+ c
A. B. C. D.
C. 5(3x + 4)4+ c D. (3x + 4)6+ c

This is a free reviewer. All rights reserved.


1000 MMR Author: Victor A. Tondo Jr., LPT
Explanation: 813. Mr. dela Cruz is going to present the growth
of a certain plant over a span of 10 weeks. Which
∫(3 + 4)5 dx = ( )( )
(3x + 4)5+1 + c graph would be best suited for this?
∫(3 + 4)5 dx = (3x + 4)6 + c A. bar graph B. line graph
C. pie graph D. pictograph
................................................
Explanation:
810. ∫(6 + 6 6) dx
A. 2x3 + 3x2 + 6x + c B. 2x3 – 3x2 + 6x + c Line graphs are best suited for presenting data
C. 2x3 + 3x2 – 6x + c D. 2x3 – 3x2 – 6x + c on development over time.
................................................
Solution:
814. Rayon is going to present the sales of six
∫(6 +6 6) dx
different teams in the company from January to
= + – 6x + c June. Which graph would be best suited for this?
= 2x3 + 3x2 – 6x + c A. bar graph B. line graph
................................................ C. pie graph D. pictograph
................................................
811. ∫ dx
815. Ms. Rowena is tasked to send 8 randomly
A. + c B. + c selected students from her class of 40 to the
C. +c D. +c clinic for drug test. She decides to let the
students count off from 1 to 5, after which she
randomly picks a number from 1 to 5. All
Solution: students whose number is that which she picked
will be sent to the clinic. What sampling method
∫ dx = ∫ dx did Ms. Rowena use?
= +c A. cluster B. systematic
C. stratified D. quota
= +c ................................................
= +c
816. What is the 7th decile of the following data?
................................................
14 15 16 20 24
25 26 26 27 29
812. ∫ dx 31 33 34 34 35
A. +c B. +c A. 31 B. 31.4 C. 31.5 D. 32
C. +c D. +c

Solution:
Solution:
(15 + 1)th data or 11.2th data
∫ dx = ∫ dx
11th data is 31, while 12th data is 33
= +c 11.2th data = 31 + 0.2(33-31)
= +c = 31 + 0.4
= 31.4
= +c

This is a free reviewer. All rights reserved.


1000 MMR Author: Victor A. Tondo Jr., LPT
817. Find the remainder when f(x) is divided by 821. ( ) = _________
(x – 3) given f(x) = x3 – 3x2 + 5x – 13.
A. 1 B. 2 C. 3 D. 4 A. ( )
B. (
Solution: )
C. (
Since the divisor is x – 3, then the remainder is )

given by f(3). D. ( )
f(3) = 33 – 3(32) + 5(3) – 13
f(3) = 27 – 27 + 15 – 13 Solution:
f(3) = 2
................................................ u = 2x + 1 du = 2
v = 3x – 4 dv = 3
818. (x2 + 3x – 2)4 = _________ ( ) =
A. (x2 + 3x – 2)3 ( )( ) ( )( )
B. 4(x2 + 3x – 2)3 = ( )
C. (8x + 12) (x2 + 3x – 2)3 ( ) ( )
= ( )
D. (2x + 3) (x2 + 3x – 2)3
=( )
Solution: ................................................
Let u = x2 + 3x – 2 .: du = 2x + 3
822. There are 17 red bags, 19 green bags, and
u4= du4u3 14 blue bags in a store. What percent of the bags
= 4 (x2 + 3x – 2)3 (2x + 3) is blue?
= (8x + 12) (x2 + 3x – 2)3 A. 38%
................................................ B. 34%
C. 28%
D. 27%
819. √ 2 + 3 = _________
Solution:
A. √
B. √
14/(17 + 19 + 14) = 14/50
C. √ D. √ 14/50 = 28%
................................................
Solution:
Let u = x3 – 2x + 3 .: du = 3x2 – 2 823. Which of the following is true?
A. (sin x) (cot x) = sec x
u1/2 = ½ u-1/2 du B. (cos x) (csc x) = tan x
C. (tan x) (csc x) = sec x
= (√ )(3x2 – 2)
D. (cot x) (sec x) = cos x
= √
................................................

820. (2x + 3)(4x – 5) = _________


A. 8x2 + 2x + 15
B. 8x2 + 2x – 15
C. 8x2 – 2x – 15
D. 8x2 – 2x + 15

This is a free reviewer. All rights reserved.


1000 MMR Author: Victor A. Tondo Jr., LPT
Explanation: 827. If f(x) = x2 – 2x + 3 and g(x) = x + 1, find
fog(x).
Refer to the trigonometric hexagon. Any
A. x2 – 1 B. x2 – 2x + 4
trigonometric function is the product of the two
C. x2 – 3x + 5 D. x2 + 2
functions beside it.
Solution:
f(g(x)) = f(x + 1)
= (x + 1)2 – 2 (x + 1) + 3
= (x2 + 2x + 1) – (2x + 2) + 3
= x2 + 2
................................................

828. How many 3-digit numbers can be formed


That means: using the digits 0, 1, 2, 3, 4, 5, and 6 if repetition
(sin x) (cot x) = cos x; is not allowed?
(cos x) (csc x) = cot x; A. 160 B. 180 C. 200 D. 210
(tan x) (csc x) = sec x; and
(cot x) (sec x) = csc x. Solution:
................................................
Use FCP (Fundamental Counting Principle):
__ x __ x __
824. Which of the following could be the value of
For the first digit, we cannot use 0. That means
x if x 4 (mod 10)?
we only have 6 choices for the first digit.
A. 46 B. 49 C. 52 D. 54
For the second digit, we can now use 0. Since we
have already used one digit for the first, that
Explanation:
means we have 6 choices for the second digit.
We are looking for a number x such that x leaves For the last digit, since we have already used two
a remainder of 4 when divided by 10. Simply put, digits, we only have 5 choices.
the number’s units digit is 4. 6 x 6 x 5 = 180
................................................ ................................................

825. Which of the following could be the value of 829. What is the longest side of ∆VAC if m V =
x if x 7 (mod 9)? 45o and m C = 65o?
A. 146 B. 149 C. 151 D. 154 ̅̅̅̅
A. VC ̅̅̅̅
B. AC ̅̅̅̅
C. VA ̅̅̅̅
D. CA

Explanation: Explanation:
We are looking for a number x such that x leaves m A = 180 – (45 + 65) = 70
a remainder of 7 when divided by 9. The longest side is opposite the largest angle, A.
................................................ ................................................

̅̅̅̅ bisects ABC and m ABF = 34o,


826. Given BF 830. Which quadrilateral has two congruent
find m ABC. diagonals that are perpendicular to each other?
A. 17o B. 34o C. 51o D. 68o A. kite B. isosceles trapezoid
C. rectangle D. rhombus
Explanation:
ABF is formed after the bisection of ABC. That
means ABF is half of ABC, or ABC is twice of
ABF.

This is a free reviewer. All rights reserved.


1000 MMR Author: Victor A. Tondo Jr., LPT
831. If the length of a rectangle is increased by 835. The salary of 5 men for 6 days is P9,000.
30% while the width is decreased by 30%, what How much is the salary of 7 men for 8 days?
will happen to its area? A. P15,000 B. P15,600
A. It stays the same. C. P16,800 D. P17,400
B. It is increased by 9%.
C. It is decreased by 9%. Solution:
D. It is decreased by 6%.
5 men x 6 days = 30 units worth a total of P9000.
That means each unit is worth P300.
Solution:
7 men x 8 days = 56 units.
(L x 1.3) (W x 0.7) = (0.91 x LW) 56 units at P300 per unit costs P16800.
................................................ ................................................

832. Rayon had an average of 93 on his first five 836. The average grade of eleven students is 84.
Math tests. After taking the next test, his average If the average of seven of these students is 82,
increased to 94. Find his most recent grade. what is the average of the other four students?
A. 96 B. 97 C. 98 D. 99 A. 88 B. 87.5 C. 86.5 D. 86

Solution: Solution:
New Score = (New Number)(New Average) – Sum of grades of 11 students: 11 x 84 = 924
(Old Number)(Old Average) Sum of grades of 7 students: 7 x 82 = 574
New Score = 6(94) – 5(93) = 564 – 465 = 99 Sum of grades of other four: 924 – 574 = 350
................................................ Average of grades of other four: 350 ÷ 4 = 87.5
................................................
833. A bus drove for 7 hours at 73 kph and 3
hours at 88 kph. What was its average speed? 837. A radius of a circle is 25 cm long. How long
A. 75.5 kph B. 76.5 kph is its longest chord?
C. 77.5 kph D. 78.5 kph A. 20√2 cm B. 40 cm
C. 50 cm D. It depends,
Solution:
Get the total distance and the total time first. Explanation:
7 hrs x 73 kph = 511 km The longest chord is the diameter, and the radius
3 hrs x 88 kph = 264 km is half the diameter.
Total distance = 775 km, total time = 10 hrs ................................................
Average spd = = = 77.5 kph
838. Find the largest area of a rectangular piece
................................................
of land that can be enclosed with 800 meters of
fencing material.
834. Fifteen guests shake hands with each other.
A. 40,000 m2 B. 56,250 m2
If each guest is to shake hands with all the other
C. 62,500 m2 D. 80,000 m2
guests, how many handshakes will be made?
A. 105 B. 150 C. 210 D. 225
Solution:
Solution: Instead of jumping to differential calculus
(minima and maxima) to solve this, simply make
15C2 = 105
the rectangle a square. That’s the shortcut for
this kind of question.

This is a free reviewer. All rights reserved.


1000 MMR Author: Victor A. Tondo Jr., LPT
839. Which statistical test is used for testing for 842. In which non-Euclidean model for geometry
relationship between two variables? can we have any given line ℓ and a point A which
A. ANOVA B. t-test is not on ℓ, wherein all lines through A will
C. Pearson R D. Chi Square intersect ℓ?
A. hyperbolic B. elliptic
Explanation: C. Saccheri D. Pythagorean
Observed vs Expected: Chi Square
Explanation:
Relationship: Pearson R (R for relationship)
Group differences: ANOVA (variance = In Euclidean geometry, only one line will pass
differences) through A. In elliptic geometry, all lines will pass
Comparing sets of normal distributions: T-test through A and intersect ℓ.
................................................ ................................................

2 +4 5 843. If A is at (-9, 11) and B is at (6,-9), find C if C


840. Given ( ) = { 5 = 5, is three-fifths the way from A to B.
9 5 A. (1, -1) B. (0, 1)
find lim ( ). C. (1, 1) D. (0, -1)
A. 5 B. 14
C. 16 D. limit does not exist Solution:

Solution: Since C is three-fifths the way from A to B, then


its coordinates are:
Limit from the left: 2(5) + 4 = 14 x = -9 + [6 – (-9)] = -9 + 9 = 0
Limit from the right: 52 – 9 = 16
Since the limits are not equal, then the limit does y = 11 + [(-9) – 11] = 11 + (-12) = -1
not exist. ................................................
................................................
844. Find the vertex of y = 5x2 – 4x – 7.
841. Find the equation of the line passing A. ( , ) B. ( , )
through (3, 7) and (-3, -5).
A. y = 2x + 1 B. y = 3x + 1 C. ( , ) D. ( , )
C. y = 2x – 1 D. y = 3x – 1
Solution:
Solution:
The vertex is at (h,k) where
Two-point form of linear equations: h= ; k=c-
y – y1 = (x x ) ( ) ( )
h= ; k = (-7)
y–7= (x 3) ( ) ( )

y – 7 = 2(x – 3) h= ; k = (-7)
y – 7 = 2x – 6 h= ; k = (-7)
y = 2x + 1
h= ; k = (-7)
................................................
h= ; k=

This is a free reviewer. All rights reserved.


1000 MMR Author: Victor A. Tondo Jr., LPT
845. Mr. G sold 70% of his chickens and still had Solution:
129 chickens left. How many chickens did he
Let x = larger number
have originally?
y = smaller number
A. 344 B. 430
C. 598 D. 860 x + y = 73
(+) x – y = 41
Solution: 2x = 114
x = 57
129 = 30% x Original number of chickens
................................................
129 ÷ 0.3 = Original number of chickens = 430
................................................ 850. An item is sold for P7,280 after being
marked down by 30%. What was its original
846. Find the average rate of change of price?
y = x3 – 6x2 + 5x – 20 from x = 1 to x = 6. A. P11,400 B. P10,900
A. 5 B. 6 C. P10,400 D. P9,900
C. 7 D. 8
Solution:
Solution:
( ) ( ) (100% - 30%) OP = 7280
Average Rate of Change =
70% OP = 7280
( )
Average Rate of Change = =6 0.7 OP = 7280
................................................ OP = 7280 ÷ 0.7
OP = 10,400
847. What is the remainder when ................................................
534,214,557,989,215 is divided by 4?
A. 1 B. 2 851. Which of the following lines passes through
C. 3 D. 0 the point (-4, 5)?
A. y = x + 1 B. y = 2x – 3
Explanation: C. y = 1 – x D. y = 3 – 2x
Simply divide the last two digits, 15, by 4 and get
the remainder. Solution:
................................................ Simply substitute x and y from your point. If the
equation holds true, then the given line passes
848. What do you call an equation in which only through your point.
integer solutions are allowed?
A. Diophantine equations y=x+1  5 -4 + 1
B. Euclidean equations y = 2x – 3  5 2(-4) – 3
C. Fibonacci equations y=1–x  5 = 1 – (-4)
D. Newtonian equations y = 3 – 2x  5 3 – 2(-4)
................................................ ................................................
849. The sum of two numbers is 73 and their 852. Which of the following is outside the circle
difference is 41. What is the larger number? defined by the equation (x + 2)2 + y2 = 50?
A. 65 B. 61 A. (-9, 1) B. (-7, 5)
C. 57 D. 53 C. (-5, 7) D. (2, 4)

This is a free reviewer. All rights reserved.


1000 MMR Author: Victor A. Tondo Jr., LPT
Solution: Explanation:
A point is outside the circle when its distance The third side of any triangle must be between
from the center is greater than the radius. the difference and the sum of the two other
A: (-9, 1)  (-9 + 2)2 + 12 = 50 sides.
This point is ON the circle. ................................................
B: (-7, 5)  (-7 + 2)2 + 52 = 50
856. Rayon has 46 coins in P1 and P5
This point is ON the circle.
denominations for a total worth of P186. How
C: (-5, 7)  (-5 + 2)2 + 72 > 50 many P5 coins does he have?
This point is OUTSIDE the circle. A. 25 B. 30 C. 35 D. 40
D: (2, 4)  (2 + 2)2 + 42 < 50
This point is IN the circle. Solution:
................................................ Let x = number of P5 coins
.: 46 – x = number of P1 coins
853. Which of the following equations pertain to
a parabola? 5(x) + 1(46 – x) = 186
A. y2 – 5y = x2 + 4 5x + 46 – x = 186
B. x2 + 5x – 4y = 13 5x – x = 186 – 46
( ) ( ) 4x = 140
C. + =1 x = 35
( ) ( )
D. =1 ................................................

857. If x = 2 and y = 3, what is x2 – y2 + 3xy?


Explanation:
A. 31 B. 13 C. -5 D. -13
A parabola’s equation includes two variables,
one of which is squared. Solution:
................................................
x2 – y2 + 3xy = 22 – 32 + 3(2)(3)
x2 – y2 + 3xy = 4 – 9 + 18 = 13
854. Which of the following equations pertain to
................................................
a parabola that opens to the left?
A. -2(y + 7) = (x – 3)2
858. In an arithmetic sequence, the first term is
B. 5(y – 9) = (x – 5)2
200 and the common difference is -3. What is the
C. (y – 4)2 = 2(x + 7)
64th term?
D. (y + 3)2 = -3(x + 4)
A. 20 B. 17 C. 14 D. 11
Explanation:
Solution:
The parabola opens to the left when the squared
An = A1 + (n – 1)d
variable is y and the coefficient of x is negative.
A64 = A1 + (64 – 1)(-3)
................................................
A64 = 200 + 63(-3)
A64 = 200 – 189
855. The lengths of two sides of a triangle are 7
A64 = 11
inches and 15 inches. Which of the following
................................................
represents x, the possible length in inches of the
remaining side of the triangle?
859. In an arithmetic sequence, the 23rd term is
A. 8 < x < 22 B. 8 x 22
157 and the common difference is -4. What is the
C. x < 8 or x > 22 D. x 8 or x 22
34th term?
A. 113 B. 117 C. 121 D. 125
This is a free reviewer. All rights reserved.
1000 MMR Author: Victor A. Tondo Jr., LPT
Solution: 865. Which of the following Mathematicians had
a method “Sieve” for identifying prime numbers?
An = Am + (n – m)d
A. Pythagoras B. Euclid
A34 = A23 + (34 – 23)(-4)
C. Eratosthenes D. Archimedes
A34 = 157 + 11(-4)
................................................
A34 = 157 + (-44)
A34 = 113
866. Which Mathematician developed a formula
................................................
for finding the area of a triangle using its side
lengths?
860. Find the inverse of y = x2 – 6x + 5.
A. Hipparchus B. Pythagoras
A. √ + 4 + 3 B. √ + 4 – 3 C. Heron D. Euclid
C. √ 4+3 D. √ 4–3 ................................................

Solution: 867. Who refined and perfected the decimal


y = – 6x + 5
x2 place value number system?
y – 5 = x2 – 6x A. Greeks B. Romans
y – 5 + 9 = x2 – 6x + 9 C. Chinese D. Indians
y + 4 = x2 – 6x + 9 ................................................
y + 4 = (x – 3)2
868. This Mathematician invented natural
√ + 4 = √( 3)
logarithms.
√ +4=x–3 A. Isaac Newton B. John Napier
√ +4+3=x C. Marin Mersenne D. Rene Descartes
√ + 4 + 3 = y-1 ................................................
................................................
869. Prime numbers that are 1 less than a power
861. ln = _____
e4 of 2 are called ______.
A. 24.1295 B. 12.06475 A. Pythagorean Primes B. Mersenne Primes
C. 6.032375 D. 4 C. Fermat Primes D. Pascal Primes
................................................
Explanation:
870. Which Mathematician developed the
ln ea = a triangle of binomial coefficients?
................................................ A. Gottfried Leibniz B. Isaac Newton
C. Blaise Pascal D. Johann Bernoulli
862. eln 5 = _____ ................................................
A. 5 B. 6.25 C. 12.5 D. 25
871. Two Mathematicians are known for the
Explanation: theory of hyperbolic geometry although they
eln a = a worked independently from each other. Who are
................................................ these Mathematicians?
A. Euclid and Pythagoras
863. What is ln e? B. Euclid and Descartes
A. 0 B. 1 C. e D. 10 C. Lobachevsky and Bolyai
................................................ D. Babbage and Bolyai

864. Who are the first to use papyrus?


A. Sumerians B. Egyptians
C. Chinese D. Greeks
This is a free reviewer. All rights reserved.
1000 MMR Author: Victor A. Tondo Jr., LPT
872. Which Mathematician developed the use of Solution:
AND, OR, and NOT operators in Algebra?
2 x 48 = 96
A. George Boole
3 x 58 = 174
B. Janos Bolyai
5 x 64 = 320
C. Charles Babbage
Total distance = 590
D. August Mobius
................................................ Average speed =

873. Which Mathematician is known for his last Average speed =


theorem? Average speed = 59 kph
A. Andrew Wiles ................................................
B. Pierre de Fermat
C. John Wallis 878. Find the domain of y = 3x.
D. Rene Descartes A. x | x B. x 0
................................................ C. x 3 D. x 0

874. Who have the first fully-developed base-10 Explanation:


number system in use?
The domain of any exponential function – a
A. Romans B. Egyptians
function where a polynomial in terms of x is used
C. Indians D. Sumerians
as the exponent of a constant – is x | x .
................................................
................................................
875. Notched tally bones proved their use of
Mathematics around 35000 BCE. 879. Convert rad to degrees.
A. Africans B. Asians A. 70o B. 105o
C. Europeans D. Americans C. 210o D. 420o
................................................
Solution:
876. Which sampling method is best used when
the population has subgroups? rad x = 105o
A. systematic sampling 880. Find the volume of a cylinder whose radius
B. stratified sampling is 50 cm and height is 24 cm.
C. quota sampling A. 20,000 π cm3 B. 30,000 π cm3
D. cluster sampling C. 45,000 π cm3 D. 60,000 π cm3

Explanation: Solution:
The root word of the word stratified is stratum, V = π r2 h
meaning subgroups. = 502 (24) π
................................................ = 60,000 π cm3
................................................
877. Mr. Lazada drove for 2 hours at a speed of
48 kph, 3 hours at 58 kph, and then 5 hours at 64 881. Find the altitude to the hypotenuse of a
kph. What was his average speed? right triangle whose legs measure 16 cm and 30
A. 57 kph B. 58 kph cm.
C. 59 kph D. 60 kph A. cm B. 240 cm
C. 34√2 cm D. 24√3 cm

This is a free reviewer. All rights reserved.


1000 MMR Author: Victor A. Tondo Jr., LPT
Solution: Explanation:
Find the hypotenuse first. 295o in in QIV. Therefore, its reference angle is
360 – 295 = 65o.
162 + 302 = h2
................................................
256 + 900 = h2
1156 = h2
886. Which numerical system is sexagesimal
34 = h
(base-60)?
A. Roman B. Babylonian
Altitude to the Hyp = C. Mayan D. Hindu-Arabic
( )
Altitude to the Hyp =
Explanation:
Altitude to the Hyp =
Romans and Hindu-Arabic: base-10 (decimal)
................................................
Mayans: base-20 (vigesimal)
Babylonians: base-60 (sexagesimal)
882. How many line segments can be made from
................................................
25 non-collinear points?
A. 900 B. 600 C. 450 D. 300
887. In solid geometry, what do you call a solid
bound by polygons?
Solution:
A. tessellation B. multigon
25C2 = 300 C. polyhedron D. soligon
................................................ ................................................

883. Dividing by 0.25 is the same as multiplying 888. Two triangles have a pair of congruent
by which number? angles. Which of the following is true about these
A. 2 B. 4 C. 5 D. 16 triangles?
A. They are isosceles triangles.
Explanation: B. They are equilateral triangles.
C. They are congruent triangles.
Just use 1 as your test number.
D. They are similar triangles.
1÷ 0.25 = 4 ................................................
................................................
889. Rayon deposited an amount of P500,000 in
884. Find the surface area of a sphere whose a bank that offers 4% interest compounded per
radius is 24 cm. annum. How much will he have in his account
A. 192 π cm2 B. 768 π cm2 after 3 years?
C. 1152 π cm2 D. 2304 π cm2 A. P720,000 B. P600,000
C. P560,000 D. P562,432
Solution:
Surface Area = 4 π r2 Solution:
Surface Area = 4 (242) π = 2304 π Since the interest is compounded annually,
Acct = Principal x (1 + rate)time
Acct = 500,000 x (1.043)
885. Which of the following is the reference Acct = 500,000 x 1.124864 = 562,432
angle of 295o? ................................................
A. 25o B. 45o C. 65o D. 75o

This is a free reviewer. All rights reserved.


1000 MMR Author: Victor A. Tondo Jr., LPT
890. Find the range of the following scores: 894. The amount of money you have falls under
37 40 44 45 41 39 48 what level of data?
A. nominal B. ordinal
A. 7 B. 9 C. 10 D. 11
C. interval D. ratio
................................................
Explanation:
Range = Highest Score – Lowest Score 895. To study tooth decay a researcher takes a
................................................ sample at random but with the stipulation that
all age groups are represented proportionally.
891. What is the 9th decile of the following data? What sampling method did the researcher use?
13 15 17 19 21 A. systematic B. cluster
23 24 26 28 29 C. stratified D. convenience
30 33 35 36 40
Explanation:
A. 34 B. 35.5 C. 37.6 D. 38
The age groups are the different strata or
Solution: subgroups of the population presented
proportionally.
(15 + 1)th data or 14.4th data ................................................
14th data is 36, while 15th data is 40
896. By what property do we say that when
14.4th data = 36 + 0.4(40 – 36)
A = B and B = C, then A = C?
= 36 + 1.6
A. reflexive B. symmetrical
= 37.6
C. transitive D. closure
................................................
................................................
892. -20192 is NOT equal to ___.
897. In number theory, what do we call a
A. (-2019)2 C. (-20192)
polynomial equation with integer coefficients
B. –(2019 x 2019) D. –(2019)2
that also allows the variables and solutions to be
integers only?
Explanation:
A. Integral equations
(-2019)2 = 4,076,361 B. Differential equations
-20192 = -4,076,361 C. Diophantine equations
................................................ D. Bolyai-Lobachevsky equations
................................................
893. The product of two numbers is 1425. If each
of the two numbers is doubled, the product of 898. How long is the latus rectum of the parabola
these larger numbers is _____. defined by (y + 3)2 = 20x – 196?
A. 2,850 B. 5,700 A. 3 B. 12 C. 20 D. 19
C. 8,550 D. 11.400
Explanation:
Solution:
The length of the latus rectum is equal to the
Let xy = 1425 coefficient of the non-squared variable.
(2x)(2y) = 4xy ................................................
(2x)(2y) = 4(1425)
(2x)(2y) = 5700

This is a free reviewer. All rights reserved.


1000 MMR Author: Victor A. Tondo Jr., LPT
899. In the set of real numbers, what do we call Explanation:
the set that includes only the counting numbers
The score 89 has the highest frequency.
and zero?
................................................
A. rational numbers B. integers
C. whole numbers D. irrational numbers
904. The sum of five consecutive integers is 980.
................................................
What is the value of the greatest integer?
A. 192 B. 194 C. 196 D. 198
900. Find the sum of the first 60 counting even
numbers.
Solution:
A. 3660 B. 3600 C. 1830 D. 900
x + (x + 1) + (x + 2) + (x + 3) + (x + 4) = 980
Solution: 5x + 10 = 980
5x = 970
Sum of the first N counting numbers is given as
x = 194
N2 + N.
................................................ x + 4 = 194 + 4 = 198
................................................
901. If M and N are complementary angles,
which of the following is true? 905. x varies directly as y and inversely as z. If
A. cos M = sec N B. sin M = cos N x = 20 when y = 10 and z = 3, what is x when
C. tan M = csc N D. sec M = -cos N y = 15 and z = 9?
A. 10 B. 15 C. 25 D. 30
Explanation:
Solution:
Sine and Cosine are trigonometric cofunctions.
................................................ x = ky/z 20 = k(10)/3
60 = 10k
902. The top ten students of a graduating class 6=k
got the following scores in their final x = 6y/z
examination in Calculus: x = 6(15)/9
89 84 83 89 89 x = 90/9
85 88 93 83 98 x = 10
What is their mean score? ................................................
A. 87.4 B. 87.5 C. 88.1 D. 89.3
906. Factorize 15x2 – 13x – 20.
Solution: A. (5x – 4)(3x – 5) B. (5x – 4)(3x + 5)
C. (5x + 4)(3x – 5) D. (5x + 4)(3x + 5)
Sum of scores = 881
................................................
881 ÷ 10 = 88.1
................................................ 907. Which of the following is the axis of
symmetry of the parabola defined by the
903. The top ten students of a graduating class equation y = x2 – 6x + 5?
got the following scores in their final A. x = 5 B. x = -5
examination in Calculus: C. x = 3 D. x = -3
89 84 83 89 89
85 88 93 83 98 Explanation:
What is the modal score?
A. 83 B. 85 C. 89 D. 98 The parabola opens upward. Therefore, its axis
of symmetry is x = -B/2A.

This is a free reviewer. All rights reserved.


1000 MMR Author: Victor A. Tondo Jr., LPT
908. Find the radius of x2 + y2 + 12x – 14y = 36. Solution:
A. 6 B. 7
Let x = budget
C. 11 D. 13
x + x + 1700 + 7400 = x
Solution: x + 9100 = x
Complete the squares on the left side of the 9100 = x – x
equation to return it to its center-radius form.
9100 = x
x2 + y2 + 12x – 14y = 36 21000 = x
x2 + 12x + y2 – 14y = 36 ................................................
(x + 12x + 36)+ (y2 – 14y +49)= 36 + 36 + 49
2

(x2 + 12x + 36)+ (y2 – 14y +49)= 121 912. An assistant’s response times were
................................................ recorded on the table below. What is her average
response time in minutes?
909. Find the instantaneous rate of change for Response Time Frequency
f(x) = 4x3 + 7x2 – 27x – 49 at x = 1. 1 minute 3
A. -2 B. -1 C. 1 D. 2 2 minutes 7
3 minutes 11
Solution:
4 minutes 9
Find f ’(1). 5 minutes 5
f ’(x) = 12x2 + 14x – 27 6 minutes 3
f ’(1) = 12(12) + 14(1) – 27 7 minutes 2
f ’(1) = -1
................................................ A. 3.625 B. 3.575 C. 3.525 D. 3.5

Solution:
910. log3 24 – 3 log3 2 = _____
A. 1 B. 0 First, insert a column for fx, which is the product
C. -1 D. e of the frequency and response time in each row.

Solution: Response Time Frequency fx


1 minute 3 3
log3 24 – 3 log3 2 = log3 2 minutes 7 14
= log3 3 3 minutes 11 33
=1 4 minutes 9 36
................................................ 5 minutes 5 25
6 minutes 3 18
911. After using one-sixth of her budget on bills, 7 minutes 2 14
two-fifths on groceries, and P1700 on books and
∑ = 143
magazines, Mrs. Lazada still had P7400 left. How
n = 40
much was her budget?
A. P20,000 143 ÷ 40 = 3.575
B. P21,000 ................................................
C. P22,500
D. P30,000 913. The average grade of 22 students in Section
Abaca is 95, while the average grade of 28
students in Section Acacia is 89. What is the
average grade of all 50 students in both sections?
A. 92.36 B. 91.64 C. 90.72 D. 89.8

This is a free reviewer. All rights reserved.


1000 MMR Author: Victor A. Tondo Jr., LPT
Solution: Solution:
( ) ( ) (180 – x) + (90 – x) = 146
Average = = = 91.64
270 – 2x = 146
................................................
270 – 146 = 2x
124 = 2x
914. Find the slope of 2x – 6y = 13.
62 = x
A. 3 B. C. -3 D. ................................................

Solution: 918. A bag contains some marbles. When the


marbles are grouped by 2, 3, 4, 5, or 6, there is
Convert it to its slope-intercept form.
always one marble left. Which of the following
2x – 6y = 13  -6y = -2x + 13
could be the number of marbles in the bag?
y= x+ A. 31 B. 41 C. 51 D. 61
.: m = or
................................................ Explanation:
N – 1 should be divisible by 2, 3, 4, 5, and 6.
915. Find the intersection of y = -3x – 2 and ................................................
y = 2x + 23.
A. (5, 13) B. (-5, -13) 919. Amitaf, Bernard, and Chloe share a total of
C. (5, -13) D. (-5, 13) P2,460 in the ratio 3:5:4 respectively. How much
is Amitaf’s share?
Solution: A. P600 B. P615
y = -3x – 2 C. P630 D. P660
(-) y = 2x + 23
0 = -5x – 25 Solution:
5x = -25
x = -5; y = 2(-5) + 23 3x + 5x + 4x = 2460
y = 13 12x = 2460
................................................ x = 205
Amitaf’s share is 3(205) or P615.
916. A book was sold for P630 after a 10% ................................................
discount was given. How much was the book
originally? 920. Fifty-four kilometers per hour is equal to
A. P800 B. P750 how many meters per second?
C. P700 D. P690 A. 15 B. 30 C. 54 D. 60

Solution: Solution:
SP = OP (1 – DC Rate) = =
630 = OP (0.9)
630/0.9 = OP ................................................
700 = OP
................................................ 921. Simplify .
A. 1 B. C. D. 2
917. If the sum of the supplement and the
complement of an angle is 146, what is the
angle?
A. 61 B. 62 C. 63 D. 64

This is a free reviewer. All rights reserved.


1000 MMR Author: Victor A. Tondo Jr., LPT
Solution: Solution:
( )
= = =1 =
................................................ = csc x – cot x
................................................
922. If x = 10, which of the following is equal to
219?
A. 23x – 1 B. 2x2 + 5x + 5 926. Simplify: x
C. 2x2 + 2x – 1 D. x2 + x + 2 A. B. C. D.

Explanation: Solution:
Just substitute x with 10.
x = x
A. 23(10) – 1 = 229
B. 2(10)2 + 5(10) + 5 = 255 x =
C. 2(10)2 + 2(10) – 1 = 219
D. (10)2 + (10) + 2 = 112
x =
................................................
................................................
923. In the equation 3x – 4y = 25, what is x when
y is 5? 927. Which of the following is a diagonal matrix?
A. 18 B. 15 C. 12 D. 9 1 6 9 0 2 4
A. [ 2 2 6] B. [1 0 5]
Solution: 3 4 3 2 3 0
3x – 4(5) = 25 4 0 0 3 2 3
3x – 20 = 25 C. [0 5 0 ] D. [2 3 6]
3x = 25 + 20 0 0 6 1 4 3
3x = 45
x = 15 Explanation:
................................................ A diagonal matrix is a square matrix where all its
non-diagonal elements are 0.
924. What is the measure of each interior angle ................................................
of a regular 30-sided polygon?
A. 158o B. 162o 928. Which of the following is a scalar matrix?
C. 165o D. 168o 5 7 9 0 2 3
A. [ 2 8 6] B. [4 0 7]
Solution: 3 4 3 3 8 0
MIA = 180 – 7 0 0 7 0 0
C. [0 5 0] D. [0 7 0]
MIA = 180 – 0 0 6 0 0 7
MIA = 180 – 12
MIA = 168 Explanation:
................................................ A scalar matrix is a diagonal matrix where all its
leading diagonal elements are of equal value.
925. Simplify: ................................................
A. csc x – tan x B. csc x + tan x
C. csc x – cot x D. csc x + cot x

This is a free reviewer. All rights reserved.


1000 MMR Author: Victor A. Tondo Jr., LPT
929. Which of the following is an identity matrix? Solution:
1 0 0 0 5 3
A. [0 1 0] B. [ 4 0 9] =
0 0 1 7 10 0 =2
6 1 1 0 1 1 x + 2 = 2(x – 1)
C. [1 12 1 ] D. [1 0 1] x + 2 = 2x – 2
1 1 18 1 1 0 2 + 2 = 2x – x
4=x
Explanation:
................................................
An identity matrix is a scalar matrix where the
elements on its leading diagonal are 1 and the 933. What are the zeros of 12x2 – x – 35?
rest are of value 0. A. and B. and
................................................
C. and D. and
930. Which of the following is the transposition
1 4 3 Solution:
of A = * +?
5 10 15
12x2 – x – 35 = 0
5 10 15 4 5 15 (4x – 7)(3x + 5) = 0
A. AT = * + B. AT = * +
1 4 3 10 1 3 4x – 7 = 0 3x + 5 = 0
1 5 1 4 3 4x = 7 3x = -5
C. A = [4 10]
T B. A = [5 10 15]
T
x= x=
3 15 1 1 1
................................................
Explanation:
934. What is the sum of the interior angles of a
Transpose A by interchanging the rows and regular dodecagon?
columns. A. 1620o B. 1800o
................................................ C. 1980o D. 2160o

931. When dealing with matrices A, B, and C, Solution:


which of the following is always true?
A. AB = BA SIA = 180(n – 2)
B. If AB = 0, then A = 0 or B = 0 SIA = 1800
C. If AB = AC, then B = C ................................................
D. None of the above.
................................................ 935. 45x – 20 = _____.
A. 45x – 20 B. 5(45x – 20)
932. If the numbers x-1, x+2 and 2x+4 are C. 20(45x – 20) D. (5x – 20)(45x – 20)
consecutive terms of a geometric sequence, what
is x? Solution:
A. 1 B. 2
C. 4 D. 8 au = au du

Let u = 5x – 20 .: du =5

45x – 20 = 45x – 20 (5)

This is a free reviewer. All rights reserved.


1000 MMR Author: Victor A. Tondo Jr., LPT
936. Which of the following graphs have a 937. Which of the following graphs have a
positive leading coefficient and an even degree? negative leading coefficient and an odd degree?

A. A.

B. B.

C. C.

D. D.

Explanation: Explanation:
A positive leading coefficient means the right tail A negative leading coefficient means the right
must point upward. An even degree means that tail must point downward. An odd degree means
the left tail points to the same direction (up) as that the left tail points to the opposite direction
the right tail. as the right tail.
................................................ ................................................

This is a free reviewer. All rights reserved.


1000 MMR Author: Victor A. Tondo Jr., LPT
938. Given f(x) = (x + 2)(x – 1)(2x – 3)(x – 5), 2 +1 3
where can we find f(3)? 941. If ( ) { 7 = 3 , find lim ( ).
A. On the x-axis. 3 3
B. Above the x-axis. A. 3 B. 6
C. Below the x-axis. C. 7 D. Limit does not exist.
D. On the intersection of the x- and y-axis.
Solution:
Explanation: lim ( ) = 2(3) + 1 = 7
f(3) = 5(2)(3)(-2) lim ( ) = 32 – 3 = 6
f(3) = -60
Since f(3) is negative, then it is found below the Since lim ( ) lim ( ), then
x-axis. therefore, limit does not exist.
................................................ ................................................

939. What can be said about the equation f(x) of 942. Give the domain of f(x) = √8 + 24.
the following graph? A. x | x B. x -3
C. x > 24 D. x -3

Explanation:
The given function is a radical function, more
specifically a square root function. If the
A. The leading coefficient is positive. radicand 8x + 24 is negative, f(x) becomes
B. The constant is positive. imaginary. That means 8x + 24 must be greater
C. The degree is 4. than or equal to 0.
D. The function is logarithmic. 8x + 24 0
8x -24
Explanation: x -3
Since the right tail is pointing downward, the ................................................
leading coefficient is negative.
The left tail points upward (opposite of right 943. What is the 3rd term in the expansion of
tail); therefore, the degree is odd. (A + B)5?
Since the graph touches the y-axis above the A. 5A4B B. 10A3B2
origin, then the constant is positive. C. 10A B
2 3 C. 5A3B2
................................................
Solution:
+1 4 First, use the Pascal triangle to find the
940. If ( ) {2 + 1 = 4 , find lim ( ). expansion of (A + B)5:
5 3 4
A. Limit does not exist. B. 9 1
C. 13 D. 17 1 1
1 2 1
Solution: 1 3 3 1
1 4 6 4 1
lim ( ) = 42 + 1 = 17
1 5 10 10 5 1
lim ( ) = 5(4) – 3 = 17
A5 + 5A4B + 10A3B2 + 10A2B3 + 5AB4 + B5
Since lim ( ) = lim ( ), then
lim ( ) = 17.
This is a free reviewer. All rights reserved.
1000 MMR Author: Victor A. Tondo Jr., LPT
944. Find the remainder when 946. In the arithmetic sequence -2, 1, 4, 7, …,
2x4 – 5x3 + 3x2 – 5x + 7 is divided by (x – 1). which term is 100?
A. 1 B. 2 C. 4 D. 7 A. 34th B. 35th C. 36th D. 37th

Solution: Solution:
Use the Remainder Theorem. An = A1 + (n – 1)d
2(14) – 5(13) + 3(12) – 5(1) + 7 = r 100 = -2 + (n – 1)(3)
2–5+3–5+7=r 100 = -2 + 3n – 3
2=r 100 = 3n – 5
................................................ 105 = 3n
35 = n
945. Give the range of f(x) = ................................................
A. y | y B. y 947. Find the length of the intercepted arc of a
C. y D. y 0 central angle measuring 45o given the radius is
80 cm.
Solution: A. 20 π cm B. 30 π cm
C. 40 π cm D. 50 π cm
First, express y in terms of x.
y= Solution:
(5x – 8)y = 3x + 4
Arc length = (2πr)
5xy – 8y = 3x + 4
5xy – 3x = 8y + 4 Arc length = (160 π)
x (5y – 3) = 8y + 4 Arc length = 20 π
x= ................................................

The denominator 5y – 3 cannot be equal to 0. 948. If f(x) = 5x2 – 4x + 9, find the average rate
5y – 3 0 of change from x = 1 to x = 4.
5y 3 A. 20 B. 21 C. 22 D. 23
y
Solution:
Shortcut: f(1) = 5 – 4 + 9 = 10
Since the numerator and denominator are of the f(4) = 80 – 16 + 9 = 73
same degree, just take the leading coefficients of ( ) ( )
the numerator (3) and denominator (5). ARoC = = = 21
................................................
Proof:
949. If f(x) = 4x2 + x – 5, find the instantaneous
 Cross-multiply.
rate of change at x = 5.
5(3x + 4) 3(5x – 8) A. 39 B. 40 C. 41 D. 42
15x + 20 15x – 24
15x – 15x -24 – 20 Solution:
0 -44
f ‘(x) = 8x + 1
f ‘(5) = 8(5) + 1
f ‘(5) = 41

This is a free reviewer. All rights reserved.


1000 MMR Author: Victor A. Tondo Jr., LPT
950. How is 67500 written in scientific notation? 955. Factorize: x3 + 5x2 – 9x – 45
A. 67.5 x 103 B. 6.75 x 103 A. (x + 1)(x + 9)(x – 5)
C. 67.5 x 10 4 D. 6.75 x 104 B. (x + 1)(x – 9)(x + 5)
................................................ C. (x + 3)(x + 3)(x – 5)
D. (x + 3)(x – 3)(x + 5)
951. Find the mean of the following data:
24 24 25 26 28 29 Solution:
A. 24 B. 25.5 C. 26 D. 29 x3 + 5x2 – 9x – 45
= x2 (x + 5) – 9 (x + 5)
Solution: = (x2 – 9)(x + 5)
= (x + 3)(x – 3)(x + 5)
Mean = = 26 ................................................
................................................
956. Which of these is equal to A-1 + B-1?
952. Rayon’s Social Security System (SSS) ID A. ( ) B. +
number is 02-2525255-2. What level of data is
the SSS ID? C. D.
A. nominal B. ordinal
................................................
C. interval D. ratio
957. Find the equation of the line with slope 3,
Explanation:
passing through (2, 5).
ID numbers fall under the nominal level of data. A. y = 3x – 2 B. y = 3x – 1
................................................ C. y = 3x + 1 D. y = 3x + 2

953. A tree is 4 meters tall, while the flagpole is 8


Solution:
meters tall. What level of data is used?
A. nominal B. ordinal y = mx + b
C. interval D. ratio y = 3x + b  slope is 3
5 = 3(2) + b  passing (2, 5)
Explanation: -1 = b
.: y = 3x – 1  m = 3, b = -1
We can infer that the flagpole is literally twice as
................................................
tall as the tree. Height or tallness falls under the
ratio level of data.
958. Find the distance of the point (7, 8) from
................................................
the line 2x – 3y + 4 = 0
√ √
954. The freezing temperature of water is 0o A. B.
Celsius. What level of data is temperature in C. 4√73 D. 5√13
Celsius?
A. nominal B. ordinal Solution:
C. interval D. ratio
Use the formula for distance of point (x1, y1)
Explanation: from line Ax + By + C = 0:
D= √
0o Celsius does not mean the absence of
( ) ( )
temperature or heat. Temperature in Celsius D=
√ ( )
(and also in Fahrenheit) falls under the interval √
level of data. D= =

This is a free reviewer. All rights reserved.


1000 MMR Author: Victor A. Tondo Jr., LPT
959. Find the distance between the parallel lines 962. Which of the following is false?
y = 2x – 1 and y = 2x + 4. A. sin x = B. tan x =
A. 5 B. 2√5 C. √5 D. ½
C. cos x = D. csc x =
Solution:
Explanation:
Use the distance between two parallel lines
You may use the mnemonics SohCahToa and
D=√
ChoShaCao to remember that sin x and csc x are
( ) reciprocals of each other. So are cos x and sec x,
D= = = √5
√ √ and tan x and cot x.
................................................ ................................................

960. There are 80 cows and ducks in a farm, all 963. V and R form a vertical pair. If m V = 7x
of which are healthy. If there are 190 legs in and m R = 3x + 60, find m V.
total, how many cows are there? A. 105o B. 84o C. 20o D. 15o
A. 5 B. 10 C. 15 D. 20
Solution:
Solution:
Since the angles form a vertical pair, then their
Let C = number of cows; measures are equal.
D = number of ducks m V=m R
C + D = 80 7x = 3x + 60
4C + 2D = 190  since cows have four legs 7x – 3x = 60
and ducks have two 4x = 60
x = 15 m V = 7(15) = 105
2(C + D = 80)  2C + 2D = 160
................................................
4C + 2D = 190 - 4C + 2D = 190
-2C = -30
964. V and R form a linear pair. If m V = 13x
C = 15
and m R = 5x + 72, find m R.
................................................ A. 112o B. 102o C. 78o D. 68o

961. Which of the following angles in standard Solution:


position is coterminal with 150o?
Since the angles form a linear pair, then the sum
A. 2570o B. 5490o
of their measures is 180.
C. 7830o D. 9870o
m V + m R = 180
Solution: 13x + 5x + 72 = 180
18x + 72 = 180
= 6.722…  Not a whole number 18x = 180 – 72
18x = 108
= 14.833…  Not a whole number x=6 .:m R = 5(6) + 72 = 102
= 21.333…  Not a whole number ................................................
= 27  Whole number
965. Evaluate: tan 45o – sin 60o + cos 30o
Therefore, 9870o is coterminal with 150o. A. -1 B. 0 C. 1 D. 2

Explanation:
Just use your calculator.

This is a free reviewer. All rights reserved.


1000 MMR Author: Victor A. Tondo Jr., LPT
966. Evaluate: lim Solution:
A. LDNE B. undefined C. 0 D. 27 P = 4S A = S2
=S A = ( )2
Solution:
A=
lim ................................................
( )( )
= lim
970. The distance D of
= lim ( + 3 + 9) a projectile from the
= 32 + 3(3) + 9 ground t seconds
= 27 after launch is
................................................ given as
D = 8t – t2. How
967. Which of the following circles is concentric many seconds
with x2 + y2 – 7x + 9y = 25? after launch does the projectile hit the ground?
A. (x – 3)2 + (y + 5)2 = 25 A. 4 B. 6 C. 7 D. 8
B. (x – 7)2 + (y + 9)2 = 50
C. (x + 3.5)2 + (y – 4.5)2 = 100 Solution:
D. (x – 3.5)2 + (y + 4.5)2 = 125
D = 8t – t2
Explanation: The projectile will be on the ground when D = 0.
The center of x2 + y2 – 7x + 9y = 25 is ( , ) or 0 = 8t – t2
(3.5, -4.5). 0 = t(8 – t)
................................................ t=0 8–t=0
8=t
968. Find k such that (x + 2) is a factor of the The projectile will hit the ground 8 seconds after
polynomial 5x3 + 11x2 – kx + 12. launch.
A. 11 B. 7 C. -3 D. -8 ................................................
Solution: 971. The distance D of a projectile from the
For (x + 2) to be a factor of the polynomial, the ground t seconds after launch is given as
value of f(-2) should be equal to 0. (See Factor D = 14t – t2. How many seconds after launch
Theorem) does it attain its peak?
f(-2) = 5(-2)3 + 11(-2)2 – (-2)k + 12 = 0 A. 4 B. 6 C. 7 D. 8
f(-2) = -40 + 44 + 2k + 12 = 0
16 + 2k = 0 Solution:
2k = -16 D = 14t – t2
k = -8 D’ = 14 – 2t  first derivative
................................................
Max height or peak is attained when D’ = 0.
969. If P = 4S and A = S2, express A in terms of P. 14 – 2t = 0
A. A = B. A = 14 = 2t
C. A = 16P2 D. A = 4P2 7=t

This is a free reviewer. All rights reserved.


1000 MMR Author: Victor A. Tondo Jr., LPT
972. The distance D of a projectile (in meters) 975. Which of the following is a scalar matrix?
from the ground t seconds after launch is given 5 1 2 5 0 0
as D = 16t – 2t2. What is its maximum height? A. [ 4 9 10 ] B. [ 0 5 0]
A. 16 m B. 24 m 8 5 3 0 0 5
C. 32 m D. 40 m 7 1 1 0 2 5
C. [1 5 1] D. [ 9 0 1]
Solution: 1 1 6 1 8 0
D = 16t – 2t2
Explanation:
D’ = 16 – 4t  first derivative
A scalar matrix is a diagonal matrix where all its
Max height or peak is attained when D’ = 0.
leading diagonal elements are of equal value.
16 – 4t = 0 (See item #928)
16 = 4t ................................................
4=t
976. Which of the following is an identity matrix?
Max height = 16(4) – 2(4)2 4 1 1 0 6 3
= 64 – 32 A. [1 8 1 ] B. [ 4 0 2 ]
= 32 1 1 12 7 9 0
................................................
1 0 0 0 1 1
B. [0 1 0] D. [1 0 1]
973. Find the slope of the line tangent to the
0 0 1 1 1 0
graph of f(x) = x3 – 3x2 + 5x – 1 at x = 1.
A. -1 B. 0 C. 1 D. 2 Explanation:
Solution: An identity matrix is a scalar matrix where the
elements on its leading diagonal are 1 and the
f ‘(x) = 3x2 – 6x + 5 rest are of value 0.
f ‘(1) = 3 – 6 + 5 (See item #929)
f ‘(1) = 2 ................................................
................................................
977. Find the quotient when the polynomial
974. Which of the following is a diagonal matrix? 7x5 – 3x3 + 2x2 – 9x + 3 is divided by x – 1.
9 0 0 0 1 4
A. 7x4 + 7x3 + 4x2 + 6x – 3
A. [ 0 3 0] B. [ 1 0 7]
B. 7x4 + 7x3 + 4x2 – 6x – 3
0 0 6 2 3 0
C. 7x4 + 7x3 – 4x2 – 6x – 3
7 6 9 4 2 9 D. 7x4 – 7x3 – 4x2 – 6x – 3
C. [4 3 8] D. [ 1 8 7]
5 4 5 2 4 6 Solution:
Explanation: Use synthetic division.
1 7 0 -3 2 -9 3
A diagonal matrix is a square matrix where all its
7 7 4 6 -3
non-diagonal elements are 0.
7 7 4 6 -3 0
(See item #927)
x4 x3 x2 x c rem

This is a free reviewer. All rights reserved.


1000 MMR Author: Victor A. Tondo Jr., LPT
978. Find k such that 9x2 + 36x + k = 0 has only 981. A square picture was framed and given 3 cm
one unique root. margins. If the total area of the margin is 324
A. 4 B. 16 C. 18 D. 36 cm2, what is the area of the picture?
A. 441 cm2 B. 576 cm2
Solution: C. 729 cm 2 D. 900 cm2
B2 – 4AC = 0
Solution:
362 – 4(9)(k) = 0
362 – 36k = 0
362 = 36k (x + 3 + 3)2 – x2 = 324
36 = k (x + 6)2 – x2 = 324
................................................ x + 12x + 36 – x2 = 324
2

12x + 36 = 324
979. There are 210 candies in a jar. The ratio of 12x = 288
red candies to blue is 2:3, and the ratio of blue to x = 24
yellow is 4:5. How many yellow candies are .: x2 = 576
there? ................................................
A. 35 B. 45 C. 60 D. 90
982. Given f(x) = x2 + 10x + 25 and
Solution: g(x) = 3x + 1, find f(g(x)).
A. 3x2 + 30x + 26 B. 9x2 + 36x + 36
First, make a common ratio.
C. 9x + 30x + 26
2 D. 3x2 + 36x + 36
Red Blue Yellow
2 3 Solution:
4 5 f(g(x)) = (g(x))2 + 10(g(x)) + 25
8 12 15 f(g(x)) = (3x + 1)2 + 10(3x + 1) + 25
8x + 12x + 15x = 210 f(g(x)) = (9x2 + 6x + 1) + (30x + 10) + 25
35x = 210 f(g(x)) = 9x2 + 36x + 36
x=6 ................................................
Number of yellow candies = 15x = 15(6) = 90 983. A whole number is 3 more than another
................................................ number. The sum of their squares is 4145. Find
the larger number.
980. A certain bacteria doubles its population A. 47 B. 46 C. 45 D. 44
after 3 minutes. If the bacteria in a petri dish is
512,000 at 8:45 AM, at what time was its Solution:
population count 125?
A. 6:30 AM B. 7:15 AM Let x = larger number
C. 8:09 AM D. 9:21 AM x – 3 = smaller number
x2 + (x – 3)2 = 4145
Solution: x2 + (x2 – 6x + 9) = 4145
125 x 2n = 512,000 2x2 – 6x + 9 = 4145
2n = 4096 2x2 – 6x = 4136
2n = 212 x2 – 3x = 2068
n = 12 x2 – 3x + 2.25 = 2068 + 2.25
(x – 1.5)2 = 2070.25
12(3) = 36 x – 1.5 = 45.5
36 minutes before 8:45 AM is 8:09 AM. x = 47

This is a free reviewer. All rights reserved.


1000 MMR Author: Victor A. Tondo Jr., LPT
984. The numbers x, y, z, and w have an average 988. In a certain university, a student’s grade is
of 30. If x, y and z have an average 35, what is w? computed as the sum of 25% of his prelims
A. 5 B. 10 C. 15 D. 20 grade, 30% of his midterms grade, and 45% of
his finals grade. He knows that his prelims grade
Solution: is 72 and his midterms grade is 70. What is his
grade for the finals if he got a grade of 75 in
x + y + z + w = 4(30)
Calculus?
x + y + z + w = 120
A. 75 B. 77 C. 79 D. 80
x + y + z = 3(35)
x + y + z = 105 Solution:
120 – 105 = 15 0.25(72) + 0.3(70) + 0.45(X) = 75
................................................ 18 + 21 + 0.45X = 75
39 + 0.45X = 75
985. A is a constant. Find A such that the 0.45X = 36
equation 2x + 1 = 2A + 3(x + A) has a solution X = 80
at x = 2. ................................................
A. -0.4 B. -0.2 C. 0 D. 0.2
989. Mr. Lazada bought 75 pieces of Elunium for
Solution: 8,000 per piece and sold them for a total of
720,000. What is his mark-up rate?
2(2) + 1 = 2A + 3(2 + A)
A. 20% B. 25%
5 = 2A + 6 + 3A
C. 32.5% D. 40%
-1 = 5A
-0.2 = A
Solution:
................................................
( )
MUR = x 100%
( )
986. It takes 4 men 9 days to build 2 houses. How
many days will it take 6 men to build 5 houses? MUR = x 100%
A. 12 B. 13 C. 14 D. 15 MUR = 20%
................................................
Solution:
990. Triangular numbers are numbers that can
4m (9d) = 2h 6m (kd) = 5h
be shown by triangular arrangements of dots.
36md = 2h 6m (kd) = 5(18md)
The triangular numbers are 1, 3, 6, 10, 15, 21, …
18md = h 6m (kd) = 90 md
What is the 20th triangular number?
kd = 15d
A. 200 B. 205 C. 210 D. 220
k = 15
................................................
Explanation:
987. If 860 = 32x, what is x? The Nth triangular number is .
A. 18 B. 24 C. 20 D. 36 ................................................

Solution: 991. What is a pattern of shapes that covers a


860 = 32x surface completely without overlaps or gaps?
(23)60 = (25)x A. translation B. tessellation
2180 = 25x C. constellation D. transposition
180 = 5x ................................................
36 = x

This is a free reviewer. All rights reserved.


1000 MMR Author: Victor A. Tondo Jr., LPT
992. Find the equation of the line perpendicular 995. The first term of an arithmetic sequence is 3
to 5x + 3y = 12, passing through (3, -7). and the 20th term is 98. What is the 25th term?
A. 3x – 5y = -44 B. 3x + 5y = 44 A. 108 B. 115 C. 118 D. 123
C. 3x – 5y = 44 D. 3x + 5y = -44
Solution:
Solution:
d=
The perpendicular line for Ax + By = C is given
as Bx – Ay = B(xp) – A(yp). d= =5
3x – 5y = 3(3) – 5(-7) A25 = A1 + (n – 1) d
3x – 5y = 9 – (-35) A25 = 3 + (25 – 1)(5)
3x – 5y = 44 A25 = 123
................................................ ................................................
993. Point V is two-fifths the way from A(9, -7) 996. An educational psychologist classifies
to B(-6, 13). Find the coordinates of point V. students as high, medium and low intelligence.
A. (2, 2) B. (2, 0) What kind of scale is being used?
C. (3, 1) D. (3, 2) A. nominal scale B. ordinal scale
C. interval scale D. ratio scale
Solution: ................................................
x = 9 + ((-6) – 9)
997. The GCF of two numbers is 8 and their LCM
x = 9 + (-15) is 80. What is their product?
x = 9 + (-6) A. 80 B. 160 D. 320 D. 640
x=3
Explanation:
y = (-7)+ (13 – (-7))
The product of two numbers is equal to the
y = (-7)+ (20) product of their GCF and LCM.
y = (-7)+ 8 ................................................
y=1
................................................ 998. If c1.5 – 4 = 7, what is c3?
A. 784 B. 121 C. 49 D. 16
994. Find the angle of inclination of the line
passing through V(5, -5) and T(2, 4) Solution:
A. 88.145o B. 92.429o
C. 97.593 o D. 108.435o c1.5 – 4 = 7
c1.5 = 11
Solution: (c1.5)2 = 112
c3 = 121
( )
Slope = = -3 ................................................

Angle of inclination = tan-1 (-3) 999. A certain Math challenge gives the
Angle of inclination = -71.565o or 108.435o competitors a score of 4 for each correct answer,
and a deduction of 1 point for each wrong
answer. If a contestant answered all 100 items
and got a score of 200, how many items did the
contestant answer correctly?
A. 45 B. 50 C. 60 D. 35

This is a free reviewer. All rights reserved.


1000 MMR Author: Victor A. Tondo Jr., LPT
Solution:
Let C = number of correct answers
.: 100 – C = number of wrong answers
(4)C + (-1)(100 – C) = 200
4C – 100 + C = 200
5C = 300
C = 60

1000. What is 111011012 in decimal?


A. 257 B. 237
C. 217 D. 197

Solution:
101011012  ____10
1 x 27 = 128
1 x 26 = 64
1 x 25 = 32
0 x 24 = 0
1 x 23 = 8
1 x 22 = 4
0 x 21 = 0
1x 20 = 1
237

End of 1000MMR.

Congratulations!!

This is a free reviewer. All rights reserved.

Potrebbero piacerti anche